Unnamed: 0 Prompt Answer wikipedia_link_1 wikipedia_link_2 wikipedia_link_3 wikipedia_link_4 wikipedia_link_5 wikipedia_link_6 wikipedia_link_7 wikipedia_link_8 wikipedia_link_9 wikipedia_link_10 wikipedia_link_11+ reasoning_types wiki_links 0 0 If my future wife has the same first name as the 15th first lady of the United States' mother and her surname is the same as the second assassinated president's mother's maiden name, what is my future wife's name? Jane Ballou https://en.wikipedia.org/wiki/President_of_the_United_States https://en.wikipedia.org/wiki/James_Buchanan https://en.wikipedia.org/wiki/Harriet_Lane https://en.wikipedia.org/wiki/List_of_presidents_of_the_United_States_who_died_in_office https://en.wikipedia.org/wiki/James_A._Garfield Multiple constraints ['https://en.wikipedia.org/wiki/President_of_the_United_States', 'https://en.wikipedia.org/wiki/James_Buchanan', 'https://en.wikipedia.org/wiki/Harriet_Lane', 'https://en.wikipedia.org/wiki/List_of_presidents_of_the_United_States_who_died_in_office', 'https://en.wikipedia.org/wiki/James_A._Garfield'] 1 1 Imagine there is a building called Bronte tower whose height in feet is the same number as the dewey decimal classification for the Charlotte Bronte book that was published in 1847. Where would this building rank among tallest buildings in New York City, as of August 2024? 37th https://en.wikipedia.org/wiki/Charlotte_Bront%C3%AB https://en.wikipedia.org/wiki/Jane_Eyre https://en.wikipedia.org/wiki/List_of_tallest_buildings_in_New_York_City Numerical reasoning | Tabular reasoning | Multiple constraints ['https://en.wikipedia.org/wiki/Charlotte_Bront%C3%AB', 'https://en.wikipedia.org/wiki/Jane_Eyre', 'https://en.wikipedia.org/wiki/List_of_tallest_buildings_in_New_York_City'] 2 2 How many years earlier would Punxsutawney Phil have to be canonically alive to have made a Groundhog Day prediction in the same state as the US capitol? 87 https://en.wikipedia.org/wiki/Punxsutawney_Phil https://en.wikipedia.org/wiki/United_States_Capitol Numerical reasoning | Multiple constraints | Temporal reasoning ['https://en.wikipedia.org/wiki/Punxsutawney_Phil', 'https://en.wikipedia.org/wiki/United_States_Capitol'] 3 3 As of August 1, 2024, which country were holders of the FIFA World Cup the last time the UEFA Champions League was won by a club from London? France https://en.wikipedia.org/wiki/FIFA_World_Cup https://en.wikipedia.org/wiki/London https://en.wikipedia.org/wiki/UEFA_Champions_League Tabular reasoning | Multiple constraints | Temporal reasoning ['https://en.wikipedia.org/wiki/FIFA_World_Cup', 'https://en.wikipedia.org/wiki/London', 'https://en.wikipedia.org/wiki/UEFA_Champions_League'] 4 4 What is the name of the vocalist from the first band to make it in the top 200 under the record label that produced the third studio album for Dismal Euphony? Jens Kidman https://en.wikipedia.org/wiki/Dismal_Euphony https://en.wikipedia.org/wiki/All_Little_Devils https://en.wikipedia.org/wiki/Nuclear_Blast https://en.wikipedia.org/wiki/Meshuggah Multiple constraints ['https://en.wikipedia.org/wiki/Dismal_Euphony', 'https://en.wikipedia.org/wiki/All_Little_Devils', 'https://en.wikipedia.org/wiki/Nuclear_Blast', 'https://en.wikipedia.org/wiki/Meshuggah'] 5 5 According to the 2000 United States census, what was the 2000 population of the birth city of the only 21st-century mayor of Austin, Texas who also served as mayor in the 1990s? Round your answer to the nearest thousand. 506000 https://en.wikipedia.org/wiki/Mayor_of_Austin https://en.wikipedia.org/wiki/Kirk_Watson https://en.wikipedia.org/wiki/Oklahoma_City Numerical reasoning | Tabular reasoning | Multiple constraints ['https://en.wikipedia.org/wiki/Mayor_of_Austin', 'https://en.wikipedia.org/wiki/Kirk_Watson', 'https://en.wikipedia.org/wiki/Oklahoma_City'] 6 6 I have an element in mind and would like you to identify the person it was named after. Here's a clue: The element's atomic number is 9 higher than that of an element discovered by the scientist who discovered Zirconium in the same year. Mendelevium is named after Dmitri Mendeleev. https://en.wikipedia.org/wiki/Zirconium https://en.wikipedia.org/wiki/Martin_Heinrich_Klaproth https://en.wikipedia.org/wiki/Uranium https://en.wikipedia.org/wiki/Mendelevium https://en.wikipedia.org/wiki/Periodic_table Numerical reasoning | Multiple constraints | Temporal reasoning ['https://en.wikipedia.org/wiki/Zirconium', 'https://en.wikipedia.org/wiki/Martin_Heinrich_Klaproth', 'https://en.wikipedia.org/wiki/Uranium', 'https://en.wikipedia.org/wiki/Mendelevium', 'https://en.wikipedia.org/wiki/Periodic_table'] 7 7 "As of Aug 3, 2024, the artist who released the album ""Father of Asahd"" went to the same high school as an Olympic diver. How many Olympic teams did this diver participate on?" 2 https://en.wikipedia.org/wiki/Father_of_Asahd https://en.wikipedia.org/wiki/DJ_Khaled https://en.wikipedia.org/wiki/Dr._Phillips_High_School https://en.wikipedia.org/wiki/Mark_Ruiz Multiple constraints | Temporal reasoning ['https://en.wikipedia.org/wiki/Father_of_Asahd', 'https://en.wikipedia.org/wiki/DJ_Khaled', 'https://en.wikipedia.org/wiki/Dr._Phillips_High_School', 'https://en.wikipedia.org/wiki/Mark_Ruiz'] 8 8 A general motors vehicle is named after the largest ward in the country of Monaco. How many people had walked on the moon as of the first model year of the vehicle? Note: the model year is not the same as the year the model was first produced. 4 https://en.wikipedia.org/wiki/Monaco https://en.wikipedia.org/wiki/List_of_Chevrolet_vehicles https://en.wikipedia.org/wiki/Chevrolet_Monte_Carlo https://en.wikipedia.org/wiki/Moon https://en.wikipedia.org/wiki/Apollo_program https://en.wikipedia.org/wiki/List_of_Apollo_missions Tabular reasoning | Multiple constraints | Temporal reasoning ['https://en.wikipedia.org/wiki/Monaco', 'https://en.wikipedia.org/wiki/List_of_Chevrolet_vehicles', 'https://en.wikipedia.org/wiki/Chevrolet_Monte_Carlo', 'https://en.wikipedia.org/wiki/Moon', 'https://en.wikipedia.org/wiki/Apollo_program', 'https://en.wikipedia.org/wiki/List_of_Apollo_missions'] 9 9 The Pope born Pietro Barbo ended a long-running war two years after his papacy began, which famous conflict, immortalized in tapestry took place 400 years earlier? The Battle of Hastings. https://en.wikipedia.org/wiki/Pope_Paul_II https://en.wikipedia.org/wiki/Thirteen_Years%27_War_(1454%E2%80%931466) https://en.wikipedia.org/wiki/Bayeux_Tapestry https://en.wikipedia.org/wiki/Battle_of_Hastings Temporal reasoning ['https://en.wikipedia.org/wiki/Pope_Paul_II', 'https://en.wikipedia.org/wiki/Thirteen_Years%27_War_(1454%E2%80%931466)', 'https://en.wikipedia.org/wiki/Bayeux_Tapestry', 'https://en.wikipedia.org/wiki/Battle_of_Hastings'] 10 10 An Australian artist, born the same year as artist Janet Cumbrae Stewart and fellow member of the Melbourne Society of Women Painters and Sculptors, had her painting featured on the cover of Women's World Magazine in 1923. What is the name of the painting? Reve d'Or https://en.wikipedia.org/wiki/Janet_Cumbrae_Stewart https://en.wikipedia.org/wiki/Melbourne_Society_of_Women_Painters_and_Sculptors https://en.wikipedia.org/wiki/Dora_Wilson Multiple constraints | Temporal reasoning ['https://en.wikipedia.org/wiki/Janet_Cumbrae_Stewart', 'https://en.wikipedia.org/wiki/Melbourne_Society_of_Women_Painters_and_Sculptors', 'https://en.wikipedia.org/wiki/Dora_Wilson'] 11 11 As of July 1, 2024, what is the parent company of the current record label of the singer of Edge of Seventeen? Warner Music Group https://en.wikipedia.org/wiki/Edge_of_Seventeen https://en.wikipedia.org/wiki/Stevie_Nicks https://en.wikipedia.org/wiki/Reprise_Records https://en.wikipedia.org/wiki/Atlantic_Records https://en.wikipedia.org/wiki/Modern_Records_(1980) https://en.wikipedia.org/wiki/Warner_Music_Group https://en.wikipedia.org/wiki/Warner_Records Tabular reasoning | Multiple constraints ['https://en.wikipedia.org/wiki/Edge_of_Seventeen', 'https://en.wikipedia.org/wiki/Stevie_Nicks', 'https://en.wikipedia.org/wiki/Reprise_Records', 'https://en.wikipedia.org/wiki/Atlantic_Records', 'https://en.wikipedia.org/wiki/Modern_Records_(1980)', 'https://en.wikipedia.org/wiki/Warner_Music_Group', 'https://en.wikipedia.org/wiki/Warner_Records'] 12 12 The Basibasy mine is located in Madagascar. This mine is abundant in a specific chemical element that was discovered for the first time in 1791. The person who discovered this element was born on what is now known as a major US holiday - what holiday is this? Christmas https://en.wikipedia.org/wiki/Basibasy_mine https://en.wikipedia.org/wiki/Titanium https://en.wikipedia.org/wiki/William_Gregor https://en.wikipedia.org/wiki/Christmas Multiple constraints ['https://en.wikipedia.org/wiki/Basibasy_mine', 'https://en.wikipedia.org/wiki/Titanium', 'https://en.wikipedia.org/wiki/William_Gregor', 'https://en.wikipedia.org/wiki/Christmas'] 13 13 One of Barbara Kingsolver's best known novels is about an American missionary family which moves to Africa. At the time, the country they move to was a Belgian colony. Which year did it become independent? 1960 https://en.wikipedia.org/wiki/Barbara_Kingsolver https://en.wikipedia.org/wiki/The_Poisonwood_Bible https://en.wikipedia.org/wiki/Belgian_Congo Multiple constraints ['https://en.wikipedia.org/wiki/Barbara_Kingsolver', 'https://en.wikipedia.org/wiki/The_Poisonwood_Bible', 'https://en.wikipedia.org/wiki/Belgian_Congo'] 14 15 Which football player got 15 or more assists in La Liga during the 2010-2011 season and also played for Arsenal at one point in his career? This was Mesut Ozil. https://en.wikipedia.org/wiki/2010–11_La_Liga https://en.wikipedia.org/wiki/Mesut_Özil https://en.wikipedia.org/wiki/Dani_Alves https://en.wikipedia.org/wiki/Lionel_Messi Numerical reasoning | Tabular reasoning | Multiple constraints ['https://en.wikipedia.org/wiki/2010–11_La_Liga', 'https://en.wikipedia.org/wiki/Mesut_Özil', 'https://en.wikipedia.org/wiki/Dani_Alves', 'https://en.wikipedia.org/wiki/Lionel_Messi'] 15 16 In Slovakia there is a well known Film Festival called the Bratistlava International Film Festival. What city/ town was the film editor for the Grand Prix winner of 2003 born in? Roudnice nad Labem https://en.wikipedia.org/wiki/Bratislava_International_Film_Festival https://en.wikipedia.org/wiki/Boredom_in_Brno https://en.wikipedia.org/wiki/Ji%C5%99%C3%AD_Bro%C5%BEek https://en.wikipedia.org/wiki/Roudnice_nad_Labem Tabular reasoning | Multiple constraints ['https://en.wikipedia.org/wiki/Bratislava_International_Film_Festival', 'https://en.wikipedia.org/wiki/Boredom_in_Brno', 'https://en.wikipedia.org/wiki/Ji%C5%99%C3%AD_Bro%C5%BEek', 'https://en.wikipedia.org/wiki/Roudnice_nad_Labem'] 16 17 On March 7th, 2012, the director James Cameron explored a very deep underseas trench. As of August 3, 2024, how many times would the tallest building in San Francisco fit end to end from the bottom of the New Britain Trench to the surface of the ocean? The answer should be a rounded-off whole number. 28 https://en.wikipedia.org/wiki/James_Cameron https://en.wikipedia.org/wiki/Solomon_Sea#Deepest_point https://en.wikipedia.org/wiki/List_of_tallest_buildings_in_San_Francisco Numerical reasoning | Tabular reasoning | Post processing | Temporal reasoning ['https://en.wikipedia.org/wiki/James_Cameron', 'https://en.wikipedia.org/wiki/Solomon_Sea#Deepest_point', 'https://en.wikipedia.org/wiki/List_of_tallest_buildings_in_San_Francisco'] 17 18 In August of 2024, what is the first name of the mayor of the U.S. state capital city who attended the same university as at least one U.S. president and whose city is home to an outgoing or former full member of the Big 12 Conference Leirion https://en.wikipedia.org/wiki/Big_12_Conference https://en.wikipedia.org/wiki/Lincoln,_Nebraska https://en.wikipedia.org/wiki/Leirion_Gaylor_Baird https://en.wikipedia.org/wiki/List_of_presidents_of_the_United_States_by_education Multiple constraints ['https://en.wikipedia.org/wiki/Big_12_Conference', 'https://en.wikipedia.org/wiki/Lincoln,_Nebraska', 'https://en.wikipedia.org/wiki/Leirion_Gaylor_Baird', 'https://en.wikipedia.org/wiki/List_of_presidents_of_the_United_States_by_education'] 18 19 How many years after the founding of the 50th most populous US city, based on 2023 estimate population data, did Frank Fox receive UK Patent (1344259)? 98 Years (Arlington, TX & Rubik's Cube) https://en.wikipedia.org/wiki/List_of_United_States_cities_by_population https://en.wikipedia.org/wiki/Arlington,_Texas https://en.wikipedia.org/wiki/Rubik%27s_Cube Numerical reasoning | Tabular reasoning | Multiple constraints | Temporal reasoning ['https://en.wikipedia.org/wiki/List_of_United_States_cities_by_population', 'https://en.wikipedia.org/wiki/Arlington,_Texas', 'https://en.wikipedia.org/wiki/Rubik%27s_Cube'] 19 20 As of August 4, 2024, in what state was the first secretary of the latest United States federal executive department born? Pennsylvania https://en.wikipedia.org/wiki/United_States_federal_executive_departments#Former_departments https://en.wikipedia.org/wiki/United_States_Secretary_of_Homeland_Security https://en.wikipedia.org/wiki/Tom_Ridge Tabular reasoning | Multiple constraints ['https://en.wikipedia.org/wiki/United_States_federal_executive_departments#Former_departments', 'https://en.wikipedia.org/wiki/United_States_Secretary_of_Homeland_Security', 'https://en.wikipedia.org/wiki/Tom_Ridge'] 20 21 As of August 1 2024, what is the most recently described genus of Colosteidae? Deltaherpeton, first described in 2010 https://en.wikipedia.org/wiki/Colosteidae https://en.wikipedia.org/wiki/Deltaherpeton https://en.wikipedia.org/wiki/Greererpeton https://en.wikipedia.org/wiki/Pholidogaster Tabular reasoning | Temporal reasoning ['https://en.wikipedia.org/wiki/Colosteidae', 'https://en.wikipedia.org/wiki/Deltaherpeton', 'https://en.wikipedia.org/wiki/Greererpeton', 'https://en.wikipedia.org/wiki/Pholidogaster'] 21 22 Małgorzata Rożniecka is a model who won the title of Miss International. What is the difference in title years from when she won and the pageant winner who was murdered by her stalker? 10 years https://en.wikipedia.org/wiki/Ma%C5%82gorzata_Ro%C5%BCniecka https://en.wikipedia.org/wiki/Miss_International https://en.wikipedia.org/wiki/Agnieszka_Kotlarska Numerical reasoning | Tabular reasoning ['https://en.wikipedia.org/wiki/Ma%C5%82gorzata_Ro%C5%BCniecka', 'https://en.wikipedia.org/wiki/Miss_International', 'https://en.wikipedia.org/wiki/Agnieszka_Kotlarska'] 22 23 According to the 1990 United States census, what was the total population of the cities in Oklahoma that had at least 100,000 residents according to the 2020 United States census? 950135 https://en.wikipedia.org/wiki/List_of_United_States_cities_by_population https://en.wikipedia.org/wiki/Oklahoma_City https://en.wikipedia.org/wiki/Tulsa,_Oklahoma https://en.wikipedia.org/wiki/Broken_Arrow,_Oklahoma https://en.wikipedia.org/wiki/Norman,_Oklahoma Numerical reasoning | Tabular reasoning | Multiple constraints ['https://en.wikipedia.org/wiki/List_of_United_States_cities_by_population', 'https://en.wikipedia.org/wiki/Oklahoma_City', 'https://en.wikipedia.org/wiki/Tulsa,_Oklahoma', 'https://en.wikipedia.org/wiki/Broken_Arrow,_Oklahoma', 'https://en.wikipedia.org/wiki/Norman,_Oklahoma'] 23 24 What was the political party of the person who advocated for the type of government used in Chikhali, Latur district to become the foundation of India's political system? Indian National Congress https://en.wikipedia.org/wiki/Chikhali,_Latur_district https://en.wikipedia.org/wiki/Panchayati_raj https://en.wikipedia.org/wiki/Mahatma_Gandhi Tabular reasoning | Multiple constraints ['https://en.wikipedia.org/wiki/Chikhali,_Latur_district', 'https://en.wikipedia.org/wiki/Panchayati_raj', 'https://en.wikipedia.org/wiki/Mahatma_Gandhi'] 24 25 Giorgio Rognoni was an Italian professional footballer who played as a midfielder. 10 years after his death who was the midfielder who played in Milan that was born in Besana in Brianza,? Demetrio Albertini https://en.wikipedia.org/wiki/Giorgio_Rognoni https://en.wikipedia.org/wiki/1996%E2%80%9397_AC_Milan_season https://en.wikipedia.org/wiki/Demetrio_Albertini Numerical reasoning ['https://en.wikipedia.org/wiki/Giorgio_Rognoni', 'https://en.wikipedia.org/wiki/1996%E2%80%9397_AC_Milan_season', 'https://en.wikipedia.org/wiki/Demetrio_Albertini'] 25 27 What was the age difference between Mike Tyson and Tyson Fury on the respective days on which they lost their first ever fights? Represent the figure in years only. 12 years. https://en.wikipedia.org/wiki/Tyson_Fury https://en.wikipedia.org/wiki/Mike_Tyson https://en.wikipedia.org/wiki/Mike_Tyson_vs._Buster_Douglas https://en.wikipedia.org/wiki/Tyson_Fury_vs_Oleksandr_Usyk Numerical reasoning | Post processing ['https://en.wikipedia.org/wiki/Tyson_Fury', 'https://en.wikipedia.org/wiki/Mike_Tyson', 'https://en.wikipedia.org/wiki/Mike_Tyson_vs._Buster_Douglas', 'https://en.wikipedia.org/wiki/Tyson_Fury_vs_Oleksandr_Usyk'] 26 28 Using the Pruett rule, out of all of the blue moons that occurred between the beginning of World War I and the end of World War II, how many of them occurred on the 31st of the month? 9 https://en.wikipedia.org/wiki/Blue_moon#Blue_moon_dates https://en.wikipedia.org/wiki/World_War_I# https://en.wikipedia.org/wiki/World_War_II Numerical reasoning | Tabular reasoning | Multiple constraints | Temporal reasoning ['https://en.wikipedia.org/wiki/Blue_moon#Blue_moon_dates', 'https://en.wikipedia.org/wiki/World_War_I#', 'https://en.wikipedia.org/wiki/World_War_II'] 27 29 What number would Tommy Lawton have worn playing for Chelsea FC? 9 https://en.wikipedia.org/wiki/Tommy_Lawton https://en.wikipedia.org/wiki/Squad_number_(association_football) Temporal reasoning ['https://en.wikipedia.org/wiki/Tommy_Lawton', 'https://en.wikipedia.org/wiki/Squad_number_(association_football)'] 28 30 "If you subtract the year that William McCrary ""Billy"" Ray II was born from the year Obama was first sworn in as President to the United States and multiply it by the number of administrative regions in France as of January 1, 2024, what number would you get?" 828 https://en.wikipedia.org/wiki/William_M._Ray_II https://en.wikipedia.org/wiki/Barack_Obama https://en.wikipedia.org/wiki/Regions_of_France Numerical reasoning ['https://en.wikipedia.org/wiki/William_M._Ray_II', 'https://en.wikipedia.org/wiki/Barack_Obama', 'https://en.wikipedia.org/wiki/Regions_of_France'] 29 31 If Princess Diana had been born three years earlier, who would have been Prime Minister when she was ten? Harold Wilson https://en.wikipedia.org/wiki/Diana,_Princess_of_Wales https://en.wikipedia.org/wiki/List_of_prime_ministers_of_the_United_Kingdom Numerical reasoning | Multiple constraints | Temporal reasoning ['https://en.wikipedia.org/wiki/Diana,_Princess_of_Wales', 'https://en.wikipedia.org/wiki/List_of_prime_ministers_of_the_United_Kingdom'] 30 32 "As of August 1, 2024, what is the population of the writer of the ""Culdcept Saga""'s birthplace? Write the answer to the nearest million, in characters." Two million. https://en.wikipedia.org/wiki/Culdcept_Saga https://en.wikipedia.org/wiki/Tow_Ubukata https://en.wikipedia.org/wiki/Gifu_Prefecture Numerical reasoning | Multiple constraints | Post processing ['https://en.wikipedia.org/wiki/Culdcept_Saga', 'https://en.wikipedia.org/wiki/Tow_Ubukata', 'https://en.wikipedia.org/wiki/Gifu_Prefecture'] 31 33 What is the middle name of the U.S. president who died on the same day of the year as Virginia Woolf? David https://en.wikipedia.org/wiki/Virginia_Woolf https://en.wikipedia.org/wiki/List_of_presidents_of_the_United_States_by_date_of_death https://en.wikipedia.org/wiki/Dwight_D._Eisenhower Multiple constraints ['https://en.wikipedia.org/wiki/Virginia_Woolf', 'https://en.wikipedia.org/wiki/List_of_presidents_of_the_United_States_by_date_of_death', 'https://en.wikipedia.org/wiki/Dwight_D._Eisenhower'] 32 34 As of 2010, if you added the number of times Brazil had won the World Cup to the amount of times the Chicago Bulls had won the NBA Championship and multiplied this number by the amount of times the Dallas Cowboys had won the Super Bowl, what number are you left with? 55 https://en.wikipedia.org/wiki/FIFA_World_Cup https://en.wikipedia.org/wiki/Chicago_Bulls https://en.wikipedia.org/wiki/Dallas_Cowboys Numerical reasoning | Tabular reasoning ['https://en.wikipedia.org/wiki/FIFA_World_Cup', 'https://en.wikipedia.org/wiki/Chicago_Bulls', 'https://en.wikipedia.org/wiki/Dallas_Cowboys'] 33 35 How old would the founder of the publishing company of the magazine that serialized the manga series Raw Hero have been the year the magazine ended publication? 145 https://en.m.wikipedia.org/wiki/Raw_Hero https://en.m.wikipedia.org/wiki/Evening_(magazine) https://en.m.wikipedia.org/wiki/Kodansha https://en.m.wikipedia.org/wiki/Seiji_Noma Numerical reasoning | Multiple constraints | Temporal reasoning ['https://en.m.wikipedia.org/wiki/Raw_Hero', 'https://en.m.wikipedia.org/wiki/Evening_(magazine)', 'https://en.m.wikipedia.org/wiki/Kodansha', 'https://en.m.wikipedia.org/wiki/Seiji_Noma'] 34 36 The oldest extant football team in Italy plays in a stadium. The stadium is named after a person. Who was the emperor of China when that person was 5 years old? Guangxu https://en.wikipedia.org/wiki/Oldest_football_clubs https://en.wikipedia.org/wiki/Genoa_CFC https://en.wikipedia.org/wiki/Stadio_Luigi_Ferraris https://en.wikipedia.org/wiki/Luigi_Ferraris_(footballer) https://en.wikipedia.org/wiki/List_of_Chinese_monarchs Numerical reasoning | Tabular reasoning | Temporal reasoning ['https://en.wikipedia.org/wiki/Oldest_football_clubs', 'https://en.wikipedia.org/wiki/Genoa_CFC', 'https://en.wikipedia.org/wiki/Stadio_Luigi_Ferraris', 'https://en.wikipedia.org/wiki/Luigi_Ferraris_(footballer)', 'https://en.wikipedia.org/wiki/List_of_Chinese_monarchs'] 35 37 Of the four main characters on Seinfeld, which actor is the oldest? Michael Richards https://en.wikipedia.org/wiki/Seinfeld https://en.wikipedia.org/wiki/Jerry_Seinfeld https://en.wikipedia.org/wiki/Jason_Alexander https://en.wikipedia.org/wiki/Julia_Louis-Dreyfus https://en.wikipedia.org/wiki/Michael_Richards Numerical reasoning | Multiple constraints ['https://en.wikipedia.org/wiki/Seinfeld', 'https://en.wikipedia.org/wiki/Jerry_Seinfeld', 'https://en.wikipedia.org/wiki/Jason_Alexander', 'https://en.wikipedia.org/wiki/Julia_Louis-Dreyfus', 'https://en.wikipedia.org/wiki/Michael_Richards'] 36 38 How old was Harvard University, when the person whom the longest river in British Columbia is named after, was born? The river in question only flows within the confines of British Columbia and does not enter any other province or territory. 140 years old. https://en.wikipedia.org/wiki/Harvard_University https://en.wikipedia.org/wiki/List_of_longest_rivers_of_Canada https://en.wikipedia.org/wiki/Fraser_River https://en.wikipedia.org/wiki/Simon_Fraser_(explorer) Numerical reasoning | Tabular reasoning | Multiple constraints | Temporal reasoning ['https://en.wikipedia.org/wiki/Harvard_University', 'https://en.wikipedia.org/wiki/List_of_longest_rivers_of_Canada', 'https://en.wikipedia.org/wiki/Fraser_River', 'https://en.wikipedia.org/wiki/Simon_Fraser_(explorer)'] 37 39 On the same day that the The Mercedes-Benz W222 arrived at dealerships, a star of the sit-com Modern Family was wed. Who did the star marry? Justin Mikita https://en.wikipedia.org/wiki/Mercedes-Benz_S-Class_(W222) https://en.wikipedia.org/wiki/Modern_Family https://en.wikipedia.org/wiki/Jesse_Tyler_Ferguson Multiple constraints ['https://en.wikipedia.org/wiki/Mercedes-Benz_S-Class_(W222)', 'https://en.wikipedia.org/wiki/Modern_Family', 'https://en.wikipedia.org/wiki/Jesse_Tyler_Ferguson'] 38 40 Which species from the genus mulona are both found in the same country? Mulona barnesi and mulona schausi https://en.wikipedia.org/wiki/Mulona https://en.wikipedia.org/wiki/Mulona_barnesi https://en.wikipedia.org/wiki/Mulona_grisea https://en.wikipedia.org/wiki/Mulona_lapidaria https://en.wikipedia.org/wiki/Mulona_manni https://en.wikipedia.org/wiki/Mulona_piperita https://en.wikipedia.org/wiki/Mulona_schausi Multiple constraints ['https://en.wikipedia.org/wiki/Mulona', 'https://en.wikipedia.org/wiki/Mulona_barnesi', 'https://en.wikipedia.org/wiki/Mulona_grisea', 'https://en.wikipedia.org/wiki/Mulona_lapidaria', 'https://en.wikipedia.org/wiki/Mulona_manni', 'https://en.wikipedia.org/wiki/Mulona_piperita', 'https://en.wikipedia.org/wiki/Mulona_schausi'] 39 41 As of July 1, 2024, if I wanted to give my daughter the middle name of the American woman who is the most decorated female in the history of American gymnastics as her first name and the full first name of the American woman who holds the world record in the 800-meter freestyle as her middle name, what would I name my daughter? Arianne Kathleen https://en.wikipedia.org/wiki/Simone_Biles https://en.wikipedia.org/wiki/Katie_Ledecky Post processing ['https://en.wikipedia.org/wiki/Simone_Biles', 'https://en.wikipedia.org/wiki/Katie_Ledecky'] 40 42 I am thinking of a Ancient Roman City. The city was destroyed by volcanic eruption. The eruption occurred in the year 79 AD. The volcano was a stratovolcano. Where was the session held where it was decided that the city would be named a UNESCO world heritage site? Naples https://en.wikipedia.org/wiki/Stratovolcano https://en.wikipedia.org/wiki/Eruption_of_Mount_Vesuvius_in_79_AD https://en.wikipedia.org/wiki/Pompeii https://en.wikipedia.org/wiki/World_Heritage_Committee Tabular reasoning ['https://en.wikipedia.org/wiki/Stratovolcano', 'https://en.wikipedia.org/wiki/Eruption_of_Mount_Vesuvius_in_79_AD', 'https://en.wikipedia.org/wiki/Pompeii', 'https://en.wikipedia.org/wiki/World_Heritage_Committee'] 41 43 What Formula One car was driven in 1994 by the nephew of a racing driver from Italy who drove a Ferrari 312T and shares a last name with a common cocktail drink? Minardi M194 https://en.wikipedia.org/wiki/Minardi_M194 https://en.wikipedia.org/wiki/Pierluigi_Martini https://en.wikipedia.org/wiki/Giancarlo_Martini https://en.wikipedia.org/wiki/List_of_cocktails Multiple constraints ['https://en.wikipedia.org/wiki/Minardi_M194', 'https://en.wikipedia.org/wiki/Pierluigi_Martini', 'https://en.wikipedia.org/wiki/Giancarlo_Martini', 'https://en.wikipedia.org/wiki/List_of_cocktails'] 42 44 As of August 1, 2024, who is the president of the team that inspired the original name of the Washington Commanders? Derek Schiller https://en.wikipedia.org/wiki/Washington_Commanders https://en.wikipedia.org/wiki/Boston_Braves https://en.wikipedia.org/wiki/Atlanta_Braves Multiple constraints ['https://en.wikipedia.org/wiki/Washington_Commanders', 'https://en.wikipedia.org/wiki/Boston_Braves', 'https://en.wikipedia.org/wiki/Atlanta_Braves'] 43 46 As of 2023, how many more employees does the company alphabetically first by ticker symbol in the S&P500 have than the company alphabetically 2nd to last by ticker symbol in the S&P500? 8,350 https://en.wikipedia.org/wiki/List_of_S%26P_500_companies https://en.wikipedia.org/wiki/Agilent_Technologies https://en.wikipedia.org/wiki/Zebra_Technologies Numerical reasoning | Tabular reasoning | Multiple constraints ['https://en.wikipedia.org/wiki/List_of_S%26P_500_companies', 'https://en.wikipedia.org/wiki/Agilent_Technologies', 'https://en.wikipedia.org/wiki/Zebra_Technologies'] 44 47 I am moving to the G40 postcode area - what train stations are nearby, as of 2024? Bridgeton Railway Station and Dalmarnock Railway Station. https://en.wikipedia.org/wiki/List_of_postcode_areas_in_the_United_Kingdom https://en.wikipedia.org/wiki/G_postcode_area https://en.wikipedia.org/wiki/Bridgeton,_Glasgow https://en.wikipedia.org/wiki/Bridgeton_railway_station https://en.wikipedia.org/wiki/Calton,_Glasgow https://en.wikipedia.org/wiki/Dalmarnock Multiple constraints ['https://en.wikipedia.org/wiki/List_of_postcode_areas_in_the_United_Kingdom', 'https://en.wikipedia.org/wiki/G_postcode_area', 'https://en.wikipedia.org/wiki/Bridgeton,_Glasgow', 'https://en.wikipedia.org/wiki/Bridgeton_railway_station', 'https://en.wikipedia.org/wiki/Calton,_Glasgow ', 'https://en.wikipedia.org/wiki/Dalmarnock'] 45 48 How old was Stephen Baldwin when Hailey and Justin got married? 52 https://en.wikipedia.org/wiki/Hailey_Bieber https://en.wikipedia.org/wiki/Stephen_Baldwin Numerical reasoning ['https://en.wikipedia.org/wiki/Hailey_Bieber', 'https://en.wikipedia.org/wiki/Stephen_Baldwin'] 46 49 As of August 1, 2024, what is the largest city of the 9th largest country by land area in Europe? The largest city of the 9th largest country in Europe is Warsaw. https://en.wikipedia.org/wiki/List_of_European_countries_by_area https://en.wikipedia.org/wiki/Poland Tabular reasoning | Multiple constraints ['https://en.wikipedia.org/wiki/List_of_European_countries_by_area', 'https://en.wikipedia.org/wiki/Poland'] 47 50 What was the running time of the first cartoon in the series that inspired the name of the Looney Tunes franchise? 5 minutes and 31 seconds https://en.wikipedia.org/wiki/Looney_Tunes https://en.wikipedia.org/wiki/Silly_Symphony Tabular reasoning | Multiple constraints ['https://en.wikipedia.org/wiki/Looney_Tunes', 'https://en.wikipedia.org/wiki/Silly_Symphony'] 48 51 The state, whose motto was adopted March 26, 1928, has 0.94% of the population in 2024 speaking a language that is native to which country? Philippines https://en.wikipedia.org/wiki/List_of_U.S._state_and_territory_mottos https://en.wikipedia.org/wiki/New_Jersey https://en.wikipedia.org/wiki/Tagalog_language Tabular reasoning | Multiple constraints ['https://en.wikipedia.org/wiki/List_of_U.S._state_and_territory_mottos', 'https://en.wikipedia.org/wiki/New_Jersey', 'https://en.wikipedia.org/wiki/Tagalog_language'] 49 52 As of 2024, at the time of his birth, what was the middle name of the U.S. president who won Alaska, graduated from Yale University, and had a son named Michael? Lynch https://en.wikipedia.org/wiki/United_States_presidential_elections_in_Alaska https://en.wikipedia.org/wiki/Gerald_Ford Multiple constraints ['https://en.wikipedia.org/wiki/United_States_presidential_elections_in_Alaska', 'https://en.wikipedia.org/wiki/Gerald_Ford'] 50 53 The first white man to visit the indigenous people who were forced on a reserve during the Klondike Gold Rush, worked for a company who officially stopped selling what in 2023? Animal Fur Products https://en.wikipedia.org/wiki/Klondike_Gold_Rush https://en.wikipedia.org/wiki/H%C3%A4n https://en.wikipedia.org/wiki/Hudson%27s_Bay_Company Multiple constraints ['https://en.wikipedia.org/wiki/Klondike_Gold_Rush', 'https://en.wikipedia.org/wiki/H%C3%A4n', 'https://en.wikipedia.org/wiki/Hudson%27s_Bay_Company'] 51 54 "What state is the home of the losing team of the World Series three years before ""Old Shufflefoot"" managed his team to victory? " Illinois https://en.wikipedia.org/wiki/Lou_Boudreau https://en.wikipedia.org/wiki/List_of_World_Series_champions https://en.wikipedia.org/wiki/Chicago Multiple constraints | Temporal reasoning ['https://en.wikipedia.org/wiki/Lou_Boudreau', 'https://en.wikipedia.org/wiki/List_of_World_Series_champions', 'https://en.wikipedia.org/wiki/Chicago'] 52 55 Was the person who served as president of the Scottish National Party from 1987 to 2005 alive when the party was founded? Yes https://en.wikipedia.org/wiki/Scottish_National_Party https://en.wikipedia.org/wiki/Winnie_Ewing Temporal reasoning ['https://en.wikipedia.org/wiki/Scottish_National_Party', 'https://en.wikipedia.org/wiki/Winnie_Ewing'] 53 56 In series six of Downton Abbey, Lord Merton is diagnosed with a terminal condition. A deficiency in which vitamin causes this condition? Vitamin B12 https://en.wikipedia.org/wiki/Downton_Abbey https://en.wikipedia.org/wiki/Pernicious_anemia Multiple constraints ['https://en.wikipedia.org/wiki/Downton_Abbey', 'https://en.wikipedia.org/wiki/Pernicious_anemia'] 54 57 How many letters long is the title of the first movie composed by the composer of the first American Godzilla movie? 17 https://en.wikipedia.org/wiki/Godzilla_(franchise) https://en.wikipedia.org/wiki/Godzilla_(1998_film) https://en.wikipedia.org/wiki/David_Arnold Tabular reasoning | Multiple constraints | Post processing ['https://en.wikipedia.org/wiki/Godzilla_(franchise)', 'https://en.wikipedia.org/wiki/Godzilla_(1998_film)', 'https://en.wikipedia.org/wiki/David_Arnold'] 55 60 "The author of the book ""A Good Woman""'s author was married to a man in 2008, who resigned from the board of HP due to the actions taken by the board's chair. What types of cancer did the chair survive?" Breast and skin https://en.wikipedia.org/wiki/A_Good_Woman_(novel) https://en.wikipedia.org/wiki/Danielle_Steel https://en.wikipedia.org/wiki/Thomas_Perkins_(businessman) https://en.wikipedia.org/wiki/Patricia_C._Dunn Multiple constraints ['https://en.wikipedia.org/wiki/A_Good_Woman_(novel)', 'https://en.wikipedia.org/wiki/Danielle_Steel', 'https://en.wikipedia.org/wiki/Thomas_Perkins_(businessman)', 'https://en.wikipedia.org/wiki/Patricia_C._Dunn'] 56 61 Benjamin Waterhouse Hawkins was commissioned to sculpt a series of life-size dinosaurs between 1852-54. In this location, one species only had its head and back built with the rest of its body submerged underwater because they didn't know what it looked like then. Where is this display located in 2024, which dinosaur species was this, and what did the rest of its body look like after all? Crystal Palace Park. The dinosaur is Mosasaurus, it had a streamlined body, an elongated tail ending with a downturn supporting a two-lobed fin, and two pairs of flippers. https://en.wikipedia.org/wiki/Benjamin_Waterhouse_Hawkins https://en.wikipedia.org/wiki/Crystal_Palace_Dinosaurs https://en.wikipedia.org/wiki/Mosasaurus Multiple constraints ['https://en.wikipedia.org/wiki/Benjamin_Waterhouse_Hawkins', 'https://en.wikipedia.org/wiki/Crystal_Palace_Dinosaurs', 'https://en.wikipedia.org/wiki/Mosasaurus'] 57 62 The Assistant to the Regional Manager on The Office TV show (US version) has a farm. You can obtain what food colorant from the crop he grows? Betanin https://en.wikipedia.org/wiki/The_Office https://en.wikipedia.org/wiki/Dwight_Schrute https://en.wikipedia.org/wiki/Beetroot Multiple constraints ['https://en.wikipedia.org/wiki/The_Office', 'https://en.wikipedia.org/wiki/Dwight_Schrute', 'https://en.wikipedia.org/wiki/Beetroot'] 58 63 How many Pokemon World Championships occurred in the contiguous United States during the presidency of Barack Obama? - Five Pokemon World Championships took place in the contiguous United States during Barack Obama's presidency - The championships tournaments were 2009 (San Diego, California), 2011 (San Diego, California), 2014 (Washington, D.C.), 2015 (Boston, Massachusetts) and 2016 (San Francisco, California) https://en.wikipedia.org/wiki/Pok%C3%A9mon_World_Championships https://en.wikipedia.org/wiki/Barack_Obama https://en.wikipedia.org/wiki/Contiguous_United_States Multiple constraints | Temporal reasoning ['https://en.wikipedia.org/wiki/Pok%C3%A9mon_World_Championships', 'https://en.wikipedia.org/wiki/Barack_Obama', 'https://en.wikipedia.org/wiki/Contiguous_United_States'] 59 64 Put these historical events in chronological order, starting with the earliest: The Beatles play Ed Sullivan, the fall of the Berlin Wall, The Great Depression, Atlanta Summer Games, World War I. World War I, The Great Depression, The Beatles play Ed Sullivan, the fall of the Berlin Wall, Atlanta Summer Games. https://en.wikipedia.org/wiki/World_War_I https://en.wikipedia.org/wiki/1996_Summer_Olympics https://en.wikipedia.org/wiki/Fall_of_the_Berlin_Wall#:~:text=The%20fall%20of%20the%20Berlin,restrictions%20were%20overwhelmed%20and%20discarded. https://en.wikipedia.org/wiki/The_Beatles https://en.wikipedia.org/wiki/Great_Depression Temporal reasoning ['https://en.wikipedia.org/wiki/World_War_I', 'https://en.wikipedia.org/wiki/1996_Summer_Olympics', 'https://en.wikipedia.org/wiki/Fall_of_the_Berlin_Wall#:~:text=The%20fall%20of%20the%20Berlin,restrictions%20were%20overwhelmed%20and%20discarded.', 'https://en.wikipedia.org/wiki/The_Beatles', 'https://en.wikipedia.org/wiki/Great_Depression'] 60 65 This individual won a Best Director Award at the 33rd Japan Academy Prize ceremony and is known for having directed a film that briefly surpassed the Godfather as the highest-grossing film in Japan for a short time. Which film was longer - The Godfather or his film - and by how many minutes? The Godfather (1972) was longer than Submersion in Japan (1973) by 32 minutes. https://en.wikipedia.org/wiki/Japan_Academy_Film_Prize https://en.wikipedia.org/wiki/Daisaku_Kimura https://en.wikipedia.org/wiki/Submersion_of_Japan https://en.wikipedia.org/wiki/The_Godfather Numerical reasoning | Tabular reasoning | Multiple constraints ['https://en.wikipedia.org/wiki/Japan_Academy_Film_Prize', 'https://en.wikipedia.org/wiki/Daisaku_Kimura', 'https://en.wikipedia.org/wiki/Submersion_of_Japan', 'https://en.wikipedia.org/wiki/The_Godfather'] 61 66 The manga 'Sailor Moon' was authored by a woman. What manga did her husband win the Shogakukan Manga Award for authoring? YuYu Hakusho https://en.wikipedia.org/wiki/Sailor_Moon https://en.wikipedia.org/wiki/Naoko_Takeuchi https://en.wikipedia.org/wiki/Yoshihiro_Togashi Multiple constraints ['https://en.wikipedia.org/wiki/Sailor_Moon', 'https://en.wikipedia.org/wiki/Naoko_Takeuchi', 'https://en.wikipedia.org/wiki/Yoshihiro_Togashi'] 62 67 Where did the daughter of the winner of the first US presidential election to occur after the official end of WWII attend university? George Washington University https://en.wikipedia.org/wiki/World_War_II https://en.wikipedia.org/wiki/United_States_presidential_election https://en.wikipedia.org/wiki/Harry_S._Truman https://en.wikipedia.org/wiki/Margaret_Truman Tabular reasoning | Multiple constraints | Temporal reasoning ['https://en.wikipedia.org/wiki/World_War_II', 'https://en.wikipedia.org/wiki/United_States_presidential_election', 'https://en.wikipedia.org/wiki/Harry_S._Truman', 'https://en.wikipedia.org/wiki/Margaret_Truman'] 63 68 In the style of wrestling performed by former Greek wrestler Mikhail Theodoropoulos, who won the inaugural olympic medal? Carl Schuhmann. https://en.wikipedia.org/wiki/Mikhail_Theodoropoulos https://en.wikipedia.org/wiki/Wrestling_at_the_1960_Summer_Olympics_%E2%80%93_Men%27s_Greco-Roman_bantamweight https://en.wikipedia.org/wiki/Greco-Roman_wrestling https://en.wikipedia.org/wiki/List_of_World_and_Olympic_Champions_in_Greco-Roman_wrestling Multiple constraints ['https://en.wikipedia.org/wiki/Mikhail_Theodoropoulos', 'https://en.wikipedia.org/wiki/Wrestling_at_the_1960_Summer_Olympics_%E2%80%93_Men%27s_Greco-Roman_bantamweight', 'https://en.wikipedia.org/wiki/Greco-Roman_wrestling', 'https://en.wikipedia.org/wiki/List_of_World_and_Olympic_Champions_in_Greco-Roman_wrestling'] 64 69 What is the difference in elevation between the respective peaks of Eggstock (Uri Alps) and Eggstock (Schwyzer Alps) in Switzerland? Convert the final figure to centimetres and round up to the nearest 1000. 110,000cm. https://en.wikipedia.org/wiki/Eggstock_(Uri_Alps) https://en.wikipedia.org/wiki/Eggstock_(Schwyzer_Alps) Numerical reasoning | Multiple constraints | Post processing ['https://en.wikipedia.org/wiki/Eggstock_(Uri_Alps)', 'https://en.wikipedia.org/wiki/Eggstock_(Schwyzer_Alps)'] 65 70 "How many films had the actress who played Trudi Frazer in ""Once Upon a Time in Hollywood"" acted in before?" 3 https://en.wikipedia.org/wiki/Once_Upon_a_Time_in_Hollywood https://en.wikipedia.org/wiki/Julia_Butters Numerical reasoning | Multiple constraints | Temporal reasoning ['https://en.wikipedia.org/wiki/Once_Upon_a_Time_in_Hollywood', 'https://en.wikipedia.org/wiki/Julia_Butters'] 66 71 The founder of the production company at which Tim Allsop and Stewart Williams met received a bachelor's degree from a college in the state of New York. In what year was this college's sibling institution founded? 1701 https://en.wikipedia.org/wiki/Tim_Allsop_&_Stewart_Williams https://en.wikipedia.org/wiki/Elisabeth_Murdoch_(businesswoman) https://en.wikipedia.org/wiki/Vassar_College https://en.wikipedia.org/wiki/Yale_University Multiple constraints ['https://en.wikipedia.org/wiki/Tim_Allsop_&_Stewart_Williams', 'https://en.wikipedia.org/wiki/Elisabeth_Murdoch_(businesswoman)', 'https://en.wikipedia.org/wiki/Vassar_College', 'https://en.wikipedia.org/wiki/Yale_University'] 67 72 Which player scored more than 15 goals in Eredevisie during the 21-22 season and had previously played for Auxerre? Sébastien Haller scored 21 goals that season and previously played for Auxerre. https://en.wikipedia.org/wiki/2021–22_Eredivisie https://en.wikipedia.org/wiki/Sébastien_Haller https://en.wikipedia.org/wiki/Loïs_Openda https://en.wikipedia.org/wiki/Vangelis_Pavlidis https://en.wikipedia.org/wiki/Ricky_van_Wolfswinkel Tabular reasoning | Multiple constraints | Temporal reasoning ['https://en.wikipedia.org/wiki/2021–22_Eredivisie', 'https://en.wikipedia.org/wiki/Sébastien_Haller', 'https://en.wikipedia.org/wiki/Loïs_Openda', 'https://en.wikipedia.org/wiki/Vangelis_Pavlidis', 'https://en.wikipedia.org/wiki/Ricky_van_Wolfswinkel'] 68 73 The latest game, as of August 4, 2024, from the creator of Kirby won an award at The Game Awards. What won Game of the Year the following year? The Last of Us Part II https://en.wikipedia.org/wiki/Kirby_(series) https://en.wikipedia.org/wiki/Masahiro_Sakurai https://en.wikipedia.org/wiki/Super_Smash_Bros._Ultimate https://en.wikipedia.org/wiki/The_Game_Awards_2020 Multiple constraints ['https://en.wikipedia.org/wiki/Kirby_(series)', 'https://en.wikipedia.org/wiki/Masahiro_Sakurai', 'https://en.wikipedia.org/wiki/Super_Smash_Bros._Ultimate', 'https://en.wikipedia.org/wiki/The_Game_Awards_2020'] 69 74 By how many years does the inception of the party to which former Finnish MP Lea Rakel Hiltunen last belonged predate the foundation of the Parliament of Finland itself? 7 years. https://en.wikipedia.org/wiki/Rakel_Hiltunen https://en.wikipedia.org/wiki/Parliament_of_Finland https://en.wikipedia.org/wiki/Social_Democratic_Party_of_Finland Numerical reasoning | Multiple constraints | Temporal reasoning ['https://en.wikipedia.org/wiki/Rakel_Hiltunen', 'https://en.wikipedia.org/wiki/Parliament_of_Finland', 'https://en.wikipedia.org/wiki/Social_Democratic_Party_of_Finland'] 70 75 According to the population data in their respective Wikipedia articles in August 2024, what is the difference between the population of Seattle, WA, and Portland, OR, according to the data from 2020? 84,512 https://en.wikipedia.org/wiki/Seattle https://en.wikipedia.org/wiki/Portland,_Oregon Numerical reasoning | Tabular reasoning ['https://en.wikipedia.org/wiki/Seattle', 'https://en.wikipedia.org/wiki/Portland,_Oregon'] 71 76 What was the Enhanced Fujita Scale rating of the 2011 tornado that hit the hometown of the band who had a controversial interview with Bryan Odell on June 3, 2012? EF5- Joplin, Missouri https://en.wikipedia.org/wiki/BryanStars https://en.wikipedia.org/wiki/Never_Shout_Never https://en.wikipedia.org/wiki/2011_Joplin_tornado Multiple constraints ['https://en.wikipedia.org/wiki/BryanStars', 'https://en.wikipedia.org/wiki/Never_Shout_Never', 'https://en.wikipedia.org/wiki/2011_Joplin_tornado'] 72 77 Which MP standing as the leader of a major party in the 2019 United Kingdom General Election was also an MP for Henley? Boris Johnson was leader of the Conservative Party and a former MP for Henley. https://en.wikipedia.org/wiki/2019_United_Kingdom_general_election https://en.wikipedia.org/wiki/Boris_Johnson https://en.wikipedia.org/wiki/Jeremy_Corbyn https://en.wikipedia.org/wiki/Nicola_Sturgeon https://en.wikipedia.org/wiki/Jo_Swinson Multiple constraints ['https://en.wikipedia.org/wiki/2019_United_Kingdom_general_election', 'https://en.wikipedia.org/wiki/Boris_Johnson', 'https://en.wikipedia.org/wiki/Jeremy_Corbyn', 'https://en.wikipedia.org/wiki/Nicola_Sturgeon', 'https://en.wikipedia.org/wiki/Jo_Swinson'] 73 78 Who was the author of the novel whose film adaptation lead singer Mark Arm took the name of his band from? Raymond Friday Locke https://en.wikipedia.org/wiki/Mark_Arm https://en.wikipedia.org/wiki/Mudhoney https://en.wikipedia.org/wiki/Mudhoney_(film) Multiple constraints ['https://en.wikipedia.org/wiki/Mark_Arm', 'https://en.wikipedia.org/wiki/Mudhoney', 'https://en.wikipedia.org/wiki/Mudhoney_(film)'] 74 79 As of August 3, 2024, how much taller was the tsunami wave of the most powerful earthquake in North America than the most powerful earthquake ever recorded in Japan? 26.5 meters of 87 feet https://en.wikipedia.org/wiki/2011_T%C5%8Dhoku_earthquake_and_tsunami#Nuclear_power_plants https://en.wikipedia.org/wiki/Lists_of_earthquakes#Largest_earthquakes_by_magnitude https://en.wikipedia.org/wiki/1964_Alaska_earthquake Numerical reasoning | Tabular reasoning ['https://en.wikipedia.org/wiki/2011_T%C5%8Dhoku_earthquake_and_tsunami#Nuclear_power_plants', 'https://en.wikipedia.org/wiki/Lists_of_earthquakes#Largest_earthquakes_by_magnitude', 'https://en.wikipedia.org/wiki/1964_Alaska_earthquake'] 75 80 "How old would the 1975 winner of the Lenore Marshall Poetry Prize have been if they were still alive on the date when Rupi Kaur released her book titled, ""Milk and Honey""?" 90 https://en.wikipedia.org/wiki/List_of_winners_of_the_Lenore_Marshall_Poetry_Prize https://en.wikipedia.org/wiki/Cid_Corman https://en.wikipedia.org/wiki/Milk_and_Honey_(poetry_collection) Numerical reasoning | Multiple constraints | Post processing | Temporal reasoning ['https://en.wikipedia.org/wiki/List_of_winners_of_the_Lenore_Marshall_Poetry_Prize', 'https://en.wikipedia.org/wiki/Cid_Corman', 'https://en.wikipedia.org/wiki/Milk_and_Honey_(poetry_collection)'] 76 81 A united states island that is only 90 miles away from Cuba has been the home of several famous people. In what year did the famous author who owns a book store there first start writing? 1959 https://en.wikipedia.org/wiki/Key_West#:~:text=The%20southernmost%20location%20that%20the,apart%20at%20their%20closest%20points. https://en.wikipedia.org/wiki/Judy_Blume Post processing | Temporal reasoning ['https://en.wikipedia.org/wiki/Key_West#:~:text=The%20southernmost%20location%20that%20the,apart%20at%20their%20closest%20points.', 'https://en.wikipedia.org/wiki/Judy_Blume'] 77 82 According to the 2011 census, what is total population of the cities of the birthplaces of author Clive Barker, Prince William, and Sir Malcolm Stanley Bradbury? Round to the nearest 100,000. 11,300,000 https://en.wikipedia.org/wiki/Clive_Barker https://en.wikipedia.org/wiki/William,_Prince_of_Wales https://en.wikipedia.org/wiki/Malcolm_Bradbury https://en.wikipedia.org/wiki/England#Geography Numerical reasoning | Tabular reasoning | Post processing ['https://en.wikipedia.org/wiki/Clive_Barker', 'https://en.wikipedia.org/wiki/William,_Prince_of_Wales', 'https://en.wikipedia.org/wiki/Malcolm_Bradbury', 'https://en.wikipedia.org/wiki/England#Geography'] 78 83 Of the two wonders of the ancient world that were statues, how much shorter was the taller of the two compared to the tallest statue in Japan as of 2024? 222 ft https://en.wikipedia.org/wiki/Seven_Wonders_of_the_Ancient_World#Wonders https://en.wikipedia.org/wiki/Statue_of_Zeus_at_Olympia https://en.wikipedia.org/wiki/Colossus_of_Rhodes https://en.wikipedia.org/wiki/List_of_tallest_statues Numerical reasoning | Tabular reasoning | Multiple constraints ['https://en.wikipedia.org/wiki/Seven_Wonders_of_the_Ancient_World#Wonders', 'https://en.wikipedia.org/wiki/Statue_of_Zeus_at_Olympia', 'https://en.wikipedia.org/wiki/Colossus_of_Rhodes', 'https://en.wikipedia.org/wiki/List_of_tallest_statues'] 79 85 "The actor known for playing Kenny ""Shammy"" Shamberg in Magnum P.I. was born how days before the Apollo 11 moon landing?" 844 days. https://en.wikipedia.org/wiki/Magnum_P.I._(2018_TV_series) https://en.wikipedia.org/wiki/Christopher_Thornton https://en.wikipedia.org/wiki/Apollo_11#Mission Numerical reasoning | Tabular reasoning | Multiple constraints | Post processing ['https://en.wikipedia.org/wiki/Magnum_P.I._(2018_TV_series)', 'https://en.wikipedia.org/wiki/Christopher_Thornton', 'https://en.wikipedia.org/wiki/Apollo_11#Mission'] 80 86 As of January 1, 2024, what was the warmest decade, since the 17th century, around the ocean that surrounds Litke Deep? The warmest decade for the Arctic Ocean since the 17th century was during the period of 1995–2005. https://en.wikipedia.org/wiki/Litke_Deep https://en.wikipedia.org/wiki/Arctic_Ocean#Climate https://en.wikipedia.org/wiki/Climate_change_in_the_Arctic Multiple constraints ['https://en.wikipedia.org/wiki/Litke_Deep', 'https://en.wikipedia.org/wiki/Arctic_Ocean#Climate', 'https://en.wikipedia.org/wiki/Climate_change_in_the_Arctic'] 81 87 Which of the bridges in Halifax, Nova Scotia is longer, and by how much? The MacKay, or the MacDonald? The MacDonald Bridge (1300 metres) is 100 metres longer than the MacKay (1200 metres). https://en.wikipedia.org/wiki/Angus_L._Macdonald_Bridge https://en.wikipedia.org/wiki/A._Murray_MacKay_Bridge Numerical reasoning ['https://en.wikipedia.org/wiki/Angus_L._Macdonald_Bridge', 'https://en.wikipedia.org/wiki/A._Murray_MacKay_Bridge'] 82 88 Which section of IPSC Australia Inc. is larger than Honshu and smaller than Sumatra by area? Victoria and Tasmania https://en.wikipedia.org/wiki/IPSC_Australia_Inc https://en.wikipedia.org/wiki/Western_Australia https://en.wikipedia.org/wiki/Victoria_(state) https://en.wikipedia.org/wiki/Tasmania https://en.wikipedia.org/wiki/South_Australia https://en.wikipedia.org/wiki/New_South_Wales https://en.wikipedia.org/wiki/Australian_Capital_Territory https://en.wikipedia.org/wiki/Queensland https://en.wikipedia.org/wiki/Northern_Territory https://en.wikipedia.org/wiki/Honshu https://en.wikipedia.org/wiki/Sumatra Numerical reasoning | Tabular reasoning | Multiple constraints ['https://en.wikipedia.org/wiki/IPSC_Australia_Inc', 'https://en.wikipedia.org/wiki/Western_Australia', 'https://en.wikipedia.org/wiki/Victoria_(state)', 'https://en.wikipedia.org/wiki/Tasmania', 'https://en.wikipedia.org/wiki/South_Australia', 'https://en.wikipedia.org/wiki/New_South_Wales', 'https://en.wikipedia.org/wiki/Australian_Capital_Territory', 'https://en.wikipedia.org/wiki/Queensland', 'https://en.wikipedia.org/wiki/Northern_Territory', 'https://en.wikipedia.org/wiki/Honshu', 'https://en.wikipedia.org/wiki/Sumatra'] 83 89 As of August 1, 2024, are there any cities in England that are more populated than Philadelphia, and which cities are they? Yes, London https://en.wikipedia.org/wiki/Philadelphia https://en.wikipedia.org/wiki/List_of_ONS_built-up_areas_in_England_by_population Numerical reasoning ['https://en.wikipedia.org/wiki/Philadelphia', 'https://en.wikipedia.org/wiki/List_of_ONS_built-up_areas_in_England_by_population'] 84 90 Which event predates the other: the amalgamation of the Province of Betanzos with Mondonedo or the completion of a cathedral marking the destination of a prominent Spanish Catholic pilgrimage in the same province? Provide the year of the earlier event. The completion of Santiago de Compostela Cathedral in 1211. https://en.wikipedia.org/wiki/Province_of_A_Coru%C3%B1a https://en.wikipedia.org/wiki/Santiago_de_Compostela_Cathedral Tabular reasoning | Temporal reasoning ['https://en.wikipedia.org/wiki/Province_of_A_Coru%C3%B1a', 'https://en.wikipedia.org/wiki/Santiago_de_Compostela_Cathedral'] 85 91 "How many films with titles including the letter ""g"" did the director of ""Sword of the Valiant"" direct after the release of said film, but before the year 2000?" 1 https://en.wikipedia.org/wiki/Sword_of_the_Valiant https://en.wikipedia.org/wiki/Stephen_Weeks Numerical reasoning | Multiple constraints | Temporal reasoning ['https://en.wikipedia.org/wiki/Sword_of_the_Valiant', 'https://en.wikipedia.org/wiki/Stephen_Weeks'] 86 92 What medal did the woman who tied for 19th in the 2011 Pan American Games women's individual bowling event win as part of a women's bowling doubles team in the 2017 World Games? Bronze medalist https://en.wikipedia.org/wiki/Bowling_at_the_2011_Pan_American_Games_%E2%80%93_Women's_individual https://en.wikipedia.org/wiki/Bowling_at_the_2017_World_Games Tabular reasoning "[""https://en.wikipedia.org/wiki/Bowling_at_the_2011_Pan_American_Games_%E2%80%93_Women's_individual"", 'https://en.wikipedia.org/wiki/Bowling_at_the_2017_World_Games']" 87 93 Did the entomologist who first described lesticus purpurascens come from the same country as the entomologist who first described the subfamily of that species? Yes, Stefano Ludovico Straneo and Franco Andrea Bonelli are both Italian https://en.wikipedia.org/wiki/Lesticus_purpurascens https://en.wikipedia.org/wiki/Stefano_Ludovico_Straneo https://en.wikipedia.org/wiki/Pterostichinae https://en.wikipedia.org/wiki/Franco_Andrea_Bonelli Tabular reasoning ['https://en.wikipedia.org/wiki/Lesticus_purpurascens', 'https://en.wikipedia.org/wiki/Stefano_Ludovico_Straneo', 'https://en.wikipedia.org/wiki/Pterostichinae', 'https://en.wikipedia.org/wiki/Franco_Andrea_Bonelli'] 88 94 If you were to combine two words, the first of which has a species called Polytrichum piliferum, and the second which has a breed type called Fleckvieh, you'd get the name of what countries capital city? Russia https://en.wikipedia.org/wiki/Moss https://en.wikipedia.org/w/index.php?search=Polytrichum+piliferum&title=Special:Search&profile=advanced&fulltext=1&ns0=1 https://en.wikipedia.org/wiki/Cattle https://en.wikipedia.org/wiki/Fleckvieh Multiple constraints ['https://en.wikipedia.org/wiki/Moss', 'https://en.wikipedia.org/w/index.php?search=Polytrichum+piliferum&title=Special:Search&profile=advanced&fulltext=1&ns0=1', 'https://en.wikipedia.org/wiki/Cattle', 'https://en.wikipedia.org/wiki/Fleckvieh'] 89 95 What was the birthday of the man who was mayor of New York City the year Snoopy debuted in the Macy's Thanksgiving Day Parade? November 24, 1921 https://en.wikipedia.org/wiki/Snoopy https://en.wikipedia.org/wiki/List_of_mayors_of_New_York_City https://en.wikipedia.org/wiki/John_Lindsay Tabular reasoning | Multiple constraints | Temporal reasoning ['https://en.wikipedia.org/wiki/Snoopy', 'https://en.wikipedia.org/wiki/List_of_mayors_of_New_York_City', 'https://en.wikipedia.org/wiki/John_Lindsay'] 90 96 What is the average distance for the left field line in MLB stadiums with a retractable roof as of August 2024? Round to the nearest whole number. 331 feet https://en.wikipedia.org/wiki/List_of_current_Major_League_Baseball_stadiums https://en.wikipedia.org/wiki/Rogers_Centre https://en.wikipedia.org/wiki/Chase_Field https://en.wikipedia.org/wiki/T-Mobile_Park https://en.wikipedia.org/wiki/Minute_Maid_Park https://en.wikipedia.org/wiki/American_Family_Field, https://en.wikipedia.org/wiki/LoanDepot_Park, https://en.wikipedia.org/wiki/Globe_Life_Field, Numerical reasoning | Tabular reasoning | Multiple constraints ['https://en.wikipedia.org/wiki/List_of_current_Major_League_Baseball_stadiums', 'https://en.wikipedia.org/wiki/Rogers_Centre', 'https://en.wikipedia.org/wiki/Chase_Field', 'https://en.wikipedia.org/wiki/T-Mobile_Park', 'https://en.wikipedia.org/wiki/Minute_Maid_Park', 'https://en.wikipedia.org/wiki/American_Family_Field, https://en.wikipedia.org/wiki/LoanDepot_Park, https://en.wikipedia.org/wiki/Globe_Life_Field, '] 91 97 As of August 3, 2024, which rabbi worked for both Reform Congregation Keneseth Israel in Philadelphia and Congregation Beth Israel in West Hartford, Connecticut? Abraham J. Feldman worked for both congregations serving as an interim associate rabbi at the Reform Congregation Keneseth Israel and the leader of the Congregation Beth Israel. https://en.wikipedia.org/wiki/Reform_Congregation_Keneseth_Israel_(Philadelphia) https://en.wikipedia.org/wiki/Congregation_Beth_Israel_(West_Hartford,_Connecticut) Multiple constraints ['https://en.wikipedia.org/wiki/Reform_Congregation_Keneseth_Israel_(Philadelphia)', 'https://en.wikipedia.org/wiki/Congregation_Beth_Israel_(West_Hartford,_Connecticut)'] 92 99 Where was the Winter Olympics held the year that the girl who was Wheaties first official spokeswoman turned 20 years old? Calgary, Alberta, Canada https://en.wikipedia.org/wiki/1988_Winter_Olympics https://en.wikipedia.org/wiki/Mary_Lou_Retton Multiple constraints | Temporal reasoning ['https://en.wikipedia.org/wiki/1988_Winter_Olympics', 'https://en.wikipedia.org/wiki/Mary_Lou_Retton'] 93 100 How many of Hitler's three couriers of his last will and other political documents died after 1980? 1 https://en.wikipedia.org/wiki/Last_will_and_testament_of_Adolf_Hitler https://en.wikipedia.org/wiki/Willy_Johannmeyer https://en.wikipedia.org/wiki/Wilhelm_Zander https://en.wikipedia.org/wiki/Heinz_Lorenz Temporal reasoning ['https://en.wikipedia.org/wiki/Last_will_and_testament_of_Adolf_Hitler', 'https://en.wikipedia.org/wiki/Willy_Johannmeyer', 'https://en.wikipedia.org/wiki/Wilhelm_Zander', 'https://en.wikipedia.org/wiki/Heinz_Lorenz'] 94 101 The inventor of the first true pinhole camera was also the first to correctly explain what theory? The Theory of Vision https://en.wikipedia.org/wiki/Photography https://en.wikipedia.org/wiki/Ibn_al-Haytham Multiple constraints ['https://en.wikipedia.org/wiki/Photography', 'https://en.wikipedia.org/wiki/Ibn_al-Haytham'] 95 102 As of August 2024, the Atlanta Braves beat the Houston Astros the last time they won the World Series. How many years before this did Jackie Robinson join the Brooklyn Dodgers? 74 years https://en.wikipedia.org/wiki/List_of_World_Series_champions#World_Series_results https://en.wikipedia.org/wiki/2021_Atlanta_Braves_season https://en.wikipedia.org/wiki/2021_Houston_Astros_season https://en.wikipedia.org/wiki/Jackie_Robinson Numerical reasoning | Temporal reasoning ['https://en.wikipedia.org/wiki/List_of_World_Series_champions#World_Series_results', 'https://en.wikipedia.org/wiki/2021_Atlanta_Braves_season', 'https://en.wikipedia.org/wiki/2021_Houston_Astros_season', 'https://en.wikipedia.org/wiki/Jackie_Robinson'] 96 103 In which of the three Intertidal zones would you most likely find the Septifer bilocularis? Low Intertidal Zone https://en.wikipedia.org/wiki/Septifer_bilocularis https://en.wikipedia.org/wiki/Mytilidae https://en.wikipedia.org/wiki/Intertidal_zone Multiple constraints ['https://en.wikipedia.org/wiki/Septifer_bilocularis', 'https://en.wikipedia.org/wiki/Mytilidae', 'https://en.wikipedia.org/wiki/Intertidal_zone'] 97 104 In the Eurovision Song Contest 2024, one country scored a combined total (jury and televoting results) of 268 - as did another country in the contest the year before. Which countries are they? Italy and Norway https://en.wikipedia.org/wiki/Eurovision_Song_Contest_2024 https://en.wikipedia.org/wiki/Eurovision_Song_Contest_2023 Tabular reasoning ['https://en.wikipedia.org/wiki/Eurovision_Song_Contest_2024', 'https://en.wikipedia.org/wiki/Eurovision_Song_Contest_2023'] 98 105 The quarterback who was selected first overall in the 1998 NFL draft, won the Superbowl with 2 different teams, both named after what type of animal? A horse https://en.wikipedia.org/wiki/1998_NFL_draft#Player_selections https://en.wikipedia.org/wiki/Peyton_Manning https://en.wikipedia.org/wiki/Colt https://en.wikipedia.org/w/index.php?title=Bronco&redirect=no Tabular reasoning ['https://en.wikipedia.org/wiki/1998_NFL_draft#Player_selections', 'https://en.wikipedia.org/wiki/Peyton_Manning', 'https://en.wikipedia.org/wiki/Colt', 'https://en.wikipedia.org/w/index.php?title=Bronco&redirect=no'] 99 106 As of 1st August 2024, How much younger is the current youngest US House Representative than the American folk hero who died at the Alamo when they were elected to the US House of Representatives? 14 years younger https://en.wikipedia.org/wiki/List_of_current_members_of_the_United_States_House_of_Representatives https://en.wikipedia.org/wiki/Battle_of_the_Alamo https://en.wikipedia.org/wiki/Davy_Crockett Numerical reasoning | Tabular reasoning | Multiple constraints | Temporal reasoning ['https://en.wikipedia.org/wiki/List_of_current_members_of_the_United_States_House_of_Representatives', 'https://en.wikipedia.org/wiki/Battle_of_the_Alamo', 'https://en.wikipedia.org/wiki/Davy_Crockett'] 100 107 Who was older, the guitar player for the Dugites from 1982-1983 or the lead singer of The Sports? Andrew Pendlebury https://en.wikipedia.org/wiki/The_Dugites https://en.wikipedia.org/wiki/Andrew_Pendlebury https://en.wikipedia.org/wiki/Stephen_Cummings Numerical reasoning | Multiple constraints ['https://en.wikipedia.org/wiki/The_Dugites', 'https://en.wikipedia.org/wiki/Andrew_Pendlebury', 'https://en.wikipedia.org/wiki/Stephen_Cummings'] 101 108 In the first movie that Emma Stone won an Academy Award for Best Actress in, did her costar win an Academy Award for Best Actor? Ryan Gosling was nominated for an Academy Award for Best Actor in La La Land, but didn't win. https://en.wikipedia.org/wiki/Emma_Stone https://en.wikipedia.org/wiki/La_La_Land#Cast https://en.wikipedia.org/wiki/89th_Academy_Awards Multiple constraints | Post processing | Temporal reasoning ['https://en.wikipedia.org/wiki/Emma_Stone', 'https://en.wikipedia.org/wiki/La_La_Land#Cast', 'https://en.wikipedia.org/wiki/89th_Academy_Awards'] 102 109 As of August 4, 2024, what is the first initial and surname of the cricketer who became the top-rated test batsman in the 2020s, is the fastest player of their country to 6 1000 run milestones in tests, and became their country's all-time leading run scorer in tests in the same year? K. Williamson https://en.wikipedia.org/wiki/ICC_men%27s_player_rankings#Top_10_Test_batsmen https://en.wikipedia.org/wiki/Kane_Williamson https://en.wikipedia.org/wiki/List_of_New_Zealand_Test_cricket_records#Most_career_runs Tabular reasoning | Multiple constraints | Post processing ['https://en.wikipedia.org/wiki/ICC_men%27s_player_rankings#Top_10_Test_batsmen', 'https://en.wikipedia.org/wiki/Kane_Williamson', 'https://en.wikipedia.org/wiki/List_of_New_Zealand_Test_cricket_records#Most_career_runs'] 103 110 As of 2024, what percentage of Afroasiatic language speakers speak Central Atlas Tamazight? 0.49% https://en.wikipedia.org/wiki/Afroasiatic_languages https://en.wikipedia.org/wiki/Central_Atlas_Tamazight Numerical reasoning ['https://en.wikipedia.org/wiki/Afroasiatic_languages', 'https://en.wikipedia.org/wiki/Central_Atlas_Tamazight'] 104 111 " ""The Terminator"" was released on October 26th exactly how many years after the famous gunfight at the O.K. Corral occurred?" 103 https://en.wikipedia.org/wiki/Gunfight_at_the_O.K._Corral https://en.wikipedia.org/wiki/The_Terminator Numerical reasoning | Multiple constraints ['https://en.wikipedia.org/wiki/Gunfight_at_the_O.K._Corral', 'https://en.wikipedia.org/wiki/The_Terminator'] 105 112 If an Ixodes scapularis tick in its nymph stage feeds on a host in the Spring, how many seasons later is it most likely to transmit Lyme disease if it becomes an adult in the same year? Two seasons. https://en.wikipedia.org/wiki/Tick https://en.wikipedia.org/wiki/Lyme_disease Numerical reasoning | Temporal reasoning ['https://en.wikipedia.org/wiki/Tick', 'https://en.wikipedia.org/wiki/Lyme_disease'] 106 113 What percentage of his total league appearances did footballer Derek Smith (born 1946) make with the team whose original name is shared by a bird impressionist born in the nineteenth century? Give your answer to two decimal places. 95.35% https://en.wikipedia.org/wiki/Derek_Smith_(footballer,_born_1946) https://en.wikipedia.org/wiki/Tranmere_Rovers_F.C. https://en.wikipedia.org/wiki/Ellesmere_Port_Town_F.C. https://en.wikipedia.org/wiki/Joe_Belmont_(bird_impressionist) Numerical reasoning | Tabular reasoning | Multiple constraints | Post processing | Temporal reasoning ['https://en.wikipedia.org/wiki/Derek_Smith_(footballer,_born_1946)', 'https://en.wikipedia.org/wiki/Tranmere_Rovers_F.C.', 'https://en.wikipedia.org/wiki/Ellesmere_Port_Town_F.C.', 'https://en.wikipedia.org/wiki/Joe_Belmont_(bird_impressionist)'] 107 114 Was the founder of the bank that was established 42 years before the National Banking Act was expanded to include the Territory of Hawai'i still alive when it acquired Pioneer Federal Savings Bank? No https://en.wikipedia.org/wiki/U.S._national_banks_of_Hawaii https://en.wikipedia.org/wiki/Charles_Reed_Bishop https://en.wikipedia.org/wiki/First_Hawaiian_Bank Numerical reasoning | Multiple constraints | Temporal reasoning ['https://en.wikipedia.org/wiki/U.S._national_banks_of_Hawaii', 'https://en.wikipedia.org/wiki/Charles_Reed_Bishop', 'https://en.wikipedia.org/wiki/First_Hawaiian_Bank'] 108 115 If you take the height of the Eiffel Tower in metres, add the number of arrondissements in Paris and subtract the street number of the official residence of the French prime minister, is the final answer a prime number? Yes https://en.wikipedia.org/wiki/Eiffel_Tower https://en.wikipedia.org/wiki/Arrondissements_of_Paris https://en.wikipedia.org/wiki/H%C3%B4tel_Matignon Numerical reasoning | Post processing ['https://en.wikipedia.org/wiki/Eiffel_Tower', 'https://en.wikipedia.org/wiki/Arrondissements_of_Paris', 'https://en.wikipedia.org/wiki/H%C3%B4tel_Matignon'] 109 116 Of the 3 largest canary islands, which has the hottest mean temperature in July as of 2024? Tenerife https://en.wikipedia.org/wiki/Canary_Islands https://en.wikipedia.org/wiki/Tenerife https://en.wikipedia.org/wiki/Fuerteventura https://en.wikipedia.org/wiki/Gran_Canaria Numerical reasoning | Multiple constraints ['https://en.wikipedia.org/wiki/Canary_Islands', 'https://en.wikipedia.org/wiki/Tenerife', 'https://en.wikipedia.org/wiki/Fuerteventura', 'https://en.wikipedia.org/wiki/Gran_Canaria'] 110 117 As of 1st August 2024, Are the actors who play Summer and Luke in the OC in anything else together? Yes, Nashville series 5. https://en.wikipedia.org/wiki/The_O.C. https://en.wikipedia.org/wiki/Rachel_Bilson https://en.wikipedia.org/wiki/Chris_Carmack Multiple constraints ['https://en.wikipedia.org/wiki/The_O.C.', 'https://en.wikipedia.org/wiki/Rachel_Bilson', 'https://en.wikipedia.org/wiki/Chris_Carmack'] 111 118 How many years was the first vessel Stephen Etnier commanded constructed after the novel that first inspired him to pursue painting was published? 7 https://en.wikipedia.org/wiki/Stephen_Etnier https://en.wikipedia.org/wiki/The_Moon_and_Sixpence https://en.wikipedia.org/wiki/USS_Mizpah Numerical reasoning | Multiple constraints | Temporal reasoning ['https://en.wikipedia.org/wiki/Stephen_Etnier', 'https://en.wikipedia.org/wiki/The_Moon_and_Sixpence', 'https://en.wikipedia.org/wiki/USS_Mizpah'] 112 119 As of the financial year ending July 31st 2023, what was the size of the endowment at the university attended by rugby player Fred McLeod? £559.8 million https://en.wikipedia.org/wiki/Fred_McLeod_(rugby_union) https://en.wikipedia.org/wiki/University_of_Edinburgh Tabular reasoning ['https://en.wikipedia.org/wiki/Fred_McLeod_(rugby_union)', 'https://en.wikipedia.org/wiki/University_of_Edinburgh'] 113 120 What medal was the captain of The RMS Titanic awarded by King Edward VII? The Transport Medal https://en.wikipedia.org/wiki/Titanic https://en.wikipedia.org/wiki/Edward_Smith_(sea_captain) Multiple constraints ['https://en.wikipedia.org/wiki/Titanic', 'https://en.wikipedia.org/wiki/Edward_Smith_(sea_captain)'] 114 121 What painting was stolen from The Louvre exactly 56 years before the birth of activist and songwriter Serj Tankian? The Mona Lisa https://en.wikipedia.org/wiki/Mona_Lisa#Refuge,_theft,_and_vandalism https://en.wikipedia.org/wiki/Serj_Tankian Numerical reasoning | Temporal reasoning ['https://en.wikipedia.org/wiki/Mona_Lisa#Refuge,_theft,_and_vandalism', 'https://en.wikipedia.org/wiki/Serj_Tankian'] 115 122 As of August 1, 2024, which player who scored more than 10 goals in the 2022 Argentine Premier League season also played for Elche in Spain? Franco Cristaldo scored more than 10 goals that year and also played for Elche. https://en.wikipedia.org/wiki/2022_Argentine_Primera_División https://en.wikipedia.org/wiki/Mateo_Retegui https://en.wikipedia.org/wiki/Franco_Cristaldo https://en.wikipedia.org/wiki/Enzo_Copetti Tabular reasoning | Multiple constraints ['https://en.wikipedia.org/wiki/2022_Argentine_Primera_División', 'https://en.wikipedia.org/wiki/Mateo_Retegui', 'https://en.wikipedia.org/wiki/Franco_Cristaldo', 'https://en.wikipedia.org/wiki/Enzo_Copetti'] 116 123 As of 2024, how many of the Star Wars actors whose first or last names are 'Jack' have starred in more than 2 Star wars movies? 2 https://en.wikipedia.org/wiki/List_of_Star_Wars_film_actors#Introduced_in_The_Skywalker_Saga https://en.wikipedia.org/wiki/Jack_Purvis_(actor) https://en.wikipedia.org/wiki/Jack_Thompson_(actor) https://en.wikipedia.org/wiki/Andrew_Jack_(dialect_coach) Multiple constraints ['https://en.wikipedia.org/wiki/List_of_Star_Wars_film_actors#Introduced_in_The_Skywalker_Saga', 'https://en.wikipedia.org/wiki/Jack_Purvis_(actor)', 'https://en.wikipedia.org/wiki/Jack_Thompson_(actor)', 'https://en.wikipedia.org/wiki/Andrew_Jack_(dialect_coach)'] 117 125 I am thinking of a province that has the smallest land area in it's particular country, but also has the the 10th largest population. This country has 10 provinces. This province joined the country in 1873. What is the scientific name of the provincial flower? Cypripedium Acaule https://en.wikipedia.org/wiki/Province https://en.wikipedia.org/wiki/Provinces_and_territories_of_Canada https://en.wikipedia.org/wiki/Prince_Edward_Island https://en.wikipedia.org/wiki/Cypripedium_acaule Numerical reasoning | Tabular reasoning | Multiple constraints ['https://en.wikipedia.org/wiki/Province', 'https://en.wikipedia.org/wiki/Provinces_and_territories_of_Canada', 'https://en.wikipedia.org/wiki/Prince_Edward_Island', 'https://en.wikipedia.org/wiki/Cypripedium_acaule'] 118 126 As of 1st January 2023, If I am 7 years younger than the eldest granddaughter of the female monarch with the longest reign in confirmed history was at the time of the monarch's death, how old am I? 34 https://en.wikipedia.org/wiki/List_of_longest-reigning_monarchs https://en.wikipedia.org/wiki/Elizabeth_II https://en.wikipedia.org/wiki/Anne,_Princess_Royal https://en.wikipedia.org/wiki/Zara_Tindall https://en.wikipedia.org/wiki/Charles_III https://en.wikipedia.org/wiki/Prince_Andrew,_Duke_of_York https://en.wikipedia.org/wiki/Prince_Edward,_Duke_of_Edinburgh Numerical reasoning | Tabular reasoning | Multiple constraints | Post processing | Temporal reasoning ['https://en.wikipedia.org/wiki/List_of_longest-reigning_monarchs', 'https://en.wikipedia.org/wiki/Elizabeth_II', 'https://en.wikipedia.org/wiki/Anne,_Princess_Royal', 'https://en.wikipedia.org/wiki/Zara_Tindall', 'https://en.wikipedia.org/wiki/Charles_III', 'https://en.wikipedia.org/wiki/Prince_Andrew,_Duke_of_York', 'https://en.wikipedia.org/wiki/Prince_Edward,_Duke_of_Edinburgh'] 119 127 The Office is an American mockumentary sitcom television series that first aired in 2005. Who won the Academy Award for Best Director the same year that the show had its series finale? Ang Lee won the award for best director for Life of Pi. https://en.wikipedia.org/wiki/The_Office_(American_TV_series) https://en.wikipedia.org/wiki/85th_Academy_Awards Multiple constraints ['https://en.wikipedia.org/wiki/The_Office_(American_TV_series)', 'https://en.wikipedia.org/wiki/85th_Academy_Awards'] 120 128 In square KM, how much bigger is the total area of La Casita-Garciasville, Texas compared to that of the independent state that was recognised in the 1929 Lateran Treaty? 10.81 https://en.wikipedia.org/wiki/La_Casita-Garciasville,_Texas https://en.wikipedia.org/wiki/Lateran_Treaty https://en.wikipedia.org/wiki/Vatican_City Tabular reasoning | Multiple constraints ['https://en.wikipedia.org/wiki/La_Casita-Garciasville,_Texas', 'https://en.wikipedia.org/wiki/Lateran_Treaty', 'https://en.wikipedia.org/wiki/Vatican_City'] 121 129 Which fast food restaurant opened first, McDonald's, Wendy's or In-and-Out? McDonald's in 1940 https://en.wikipedia.org/wiki/McDonald%27s https://en.wikipedia.org/wiki/In-N-Out_Burger https://en.wikipedia.org/wiki/Wendy%27s Multiple constraints ['https://en.wikipedia.org/wiki/McDonald%27s', 'https://en.wikipedia.org/wiki/In-N-Out_Burger', 'https://en.wikipedia.org/wiki/Wendy%27s'] 122 130 How much wider in centimeters is the painting that inspired a Stephen Sondheim musical than the 10th most expensive painting ever sold? 211.2 cm https://en.wikipedia.org/wiki/Works_of_Stephen_Sondheim https://en.wikipedia.org/wiki/A_Sunday_Afternoon_on_the_Island_of_La_Grande_Jatte https://en.wikipedia.org/wiki/List_of_most_expensive_paintings https://en.wikipedia.org/wiki/The_Standard_Bearer_(Rembrandt,_1636) Numerical reasoning | Tabular reasoning | Multiple constraints ['https://en.wikipedia.org/wiki/Works_of_Stephen_Sondheim', 'https://en.wikipedia.org/wiki/A_Sunday_Afternoon_on_the_Island_of_La_Grande_Jatte', 'https://en.wikipedia.org/wiki/List_of_most_expensive_paintings', 'https://en.wikipedia.org/wiki/The_Standard_Bearer_(Rembrandt,_1636)'] 123 131 What is the birthplace and hometown of the winning goal scorer of the 2010 Vancouver Olympics, Men's Ice Hockey event? Halifax https://en.wikipedia.org/wiki/Ice_hockey_at_the_2010_Winter_Olympics_%E2%80%93_Men%27s_tournament https://en.wikipedia.org/wiki/Sidney_Crosby Tabular reasoning ['https://en.wikipedia.org/wiki/Ice_hockey_at_the_2010_Winter_Olympics_%E2%80%93_Men%27s_tournament', 'https://en.wikipedia.org/wiki/Sidney_Crosby'] 124 132 Who won the World series the year Happy Days premiered? The Oakland Athletics https://en.wikipedia.org/wiki/Happy_Days#Characters https://en.wikipedia.org/wiki/1974_World_Series Numerical reasoning | Multiple constraints ['https://en.wikipedia.org/wiki/Happy_Days#Characters', 'https://en.wikipedia.org/wiki/1974_World_Series'] 125 133 How many days after the United States release of the record holder for largest sweep at the 2004 Oscars was the death of that movie's cinematographer? 4149 days https://en.wikipedia.org/wiki/List_of_Academy_Award_records https://en.wikipedia.org/wiki/The_Lord_of_the_Rings:_The_Return_of_the_King https://en.wikipedia.org/wiki/Andrew_Lesnie Numerical reasoning | Multiple constraints | Temporal reasoning ['https://en.wikipedia.org/wiki/List_of_Academy_Award_records', 'https://en.wikipedia.org/wiki/The_Lord_of_the_Rings:_The_Return_of_the_King', 'https://en.wikipedia.org/wiki/Andrew_Lesnie'] 126 134 In the Belgian capital, there is a street named after King Leopold II's oldest daughter which is lined with chestnut trees and is the home to many embassies. What is the capital of the country whose embassy is found at number 425? Zagreb https://en.wikipedia.org/wiki/Belgium https://en.wikipedia.org/wiki/City_of_Brussels https://en.wikipedia.org/wiki/Avenue_Louise https://en.wikipedia.org/wiki/Croatia Multiple constraints ['https://en.wikipedia.org/wiki/Belgium', 'https://en.wikipedia.org/wiki/City_of_Brussels', 'https://en.wikipedia.org/wiki/Avenue_Louise', 'https://en.wikipedia.org/wiki/Croatia'] 127 135 I'm thinking of a man whose first name is Hart. He acted in a miniseries (based on a historical novel about WW2 with three words in its title) by an author who also wrote a book about a kid with the last name of Bookbinder. Hart Bochner https://en.wikipedia.org/wiki/City_Boy:_The_Adventures_of_Herbie_Bookbinder https://en.wikipedia.org/wiki/Herman_Wouk https://en.wikipedia.org/wiki/War_and_Remembrance_(miniseries) https://en.wikipedia.org/wiki/Hart_Bochner Multiple constraints ['https://en.wikipedia.org/wiki/City_Boy:_The_Adventures_of_Herbie_Bookbinder', 'https://en.wikipedia.org/wiki/Herman_Wouk', 'https://en.wikipedia.org/wiki/War_and_Remembrance_(miniseries)', 'https://en.wikipedia.org/wiki/Hart_Bochner'] 128 136 Name the teams in alphabetical order that every AL MVP from Texas Rangers retired from as of August 2024. Cleveland Indians, New York Yankees, Texas Rangers, Toronto Blue Jays https://en.wikipedia.org/wiki/Texas_Rangers_award_winners_and_league_leaders https://en.wikipedia.org/wiki/Jeff_Burroughs https://en.wikipedia.org/wiki/Juan_Gonz%C3%A1lez_(baseball) https://en.wikipedia.org/wiki/Iv%C3%A1n_Rodr%C3%ADguez https://en.wikipedia.org/wiki/Alex_Rodriguez https://en.wikipedia.org/wiki/Josh_Hamilton Post processing | Temporal reasoning ['https://en.wikipedia.org/wiki/Texas_Rangers_award_winners_and_league_leaders', 'https://en.wikipedia.org/wiki/Jeff_Burroughs', 'https://en.wikipedia.org/wiki/Juan_Gonz%C3%A1lez_(baseball)', 'https://en.wikipedia.org/wiki/Iv%C3%A1n_Rodr%C3%ADguez', 'https://en.wikipedia.org/wiki/Alex_Rodriguez', 'https://en.wikipedia.org/wiki/Josh_Hamilton'] 129 137 What films did Big Hit Music's 7-member boy group release in the year that the company's 5-member boy group first debuted? Love Yourself in Seoul (2019) and Bring the Soul: The Movie (2019) https://en.wikipedia.org/wiki/Big_Hit_Music#Groups https://en.wikipedia.org/wiki/BTS# https://en.wikipedia.org/wiki/Tomorrow_X_Together Multiple constraints | Temporal reasoning ['https://en.wikipedia.org/wiki/Big_Hit_Music#Groups', 'https://en.wikipedia.org/wiki/BTS#', 'https://en.wikipedia.org/wiki/Tomorrow_X_Together'] 130 138 What year was the University that gave Taylor Swift an honorary doctorate founded? 1831 (New York University) https://en.wikipedia.org/wiki/List_of_awards_and_nominations_received_by_Taylor_Swift#Honorary_degree https://en.wikipedia.org/wiki/New_York_University Multiple constraints ['https://en.wikipedia.org/wiki/List_of_awards_and_nominations_received_by_Taylor_Swift#Honorary_degree', 'https://en.wikipedia.org/wiki/New_York_University'] 131 139 What is the etymology of the name of the province to the east of the province in which Hazrati Sultan District is located? The Dari name 'Baghlan' comes from the Bactrian 'Bagolango', meaning 'image-temple' https://en.wikipedia.org/wiki/Hazrati_Sultan_District https://en.wikipedia.org/wiki/Samangan_Province https://en.wikipedia.org/wiki/Baghlan_Province https://en.wikipedia.org/wiki/Kushan_Empire https://en.wikipedia.org/wiki/Bactrian_language Multiple constraints ['https://en.wikipedia.org/wiki/Hazrati_Sultan_District', 'https://en.wikipedia.org/wiki/Samangan_Province', 'https://en.wikipedia.org/wiki/Baghlan_Province', 'https://en.wikipedia.org/wiki/Kushan_Empire', 'https://en.wikipedia.org/wiki/Bactrian_language'] 132 140 "Consider the number of months lapsing between the major earthquake that caused widespread destruction around the Bay of Naples in 62 CE to the eruption of Mount Vesuvius in 79 CE which buried Pompeii in ash. If Mount Vesuvius were to have erupted once whenever that number of months came to pass between its 79 CE eruption and the date on which ""Pompeii"" by Bastille was officially released, how many times would the volcano have erupted between those two events?" 109 times https://en.wikipedia.org/wiki/Eruption_of_Mount_Vesuvius_in_79_AD https://en.wikipedia.org/wiki/Pompeii_(song) Numerical reasoning | Temporal reasoning ['https://en.wikipedia.org/wiki/Eruption_of_Mount_Vesuvius_in_79_AD', 'https://en.wikipedia.org/wiki/Pompeii_(song)'] 133 141 "Multiple the number of Tony's won by the guest host of SNL 12/6/1997 by the number of Oscar nominations received by the 2023 film directed by Greta Gerwig. Then divide this number by the number of Grammy's won by the band behind the 1979 album ""Tusk""." 12 https://en.wikipedia.org/wiki/Saturday_Night_Live_season_23 https://en.wikipedia.org/wiki/Nathan_Lane https://en.wikipedia.org/wiki/List_of_awards_and_nominations_received_by_Nathan_Lane https://en.wikipedia.org/wiki/Barbie_(film) https://en.wikipedia.org/wiki/List_of_accolades_received_by_Barbie_(film) https://en.wikipedia.org/wiki/Tusk_(album) https://en.wikipedia.org/wiki/Fleetwood_Mac#Grammy_Awards Numerical reasoning | Tabular reasoning | Multiple constraints | Post processing | Temporal reasoning ['https://en.wikipedia.org/wiki/Saturday_Night_Live_season_23', 'https://en.wikipedia.org/wiki/Nathan_Lane', 'https://en.wikipedia.org/wiki/List_of_awards_and_nominations_received_by_Nathan_Lane', 'https://en.wikipedia.org/wiki/Barbie_(film)', 'https://en.wikipedia.org/wiki/List_of_accolades_received_by_Barbie_(film)', 'https://en.wikipedia.org/wiki/Tusk_(album)', 'https://en.wikipedia.org/wiki/Fleetwood_Mac#Grammy_Awards'] 134 142 On March 3rd during the year of Mariah Carey's birth, a famous space launch occurred and the mission lasted for how many days? 10 days https://en.wikipedia.org/wiki/Mariah_Carey https://en.wikipedia.org/wiki/1969 https://en.wikipedia.org/wiki/Apollo_9 Multiple constraints ['https://en.wikipedia.org/wiki/Mariah_Carey', 'https://en.wikipedia.org/wiki/1969', 'https://en.wikipedia.org/wiki/Apollo_9'] 135 143 What is the Chinese name for the bodhisattva that the Sensoji temple is dedicated to? Guanyin https://en.wikipedia.org/wiki/Sens%C5%8D-ji https://en.wikipedia.org/wiki/Avalokite%C5%9Bvara Multiple constraints ['https://en.wikipedia.org/wiki/Sens%C5%8D-ji', 'https://en.wikipedia.org/wiki/Avalokite%C5%9Bvara'] 136 144 Which president of the United States is represented by the sum of the ordinal numbers of the presidencies of the four men depicted on Mount Rushmore? Joe Biden https://en.wikipedia.org/wiki/Mount_Rushmore#History https://en.wikipedia.org/wiki/George_Washington https://en.wikipedia.org/wiki/Thomas_Jefferson https://en.wikipedia.org/wiki/Theodore_Roosevelt https://en.wikipedia.org/wiki/Abraham_Lincoln https://en.wikipedia.org/wiki/Presidency_of_Joe_Biden https://en.wikipedia.org/wiki/Ordinal_number Numerical reasoning | Post processing ['https://en.wikipedia.org/wiki/Mount_Rushmore#History', 'https://en.wikipedia.org/wiki/George_Washington', 'https://en.wikipedia.org/wiki/Thomas_Jefferson', 'https://en.wikipedia.org/wiki/Theodore_Roosevelt', 'https://en.wikipedia.org/wiki/Abraham_Lincoln', 'https://en.wikipedia.org/wiki/Presidency_of_Joe_Biden', 'https://en.wikipedia.org/wiki/Ordinal_number'] 137 145 If we consider their inception being the date of declared independence, how many years older is the US than Mexico? 45 years https://en.wikipedia.org/wiki/Mexico https://en.wikipedia.org/wiki/United_States Numerical reasoning | Temporal reasoning ['https://en.wikipedia.org/wiki/Mexico', 'https://en.wikipedia.org/wiki/United_States'] 138 146 The manager of the Schenectady Blue Jays in 1953 also played Major League Baseball for which teams? Skeeter Newsome - Philadelphia Athletics, Boston Red Sox, and Philadelphia Phillies https://en.wikipedia.org/wiki/Schenectady_Blue_Jays https://en.wikipedia.org/wiki/Skeeter_Newsome Multiple constraints ['https://en.wikipedia.org/wiki/Schenectady_Blue_Jays', 'https://en.wikipedia.org/wiki/Skeeter_Newsome'] 139 147 "What is the name of the play written in May 2016 by a playwright who won the MacArthur Fellowship the same year as the poet who wrote ""Postcolonial Love Poem""?" Skeleton Crew https://en.wikipedia.org/wiki/Postcolonial_Love_Poem https://en.wikipedia.org/wiki/Natalie_Diaz https://en.wikipedia.org/wiki/MacArthur_Fellows_Program# https://en.wikipedia.org/wiki/Dominique_Morisseau Multiple constraints | Temporal reasoning ['https://en.wikipedia.org/wiki/Postcolonial_Love_Poem', 'https://en.wikipedia.org/wiki/Natalie_Diaz', 'https://en.wikipedia.org/wiki/MacArthur_Fellows_Program#', 'https://en.wikipedia.org/wiki/Dominique_Morisseau'] 140 148 What is the birth date of the person picked right after Lee Vaughn in the 1997 NFL draft? August 24, 1974 https://en.wikipedia.org/wiki/Lee_Vaughn https://en.wikipedia.org/wiki/1997_NFL_draft#Round_6 https://en.wikipedia.org/wiki/Tony_McCombs Temporal reasoning ['https://en.wikipedia.org/wiki/Lee_Vaughn', 'https://en.wikipedia.org/wiki/1997_NFL_draft#Round_6', 'https://en.wikipedia.org/wiki/Tony_McCombs'] 141 150 "Suppose Egon Sendler's book ""Les mystères du Christ: Icônes de la liturgie"" was written in the same year as the birth of Nemanja Markovic. How old would the book be when the New Hampshire state election results for the Democratic party were 53.9% and 125,822 votes?" 51 years. https://en.wikipedia.org/wiki/Egon_Sendler https://en.wikipedia.org/wiki/Elections_in_New_Hampshire https://en.wikipedia.org/wiki/Nemanja_Markovi%C4%87 Numerical reasoning | Tabular reasoning ['https://en.wikipedia.org/wiki/Egon_Sendler', 'https://en.wikipedia.org/wiki/Elections_in_New_Hampshire', 'https://en.wikipedia.org/wiki/Nemanja_Markovi%C4%87'] 142 151 As of August 5, 2024, what is the name of the federal law that was found to be violated by the company that Brian Bergstein is employed by? The Sherman Antitrust Act https://en.wikipedia.org/wiki/Brian_Bergstein https://en.wikipedia.org/wiki/Associated_Press https://en.wikipedia.org/wiki/Associated_Press_v._United_States https://en.wikipedia.org/wiki/Sherman_Antitrust_Act Multiple constraints ['https://en.wikipedia.org/wiki/Brian_Bergstein', 'https://en.wikipedia.org/wiki/Associated_Press', 'https://en.wikipedia.org/wiki/Associated_Press_v._United_States', 'https://en.wikipedia.org/wiki/Sherman_Antitrust_Act'] 143 152 How many more votes did the Conservatives receive in the Highlands and Islands region in the 2021 Scottish Parliamentary Elections than in 2016? 16,086 https://en.wikipedia.org/wiki/2021_Scottish_Parliament_election https://en.wikipedia.org/wiki/2016_Scottish_Parliament_election Numerical reasoning ['https://en.wikipedia.org/wiki/2021_Scottish_Parliament_election', 'https://en.wikipedia.org/wiki/2016_Scottish_Parliament_election'] 144 154 What lake in Isreal supports a population of the state bird of the Indian state Haryana? The Sea of Galilee https://en.wikipedia.org/wiki/Sea_of_Galilee https://en.wikipedia.org/wiki/Black_francolin Multiple constraints ['https://en.wikipedia.org/wiki/Sea_of_Galilee', 'https://en.wikipedia.org/wiki/Black_francolin'] 145 155 As of August 3, 2024, what is the sum of the birth years of every tennis player to both represent the country that tennis was first played and complete a Grand Slam. 5980 https://en.wikipedia.org/wiki/Tennis https://en.wikipedia.org/wiki/Grand_Slam_(tennis) https://en.wikipedia.org/wiki/Jordanne_Whiley https://en.wikipedia.org/wiki/Gordon_Reid_(tennis) https://en.wikipedia.org/wiki/Alfie_Hewett Tabular reasoning | Multiple constraints | Post processing ['https://en.wikipedia.org/wiki/Tennis', 'https://en.wikipedia.org/wiki/Grand_Slam_(tennis)', 'https://en.wikipedia.org/wiki/Jordanne_Whiley', 'https://en.wikipedia.org/wiki/Gordon_Reid_(tennis)', 'https://en.wikipedia.org/wiki/Alfie_Hewett'] 146 156 Rosie Ruiz was disqualified from the Boston Marathon. The subsequent winner placed in what position of the 1988 Grandma's Marathon? Jacqueline Gareau placed first in the 1988 Grandma's Marathon. https://en.wikipedia.org/wiki/Boston_Marathon#Rosie_Ruiz,_the_impostor https://en.wikipedia.org/wiki/Jacqueline_Gareau Tabular reasoning | Multiple constraints ['https://en.wikipedia.org/wiki/Boston_Marathon#Rosie_Ruiz,_the_impostor', 'https://en.wikipedia.org/wiki/Jacqueline_Gareau'] 147 157 On Nov 15, 2017 a painting was sold for US $450 million setting a new record for the most expensive painting ever sold at public auction. What year was the auction house where this purchase took place founded? 1766 https://en.wikipedia.org/wiki/List_of_most_expensive_paintings https://en.wikipedia.org/wiki/Christie%27s Multiple constraints ['https://en.wikipedia.org/wiki/List_of_most_expensive_paintings', 'https://en.wikipedia.org/wiki/Christie%27s'] 148 158 A disease that had millions of dollars raised for on April 20, 1992, was first recognized by the Center for Disease Control and Prevention (CDC) in what year? 1981 https://en.wikipedia.org/wiki/1992 https://en.wikipedia.org/wiki/HIV/AIDS Multiple constraints ['https://en.wikipedia.org/wiki/1992', 'https://en.wikipedia.org/wiki/HIV/AIDS'] 149 160 As of July 1, 2023, what is the total number of letters in the names of the capital cities of the 5 most populated countries in the world? 43 https://en.wikipedia.org/wiki/List_of_countries_by_population_(United_Nations) https://en.wikipedia.org/wiki/India https://en.wikipedia.org/wiki/China https://en.wikipedia.org/wiki/United_States https://en.wikipedia.org/wiki/Indonesia https://en.wikipedia.org/wiki/Pakistan Numerical reasoning | Post processing ['https://en.wikipedia.org/wiki/List_of_countries_by_population_(United_Nations)', 'https://en.wikipedia.org/wiki/India', 'https://en.wikipedia.org/wiki/China', 'https://en.wikipedia.org/wiki/United_States', 'https://en.wikipedia.org/wiki/Indonesia', 'https://en.wikipedia.org/wiki/Pakistan'] 150 161 "Where was the rapper behind the song ""Hind's Hall"" born? " Seattle, Washington https://en.wikipedia.org/wiki/Hind%27s_Hall https://en.wikipedia.org/wiki/Macklemore Multiple constraints ['https://en.wikipedia.org/wiki/Hind%27s_Hall', 'https://en.wikipedia.org/wiki/Macklemore'] 151 162 Based on the information available on Wikipedia on August 4, 2024 at 2:42 AM Greenwich Mean Time, which of the following areas, Minamidaitōjima, Nuapada district, or Vostochnaya Niva has the highest population and how many more citizens does it contain than the other two provided cities? The Nuapada district contains 608,269 more people than the other two areas combined. https://en.wikipedia.org/wiki/Nuapada_district https://en.wikipedia.org/wiki/Vostochnaya_Niva https://en.wikipedia.org/wiki/Minamidait%C5%8Djima Numerical reasoning | Post processing ['https://en.wikipedia.org/wiki/Nuapada_district', 'https://en.wikipedia.org/wiki/Vostochnaya_Niva', 'https://en.wikipedia.org/wiki/Minamidait%C5%8Djima'] 152 163 What are the first three letters of the capital city of the country where Shakespeare's longest play is set? Cop https://en.wikipedia.org/wiki/Hamlet https://en.wikipedia.org/wiki/Denmark Multiple constraints | Post processing ['https://en.wikipedia.org/wiki/Hamlet', 'https://en.wikipedia.org/wiki/Denmark'] 153 164 How many of Ron Hutchinson's teams won League Championships while he was on them? Of the six hockey teams that Hutchinson played on during his career, two, the Flin Flon Bombers and the Vancouver Canucks, took home League Championships during his time on the roster. https://en.wikipedia.org/wiki/Ron_Hutchinson_(ice_hockey) https://en.wikipedia.org/wiki/Flin_Flon_Bombers https://en.wikipedia.org/wiki/Vancouver_Canucks_(WHL) https://en.wikipedia.org/wiki/New_York_Rangers https://en.wikipedia.org/wiki/Seattle_Totems https://en.wikipedia.org/wiki/Charlotte_Checkers_(1956%E2%80%931977) https://en.wikipedia.org/wiki/Cranbrook_Royals Tabular reasoning ['https://en.wikipedia.org/wiki/Ron_Hutchinson_(ice_hockey)', 'https://en.wikipedia.org/wiki/Flin_Flon_Bombers', 'https://en.wikipedia.org/wiki/Vancouver_Canucks_(WHL)', 'https://en.wikipedia.org/wiki/New_York_Rangers', 'https://en.wikipedia.org/wiki/Seattle_Totems', 'https://en.wikipedia.org/wiki/Charlotte_Checkers_(1956%E2%80%931977)', 'https://en.wikipedia.org/wiki/Cranbrook_Royals'] 154 165 What Pink Floyd album came out the year Pablo Picasso died? Dark Side of the Moon https://en.wikipedia.org/wiki/Pablo_Picasso https://en.wikipedia.org/wiki/The_Dark_Side_of_the_Moon Multiple constraints ['https://en.wikipedia.org/wiki/Pablo_Picasso', 'https://en.wikipedia.org/wiki/The_Dark_Side_of_the_Moon'] 155 166 As of August 1 2024, what books has the author of the Harry Potter series written under an alias? The Cuckoo's Calling, The Silkworm, Career of Evil, Lethal White, Troubled Blood, The Ink Black Heart, The Running Grave https://en.wikipedia.org/wiki/Harry_Potter https://en.wikipedia.org/wiki/J._K._Rowling Multiple constraints ['https://en.wikipedia.org/wiki/Harry_Potter', 'https://en.wikipedia.org/wiki/J._K._Rowling'] 156 167 What football team did Travis Kelce play for the year Taylor Swift's VMA acceptance speech was interrupted by Kanye West? The University of Cincinnati Bearcats https://en.wikipedia.org/wiki/Taylor_Swift https://en.wikipedia.org/wiki/Travis_Kelce Multiple constraints | Temporal reasoning ['https://en.wikipedia.org/wiki/Taylor_Swift', 'https://en.wikipedia.org/wiki/Travis_Kelce'] 157 168 What horror movie remake did the director who was the first to attempt and failed to make Brokeback Mountian into a film direct in the 90's? Psycho https://en.wikipedia.org/wiki/Brokeback_Mountain https://en.wikipedia.org/wiki/Gus_Van_Sant https://en.wikipedia.org/wiki/Psycho_(1998_film) Tabular reasoning | Temporal reasoning ['https://en.wikipedia.org/wiki/Brokeback_Mountain', 'https://en.wikipedia.org/wiki/Gus_Van_Sant', 'https://en.wikipedia.org/wiki/Psycho_(1998_film)'] 158 169 As of 2024, how many total Academy award nominations has the the man who won the Academy award for best actor one year before 1999 received? 12 https://en.wikipedia.org/wiki/70th_Academy_Awards https://en.wikipedia.org/wiki/Jack_Nicholson Numerical reasoning | Post processing | Temporal reasoning ['https://en.wikipedia.org/wiki/70th_Academy_Awards', 'https://en.wikipedia.org/wiki/Jack_Nicholson'] 159 170 What is the difference between the fastest recorded swimming speed of a fish, and the fastest record for swimming the 50m freestyle in the 2020 Tokyo Olympics in meters per second? 34.30 m/s https://en.wikipedia.org/wiki/Fastest_animals#Fish https://en.wikipedia.org/wiki/List_of_Olympic_records_in_swimming Numerical reasoning ['https://en.wikipedia.org/wiki/Fastest_animals#Fish', 'https://en.wikipedia.org/wiki/List_of_Olympic_records_in_swimming'] 160 171 How many years separate the birth of Alexander Graham Bell and the birth of Charles Dickens? 35 https://en.wikipedia.org/wiki/Alexander_Graham_Bell https://en.wikipedia.org/wiki/Charles_Dickens Numerical reasoning ['https://en.wikipedia.org/wiki/Alexander_Graham_Bell', 'https://en.wikipedia.org/wiki/Charles_Dickens'] 161 172 Stamatios Krimigis was named after an asteroid by the IAU. The observing site that found this asteroid is part of a large observatory known for discovering a planet. What is the difference in years between the discovery of this asteroid and the discovery of the planet? 49 years https://en.wikipedia.org/wiki/Stamatios_Krimigis https://en.wikipedia.org/wiki/List_of_minor_planets:_8001%E2%80%939000#323c https://en.wikipedia.org/wiki/Anderson_Mesa_Station https://en.wikipedia.org/wiki/Lowell_Observatory https://en.wikipedia.org/wiki/Pluto Numerical reasoning | Tabular reasoning ['https://en.wikipedia.org/wiki/Stamatios_Krimigis', 'https://en.wikipedia.org/wiki/List_of_minor_planets:_8001%E2%80%939000#323c', 'https://en.wikipedia.org/wiki/Anderson_Mesa_Station', 'https://en.wikipedia.org/wiki/Lowell_Observatory', 'https://en.wikipedia.org/wiki/Pluto'] 162 173 What two buildings can be said to have introduced the onset of the architectural style of Big Ben in London? Dromore Cathedral and The Great Hall of Lambeth Palace https://en.wikipedia.org/wiki/Big_Ben#Design https://en.wikipedia.org/wiki/Gothic_Revival_architecture#Roots Multiple constraints ['https://en.wikipedia.org/wiki/Big_Ben#Design', 'https://en.wikipedia.org/wiki/Gothic_Revival_architecture#Roots'] 163 174 How many years after Ghengis Khan died did World War II begin? 712 years https://en.wikipedia.org/wiki/Genghis_Khan https://en.wikipedia.org/wiki/World_War_II Numerical reasoning | Post processing | Temporal reasoning ['https://en.wikipedia.org/wiki/Genghis_Khan', 'https://en.wikipedia.org/wiki/World_War_II'] 164 175 "Among the singers in the 1985 version of ""That's What Friends Are For,"" which one was born in the Peach State?" Gladys Knight https://en.wikipedia.org/wiki/That%27s_What_Friends_Are_For https://en.wikipedia.org/wiki/Dionne_Warwick https://en.wikipedia.org/wiki/Elton_John https://en.wikipedia.org/wiki/Stevie_Wonder https://en.wikipedia.org/wiki/Gladys_Knight https://en.wikipedia.org/wiki/Georgia_(U.S._state) Multiple constraints ['https://en.wikipedia.org/wiki/That%27s_What_Friends_Are_For', 'https://en.wikipedia.org/wiki/Dionne_Warwick', 'https://en.wikipedia.org/wiki/Elton_John', 'https://en.wikipedia.org/wiki/Stevie_Wonder', 'https://en.wikipedia.org/wiki/Gladys_Knight', 'https://en.wikipedia.org/wiki/Georgia_(U.S._state)'] 165 176 On what station did the television show that started in 1993 and had a star who shared a name with the third wife of King Henry VIII first run? CBS https://en.wikipedia.org/wiki/Jane_Seymour https://en.wikipedia.org/wiki/Jane_Seymour_(actress) https://en.wikipedia.org/wiki/Dr._Quinn,_Medicine_Woman Tabular reasoning | Multiple constraints ['https://en.wikipedia.org/wiki/Jane_Seymour', 'https://en.wikipedia.org/wiki/Jane_Seymour_(actress)', 'https://en.wikipedia.org/wiki/Dr._Quinn,_Medicine_Woman'] 166 177 As of January 1, 2024, are any members of Vampire Weekend Capricorn? If no, what are their signs? No. Ezra Koenig is Aries, Chris Baio is Scorpio, and Chris Tomson is Pisces. https://en.wikipedia.org/wiki/Vampire_Weekend https://en.wikipedia.org/wiki/Ezra_Koenig https://en.wikipedia.org/wiki/Chris_Baio https://en.wikipedia.org/wiki/Chris_Tomson https://en.wikipedia.org/wiki/Astrological_sign Multiple constraints ['https://en.wikipedia.org/wiki/Vampire_Weekend', 'https://en.wikipedia.org/wiki/Ezra_Koenig', 'https://en.wikipedia.org/wiki/Chris_Baio', 'https://en.wikipedia.org/wiki/Chris_Tomson', 'https://en.wikipedia.org/wiki/Astrological_sign'] 167 178 Who is married to the actor who plays Imdad Khan in the film version of The Wonderful Story of Henry Sugar, as of August 1, 2024 ? Daniela Lavender https://en.wikipedia.org/wiki/The_Wonderful_Story_of_Henry_Sugar_(film) https://en.wikipedia.org/wiki/Ben_Kingsley Multiple constraints ['https://en.wikipedia.org/wiki/The_Wonderful_Story_of_Henry_Sugar_(film)', 'https://en.wikipedia.org/wiki/Ben_Kingsley'] 168 179 What is the difference in mean flow rate (in cubic feet per second) between the River Avon at Great Somerford and its tributary, the River Marden? 75.5 cubic ft/s https://en.wikipedia.org/wiki/River_Avon,_Bristol#Hydrology_and_water_quality https://en.wikipedia.org/wiki/River_Marden Numerical reasoning ['https://en.wikipedia.org/wiki/River_Avon,_Bristol#Hydrology_and_water_quality', 'https://en.wikipedia.org/wiki/River_Marden'] 169 180 I am thinking of a country. A former member of Swedish Parliament during 2002 – 2006 was born there. English is the official language but many other languages are spoken there. The Trans–West African Coastal Highway passes through this country. In June 2020, Democratic Party leaders in the United States caused controversy by wearing stoles made of cloth from this country. Ghana https://en.wikipedia.org/wiki/Joe_Frans_(politician) https://en.wikipedia.org/wiki/Languages_of_Ghana https://en.wikipedia.org/wiki/Trans%E2%80%93West_African_Coastal_Highway https://en.wikipedia.org/wiki/Kente_cloth#cite_note-CNN-2020-06-08-18 Multiple constraints ['https://en.wikipedia.org/wiki/Joe_Frans_(politician)', 'https://en.wikipedia.org/wiki/Languages_of_Ghana', 'https://en.wikipedia.org/wiki/Trans%E2%80%93West_African_Coastal_Highway', 'https://en.wikipedia.org/wiki/Kente_cloth#cite_note-CNN-2020-06-08-18'] 170 181 Which film, based loosely on the story of Frederick 'Fritz' Niland, won the Golden Globe for best drama at the 56th Golden Globes in 1999? Saving Private Ryan was loosely based on the story of Frederick 'Fritz' Niland, and won the Golden Globe for best drama at the 56th Golden Globes in 1999. https://en.wikipedia.org/wiki/Niland_brothers https://en.wikipedia.org/wiki/Saving_Private_Ryan#Reception Multiple constraints ['https://en.wikipedia.org/wiki/Niland_brothers', 'https://en.wikipedia.org/wiki/Saving_Private_Ryan#Reception'] 171 182 Which player scored 20 goals in the English Premier League in the 2006-2007 season and won 'Chelsea Players Player of the Year' award in 2007? Didier Drogba scored 20 goals in the the 2006-2007 English Premier League season and won the 'Chelsea Players Player of the Year' award in 2007. https://en.wikipedia.org/wiki/2006–07_FA_Premier_League https://en.wikipedia.org/wiki/Didier_Drogba#Honours Tabular reasoning | Multiple constraints | Temporal reasoning ['https://en.wikipedia.org/wiki/2006–07_FA_Premier_League', 'https://en.wikipedia.org/wiki/Didier_Drogba#Honours'] 172 183 Which Northern Irish footballer who started in the 90s played for 9 English clubs, 8 of which were FA Cup winners? Keith Gillespie https://en.wikipedia.org/wiki/List_of_Northern_Ireland_international_footballers#List_of_players https://en.wikipedia.org/wiki/Keith_Gillespie https://en.wikipedia.org/wiki/Manchester_United_F.C. https://en.wikipedia.org/wiki/Wigan_Athletic_F.C. https://en.wikipedia.org/wiki/Newcastle_United_F.C. https://en.wikipedia.org/wiki/Blackburn_Rovers_F.C. https://en.wikipedia.org/wiki/Leicester_City_F.C. https://en.wikipedia.org/wiki/Sheffield_United_F.C. https://en.wikipedia.org/wiki/Charlton_Athletic_F.C. https://en.wikipedia.org/wiki/Bradford_City_A.F.C. https://en.wikipedia.org/wiki/Darlington_F.C. Multiple constraints ['https://en.wikipedia.org/wiki/List_of_Northern_Ireland_international_footballers#List_of_players', 'https://en.wikipedia.org/wiki/Keith_Gillespie', 'https://en.wikipedia.org/wiki/Manchester_United_F.C.', 'https://en.wikipedia.org/wiki/Wigan_Athletic_F.C.', 'https://en.wikipedia.org/wiki/Newcastle_United_F.C.', 'https://en.wikipedia.org/wiki/Blackburn_Rovers_F.C.', 'https://en.wikipedia.org/wiki/Leicester_City_F.C.', 'https://en.wikipedia.org/wiki/Sheffield_United_F.C.', 'https://en.wikipedia.org/wiki/Charlton_Athletic_F.C.', 'https://en.wikipedia.org/wiki/Bradford_City_A.F.C.', 'https://en.wikipedia.org/wiki/Darlington_F.C.'] 173 184 In roman numerals, how many nations competed in the Olympic Games where the most gold medals was won by an athlete at a single Olympic Games, as of 1st July 2024? CCIV https://en.wikipedia.org/wiki/List_of_multiple_Olympic_gold_medalists_at_a_single_Games https://en.wikipedia.org/wiki/2008_Summer_Olympics Tabular reasoning | Post processing ['https://en.wikipedia.org/wiki/List_of_multiple_Olympic_gold_medalists_at_a_single_Games', 'https://en.wikipedia.org/wiki/2008_Summer_Olympics'] 174 185 Which jetliner first flown on June 12th, 1994 is also widely used, as of August 3, 2024, by an airline using the ICE system for entertainment on board? The Boeing 777 was first flown on the 12th of June, 1994 and is widely used by Emirates, which uses the ICE system on board. https://en.wikipedia.org/wiki/Boeing_777 https://en.wikipedia.org/wiki/Emirates_(airline)#Services Multiple constraints | Post processing ['https://en.wikipedia.org/wiki/Boeing_777', 'https://en.wikipedia.org/wiki/Emirates_(airline)#Services'] 175 186 If Andrew Fluegelman's suggested donation for his freeware program were paid at a rate of 1 per every day, how much money would he have made during his fast? $1,225 https://en.wikipedia.org/wiki/PC-Talk https://en.wikipedia.org/wiki/Andrew_Fluegelman Numerical reasoning ['https://en.wikipedia.org/wiki/PC-Talk', 'https://en.wikipedia.org/wiki/Andrew_Fluegelman'] 176 187 For the year 2020, what was the difference in total fertility rate (TFR) for East Timor and Japan? 1.92 https://en.wikipedia.org/wiki/Demographics_of_East_Timor https://en.wikipedia.org/wiki/Demographics_of_Japan Numerical reasoning ['https://en.wikipedia.org/wiki/Demographics_of_East_Timor', 'https://en.wikipedia.org/wiki/Demographics_of_Japan'] 177 188 As of August 1, 2024, if you add together the age of Ana Ollo Hualde and the age of the country of Israel what number do you get when you subtract 35 from your answer? 100 (59 + 76) - 35 https://en.wikipedia.org/wiki/Ana_Ollo https://en.wikipedia.org/wiki/Israel Numerical reasoning | Post processing ['https://en.wikipedia.org/wiki/Ana_Ollo', 'https://en.wikipedia.org/wiki/Israel'] 178 189 "I'm trying to show my daughter some of the movies I grew up with. There's one a really want to show her but I can't remember the name of it. I remember that the male lead also played Fred in a live action Scooby Doo movie and the main girl was in this crazy anti-drug commercial in the 90s where she used a pan to destroy a bunch of things in a kitchen and said ""This is your brain on drugs..."". The movie is about a guy who makes a bet with his friend that he can turn an unpopular girl into prom queen. Can you tell me the name of the film?" *She's All That* https://en.wikipedia.org/wiki/Scooby-Doo_(film) https://en.wikipedia.org/wiki/Freddie_Prinze_Jr. https://en.wikipedia.org/wiki/This_Is_Your_Brain_on_Drugs https://en.wikipedia.org/wiki/Rachael_Leigh_Cook https://en.wikipedia.org/wiki/She%27s_All_That Multiple constraints | Temporal reasoning ['https://en.wikipedia.org/wiki/Scooby-Doo_(film)', 'https://en.wikipedia.org/wiki/Freddie_Prinze_Jr.', 'https://en.wikipedia.org/wiki/This_Is_Your_Brain_on_Drugs', 'https://en.wikipedia.org/wiki/Rachael_Leigh_Cook', 'https://en.wikipedia.org/wiki/She%27s_All_That'] 179 190 Which major city in Europe can be reached from New York City, if you use the total distance calculated through GPA coordinates (give or take 18 miles) from West Quoddy Light in Maine to Cape Sarichef Light in Alaska? Prague, Czech Republic https://en.wikipedia.org/wiki/West_Quoddy_Head_Light https://en.wikipedia.org/wiki/Cape_Sarichef_Light https://en.wikipedia.org/wiki/Module:Location_map/data/USA_New_York_City https://en.wikipedia.org/wiki/Prague#:~:text=Prague%20is%20located%20approximately%20at,N%2014%C2%B025%E2%80%B2E. Numerical reasoning | Multiple constraints | Temporal reasoning ['https://en.wikipedia.org/wiki/West_Quoddy_Head_Light', 'https://en.wikipedia.org/wiki/Cape_Sarichef_Light', 'https://en.wikipedia.org/wiki/Module:Location_map/data/USA_New_York_City', 'https://en.wikipedia.org/wiki/Prague#:~:text=Prague%20is%20located%20approximately%20at,N%2014%C2%B025%E2%80%B2E.'] 180 192 When did the actress with multiple sclerosis who starred in the comedy about killing her husband receive a star on the Hollywood Walk of Fame? November 14, 2022 https://en.wikipedia.org/wiki/Dead_to_Me_(TV_series) https://en.wikipedia.org/wiki/Christina_Applegate Multiple constraints ['https://en.wikipedia.org/wiki/Dead_to_Me_(TV_series)', 'https://en.wikipedia.org/wiki/Christina_Applegate'] 181 193 What is so distinctive about the label design of an Australian record label which was purchased in 1960 by another record label which produced the debut album for The Clash? The octagonal shape https://en.wikipedia.org/wiki/The_Clash https://en.wikipedia.org/wiki/The_Clash_(album) https://en.wikipedia.org/wiki/Columbia_Records https://en.wikipedia.org/wiki/Australian_Record_Company https://en.wikipedia.org/wiki/Coronet_Records Multiple constraints ['https://en.wikipedia.org/wiki/The_Clash', 'https://en.wikipedia.org/wiki/The_Clash_(album)', 'https://en.wikipedia.org/wiki/Columbia_Records', 'https://en.wikipedia.org/wiki/Australian_Record_Company', 'https://en.wikipedia.org/wiki/Coronet_Records'] 182 194 "Concerning the 2007 book by Sofi Oksanen, the novel was described as ""not shrink from depicting rape, torture or murder."" In what year was the publication that quoted this founded?" 1872 https://en.wikipedia.org/wiki/Sofi_Oksanen https://en.wikipedia.org/wiki/Winnipeg_Free_Press https://en.wikipedia.org/wiki/Purge_(novel) Multiple constraints ['https://en.wikipedia.org/wiki/Sofi_Oksanen', 'https://en.wikipedia.org/wiki/Winnipeg_Free_Press', 'https://en.wikipedia.org/wiki/Purge_(novel)'] 183 195 How many months, rounded to the nearest whole number, did it take to construct the tallest building in the world as of January 1, 2024? 69 https://en.wikipedia.org/wiki/List_of_tallest_buildings_and_structures https://en.wikipedia.org/wiki/Burj_Khalifa Numerical reasoning | Tabular reasoning | Multiple constraints ['https://en.wikipedia.org/wiki/List_of_tallest_buildings_and_structures', 'https://en.wikipedia.org/wiki/Burj_Khalifa'] 184 196 Which player that scored more than 20 goals in the 2020-2021 Bundesliga season went on to play for a Red Bull owned club, as of August 1, 2024? Andre Silva went on to play for RB Leipzig. https://en.wikipedia.org/wiki/2020–21_Bundesliga https://en.wikipedia.org/wiki/Erling_Haaland https://en.wikipedia.org/wiki/André_Silva_(footballer,_born_1995) https://en.wikipedia.org/wiki/Robert_Lewandowski Tabular reasoning | Multiple constraints | Temporal reasoning ['https://en.wikipedia.org/wiki/2020–21_Bundesliga', 'https://en.wikipedia.org/wiki/Erling_Haaland', 'https://en.wikipedia.org/wiki/André_Silva_(footballer,_born_1995)', 'https://en.wikipedia.org/wiki/Robert_Lewandowski'] 185 197 What actor who starred in the People's Choice Award for Favorite Comedic Movie 1993 later became a U.S. President? The actor who starred in the People's Choice Award for Favorite Comedic Movie in 1993 who later became the 45th President of the United States, Donald Trump. https://en.wikipedia.org/wiki/19th_People%27s_Choice_Awards#Awards https://en.wikipedia.org/wiki/Home_Alone_2:_Lost_in_New_York#Cast https://en.wikipedia.org/wiki/List_of_presidents_of_the_United_States#Presidents Multiple constraints ['https://en.wikipedia.org/wiki/19th_People%27s_Choice_Awards#Awards', 'https://en.wikipedia.org/wiki/Home_Alone_2:_Lost_in_New_York#Cast', 'https://en.wikipedia.org/wiki/List_of_presidents_of_the_United_States#Presidents'] 186 198 If you divide the number of Papuan tribes in the Sarmi and Keerom Regencies of Papua province in Indonesia as of 2024 by the number of indigenous tribes in Brazil whose names, as they are typically written, begin with letters W, X, Y, or Z as of 2024, what is the answer to the fifth decimal place, rounding up? 1.82143 https://en.wikipedia.org/wiki/Indigenous_people_of_New_Guinea https://en.wikipedia.org/wiki/List_of_indigenous_peoples_of_Brazil Numerical reasoning | Tabular reasoning | Multiple constraints ['https://en.wikipedia.org/wiki/Indigenous_people_of_New_Guinea', 'https://en.wikipedia.org/wiki/List_of_indigenous_peoples_of_Brazil'] 187 199 What member of the 1992 Unified Olympic women's gymnastics team scored a 9.975 in the qualifier for floor and competed under three different flags in her Olympic career? Svetlana Boginskaya https://en.wikipedia.org/wiki/Svetlana_Boginskaya https://en.wikipedia.org/wiki/Gymnastics_at_the_1992_Summer_Olympics_%E2%80%93_Women%27s_artistic_team_all-around Multiple constraints ['https://en.wikipedia.org/wiki/Svetlana_Boginskaya', 'https://en.wikipedia.org/wiki/Gymnastics_at_the_1992_Summer_Olympics_%E2%80%93_Women%27s_artistic_team_all-around'] 188 201 In the country where Haribomo is located, what is the largest ethnic group as of August 3, 2024? Harimbo is located in Mali where the Bambara are the largest ethnic group. https://en.wikipedia.org/wiki/Haribomo https://en.wikipedia.org/wiki/Mali#Ethnic_groups Multiple constraints ['https://en.wikipedia.org/wiki/Haribomo', 'https://en.wikipedia.org/wiki/Mali#Ethnic_groups'] 189 202 What was the first elected position of the official that was in the office before Steven C. Johnson became the 42nd Kansas State Treasurer? Wichita School Board member https://en.wikipedia.org/wiki/Kansas_State_Treasurer https://en.wikipedia.org/wiki/Lynn_Rogers_(politician)#Career Multiple constraints | Temporal reasoning ['https://en.wikipedia.org/wiki/Kansas_State_Treasurer', 'https://en.wikipedia.org/wiki/Lynn_Rogers_(politician)#Career'] 190 203 As of 1st june 2024 Which Jonas brother has a wife who narrated a nature documentary released under the Disneynature label? Nick https://en.wikipedia.org/wiki/Jonas_Brothers#Members https://en.wikipedia.org/wiki/Nick_Jonas#Personal_life https://en.wikipedia.org/wiki/Joe_Jonas#Personal_life https://en.wikipedia.org/wiki/Kevin_Jonas#Personal_life https://en.wikipedia.org/wiki/Disneynature#Filmography https://en.wikipedia.org/wiki/Priyanka_Chopra Multiple constraints | Post processing ['https://en.wikipedia.org/wiki/Jonas_Brothers#Members', 'https://en.wikipedia.org/wiki/Nick_Jonas#Personal_life', 'https://en.wikipedia.org/wiki/Joe_Jonas#Personal_life', 'https://en.wikipedia.org/wiki/Kevin_Jonas#Personal_life', 'https://en.wikipedia.org/wiki/Disneynature#Filmography', 'https://en.wikipedia.org/wiki/Priyanka_Chopra'] 191 204 I'm thinking of a famous house, can you tell me which one from these clues? * The author of a philosophical work whose frontispiece was designed by Abraham Bosse spent time here. * The son of Francis Talbot and Mary Dacre used this house as a royal jail. Chatsworth House https://en.wikipedia.org/wiki/Chatsworth_House https://en.wikipedia.org/wiki/Thomas_Hobbes https://en.wikipedia.org/wiki/Leviathan_(Hobbes_book) https://en.wikipedia.org/wiki/Abraham_Bosse https://en.wikipedia.org/wiki/George_Talbot,_6th_Earl_of_Shrewsbury Multiple constraints ['https://en.wikipedia.org/wiki/Chatsworth_House', 'https://en.wikipedia.org/wiki/Thomas_Hobbes', 'https://en.wikipedia.org/wiki/Leviathan_(Hobbes_book)', 'https://en.wikipedia.org/wiki/Abraham_Bosse', 'https://en.wikipedia.org/wiki/George_Talbot,_6th_Earl_of_Shrewsbury'] 192 205 Out of all of the sovereign states with U.N. membership as of January 1, 2024, that recognize Abkhazia as a sovereign state, how many of them have active volcanoes? 3, Russia, Nicaragua, and Syria. https://en.wikipedia.org/wiki/List_of_sovereign_states https://en.wikipedia.org/wiki/Lists_of_volcanoes Numerical reasoning | Tabular reasoning | Multiple constraints ['https://en.wikipedia.org/wiki/List_of_sovereign_states', 'https://en.wikipedia.org/wiki/Lists_of_volcanoes'] 193 206 Do the timelines in the stories of Nier: Automata and Nier Replicant intersect? No, they do not intersect. https://en.wikipedia.org/wiki/Nier https://en.wikipedia.org/wiki/Nier:_Automata Numerical reasoning | Temporal reasoning ['https://en.wikipedia.org/wiki/Nier', 'https://en.wikipedia.org/wiki/Nier:_Automata'] 194 207 As of August 3, 2024, what is the capital of the 7th largest country in Asia? The capital of the 7th largest country in Asia is Ulaanbaatar https://en.wikipedia.org/wiki/List_of_Asian_countries_by_area https://en.wikipedia.org/wiki/Mongolia Tabular reasoning | Multiple constraints ['https://en.wikipedia.org/wiki/List_of_Asian_countries_by_area', 'https://en.wikipedia.org/wiki/Mongolia'] 195 208 As of August 3, 2024, what is the biggest religion in the country who has the best democracy in 2023, according to the democracy index? The Evangelical Lutheran Church of Norway https://en.wikipedia.org/wiki/The_Economist_Democracy_Index https://en.wikipedia.org/wiki/Norway Tabular reasoning ['https://en.wikipedia.org/wiki/The_Economist_Democracy_Index', 'https://en.wikipedia.org/wiki/Norway'] 196 209 Ben Darwin, former Australian rugby union footballer, graduated from the Australian Institute of Sport (AIS). How many years after Darwin's birth was the headquarters for the AIS opened? Ignore the month either event occurred. 5 https://en.wikipedia.org/wiki/Ben_Darwin https://en.wikipedia.org/wiki/Australian_Institute_of_Sport Numerical reasoning ['https://en.wikipedia.org/wiki/Ben_Darwin', 'https://en.wikipedia.org/wiki/Australian_Institute_of_Sport'] 197 210 What city does the band whose song spent the most weeks at No. 1 on the Billboard Hot Rock & Alternative Songs chart as of August 1, 2024 originate from? Las Vegas, Nevada https://en.wikipedia.org/wiki/Hot_Rock_%26_Alternative_Songs https://en.wikipedia.org/wiki/High_Hopes_(Panic!_at_the_Disco_song) https://en.wikipedia.org/wiki/Panic!_at_the_Disco Multiple constraints ['https://en.wikipedia.org/wiki/Hot_Rock_%26_Alternative_Songs', 'https://en.wikipedia.org/wiki/High_Hopes_(Panic!_at_the_Disco_song)', 'https://en.wikipedia.org/wiki/Panic!_at_the_Disco'] 198 211 Why didn't Harvard have calculus classes when it first opened? Calculus was not invented yet. https://en.wikipedia.org/wiki/Harvard_University https://en.wikipedia.org/wiki/Calculus Multiple constraints ['https://en.wikipedia.org/wiki/Harvard_University', 'https://en.wikipedia.org/wiki/Calculus'] 199 212 Which Lord of Montpellier had a child named Tortoseta? William VIII of Montpellier https://en.wikipedia.org/wiki/Lords_of_Montpellier https://en.wikipedia.org/wiki/William_I_of_Montpellier https://en.wikipedia.org/wiki/William_II_of_Montpellier https://en.wikipedia.org/wiki/William_III_of_Montpellier https://en.wikipedia.org/wiki/William_V_of_Montpellier https://en.wikipedia.org/wiki/William_IV_of_Montpellier https://en.wikipedia.org/wiki/William_VI_of_Montpellier https://en.wikipedia.org/wiki/William_VII_of_Montpellier https://en.wikipedia.org/wiki/William_VIII_of_Montpellier https://en.wikipedia.org/wiki/William_IX_of_Montpellier https://en.wikipedia.org/wiki/Marie_of_Montpellier, https://en.wikipedia.org/wiki/James_I_of_Aragon, https://en.wikipedia.org/wiki/James_II_of_Majorca, https://en.wikipedia.org/wiki/Sancho_of_Majorca, https://en.wikipedia.org/wiki/James_III_of_Majorca Multiple constraints ['https://en.wikipedia.org/wiki/Lords_of_Montpellier', 'https://en.wikipedia.org/wiki/William_I_of_Montpellier', 'https://en.wikipedia.org/wiki/William_II_of_Montpellier', 'https://en.wikipedia.org/wiki/William_III_of_Montpellier', 'https://en.wikipedia.org/wiki/William_V_of_Montpellier', 'https://en.wikipedia.org/wiki/William_IV_of_Montpellier', 'https://en.wikipedia.org/wiki/William_VI_of_Montpellier', 'https://en.wikipedia.org/wiki/William_VII_of_Montpellier', 'https://en.wikipedia.org/wiki/William_VIII_of_Montpellier', 'https://en.wikipedia.org/wiki/William_IX_of_Montpellier', 'https://en.wikipedia.org/wiki/Marie_of_Montpellier, https://en.wikipedia.org/wiki/James_I_of_Aragon, https://en.wikipedia.org/wiki/James_II_of_Majorca, https://en.wikipedia.org/wiki/Sancho_of_Majorca, https://en.wikipedia.org/wiki/James_III_of_Majorca'] 200 213 As of August 3rd 2024, how many Emmy Award nominations does the main cast member that was introduced in Season 2 of It's Always Sunny in Philadelphia have? 5 https://en.wikipedia.org/wiki/It%27s_Always_Sunny_in_Philadelphia https://en.wikipedia.org/wiki/Danny_DeVito#Acting_credits_and_accolades https://en.wikipedia.org/wiki/List_of_awards_and_nominations_received_by_Danny_DeVito Multiple constraints ['https://en.wikipedia.org/wiki/It%27s_Always_Sunny_in_Philadelphia', 'https://en.wikipedia.org/wiki/Danny_DeVito#Acting_credits_and_accolades', 'https://en.wikipedia.org/wiki/List_of_awards_and_nominations_received_by_Danny_DeVito'] 201 214 What is the atomic number in roman numerals for the element that has the same symbol as the first two letters of the 23rd most populous city (as of 2024) of the country represented between Italy and China at the World Showcase in Epcot? LXXIX https://en.wikipedia.org/wiki/Epcot https://en.wikipedia.org/wiki/List_of_cities_in_Germany_by_population https://en.wikipedia.org/wiki/Gold Tabular reasoning | Multiple constraints | Post processing ['https://en.wikipedia.org/wiki/Epcot', 'https://en.wikipedia.org/wiki/List_of_cities_in_Germany_by_population', 'https://en.wikipedia.org/wiki/Gold'] 202 215 The National Peace Corps Association was founded in 1979 by a United States politician. This founder then appointed the very first director of the Peace Corps, his brother-in-law. What is the first name of this director? Robert https://en.wikipedia.org/wiki/National_Peace_Corps_Association https://en.wikipedia.org/wiki/John_F._Kennedy https://en.wikipedia.org/wiki/Sargent_Shriver Multiple constraints ['https://en.wikipedia.org/wiki/National_Peace_Corps_Association', 'https://en.wikipedia.org/wiki/John_F._Kennedy', 'https://en.wikipedia.org/wiki/Sargent_Shriver'] 203 216 As of August 3, 2024, what is the hometown of the captain of the team that won the Stanley Cup three years before 2017? Manhattan Beach, CA https://en.m.wikipedia.org/wiki/2014_Stanley_Cup_Finals https://en.m.wikipedia.org/wiki/Dustin_Brown_(ice_hockey) Numerical reasoning | Post processing ['https://en.m.wikipedia.org/wiki/2014_Stanley_Cup_Finals', 'https://en.m.wikipedia.org/wiki/Dustin_Brown_(ice_hockey)'] 204 217 "Tell me the names of the two famous people I'm thinking of by using the following clues: They both have the initials M.T. Both were known by nicknames that included the word ""Iron"" One became the world champion in his sport while the other was her country's political leader" Mike Tyson and Margaret Thatcher https://en.wikipedia.org/wiki/Mike_Tyson https://en.wikipedia.org/wiki/Margaret_Thatcher Multiple constraints | Temporal reasoning ['https://en.wikipedia.org/wiki/Mike_Tyson', 'https://en.wikipedia.org/wiki/Margaret_Thatcher'] 205 218 If we added the sum of all ages as of 2010 (assuming they were still alive) of the inventors of the cotton gin, vacuum pump, and commercial toilet paper (ignoring the month) and then subtracted the ages of the inventors of the safety pin and the sewing machine what number would we have? 622 https://en.wikipedia.org/wiki/Cotton_gin#:~:text=A%20cotton%20gin%E2%80%94meaning%20%22cotton,productivity%20than%20manual%20cotton%20separation. https://en.wikipedia.org/wiki/Pneumatics https://en.wikipedia.org/wiki/Joseph_Gayetty https://en.wikipedia.org/wiki/Walter_Hunt_(inventor) Numerical reasoning | Temporal reasoning ['https://en.wikipedia.org/wiki/Cotton_gin#:~:text=A%20cotton%20gin%E2%80%94meaning%20%22cotton,productivity%20than%20manual%20cotton%20separation.', 'https://en.wikipedia.org/wiki/Pneumatics', 'https://en.wikipedia.org/wiki/Joseph_Gayetty', 'https://en.wikipedia.org/wiki/Walter_Hunt_(inventor)'] 206 219 What are the combined ages of the Guildford 4 at the time of their trial, divided by the number of the Birmingham 6 who were originally from the capital of Northern Ireland? Round it and provide the answer in binary. 10001 https://en.wikipedia.org/wiki/Guildford_Four_and_Maguire_Seven https://en.wikipedia.org/wiki/Birmingham_Six https://en.wikipedia.org/wiki/Northern_Ireland Numerical reasoning | Post processing ['https://en.wikipedia.org/wiki/Guildford_Four_and_Maguire_Seven', 'https://en.wikipedia.org/wiki/Birmingham_Six', 'https://en.wikipedia.org/wiki/Northern_Ireland'] 207 220 Who was the King of Siam during the 6th deadliest single-day terrorist attack in U.S. history? King Prajadhipok https://en.wikipedia.org/wiki/Terrorism_in_the_United_States#Deadliest_attacks https://en.wikipedia.org/wiki/Supreme_Council_of_State_of_Siam Tabular reasoning | Temporal reasoning ['https://en.wikipedia.org/wiki/Terrorism_in_the_United_States#Deadliest_attacks', 'https://en.wikipedia.org/wiki/Supreme_Council_of_State_of_Siam'] 208 221 Roberto Álamo starred in a film with Inma Cuesta, I think it was released in 2021 but I can't remember the name. What was the movie called and who did he play? El páramo / The Wasteland. He played Salvador. https://en.wikipedia.org/wiki/Roberto_%C3%81lamo https://en.wikipedia.org/wiki/Inma_Cuesta https://en.wikipedia.org/wiki/The_Wasteland_(2021_film) Multiple constraints ['https://en.wikipedia.org/wiki/Roberto_%C3%81lamo ', 'https://en.wikipedia.org/wiki/Inma_Cuesta', 'https://en.wikipedia.org/wiki/The_Wasteland_(2021_film)'] 209 223 What was the age difference, in years, between the seventh former President of Murray State University and the comic book artist who worked on *Superman & Bugs Bunny* and *New Guardians* when the comic book artist graduated from college? 7 years https://en.wikipedia.org/wiki/Superman_%26_Bugs_Bunny https://en.wikipedia.org/wiki/New_Guardians https://en.wikipedia.org/wiki/Joe_Staton https://en.wikipedia.org/wiki/Constantine_W._Curris https://en.wikipedia.org/wiki/Murray_State_University#Former_Presidents_of_the_University Numerical reasoning | Temporal reasoning ['https://en.wikipedia.org/wiki/Superman_%26_Bugs_Bunny', 'https://en.wikipedia.org/wiki/New_Guardians', 'https://en.wikipedia.org/wiki/Joe_Staton', 'https://en.wikipedia.org/wiki/Constantine_W._Curris', 'https://en.wikipedia.org/wiki/Murray_State_University#Former_Presidents_of_the_University'] 210 224 Of the top 3 women's WTA singles ranking as of 29th July 2024, which has a father who was an ice hockey player? Aryna Sablenka https://en.wikipedia.org/wiki/Women%27s_Tennis_Association https://en.wikipedia.org/wiki/Iga_%C5%9Awi%C4%85tek https://en.wikipedia.org/wiki/Aryna_Sabalenka https://en.wikipedia.org/wiki/Coco_Gauff Multiple constraints ['https://en.wikipedia.org/wiki/Women%27s_Tennis_Association', 'https://en.wikipedia.org/wiki/Iga_%C5%9Awi%C4%85tek', 'https://en.wikipedia.org/wiki/Aryna_Sabalenka', 'https://en.wikipedia.org/wiki/Coco_Gauff'] 211 225 How many times could Usain Bolt span the length of a standard Olympic-sized swimming pool if Tom Daley was standing on his head? Please answer to a whole Usain or Tom without exceeding the length of the pool. 13 https://en.wikipedia.org/wiki/Olympic-size_swimming_pool https://en.wikipedia.org/wiki/Usain_Bolt https://en.wikipedia.org/wiki/Tom_Daley Numerical reasoning | Post processing ['https://en.wikipedia.org/wiki/Olympic-size_swimming_pool', 'https://en.wikipedia.org/wiki/Usain_Bolt', 'https://en.wikipedia.org/wiki/Tom_Daley'] 212 226 Who was Prime Minister when Will Young won Pop Idol? Tony Blair https://en.wikipedia.org/wiki/Pop_Idol https://en.wikipedia.org/wiki/Timeline_of_prime_ministers_of_Great_Britain_and_the_United_Kingdom Multiple constraints | Post processing | Temporal reasoning ['https://en.wikipedia.org/wiki/Pop_Idol', 'https://en.wikipedia.org/wiki/Timeline_of_prime_ministers_of_Great_Britain_and_the_United_Kingdom'] 213 227 "I'm thinking of a diving duck, the largest found in North America, that breeds in the Prairie Pothole Region. This duck, smothered in blackcurrant sauce, was once described by Edith Wharton as an ""especially luxurious dinner"" which was served in New York City in the 1870s. Which duck am I thinking of?" The canvasback. https://en.wikipedia.org/wiki/Diving_duck https://en.wikipedia.org/wiki/Prairie_Pothole_Region https://en.wikipedia.org/wiki/Canvasback Multiple constraints ['https://en.wikipedia.org/wiki/Diving_duck', 'https://en.wikipedia.org/wiki/Prairie_Pothole_Region', 'https://en.wikipedia.org/wiki/Canvasback'] 214 228 I'm thinking of an Actor that won an Oscar for Best Supporting Actor two years after being nominated for the same award, but not winning. The same year that the actor won the Oscar, the Actor also won a BAFTA in the same category and for the same film. The Actor also won a Tony award in 1974. Christopher Plummer https://en.wikipedia.org/wiki/28th_Tony_Awards https://en.wikipedia.org/wiki/Academy_Award_for_Best_Supporting_Actor https://en.wikipedia.org/wiki/BAFTA_Award_for_Best_Actor_in_a_Supporting_Role https://en.wikipedia.org/wiki/List_of_awards_and_nominations_received_by_Christopher_Plummer Tabular reasoning | Multiple constraints | Post processing | Temporal reasoning ['https://en.wikipedia.org/wiki/28th_Tony_Awards', 'https://en.wikipedia.org/wiki/Academy_Award_for_Best_Supporting_Actor', 'https://en.wikipedia.org/wiki/BAFTA_Award_for_Best_Actor_in_a_Supporting_Role', 'https://en.wikipedia.org/wiki/List_of_awards_and_nominations_received_by_Christopher_Plummer'] 215 229 "In the George Eliot novel described by Virginia Woolf as ""one of the few English novels written for grown-up people"", one of the main characters shares a first name with a former prime minister of Burkino Faso who went on the join the board of a Canadian mining company. What forename do they share?" Tertius https://en.wikipedia.org/wiki/George_Eliot https://en.wikipedia.org/wiki/Middlemarch https://en.wikipedia.org/wiki/Tertius_Zongo Multiple constraints ['https://en.wikipedia.org/wiki/George_Eliot', 'https://en.wikipedia.org/wiki/Middlemarch', 'https://en.wikipedia.org/wiki/Tertius_Zongo'] 216 230 What is the population of the town (as of December 2020) that holds the Mountain Cheese Olympics? 5793 https://en.wikipedia.org/wiki/Mountain_Cheese_Olympics https://en.wikipedia.org/wiki/Appenzell_District Numerical reasoning | Multiple constraints ['https://en.wikipedia.org/wiki/Mountain_Cheese_Olympics', 'https://en.wikipedia.org/wiki/Appenzell_District'] 217 231 How old would James Logan have been when the estate of his descendent donated Neshaminy State Park land to the Commonwealth of Pennsylvania? 282 years https://en.wikipedia.org/wiki/Neshaminy_State_Park https://en.wikipedia.org/wiki/James_Logan_(statesman) Numerical reasoning ['https://en.wikipedia.org/wiki/Neshaminy_State_Park', 'https://en.wikipedia.org/wiki/James_Logan_(statesman)'] 218 232 Is the time between Oasis's first album and 2024 shorter or longer than between Oasis's first album and The Beatles' last album? Longer. https://en.wikipedia.org/wiki/Oasis_(band) https://en.wikipedia.org/wiki/The_Beatles Numerical reasoning ['https://en.wikipedia.org/wiki/Oasis_(band)', 'https://en.wikipedia.org/wiki/The_Beatles'] 219 233 Ysaires Restituyo was a volleyball player known for playing in the NORCECA Beach Volleyball Circuit. How much older was her partner in 2007 than her partner in 2009? 2 years https://en.wikipedia.org/wiki/Ysaires_Restituyo https://en.wikipedia.org/wiki/Cinthia_Pi%C3%B1eiro# https://en.wikipedia.org/wiki/Ana_Ligia_Fabian Numerical reasoning ['https://en.wikipedia.org/wiki/Ysaires_Restituyo', 'https://en.wikipedia.org/wiki/Cinthia_Pi%C3%B1eiro#', 'https://en.wikipedia.org/wiki/Ana_Ligia_Fabian'] 220 234 "Demi Moore shares a birthday with which costar from the movies ""Margin Call"" and ""Deconstructing Harry""?" Stanley Tucci https://en.wikipedia.org/wiki/Deconstructing_Harry https://en.wikipedia.org/wiki/Margin_Call https://en.wikipedia.org/wiki/Stanley_Tucci https://en.wikipedia.org/wiki/Demi_Moore Multiple constraints ['https://en.wikipedia.org/wiki/Deconstructing_Harry', 'https://en.wikipedia.org/wiki/Margin_Call', 'https://en.wikipedia.org/wiki/Stanley_Tucci', 'https://en.wikipedia.org/wiki/Demi_Moore'] 221 236 If the Great North Run took place on York's Medieval Walls how many circuits of them would the athletes run? Round to the nearest tenth of a circuit. 6.2 circuits. https://en.wikipedia.org/wiki/York_city_walls https://en.wikipedia.org/wiki/Great_North_Run https://en.wikipedia.org/wiki/Half_marathon Numerical reasoning ['https://en.wikipedia.org/wiki/York_city_walls', 'https://en.wikipedia.org/wiki/Great_North_Run', 'https://en.wikipedia.org/wiki/Half_marathon'] 222 238 What was the 2021 population of the birthplace of the leader of the party that won the 1869 Newfoundland general election? 9,162 was the population of Shaftesbury (the birthplace of Charles Fox Bennett) in the 2021 census. https://en.wikipedia.org/wiki/1869_Newfoundland_general_election https://en.wikipedia.org/wiki/Charles_Fox_Bennett https://en.wikipedia.org/wiki/Shaftesbury Tabular reasoning | Multiple constraints ['https://en.wikipedia.org/wiki/1869_Newfoundland_general_election', 'https://en.wikipedia.org/wiki/Charles_Fox_Bennett', 'https://en.wikipedia.org/wiki/Shaftesbury'] 223 239 After Meat Loaf legally changed his name due to a commercial, what was the first new brand launched by the company to whom the commercial belonged? Dockers https://en.wikipedia.org/wiki/Meat_Loaf#Personal_life https://en.wikipedia.org/wiki/Levi_Strauss_%26_Co.#Blue_jeans_era_(1960s%E2%80%931980s) Temporal reasoning ['https://en.wikipedia.org/wiki/Meat_Loaf#Personal_life', 'https://en.wikipedia.org/wiki/Levi_Strauss_%26_Co.#Blue_jeans_era_(1960s%E2%80%931980s)'] 224 240 I'm a concert venue in Washington, D.C. Blink-182 played here on their tour the same year Brazil won their fifth FIFA World Cup. What was my name in 2010? Verizon Center https://en.wikipedia.org/wiki/Brazil_national_football_team#FIFA_World_Cup https://en.wikipedia.org/wiki/Blink-182#Tours https://en.wikipedia.org/wiki/Pop_Disaster_Tour https://en.wikipedia.org/wiki/Capital_One_Arena Tabular reasoning | Temporal reasoning ['https://en.wikipedia.org/wiki/Brazil_national_football_team#FIFA_World_Cup', 'https://en.wikipedia.org/wiki/Blink-182#Tours', 'https://en.wikipedia.org/wiki/Pop_Disaster_Tour', 'https://en.wikipedia.org/wiki/Capital_One_Arena'] 225 241 Which prime minister has been in office between 2017 and 2022 and also shares their middle name with a former monarch of Scotland. Theresa (Mary) May https://en.wikipedia.org/wiki/List_of_prime_ministers_of_the_United_Kingdom https://en.wikipedia.org/wiki/Theresa_May https://en.wikipedia.org/wiki/Boris_Johnson https://en.wikipedia.org/wiki/Liz_Truss https://en.wikipedia.org/wiki/Rishi_Sunak https://en.wikipedia.org/wiki/List_of_Scottish_monarchs Multiple constraints | Temporal reasoning ['https://en.wikipedia.org/wiki/List_of_prime_ministers_of_the_United_Kingdom', 'https://en.wikipedia.org/wiki/Theresa_May', 'https://en.wikipedia.org/wiki/Boris_Johnson', 'https://en.wikipedia.org/wiki/Liz_Truss', 'https://en.wikipedia.org/wiki/Rishi_Sunak', 'https://en.wikipedia.org/wiki/List_of_Scottish_monarchs'] 226 242 Who were the first two women who won the Nobel Prize, in any category, who were also mothers? Marie Curie and Grazia Deledda https://en.wikipedia.org/wiki/Marie_Curie en.wikipedia.org/wiki/Grazia_Deledda en.wikipedia.org/wiki/List_of_female_Nobel_laureates https://en.wikipedia.org/wiki/Irène_Joliot-Curie Numerical reasoning | Multiple constraints ['https://en.wikipedia.org/wiki/Marie_Curie', 'en.wikipedia.org/wiki/Grazia_Deledda', 'en.wikipedia.org/wiki/List_of_female_Nobel_laureates', 'https://en.wikipedia.org/wiki/Irène_Joliot-Curie'] 227 243 Of the non-Americans who have won the Phoenix Open as of 2024, who was the youngest at the time of his win? Hideki Matsuyama https://en.wikipedia.org/wiki/Phoenix_Open https://en.wikipedia.org/wiki/Nick_Taylor_(golfer) https://en.wikipedia.org/wiki/Hideki_Matsuyama https://en.wikipedia.org/wiki/Aaron_Baddeley https://en.wikipedia.org/wiki/Vijay_Singh https://en.wikipedia.org/wiki/Jesper_Parnevik https://en.wikipedia.org/wiki/Sandy_Lyle https://en.wikipedia.org/wiki/Bruce_Crampton https://en.wikipedia.org/wiki/George_Knudson https://en.wikipedia.org/wiki/Bobby_Locke Numerical reasoning | Temporal reasoning ['https://en.wikipedia.org/wiki/Phoenix_Open', 'https://en.wikipedia.org/wiki/Nick_Taylor_(golfer)', 'https://en.wikipedia.org/wiki/Hideki_Matsuyama', 'https://en.wikipedia.org/wiki/Aaron_Baddeley', 'https://en.wikipedia.org/wiki/Vijay_Singh', 'https://en.wikipedia.org/wiki/Jesper_Parnevik', 'https://en.wikipedia.org/wiki/Sandy_Lyle', 'https://en.wikipedia.org/wiki/Bruce_Crampton', 'https://en.wikipedia.org/wiki/George_Knudson', 'https://en.wikipedia.org/wiki/Bobby_Locke'] 228 244 The US Naval ship that sunk in Havana Harbor on February 15, 1898, is named for a state that was admitted to the Union while what woman was serving as First Lady? Elizabeth Monroe https://en.wikipedia.org/wiki/USS_Maine_(1889) https://en.wikipedia.org/wiki/Maine https://en.wikipedia.org/wiki/List_of_first_ladies_of_the_United_States Multiple constraints | Temporal reasoning ['https://en.wikipedia.org/wiki/USS_Maine_(1889)', 'https://en.wikipedia.org/wiki/Maine', 'https://en.wikipedia.org/wiki/List_of_first_ladies_of_the_United_States'] 229 245 As of the 2023 Major League Baseball season, who is the only player in the top 15 lists of career home runs, career runs batted in, and career hits, who was also named (in 2007) the all-time Gold Glove team? Willie Mays https://en.wikipedia.org/wiki/List_of_Major_League_Baseball_career_home_run_leaders https://en.wikipedia.org/wiki/List_of_Major_League_Baseball_career_runs_batted_in_leaders https://en.wikipedia.org/wiki/List_of_Major_League_Baseball_career_hits_leaders https://en.wikipedia.org/wiki/Gold_Glove_Award Tabular reasoning | Multiple constraints ['https://en.wikipedia.org/wiki/List_of_Major_League_Baseball_career_home_run_leaders', 'https://en.wikipedia.org/wiki/List_of_Major_League_Baseball_career_runs_batted_in_leaders', 'https://en.wikipedia.org/wiki/List_of_Major_League_Baseball_career_hits_leaders', 'https://en.wikipedia.org/wiki/Gold_Glove_Award'] 230 246 Who had the best career batting average out of every player to hit a home run in the 2002 World Series matchup between the Anaheim Angeles and San Francisco Giants? Barry Bonds with a .298 lifetime batting average. https://en.wikipedia.org/wiki/2002_World_Series#Composite_box https://en.wikipedia.org/wiki/Barry_Bonds https://en.wikipedia.org/wiki/Darin_Erstad https://en.wikipedia.org/wiki/David_Bell_(baseball) https://en.wikipedia.org/wiki/Jeff_Kent https://en.wikipedia.org/wiki/J._T._Snow https://en.wikipedia.org/wiki/Reggie_Sanders https://en.wikipedia.org/wiki/Rich_Aurilia https://en.wikipedia.org/wiki/Scott_Spiezio https://en.wikipedia.org/wiki/Shawon_Dunston https://en.wikipedia.org/wiki/Tim_Salmon, https://en.wikipedia.org/wiki/Troy_Glaus Tabular reasoning ['https://en.wikipedia.org/wiki/2002_World_Series#Composite_box', 'https://en.wikipedia.org/wiki/Barry_Bonds', 'https://en.wikipedia.org/wiki/Darin_Erstad', 'https://en.wikipedia.org/wiki/David_Bell_(baseball)', 'https://en.wikipedia.org/wiki/Jeff_Kent', 'https://en.wikipedia.org/wiki/J._T._Snow', 'https://en.wikipedia.org/wiki/Reggie_Sanders', 'https://en.wikipedia.org/wiki/Rich_Aurilia', 'https://en.wikipedia.org/wiki/Scott_Spiezio', 'https://en.wikipedia.org/wiki/Shawon_Dunston', 'https://en.wikipedia.org/wiki/Tim_Salmon, https://en.wikipedia.org/wiki/Troy_Glaus'] 231 247 Out of all of the feature-length theatrical films that John Carpenter directed before 2015, which has the longest running time? Starman (1984) https://en.wikipedia.org/wiki/John_Carpenter_filmography https://en.wikipedia.org/wiki/Dark_Star_(film) https://en.wikipedia.org/wiki/Assault_on_Precinct_13_(1976_film) https://en.wikipedia.org/wiki/Halloween_(1978_film) https://en.wikipedia.org/wiki/The_Fog https://en.wikipedia.org/wiki/Escape_from_New_York https://en.wikipedia.org/wiki/The_Thing_(1982_film) https://en.wikipedia.org/wiki/Christine_(1983_film) https://en.wikipedia.org/wiki/Starman_(film) https://en.wikipedia.org/wiki/Big_Trouble_in_Little_China https://en.wikipedia.org/wiki/Prince_of_Darkness_(film), https://en.wikipedia.org/wiki/They_Live, https://en.wikipedia.org/wiki/Memoirs_of_an_Invisible_Man_(film), https://en.wikipedia.org/wiki/In_the_Mouth_of_Madness, https://en.wikipedia.org/wiki/Village_of_the_Damned_(1995_film), https://en.wikipedia.org/wiki/Escape_from_L.A., https://en.wikipedia.org/wiki/Vampires_(1998_film), https://en.wikipedia.org/wiki/Ghosts_of_Mars, https://en.wikipedia.org/wiki/The_Ward_(film) Tabular reasoning ['https://en.wikipedia.org/wiki/John_Carpenter_filmography', 'https://en.wikipedia.org/wiki/Dark_Star_(film)', 'https://en.wikipedia.org/wiki/Assault_on_Precinct_13_(1976_film)', 'https://en.wikipedia.org/wiki/Halloween_(1978_film)', 'https://en.wikipedia.org/wiki/The_Fog', 'https://en.wikipedia.org/wiki/Escape_from_New_York', 'https://en.wikipedia.org/wiki/The_Thing_(1982_film)', 'https://en.wikipedia.org/wiki/Christine_(1983_film)', 'https://en.wikipedia.org/wiki/Starman_(film)', 'https://en.wikipedia.org/wiki/Big_Trouble_in_Little_China', 'https://en.wikipedia.org/wiki/Prince_of_Darkness_(film), https://en.wikipedia.org/wiki/They_Live, https://en.wikipedia.org/wiki/Memoirs_of_an_Invisible_Man_(film), https://en.wikipedia.org/wiki/In_the_Mouth_of_Madness, https://en.wikipedia.org/wiki/Village_of_the_Damned_(1995_film), https://en.wikipedia.org/wiki/Escape_from_L.A., https://en.wikipedia.org/wiki/Vampires_(1998_film), https://en.wikipedia.org/wiki/Ghosts_of_Mars, https://en.wikipedia.org/wiki/The_Ward_(film)'] 232 248 How much taller (in centimetres) is Mark O'Halloran (ex West Tigers rugby league player) than the London Broncos' player with the heritage number 341? 11cm https://en.wikipedia.org/wiki/Mark_O%27Halloran_(rugby_league) https://en.wikipedia.org/wiki/Robert_Fico https://en.wikipedia.org/wiki/London_Broncos#1994%E2%80%932005:_Broncos_and_Super_League https://en.wikipedia.org/wiki/Steele_Retchless Numerical reasoning | Multiple constraints | Post processing | Temporal reasoning ['https://en.wikipedia.org/wiki/Mark_O%27Halloran_(rugby_league)', 'https://en.wikipedia.org/wiki/Robert_Fico', 'https://en.wikipedia.org/wiki/London_Broncos#1994%E2%80%932005:_Broncos_and_Super_League', 'https://en.wikipedia.org/wiki/Steele_Retchless'] 233 249 This founder of the Academy of Science, St. Louis became established as a botonist for a monograph he did in 1842. What is the scientic name for plant featured in that monograph? Cuscuta https://en.wikipedia.org/wiki/Academy_of_Science,_St._Louis https://en.wikipedia.org/wiki/George_Engelmann https://en.wikipedia.org/wiki/Cuscuta Multiple constraints ['https://en.wikipedia.org/wiki/Academy_of_Science,_St._Louis', 'https://en.wikipedia.org/wiki/George_Engelmann', 'https://en.wikipedia.org/wiki/Cuscuta'] 234 250 "Which American actress, born the same year Denzel Washington won his first Best Actor in a Leading Role Academy Award, had a supporting role in the second season of the Netflix series ""You""?" Jenna Ortega https://en.wikipedia.org/wiki/List_of_awards_and_nominations_received_by_Denzel_Washington https://en.wikipedia.org/wiki/Jenna_Ortega https://en.wikipedia.org/wiki/Victoria_Pedretti https://en.wikipedia.org/wiki/Carmela_Zumbado https://en.wikipedia.org/wiki/Ambyr_Childers https://en.wikipedia.org/wiki/Saffron_Burrows https://en.wikipedia.org/wiki/Tati_Gabrielle https://en.wikipedia.org/wiki/You_(TV_series)#Season_2_(2019) Numerical reasoning | Multiple constraints | Post processing | Temporal reasoning ['https://en.wikipedia.org/wiki/List_of_awards_and_nominations_received_by_Denzel_Washington', 'https://en.wikipedia.org/wiki/Jenna_Ortega', 'https://en.wikipedia.org/wiki/Victoria_Pedretti', 'https://en.wikipedia.org/wiki/Carmela_Zumbado', 'https://en.wikipedia.org/wiki/Ambyr_Childers', 'https://en.wikipedia.org/wiki/Saffron_Burrows', 'https://en.wikipedia.org/wiki/Tati_Gabrielle', 'https://en.wikipedia.org/wiki/You_(TV_series)#Season_2_(2019)'] 235 251 What is the birthday of the man who produced the pop song named after one of the largest bird in the Procellariiformes species, off the 2024 album The Tortured Poets Department? Aaron Dessner was born April 23, 1976. https://en.wikipedia.org/wiki/Procellariiformes https://en.wikipedia.org/wiki/Aaron_Dessner https://en.wikipedia.org/wiki/The_Tortured_Poets_Department Tabular reasoning | Multiple constraints | Post processing ['https://en.wikipedia.org/wiki/Procellariiformes', 'https://en.wikipedia.org/wiki/Aaron_Dessner', 'https://en.wikipedia.org/wiki/The_Tortured_Poets_Department'] 236 252 What Jeep model shares its name with the Secret Service codename for a 21st-century US president? Renegade https://en.wikipedia.org/wiki/Secret_Service_code_name https://en.wikipedia.org/wiki/List_of_Jeep_vehicles Multiple constraints ['https://en.wikipedia.org/wiki/Secret_Service_code_name', 'https://en.wikipedia.org/wiki/List_of_Jeep_vehicles'] 237 253 Who was the Prime Minister of Canada in the year that the 1965 winner of the Best New Artist Grammy Award made their first appearance on US television? Lester Pearson https://en.wikipedia.org/wiki/Grammy_Award_for_Best_New_Artist#1960s https://en.wikipedia.org/wiki/The_Beatles https://en.wikipedia.org/wiki/List_of_prime_ministers_of_Canada#Prime_ministers Multiple constraints ['https://en.wikipedia.org/wiki/Grammy_Award_for_Best_New_Artist#1960s', 'https://en.wikipedia.org/wiki/The_Beatles', 'https://en.wikipedia.org/wiki/List_of_prime_ministers_of_Canada#Prime_ministers'] 238 254 Part of the dynamic duo who broke a record set by Mickey Lolich and Bill Freehan in 1975 for most starts together, this MLB pitcher's battery mate ranked first all-time among catchers in putouts as of 2022. In what year did he (the pitcher) make his Grand Ole Opry debut? 2024 https://en.wikipedia.org/wiki/St._Louis_Cardinals https://en.wikipedia.org/wiki/Yadier_Molina https://en.wikipedia.org/wiki/Adam_Wainwright Multiple constraints | Temporal reasoning ['https://en.wikipedia.org/wiki/St._Louis_Cardinals', 'https://en.wikipedia.org/wiki/Yadier_Molina', 'https://en.wikipedia.org/wiki/Adam_Wainwright'] 239 255 How much time passed between the release of the blockchain platform founded by the individual who resides in the town of the same name as the James Buchanan House and the release of the prior blockchain platform co-founded by the same individual? Wheatland, also known as the James Buchanan House, is the same name as the town in Wyoming where Charles Hoskinson resides. Charles Hoskinson cofounded Ethereum and founded Cardano. Cardano was released in 2017; Ethereum was released in 2015. Two years elapsed between the two platforms' releases. https://en.wikipedia.org/wiki/Wheatland_(James_Buchanan_House) https://en.wikipedia.org/wiki/Wheatland,_Wyoming https://en.wikipedia.org/wiki/Cardano_(blockchain_platform) https://en.wikipedia.org/wiki/Ethereum#Ether Numerical reasoning | Multiple constraints | Temporal reasoning ['https://en.wikipedia.org/wiki/Wheatland_(James_Buchanan_House)', 'https://en.wikipedia.org/wiki/Wheatland,_Wyoming', 'https://en.wikipedia.org/wiki/Cardano_(blockchain_platform)', 'https://en.wikipedia.org/wiki/Ethereum#Ether'] 240 256 Who was the character in the roster of the Marvel vs. Capcom entry that was ported to the PlayStation 4 in 2016 that represented a video game franchise that has zombies as main antagonists and is the player character of a game released in 1999? Jill Valentine https://en.wikipedia.org/wiki/Marvel_vs._Capcom https://en.wikipedia.org/wiki/Ultimate_Marvel_vs._Capcom_3 https://en.wikipedia.org/wiki/Jill_Valentine https://en.wikipedia.org/wiki/Nemesis_(Resident_Evil) https://en.wikipedia.org/wiki/Albert_Wesker https://en.wikipedia.org/wiki/Chris_Redfield https://en.wikipedia.org/wiki/Frank_West_(Dead_Rising) https://en.wikipedia.org/wiki/Resident_Evil_3:_Nemesis Multiple constraints ['https://en.wikipedia.org/wiki/Marvel_vs._Capcom', 'https://en.wikipedia.org/wiki/Ultimate_Marvel_vs._Capcom_3', 'https://en.wikipedia.org/wiki/Jill_Valentine', 'https://en.wikipedia.org/wiki/Nemesis_(Resident_Evil)', 'https://en.wikipedia.org/wiki/Albert_Wesker', 'https://en.wikipedia.org/wiki/Chris_Redfield', 'https://en.wikipedia.org/wiki/Frank_West_(Dead_Rising)', 'https://en.wikipedia.org/wiki/Resident_Evil_3:_Nemesis'] 241 257 The screenwriter of the film, which received nominations for Best Screenplay and Best Actor in a Motion Picture - Drama at the 1995 Golden Globes, attended which Michigan university? Grand Valley State University https://en.wikipedia.org/wiki/Golden_Globe_Award_for_Best_Actor_in_a_Motion_Picture_%E2%80%93_Drama https://en.wikipedia.org/wiki/Golden_Globe_Award_for_Best_Screenplay https://en.wikipedia.org/wiki/Tim_Robbins https://en.wikipedia.org/wiki/Patrick_Sheane_Duncan Tabular reasoning | Multiple constraints ['https://en.wikipedia.org/wiki/Golden_Globe_Award_for_Best_Actor_in_a_Motion_Picture_%E2%80%93_Drama', 'https://en.wikipedia.org/wiki/Golden_Globe_Award_for_Best_Screenplay', 'https://en.wikipedia.org/wiki/Tim_Robbins', 'https://en.wikipedia.org/wiki/Patrick_Sheane_Duncan'] 242 258 NASA launched an Apollo mission a year after the Stonewall Riots. How many collective children did the astronauts onboard that mission have? 8 https://en.wikipedia.org/wiki/Stonewall_riots https://en.wikipedia.org/wiki/List_of_Apollo_missions https://en.wikipedia.org/wiki/Jim_Lovell https://en.wikipedia.org/wiki/Jack_Swigert https://en.wikipedia.org/wiki/Fred_Haise Numerical reasoning | Tabular reasoning | Multiple constraints | Post processing | Temporal reasoning ['https://en.wikipedia.org/wiki/Stonewall_riots', 'https://en.wikipedia.org/wiki/List_of_Apollo_missions', 'https://en.wikipedia.org/wiki/Jim_Lovell', 'https://en.wikipedia.org/wiki/Jack_Swigert', 'https://en.wikipedia.org/wiki/Fred_Haise'] 243 259 A United States women's national soccer team player scored her first career international goal during the semi-final match of the 2015 FIFA Women's World Cup. This same player scored her second goal the next year. Tell me the difference in attendance between these two games. 43,518 https://en.wikipedia.org/wiki/2015_FIFA_Women%27s_World_Cup_knockout_stage https://en.wikipedia.org/wiki/Kelley_O%27Hara https://en.wikipedia.org/wiki/Carli_Lloyd https://en.wikipedia.org/wiki/2016_CONCACAF_Women%27s_Olympic_Qualifying_Championship#Group_A Numerical reasoning | Tabular reasoning | Multiple constraints | Post processing | Temporal reasoning ['https://en.wikipedia.org/wiki/2015_FIFA_Women%27s_World_Cup_knockout_stage', 'https://en.wikipedia.org/wiki/Kelley_O%27Hara', 'https://en.wikipedia.org/wiki/Carli_Lloyd', 'https://en.wikipedia.org/wiki/2016_CONCACAF_Women%27s_Olympic_Qualifying_Championship#Group_A'] 244 260 Who is the Formula One driver who won their first Driver's Championship in the 46th season, what team did they race for that year, and how many years after that team's first race was it? Nigel Mansell, Williams Grand Prix Engineering, 15 years https://en.wikipedia.org/wiki/History_of_Formula_One https://en.wikipedia.org/wiki/1992_Formula_One_World_Championship https://en.m.wikipedia.org/wiki/Williams_Grand_Prix_Engineering Numerical reasoning | Multiple constraints | Temporal reasoning ['https://en.wikipedia.org/wiki/History_of_Formula_One', 'https://en.wikipedia.org/wiki/1992_Formula_One_World_Championship', 'https://en.m.wikipedia.org/wiki/Williams_Grand_Prix_Engineering'] 245 261 There is only one existing lighthouse with attached living quarters in the ninth-largest US state by area, as of August 1, 2024. This lighthouse is located on the north side of a bay named for which tribe? The Yaquina Tribe https://en.wikipedia.org/wiki/Yaquina_Bay_Light https://en.wikipedia.org/wiki/Oregon https://en.wikipedia.org/wiki/Yaquina_Bay Multiple constraints ['https://en.wikipedia.org/wiki/Yaquina_Bay_Light', 'https://en.wikipedia.org/wiki/Oregon', 'https://en.wikipedia.org/wiki/Yaquina_Bay'] 246 262 Consider the following three people: 1. Edmund, who turned 10 on the day of the Battle of Hastings 2. Edward, who turned 12 on the day that Guy Fawkes was executed 3. Eddie, who turned 14 on the day of the London 2012 Summer Olympics opening ceremony Who would be oldest: Edmund on the day King Henry I of England died, Edward on the day of the Battle of Naseby, or Eddie on the day Liz Truss announced her resignation as Conservative Party leader? Edmund https://en.wikipedia.org/wiki/Battle_of_Hastings https://en.wikipedia.org/wiki/Guy_Fawkes https://en.wikipedia.org/wiki/2012_Summer_Olympics_opening_ceremony https://en.wikipedia.org/wiki/Henry_I_of_England https://en.wikipedia.org/wiki/Battle_of_Naseby https://en.wikipedia.org/wiki/Liz_Truss Numerical reasoning | Temporal reasoning ['https://en.wikipedia.org/wiki/Battle_of_Hastings', 'https://en.wikipedia.org/wiki/Guy_Fawkes', 'https://en.wikipedia.org/wiki/2012_Summer_Olympics_opening_ceremony', 'https://en.wikipedia.org/wiki/Henry_I_of_England', 'https://en.wikipedia.org/wiki/Battle_of_Naseby', 'https://en.wikipedia.org/wiki/Liz_Truss'] 247 263 What was the final league position of the football team found in the city where José Loiola won his first World Championship gold medal for the season which began in that same year when he won gold? 15th https://en.wikipedia.org/wiki/Jos%C3%A9_Loiola https://en.wikipedia.org/wiki/1999%E2%80%932000_Olympique_de_Marseille_season Temporal reasoning ['https://en.wikipedia.org/wiki/Jos%C3%A9_Loiola', 'https://en.wikipedia.org/wiki/1999%E2%80%932000_Olympique_de_Marseille_season'] 248 264 Which is bigger based on their maximum average lengths multiplied by their number of legs: an elephant beetle, a brown rhinoceros beetle, or a bee hummingbird? Elephant beetle https://en.wikipedia.org/wiki/Elephant_beetle https://en.wikipedia.org/wiki/Xylotrupes_gideon https://en.wikipedia.org/wiki/Bee_hummingbird https://en.wikipedia.org/wiki/Bird#Anatomy_and_physiology https://en.wikipedia.org/wiki/Insect Numerical reasoning | Multiple constraints ['https://en.wikipedia.org/wiki/Elephant_beetle', 'https://en.wikipedia.org/wiki/Xylotrupes_gideon', 'https://en.wikipedia.org/wiki/Bee_hummingbird', 'https://en.wikipedia.org/wiki/Bird#Anatomy_and_physiology', 'https://en.wikipedia.org/wiki/Insect'] 249 265 "What is the name of the Japanese man who protested the U.S.'s involvement in the Iraq War, and who has also been awarded multiple times the same award that ""Shrek"" won in 2002, beating ""Jimmy Neutron: Boy Genius""?" Hayao Miyazaki https://en.wikipedia.org/wiki/Shrek#Accolades https://en.wikipedia.org/wiki/74th_Academy_Awards#Winners_and_nominees https://en.wikipedia.org/wiki/Academy_Award_for_Best_Animated_Feature#Multiple_wins_and_nominations https://en.wikipedia.org/wiki/Hayao_Miyazaki#Views Multiple constraints ['https://en.wikipedia.org/wiki/Shrek#Accolades', 'https://en.wikipedia.org/wiki/74th_Academy_Awards#Winners_and_nominees', 'https://en.wikipedia.org/wiki/Academy_Award_for_Best_Animated_Feature#Multiple_wins_and_nominations', 'https://en.wikipedia.org/wiki/Hayao_Miyazaki#Views'] 250 266 The Sikh empire's capital at the time of the Battle of Sobraon came under the rule of the British Crown in what year? 1858 https://en.wikipedia.org/wiki/Battle_of_Sobraon https://en.wikipedia.org/wiki/Lahore https://en.wikipedia.org/wiki/Punjab_Province_(British_India) Temporal reasoning ['https://en.wikipedia.org/wiki/Battle_of_Sobraon', 'https://en.wikipedia.org/wiki/Lahore', 'https://en.wikipedia.org/wiki/Punjab_Province_(British_India)'] 251 267 I remember reading a book in elementary school that I LOVED and I want to read it to my daughter. The problem is I can't remember the title. I know that when I first read it, it had recently won a Caldecott Medal. I have another memory from around the same time period of watching the Sydney Summer Olympics. All I remember about the plot is that it was based on a true story and set in the winter time. Can you help me remember the title? Snowflake Bentley https://en.wikipedia.org/wiki/Caldecott_Medal#Recipients https://en.wikipedia.org/wiki/2000_Summer_Olympics https://en.wikipedia.org/wiki/Snowflake_Bentley_(book) Tabular reasoning | Multiple constraints ['https://en.wikipedia.org/wiki/Caldecott_Medal#Recipients', 'https://en.wikipedia.org/wiki/2000_Summer_Olympics', 'https://en.wikipedia.org/wiki/Snowflake_Bentley_(book)'] 252 268 What is the name of the town or city of birth of the player who won the men's singles at the US Open on the year after Venus & Serena Williams played each other for the 8th time as pros? Omaha, Nebraska https://en.wikipedia.org/wiki/Williams_sisters_rivalry https://en.wikipedia.org/wiki/2003_US_Open_(tennis) https://en.wikipedia.org/wiki/Andy_Roddick Multiple constraints | Temporal reasoning ['https://en.wikipedia.org/wiki/Williams_sisters_rivalry', 'https://en.wikipedia.org/wiki/2003_US_Open_(tennis)', 'https://en.wikipedia.org/wiki/Andy_Roddick'] 253 269 Which fast food chain did the sponsor of the Women's 2018 Volleyball Thailand League acquire rights to in 1987? Kentucky Fried Chicken https://en.wikipedia.org/wiki/2018%E2%80%9319_Women%27s_Volleyball_Thailand_League https://en.wikipedia.org/wiki/Charoen_Pokphand Temporal reasoning ['https://en.wikipedia.org/wiki/2018%E2%80%9319_Women%27s_Volleyball_Thailand_League', 'https://en.wikipedia.org/wiki/Charoen_Pokphand'] 254 270 How many calories are in 7 oz. of the fruit given as a present in the musical that won Best Revival of a Musical at the 52nd Annual Tony Awards? 100 calories https://en.wikipedia.org/wiki/52nd_Tony_Awards https://en.wikipedia.org/wiki/Cabaret_(musical) https://en.wikipedia.org/wiki/Pineapple Numerical reasoning | Tabular reasoning ['https://en.wikipedia.org/wiki/52nd_Tony_Awards', 'https://en.wikipedia.org/wiki/Cabaret_(musical)', 'https://en.wikipedia.org/wiki/Pineapple'] 255 271 In the second album of a synthpop-rock band from the county seat city of Utah County, which song has a length of under 3 minutes, not counting any bonus tracks? Everybody Talks https://en.wikipedia.org/wiki/Utah_County,_Utah https://en.wikipedia.org/wiki/Provo,_Utah https://en.wikipedia.org/wiki/List_of_people_from_Provo,_Utah https://en.wikipedia.org/wiki/Neon_Trees https://en.wikipedia.org/wiki/Picture_Show_(album) Numerical reasoning | Tabular reasoning | Multiple constraints ['https://en.wikipedia.org/wiki/Utah_County,_Utah', 'https://en.wikipedia.org/wiki/Provo,_Utah', 'https://en.wikipedia.org/wiki/List_of_people_from_Provo,_Utah', 'https://en.wikipedia.org/wiki/Neon_Trees', 'https://en.wikipedia.org/wiki/Picture_Show_(album)'] 256 272 "The name of which supernova remnant nebula spelled backwards (not including the word ""nubula"") is a homonym for a large sailing ship?" "Crab Nebula (""barc"" is a homonym for ""barque"")" https://en.wikipedia.org/wiki/List_of_supernova_remnants https://en.wikipedia.org/wiki/Barque Post processing ['https://en.wikipedia.org/wiki/List_of_supernova_remnants', 'https://en.wikipedia.org/wiki/Barque'] 257 273 What award did the arachnologist who discovered several species of pseudoscorpions endemic to Australia receive in 2013? The species he discovered in 1987 belong to the Garypidae family and the Synsphyronus genus. The Bonnet Award by the International Society of Arachnology https://en.wikipedia.org/wiki/Pseudoscorpion#Classification https://en.wikipedia.org/wiki/Garypidae#Genera https://en.wikipedia.org/wiki/Synsphyronus#Species https://en.wikipedia.org/wiki/Mark_Harvey_(arachnologist)#Achievements,_awards_and_recognition Multiple constraints ['https://en.wikipedia.org/wiki/Pseudoscorpion#Classification', 'https://en.wikipedia.org/wiki/Garypidae#Genera', 'https://en.wikipedia.org/wiki/Synsphyronus#Species', 'https://en.wikipedia.org/wiki/Mark_Harvey_(arachnologist)#Achievements,_awards_and_recognition'] 258 274 "As of August 2, 2024, what is the title of the most viewed episode in the second most viewed season of the TV show that Zooey Deschanel stars in as a character named ""Jess Day""?" "The most viewed episode of the second season (second most viewed) is its first episode, ""Re-Launch"". " https://en.wikipedia.org/wiki/Zooey_Deschanel https://en.wikipedia.org/wiki/New_Girl#Episodes https://en.wikipedia.org/wiki/List_of_New_Girl_episodes#Season_2_(2012%E2%80%9313) Numerical reasoning | Multiple constraints ['https://en.wikipedia.org/wiki/Zooey_Deschanel', 'https://en.wikipedia.org/wiki/New_Girl#Episodes', 'https://en.wikipedia.org/wiki/List_of_New_Girl_episodes#Season_2_(2012%E2%80%9313)'] 259 275 American author Joan Didion's second fiction novel has a film adaptation. The director of this film is the uncle of a famous pop singer This pop singer once famously sang a song with lyrics describing a combustible-containing device. This song received nominations for Best Pop Solo Performance and Record of the Year at the 54th Annual Grammy Awards. This song was inspired by another book. What is the name of the song and the book that inspired it? "Firework by Katy Perry was inspired by ""On the Road"" by Jack Kerouac " https://en.wikipedia.org/wiki/Joan_Didion https://en.wikipedia.org/wiki/Play_It_as_It_Lays https://en.wikipedia.org/wiki/Play_It_as_It_Lays_(film) https://en.wikipedia.org/wiki/Frank_Perry https://en.wikipedia.org/wiki/Katy_Perry https://en.wikipedia.org/wiki/List_of_awards_and_nominations_received_by_Katy_Perry#Awards_and_nominations Multiple constraints | Post processing | Temporal reasoning ['https://en.wikipedia.org/wiki/Joan_Didion', 'https://en.wikipedia.org/wiki/Play_It_as_It_Lays', 'https://en.wikipedia.org/wiki/Play_It_as_It_Lays_(film)', 'https://en.wikipedia.org/wiki/Frank_Perry', 'https://en.wikipedia.org/wiki/Katy_Perry', 'https://en.wikipedia.org/wiki/List_of_awards_and_nominations_received_by_Katy_Perry#Awards_and_nominations'] 260 276 Canadian politician Keir Clark attended a now defunct university college, which shut its doors in 1969. Who served as the first President of its successor institution? Ronald James Baker https://en.wikipedia.org/wiki/Keir_Clark https://en.wikipedia.org/wiki/Prince_of_Wales_College https://en.wikipedia.org/wiki/University_of_Prince_Edward_Island Multiple constraints ['https://en.wikipedia.org/wiki/Keir_Clark', 'https://en.wikipedia.org/wiki/Prince_of_Wales_College', 'https://en.wikipedia.org/wiki/University_of_Prince_Edward_Island'] 261 277 As of August 2024, who was president of the United States the last time The Beach Boys topped the chart on the Billboard Hot 100? Ronald Reagan https://en.wikipedia.org/wiki/The_Beach_Boys#History https://en.wikipedia.org/wiki/List_of_Billboard_Hot_100_number_ones_of_1988 https://en.wikipedia.org/wiki/President_of_the_United_States#History_and_development https://en.wikipedia.org/wiki/Ronald_Reagan Multiple constraints | Temporal reasoning ['https://en.wikipedia.org/wiki/The_Beach_Boys#History', 'https://en.wikipedia.org/wiki/List_of_Billboard_Hot_100_number_ones_of_1988', 'https://en.wikipedia.org/wiki/President_of_the_United_States#History_and_development', 'https://en.wikipedia.org/wiki/Ronald_Reagan'] 262 278 As of August 1, 2024, which NFL players were both league MVP and Super Bowl MVP in the same season? Bart Starr (66), Terry Bradshaw (78), Joe Montana (89), Emmit Smith (93), Steve Young (94), Kurt Warner (99), and Patrick Mahomes (22). https://en.wikipedia.org/wiki/Associated_Press_NFL_Most_Valuable_Player_Award https://en.wikipedia.org/wiki/Super_Bowl_Most_Valuable_Player_Award Tabular reasoning ['https://en.wikipedia.org/wiki/Associated_Press_NFL_Most_Valuable_Player_Award', 'https://en.wikipedia.org/wiki/Super_Bowl_Most_Valuable_Player_Award'] 263 279 Who was the wife of the founder of the city where the 2023 Tour De France started? Violant of Castile https://en.wikipedia.org/wiki/2023_Tour_de_France https://en.wikipedia.org/wiki/Bilbao https://en.wikipedia.org/wiki/Diego_L%C3%B3pez_V_de_Haro Multiple constraints ['https://en.wikipedia.org/wiki/2023_Tour_de_France', 'https://en.wikipedia.org/wiki/Bilbao', 'https://en.wikipedia.org/wiki/Diego_L%C3%B3pez_V_de_Haro'] 264 280 What was the military rank of the employer of the astronomer who discovered the globular cluster of stars called NGC 6441? Major General was the rank of Thomas Brisbane. https://en.wikipedia.org/wiki/NGC_6441 https://en.wikipedia.org/wiki/James_Dunlop https://en.wikipedia.org/wiki/Thomas_Brisbane Multiple constraints ['https://en.wikipedia.org/wiki/NGC_6441', 'https://en.wikipedia.org/wiki/James_Dunlop', 'https://en.wikipedia.org/wiki/Thomas_Brisbane'] 265 281 What is the average height of Mount Everest, Mount Thor, Mount Denali and The Matterhorn? 17382 ft. https://en.wikipedia.org/wiki/Denali https://en.wikipedia.org/wiki/Matterhorn https://en.wikipedia.org/wiki/Mount_Thor https://en.wikipedia.org/wiki/Mount_Everest Numerical reasoning ['https://en.wikipedia.org/wiki/Denali', 'https://en.wikipedia.org/wiki/Matterhorn', 'https://en.wikipedia.org/wiki/Mount_Thor', 'https://en.wikipedia.org/wiki/Mount_Everest'] 266 282 "How old was the famous composer who taught at the Ospedale della Pietà and was known as ""The Red Priest"" when his first opera premiered? " Antonio Vivaldi was 35 when his first opera, Ottone in villa, premiered. https://en.wikipedia.org/wiki/Ospedale_della_Piet%C3%A0 https://en.wikipedia.org/wiki/Antonio_Vivaldi https://en.wikipedia.org/wiki/List_of_operas_by_Antonio_Vivaldi https://en.wikipedia.org/wiki/Ottone_in_villa Numerical reasoning | Multiple constraints | Temporal reasoning ['https://en.wikipedia.org/wiki/Ospedale_della_Piet%C3%A0', 'https://en.wikipedia.org/wiki/Antonio_Vivaldi', 'https://en.wikipedia.org/wiki/List_of_operas_by_Antonio_Vivaldi', 'https://en.wikipedia.org/wiki/Ottone_in_villa'] 267 283 Twenty-three years after the deadliest battle in the US Civil War, who served as governor in the state in which this battle was fought? Robert E Pattison https://en.wikipedia.org/wiki/List_of_costliest_American_Civil_War_land_battles https://en.wikipedia.org/wiki/Battle_of_Gettysburg https://en.wikipedia.org/wiki/List_of_governors_of_Pennsylvania Numerical reasoning | Tabular reasoning | Multiple constraints | Temporal reasoning ['https://en.wikipedia.org/wiki/List_of_costliest_American_Civil_War_land_battles', 'https://en.wikipedia.org/wiki/Battle_of_Gettysburg', 'https://en.wikipedia.org/wiki/List_of_governors_of_Pennsylvania'] 268 284 Which president featured on a U.S. coin in 1972 served the longest as President of the United States of America? Franklin Delano Roosevelt (12 years). https://en.wikipedia.org/wiki/Penny_(United_States_coin) https://en.wikipedia.org/wiki/Nickel_(United_States_coin) https://en.wikipedia.org/wiki/Dime_(United_States_coin) https://en.wikipedia.org/wiki/Quarter_(United_States_coin) https://en.wikipedia.org/wiki/Half_dollar_(United_States_coin) https://en.wikipedia.org/wiki/Dollar_coin_(United_States) https://en.wikipedia.org/wiki/List_of_presidents_of_the_United_States Numerical reasoning | Tabular reasoning | Multiple constraints ['https://en.wikipedia.org/wiki/Penny_(United_States_coin)', 'https://en.wikipedia.org/wiki/Nickel_(United_States_coin)', 'https://en.wikipedia.org/wiki/Dime_(United_States_coin)', 'https://en.wikipedia.org/wiki/Quarter_(United_States_coin)', 'https://en.wikipedia.org/wiki/Half_dollar_(United_States_coin)', 'https://en.wikipedia.org/wiki/Dollar_coin_(United_States)', 'https://en.wikipedia.org/wiki/List_of_presidents_of_the_United_States'] 269 285 Tiny Tina's Wonderlands' developer released a game 10 years prior to Tiny Tina's Wonderlands' release, this game was released on PC and Consoles. What is the name of the central antagonist of that game? Handsome Jack. https://en.wikipedia.org/wiki/Tiny_Tina%27s_Wonderlands https://en.wikipedia.org/wiki/Gearbox_Software https://en.wikipedia.org/wiki/Borderlands_2 Numerical reasoning | Tabular reasoning | Multiple constraints | Temporal reasoning ['https://en.wikipedia.org/wiki/Tiny_Tina%27s_Wonderlands', 'https://en.wikipedia.org/wiki/Gearbox_Software', 'https://en.wikipedia.org/wiki/Borderlands_2'] 270 286 In 2023, Beavers are known to be the second-largest living rodents. The largest extant rodent species are natively most prevalent in a single continent. What is the largest country in this continent? Brazil. https://en.wikipedia.org/wiki/Beaver https://en.wikipedia.org/wiki/Hydrochoerus https://en.wikipedia.org/wiki/Capybara https://en.wikipedia.org/wiki/South_America Numerical reasoning | Tabular reasoning | Multiple constraints | Temporal reasoning ['https://en.wikipedia.org/wiki/Beaver', 'https://en.wikipedia.org/wiki/Hydrochoerus', 'https://en.wikipedia.org/wiki/Capybara', 'https://en.wikipedia.org/wiki/South_America'] 271 287 Which family duo both made an appearance at the 2017 Billboard Music Awards and starred together on a Disney Channel Original Series? Miley Cyrus and Billy Ray Cyrus https://en.wikipedia.org/wiki/2017_Billboard_Music_Awards https://en.wikipedia.org/wiki/List_of_programs_broadcast_by_Disney_Channel https://en.wikipedia.org/wiki/Hannah_Montana Tabular reasoning | Multiple constraints ['https://en.wikipedia.org/wiki/2017_Billboard_Music_Awards', 'https://en.wikipedia.org/wiki/List_of_programs_broadcast_by_Disney_Channel', 'https://en.wikipedia.org/wiki/Hannah_Montana'] 272 288 Two seasons after Demar Derozan was traded to the San Antonio Spurs, who was the leading scorer for the fifth place team in the Western Conference? Luka Dončić https://en.wikipedia.org/wiki/DeMar_DeRozan https://en.wikipedia.org/wiki/2020%E2%80%9321_NBA_season https://en.wikipedia.org/wiki/2020%E2%80%9321_Dallas_Mavericks_season Numerical reasoning | Tabular reasoning | Multiple constraints | Temporal reasoning ['https://en.wikipedia.org/wiki/DeMar_DeRozan', 'https://en.wikipedia.org/wiki/2020%E2%80%9321_NBA_season', 'https://en.wikipedia.org/wiki/2020%E2%80%9321_Dallas_Mavericks_season'] 273 289 What William Wyler movie debuted the same year that the chairman for the 1982–83 Wolverhampton Wanderers season was born? Jezebel https://en.wikipedia.org/wiki/1982%E2%80%9383_Wolverhampton_Wanderers_F.C._season https://en.wikipedia.org/wiki/Derek_Dougan https://en.wikipedia.org/wiki/William_Wyler Multiple constraints ['https://en.wikipedia.org/wiki/1982%E2%80%9383_Wolverhampton_Wanderers_F.C._season', 'https://en.wikipedia.org/wiki/Derek_Dougan', 'https://en.wikipedia.org/wiki/William_Wyler'] 274 290 "How many more letters does the name of the city that the director of ""Whiplash"" (2014) was born in have compared the name of the city in which the film first premiered?" 2 https://en.wikipedia.org/wiki/Whiplash_(2014_film)# https://en.wikipedia.org/wiki/Damien_Chazelle https://en.wikipedia.org/wiki/2014_Sundance_Film_Festival Post processing ['https://en.wikipedia.org/wiki/Whiplash_(2014_film)#', 'https://en.wikipedia.org/wiki/Damien_Chazelle', 'https://en.wikipedia.org/wiki/2014_Sundance_Film_Festival'] 275 291 The Brihadeeswarar Temple was built by an Indian emperor. The emperor’s only sister‘s husband is a king of another Dynasty. Name the Dynasty and state how many known kings ruled within that Dynasty. Bana Kingdom. 10 Kings. https://en.wikipedia.org/wiki/Brihadisvara_Temple https://en.wikipedia.org/wiki/Rajaraja_I https://en.wikipedia.org/wiki/Kundavai_Pir%C4%81ttiy%C4%81r https://en.wikipedia.org/wiki/Bana_Kingdom Multiple constraints ['https://en.wikipedia.org/wiki/Brihadisvara_Temple', 'https://en.wikipedia.org/wiki/Rajaraja_I', 'https://en.wikipedia.org/wiki/Kundavai_Pir%C4%81ttiy%C4%81r', 'https://en.wikipedia.org/wiki/Bana_Kingdom'] 276 292 Of the 'Big Four' of Thrash Metal, whose debut full-length was released first? Metallica https://en.wikipedia.org/wiki/Thrash_metal https://en.wikipedia.org/wiki/Metallica https://en.wikipedia.org/wiki/Slayer https://en.wikipedia.org/wiki/Megadeth https://en.wikipedia.org/wiki/Anthrax_(American_band) https://en.wikipedia.org/wiki/Kill_%27Em_All https://en.wikipedia.org/wiki/Show_No_Mercy Multiple constraints ['https://en.wikipedia.org/wiki/Thrash_metal', 'https://en.wikipedia.org/wiki/Metallica', 'https://en.wikipedia.org/wiki/Slayer', 'https://en.wikipedia.org/wiki/Megadeth', 'https://en.wikipedia.org/wiki/Anthrax_(American_band)', 'https://en.wikipedia.org/wiki/Kill_%27Em_All', 'https://en.wikipedia.org/wiki/Show_No_Mercy'] 277 293 What is the date of the movie directed by Gordon Douglas that featured an American decathlete who was a part of the 1984 Summer Olympic Torch Relay and first African American to light the cauldron? April 2, 1961 https://en.wikipedia.org/wiki/1984_Summer_Olympics https://en.wikipedia.org/wiki/Rafer_Johnson https://en.wikipedia.org/wiki/The_Sins_of_Rachel_Cade Multiple constraints ['https://en.wikipedia.org/wiki/1984_Summer_Olympics', 'https://en.wikipedia.org/wiki/Rafer_Johnson', 'https://en.wikipedia.org/wiki/The_Sins_of_Rachel_Cade'] 278 294 A 1986 song by Peter Gabriel shares the name with a tool. What is the name of the tool and how does it look? Sledgehammer: A tool with a large, flat, often metal head, attached to a long handle. https://en.wikipedia.org/wiki/Peter_Gabriel https://en.wikipedia.org/wiki/So_(album) https://en.wikipedia.org/wiki/Sledgehammer Multiple constraints ['https://en.wikipedia.org/wiki/Peter_Gabriel', 'https://en.wikipedia.org/wiki/So_(album)', 'https://en.wikipedia.org/wiki/Sledgehammer'] 279 295 How old was the author of Dragon Ball when the manga was first released? 29 https://en.wikipedia.org/wiki/Dragon_Ball https://en.wikipedia.org/wiki/Akira_Toriyama Temporal reasoning ['https://en.wikipedia.org/wiki/Dragon_Ball', 'https://en.wikipedia.org/wiki/Akira_Toriyama'] 280 296 I'm thinking of a dam. Here are some clues: -It had two official names in its history. -Construction was ordered to begin by the husband of the President of The Girl Scouts of the USA in 1936. Hoover Dam (briefly known as Boulder Dam) https://en.wikipedia.org/wiki/Girl_Scouts_of_the_USA#Presidents https://en.wikipedia.org/wiki/Lou_Henry_Hoover https://en.wikipedia.org/wiki/Herbert_Hoover https://en.wikipedia.org/wiki/Hoover_Dam Multiple constraints ['https://en.wikipedia.org/wiki/Girl_Scouts_of_the_USA#Presidents', 'https://en.wikipedia.org/wiki/Lou_Henry_Hoover', 'https://en.wikipedia.org/wiki/Herbert_Hoover', 'https://en.wikipedia.org/wiki/Hoover_Dam'] 281 297 I am the narrator character in the final novel written by the recipient of the 1963 Hugo Award for Best Novel. Who am I? Angel Archer. https://en.wikipedia.org/wiki/Hugo_Award_for_Best_Novel https://en.wikipedia.org/wiki/Philip_K._Dick#Career https://en.wikipedia.org/wiki/The_Transmigration_of_Timothy_Archer Tabular reasoning | Multiple constraints ['https://en.wikipedia.org/wiki/Hugo_Award_for_Best_Novel', 'https://en.wikipedia.org/wiki/Philip_K._Dick#Career', 'https://en.wikipedia.org/wiki/The_Transmigration_of_Timothy_Archer'] 282 298 Who developed the first effective vaccine against the disease that killed the father of a famous Hungarian composer born in 1811? Almroth Edward Wright https://en.wikipedia.org/wiki/List_of_Hungarian_composers https://en.wikipedia.org/wiki/Franz_Liszt https://en.wikipedia.org/wiki/Typhoid_fever Multiple constraints ['https://en.wikipedia.org/wiki/List_of_Hungarian_composers', 'https://en.wikipedia.org/wiki/Franz_Liszt', 'https://en.wikipedia.org/wiki/Typhoid_fever'] 283 299 "What was the last prose book written by the poet who referred to Wyndham Lewis as ""that lonely old volcano of the Right.""?" Forewords and Afterwords (1973) https://en.wikipedia.org/wiki/Wyndham_Lewis https://en.wikipedia.org/wiki/W._H._Auden Multiple constraints ['https://en.wikipedia.org/wiki/Wyndham_Lewis', 'https://en.wikipedia.org/wiki/W._H._Auden'] 284 300 What was the age difference between the inventor of Barbie and the inventor of Hot Wheels? 6 months and 26 days. https://en.wikipedia.org/wiki/Barbie https://en.wikipedia.org/wiki/Hot_Wheels https://en.wikipedia.org/wiki/Ruth_Handler https://en.wikipedia.org/wiki/Elliot_Handler Numerical reasoning | Post processing | Temporal reasoning ['https://en.wikipedia.org/wiki/Barbie', 'https://en.wikipedia.org/wiki/Hot_Wheels', 'https://en.wikipedia.org/wiki/Ruth_Handler', 'https://en.wikipedia.org/wiki/Elliot_Handler'] 285 301 This deceased American singer and songwriter who's song about censorship was nominated for a Grammy Award (for Best Hard Rock Performance) in the same year that was designated as International Space Year by the United Nations. How many letters are in his name? 11 (Layne Staley) https://en.wikipedia.org/wiki/Layne_Staley https://en.wikipedia.org/wiki/Man_in_the_Box https://en.wikipedia.org/wiki/1992 https://en.wikipedia.org/wiki/Grammy_Award_for_Best_Hard_Rock_Performance Numerical reasoning | Tabular reasoning | Multiple constraints | Temporal reasoning ['https://en.wikipedia.org/wiki/Layne_Staley', 'https://en.wikipedia.org/wiki/Man_in_the_Box', 'https://en.wikipedia.org/wiki/1992', 'https://en.wikipedia.org/wiki/Grammy_Award_for_Best_Hard_Rock_Performance'] 286 302 In 2020, the Italian curling team won the Moscow Classic annual bonspiel. One of the members of that four-person team was not born in Italy. As of August 1, 2024, what are the colors of the national flag of their country of origin? Red and White (Switzerland, Joël Retornaz) https://en.wikipedia.org/wiki/Red_Square_Classic https://en.wikipedia.org/wiki/Jo%C3%ABl_Retornaz https://en.wikipedia.org/wiki/Amos_Mosaner https://en.wikipedia.org/wiki/Sebastiano_Arman https://en.wikipedia.org/wiki/Simone_Gonin https://en.wikipedia.org/wiki/Flag_of_Switzerland Multiple constraints ['https://en.wikipedia.org/wiki/Red_Square_Classic', 'https://en.wikipedia.org/wiki/Jo%C3%ABl_Retornaz', 'https://en.wikipedia.org/wiki/Amos_Mosaner', 'https://en.wikipedia.org/wiki/Sebastiano_Arman', 'https://en.wikipedia.org/wiki/Simone_Gonin', 'https://en.wikipedia.org/wiki/Flag_of_Switzerland'] 287 303 When was the song Cold Blow and the Rainy Night featured on a daily list of folk songs recorded by Jon Boden? February 5, 2011 https://en.wikipedia.org/wiki/Cold_Blow_and_the_Rainy_Night_(song) https://en.wikipedia.org/wiki/A_Folk_Song_A_Day Numerical reasoning ['https://en.wikipedia.org/wiki/Cold_Blow_and_the_Rainy_Night_(song)', 'https://en.wikipedia.org/wiki/A_Folk_Song_A_Day'] 288 304 What album by the band named 'The Band' was released on the same day as the Nuclear Non-Proliferation Treaty was opened for signature? Music from Big Pink https://en.wikipedia.org/wiki/Treaty_on_the_Non-Proliferation_of_Nuclear_Weapons https://en.wikipedia.org/wiki/The_Band#Discography https://en.wikipedia.org/wiki/Music_from_Big_Pink Tabular reasoning ['https://en.wikipedia.org/wiki/Treaty_on_the_Non-Proliferation_of_Nuclear_Weapons', 'https://en.wikipedia.org/wiki/The_Band#Discography', 'https://en.wikipedia.org/wiki/Music_from_Big_Pink'] 289 305 In the year the first UK winner of Big Brother was born, who was the Prime Minister? Sir Edward Heath https://en.wikipedia.org/wiki/Big_Brother_(franchise) https://en.wikipedia.org/wiki/Craig_Phillips https://en.wikipedia.org/wiki/Edward_Heath Post processing | Temporal reasoning ['https://en.wikipedia.org/wiki/Big_Brother_(franchise)', 'https://en.wikipedia.org/wiki/Craig_Phillips', 'https://en.wikipedia.org/wiki/Edward_Heath'] 290 306 The lead actress of the television show The Good Place who played protagonist Eleanor Shellstop, is married to a man who has been in many TV series and Films in his career. What was the name of the character he provided the voice for in a 2021 animated movie? Ruben https://en.wikipedia.org/wiki/The_Good_Place https://en.wikipedia.org/wiki/Kristen_Bell https://en.wikipedia.org/wiki/Dax_Shepard#Film https://en.wikipedia.org/wiki/PAW_Patrol:_The_Movie Tabular reasoning | Multiple constraints ['https://en.wikipedia.org/wiki/The_Good_Place', 'https://en.wikipedia.org/wiki/Kristen_Bell', 'https://en.wikipedia.org/wiki/Dax_Shepard#Film', 'https://en.wikipedia.org/wiki/PAW_Patrol:_The_Movie'] 291 307 Andy Warhol made the painting Marilyn Diptych from a publicity photo of Marilyn Monroe for a film. What was the name of Monroe's character in that film? Rose Loomis https://en.wikipedia.org/wiki/Marilyn_Diptych https://en.wikipedia.org/wiki/Niagara_(1953_film) Multiple constraints ['https://en.wikipedia.org/wiki/Marilyn_Diptych', 'https://en.wikipedia.org/wiki/Niagara_(1953_film)'] 292 308 Out of every team that has won at least one official global Valorant tournament, which team has also won at least one League of Legends World Championship and at least one CS:GO Major Championship as of 2024? Fnatic https://en.wikipedia.org/wiki/Valorant_Champions_Tour https://en.wikipedia.org/wiki/League_of_Legends_World_Championship https://en.wikipedia.org/wiki/Counter-Strike_Major_Championships Tabular reasoning | Multiple constraints ['https://en.wikipedia.org/wiki/Valorant_Champions_Tour', 'https://en.wikipedia.org/wiki/League_of_Legends_World_Championship', 'https://en.wikipedia.org/wiki/Counter-Strike_Major_Championships'] 293 309 How old was the New-York born comic book writer who created the character Catwoman when he died? 83 https://en.wikipedia.org/wiki/Catwoman https://en.wikipedia.org/wiki/Bill_Finger https://en.wikipedia.org/wiki/Bob_Kane Multiple constraints ['https://en.wikipedia.org/wiki/Catwoman', 'https://en.wikipedia.org/wiki/Bill_Finger', 'https://en.wikipedia.org/wiki/Bob_Kane'] 294 310 How long did Steve Jobs live, percentage wise, compared to the average lifespan of a person in the United States in 1984? Round each number to the nearest whole integer before calculating your answer. Then round your answer the nearest hundredth. 75.68% https://en.wikipedia.org/wiki/List_of_countries_by_past_life_expectancy https://en.wikipedia.org/wiki/Steve_Jobs Numerical reasoning | Tabular reasoning | Multiple constraints | Post processing | Temporal reasoning ['https://en.wikipedia.org/wiki/List_of_countries_by_past_life_expectancy', 'https://en.wikipedia.org/wiki/Steve_Jobs'] 295 311 What Indian Empire was overrun by Huns 24 years after conventionally accepted date of the fall of the Western Roman Empire? Gupta Empire https://en.wikipedia.org/wiki/Roman_Empire https://en.wikipedia.org/wiki/List_of_ancient_great_powers https://en.wikipedia.org/wiki/Gupta_Empire Numerical reasoning | Multiple constraints ['https://en.wikipedia.org/wiki/Roman_Empire', 'https://en.wikipedia.org/wiki/List_of_ancient_great_powers', 'https://en.wikipedia.org/wiki/Gupta_Empire'] 296 312 Which album by the band Paramore came out after the death of Michael Jackson and before the death of Amy Winehouse? Brand New Eyes https://en.wikipedia.org/wiki/Michael_Jackson https://en.wikipedia.org/wiki/Amy_Winehouse https://en.wikipedia.org/wiki/Paramore https://en.wikipedia.org/wiki/Brand_New_Eyes Multiple constraints ['https://en.wikipedia.org/wiki/Michael_Jackson', 'https://en.wikipedia.org/wiki/Amy_Winehouse', 'https://en.wikipedia.org/wiki/Paramore', 'https://en.wikipedia.org/wiki/Brand_New_Eyes'] 297 313 What song topped Billboard magazine's Top 30 chart in the same year that the first documented case of a person being hit and surviving a meteorite occurred? Little Things Mean a Lot by Kitty Kallen https://en.wikipedia.org/wiki/Ann_Elizabeth_Fowler_Hodges https://en.wikipedia.org/wiki/Billboard_year-end_top_30_singles_of_1954 Tabular reasoning | Multiple constraints | Temporal reasoning ['https://en.wikipedia.org/wiki/Ann_Elizabeth_Fowler_Hodges', 'https://en.wikipedia.org/wiki/Billboard_year-end_top_30_singles_of_1954'] 298 314 How much bigger was Ford Motor Company's market share of US Sales the year Matthew McConaughey won his first MTV Movie Award than the year Zac Efron won his? 10.30% https://en.wikipedia.org/wiki/Ford_Motor_Company#Sales_numbers https://en.wikipedia.org/wiki/List_of_awards_and_nominations_received_by_Matthew_McConaughey https://en.wikipedia.org/wiki/Zac_Efron#Awards_and_nominations Numerical reasoning | Tabular reasoning | Multiple constraints ['https://en.wikipedia.org/wiki/Ford_Motor_Company#Sales_numbers', 'https://en.wikipedia.org/wiki/List_of_awards_and_nominations_received_by_Matthew_McConaughey', 'https://en.wikipedia.org/wiki/Zac_Efron#Awards_and_nominations'] 299 315 At the time of their publication, who was the editor of the magazine that published the two short stories that first developed the World Urslula LeGuinn would use for the first of the two books Harold Bloom would later call her masterpieces? Cele Goldsmith (or Cele Goldsmith Lalli, once she married) https://en.wikipedia.org/wiki/Ursula_K._Le_Guin https://en.wikipedia.org/wiki/A_Wizard_of_Earthsea https://en.wikipedia.org/wiki/The_Rule_of_Names https://en.wikipedia.org/wiki/Fantastic_(magazine) Multiple constraints ['https://en.wikipedia.org/wiki/Ursula_K._Le_Guin', 'https://en.wikipedia.org/wiki/A_Wizard_of_Earthsea', 'https://en.wikipedia.org/wiki/The_Rule_of_Names', 'https://en.wikipedia.org/wiki/Fantastic_(magazine)'] 300 316 Who lit the Olympic cauldron in the Summer Games immediately following the release of Van Halen's sixth studio album? Rafer Johnson lit the Olympic cauldron for the 1984 Summer Olympics, which began July 28, 1984, six months after the release of 1984 (stylized in Roman numerals as MCMLXXXIV), Van Halen's sixth studio album. https://en.wikipedia.org/wiki/1984_(Van_Halen_album) https://en.wikipedia.org/wiki/1984_Summer_Olympics https://en.wikipedia.org/wiki/Van_Halen_discography Temporal reasoning ['https://en.wikipedia.org/wiki/1984_(Van_Halen_album)', 'https://en.wikipedia.org/wiki/1984_Summer_Olympics', 'https://en.wikipedia.org/wiki/Van_Halen_discography'] 301 317 By mass, what is the largest species out of the closest living relatives to manatees and dugongs? African bush elephant (Loxodonta africana) https://en.wikipedia.org/wiki/Manatee https://en.wikipedia.org/wiki/Sirenia https://en.wikipedia.org/wiki/Tethytheria https://en.wikipedia.org/wiki/Proboscidea Multiple constraints ['https://en.wikipedia.org/wiki/Manatee', 'https://en.wikipedia.org/wiki/Sirenia', 'https://en.wikipedia.org/wiki/Tethytheria', 'https://en.wikipedia.org/wiki/Proboscidea'] 302 318 At the time of the END of the 2016 French Open, what was the head-to-head between the Men's World No. 1 at that time, and the runner-up of that year's Australian Open in the Men's Singles category? Novak Djokovic 24-10 Andy Murray https://en.wikipedia.org/wiki/2016_French_Open https://en.wikipedia.org/wiki/List_of_ATP_number_1_ranked_singles_tennis_players https://en.wikipedia.org/wiki/2016_Australian_Open_%E2%80%93_Men%27s_singles https://en.wikipedia.org/wiki/Djokovic%E2%80%93Murray_rivalry Tabular reasoning | Multiple constraints | Temporal reasoning ['https://en.wikipedia.org/wiki/2016_French_Open', 'https://en.wikipedia.org/wiki/List_of_ATP_number_1_ranked_singles_tennis_players', 'https://en.wikipedia.org/wiki/2016_Australian_Open_%E2%80%93_Men%27s_singles', 'https://en.wikipedia.org/wiki/Djokovic%E2%80%93Murray_rivalry'] 303 319 "How many more full seasons already existed of the longest running cartoon television series ever in the United States of all time than seasons of ""X-Men: The Animated Series"" when Barack Obama was elected president of the United States. Show me your reasoning using a mathematical equation. Write out your answer in words, but give me the mathematical equation in numerical form. " Fourteen 19 - 5 = 14 https://en.wikipedia.org/wiki/Barack_Obama https://en.wikipedia.org/wiki/X-Men:_The_Animated_Series https://en.wikipedia.org/wiki/The_Simpsons https://en.wikipedia.org/wiki/List_of_animated_television_series_by_episode_count Numerical reasoning | Multiple constraints | Post processing | Temporal reasoning ['https://en.wikipedia.org/wiki/Barack_Obama', 'https://en.wikipedia.org/wiki/X-Men:_The_Animated_Series', 'https://en.wikipedia.org/wiki/The_Simpsons', 'https://en.wikipedia.org/wiki/List_of_animated_television_series_by_episode_count'] 304 320 What year was the person born who was Prime Minister of The United Kingdom during the year that the first African American ran for president of the United States? 1792 https://en.wikipedia.org/wiki/List_of_African-American_United_States_presidential_and_vice_presidential_candidates#:~:text=In%201848%2C%20Frederick%20Douglass%20became,major%20party%2C%20namely%20the%20Democrats. https://en.wikipedia.org/wiki/List_of_prime_ministers_of_the_United_Kingdom https://en.wikipedia.org/wiki/John_Russell,_1st_Earl_Russell Numerical reasoning | Tabular reasoning | Multiple constraints | Temporal reasoning ['https://en.wikipedia.org/wiki/List_of_African-American_United_States_presidential_and_vice_presidential_candidates#:~:text=In%201848%2C%20Frederick%20Douglass%20became,major%20party%2C%20namely%20the%20Democrats.', 'https://en.wikipedia.org/wiki/List_of_prime_ministers_of_the_United_Kingdom', 'https://en.wikipedia.org/wiki/John_Russell,_1st_Earl_Russell'] 305 321 What is the shortest possible abbreviation in the United States for the last to be discovered of the three antileprosy drugs on the World Health Organization's List of Essential Medicines? R https://en.wikipedia.org/wiki/WHO_Model_List_of_Essential_Medicines https://en.wikipedia.org/wiki/Clofazimine https://en.wikipedia.org/wiki/Dapsone https://en.wikipedia.org/wiki/Rifampicin Temporal reasoning ['https://en.wikipedia.org/wiki/WHO_Model_List_of_Essential_Medicines', 'https://en.wikipedia.org/wiki/Clofazimine', 'https://en.wikipedia.org/wiki/Dapsone', 'https://en.wikipedia.org/wiki/Rifampicin'] 306 322 What is the 7th track on the 3rd album released by the band formed in 1981 and fronted by Jordan Luck? As I Love You https://en.wikipedia.org/wiki/Jordan_Luck https://en.wikipedia.org/wiki/The_Exponents https://en.wikipedia.org/wiki/Amplifier_(Dance_Exponents_album)#Track_listing Tabular reasoning ['https://en.wikipedia.org/wiki/Jordan_Luck', 'https://en.wikipedia.org/wiki/The_Exponents', 'https://en.wikipedia.org/wiki/Amplifier_(Dance_Exponents_album)#Track_listing'] 307 323 This athlete was the first man to run the 100 metres in under 10 seconds at an Olympic Games. Which NFL team was he drafted by? Miami Dolphins (Jim Hines) https://en.wikipedia.org/wiki/100_metres https://en.wikipedia.org/wiki/Jim_Hines Post processing ['https://en.wikipedia.org/wiki/100_metres', 'https://en.wikipedia.org/wiki/Jim_Hines'] 308 324 Which member of the Wu-Tang Clan was born on Staten Island? Ghostface Killah https://en.wikipedia.org/wiki/Wu-Tang_Clan https://en.wikipedia.org/wiki/RZA https://en.wikipedia.org/wiki/GZA https://en.wikipedia.org/wiki/Method_Man https://en.wikipedia.org/wiki/Raekwon https://en.wikipedia.org/wiki/Ghostface_Killah https://en.wikipedia.org/wiki/Inspectah_Deck https://en.wikipedia.org/wiki/U-God https://en.wikipedia.org/wiki/Masta_Killa https://en.wikipedia.org/wiki/Ol%27_Dirty_Bastard https://en.wikipedia.org/wiki/Cappadonna Multiple constraints ['https://en.wikipedia.org/wiki/Wu-Tang_Clan', 'https://en.wikipedia.org/wiki/RZA', 'https://en.wikipedia.org/wiki/GZA', 'https://en.wikipedia.org/wiki/Method_Man', 'https://en.wikipedia.org/wiki/Raekwon', 'https://en.wikipedia.org/wiki/Ghostface_Killah', 'https://en.wikipedia.org/wiki/Inspectah_Deck', 'https://en.wikipedia.org/wiki/U-God', 'https://en.wikipedia.org/wiki/Masta_Killa', 'https://en.wikipedia.org/wiki/Ol%27_Dirty_Bastard', 'https://en.wikipedia.org/wiki/Cappadonna'] 309 325 "The New Zealand author of the children's book ""Bobby the Littlest War Hero"" won a Queen's Service Medal how many years before it was renamed the King's Service Medal?" Glyn Harper won the medal 12 years before it was renamed. https://en.wikipedia.org/wiki/Glyn_Harper https://en.wikipedia.org/wiki/King%27s_Service_Medal Numerical reasoning ['https://en.wikipedia.org/wiki/Glyn_Harper', 'https://en.wikipedia.org/wiki/King%27s_Service_Medal'] 310 326 I'm thinking of a sport that a University in the midwest USA won the championship for in 1974. The University suffered a defeat in football in 2005, losing to the University where the 18th honourary citizen of Beijing obtained his Bachelor's degree. What is the sport I'm thinking of? Cross country https://en.wikipedia.org/wiki/List_of_honorary_citizens_of_Beijing https://en.wikipedia.org/wiki/George_E._Killian https://en.wikipedia.org/wiki/Ohio_Northern_University https://en.wikipedia.org/wiki/University_of_Mount_Union https://en.wikipedia.org/wiki/NCAA_Division_III_men%27s_cross_country_championships Tabular reasoning | Multiple constraints | Temporal reasoning ['https://en.wikipedia.org/wiki/List_of_honorary_citizens_of_Beijing', 'https://en.wikipedia.org/wiki/George_E._Killian', 'https://en.wikipedia.org/wiki/Ohio_Northern_University', 'https://en.wikipedia.org/wiki/University_of_Mount_Union', 'https://en.wikipedia.org/wiki/NCAA_Division_III_men%27s_cross_country_championships'] 311 327 Who was elected the United States President in the same year that a ship, which was named after the snake that some argue killed Cleopatra, wrecked after the United Kingdom captured it from France? Andrew Jackson https://en.wikipedia.org/wiki/Cleopatra https://en.wikipedia.org/wiki/HMS_Asp https://en.wikipedia.org/wiki/French_brig_Serpent_(1807) https://en.wikipedia.org/wiki/1828_United_States_presidential_election Multiple constraints | Temporal reasoning ['https://en.wikipedia.org/wiki/Cleopatra', 'https://en.wikipedia.org/wiki/HMS_Asp', 'https://en.wikipedia.org/wiki/French_brig_Serpent_(1807)', 'https://en.wikipedia.org/wiki/1828_United_States_presidential_election'] 312 328 What major historical event began 171 years before the first European Nations' Cup in the country that hosted the tournament? The French Revolution https://en.wikipedia.org/wiki/UEFA_European_Championship https://en.wikipedia.org/wiki/1960_European_Nations%27_Cup https://en.wikipedia.org/wiki/France Numerical reasoning | Multiple constraints | Temporal reasoning ['https://en.wikipedia.org/wiki/UEFA_European_Championship', 'https://en.wikipedia.org/wiki/1960_European_Nations%27_Cup', 'https://en.wikipedia.org/wiki/France'] 313 329 "For how long was the ""Father of the National Parks"", the first president to declare a national park, and the president who declared the most national parks all alive at the same time?" 26 years, 8 months, and 26 days. https://en.wikipedia.org/wiki/List_of_national_parks_of_the_United_States https://en.wikipedia.org/wiki/John_Muir https://en.wikipedia.org/wiki/Ulysses_S._Grant https://en.wikipedia.org/wiki/Theodore_Roosevelt Numerical reasoning | Multiple constraints | Temporal reasoning ['https://en.wikipedia.org/wiki/List_of_national_parks_of_the_United_States', 'https://en.wikipedia.org/wiki/John_Muir', 'https://en.wikipedia.org/wiki/Ulysses_S._Grant', 'https://en.wikipedia.org/wiki/Theodore_Roosevelt'] 314 330 Using data from the year 2020, if you were to combine the permanent human populations of Aukland Island, Rose Island, and Budelli Island, how many total people would you have? 1 https://en.wikipedia.org/wiki/Auckland_Island#Human_presence_on_the_island https://en.wikipedia.org/wiki/Rose_Island_(New_Zealand) https://en.wikipedia.org/wiki/Budelli Numerical reasoning | Multiple constraints | Post processing ['https://en.wikipedia.org/wiki/Auckland_Island#Human_presence_on_the_island', 'https://en.wikipedia.org/wiki/Rose_Island_(New_Zealand)', 'https://en.wikipedia.org/wiki/Budelli'] 315 331 I'm thinking of a famous zoologist who is alumni of Upsala University and is credited with formally describing the African Gray parrot. He published his first edition of a certain book with a Latin title in the Netherlands, in 1735, while attending the university. This book contained his system of classifying animals, and it was key to the book's title. What is the modern-day term for the classification system he famously wrote about? Binomial nomenclature https://en.wikipedia.org/wiki/Grey_parrot https://en.wikipedia.org/wiki/Carl_Linnaeus https://en.wikipedia.org/wiki/Systema_Naturae Post processing | Temporal reasoning ['https://en.wikipedia.org/wiki/Grey_parrot', 'https://en.wikipedia.org/wiki/Carl_Linnaeus', 'https://en.wikipedia.org/wiki/Systema_Naturae'] 316 332 How many copies of Coit Tower would have to be stacked on top of the Willis Tower in order to exceed the height of the Chicago Spire, had it been completed? Give your answer as the lowest possible whole number of Coit Towers. 3 https://en.wikipedia.org/wiki/Willis_Tower https://en.wikipedia.org/wiki/Chicago_Spire https://en.wikipedia.org/wiki/Coit_Tower Numerical reasoning ['https://en.wikipedia.org/wiki/Willis_Tower', 'https://en.wikipedia.org/wiki/Chicago_Spire', 'https://en.wikipedia.org/wiki/Coit_Tower'] 317 333 The man who owned The Washington Post in 1932 broke the trust established by his father to gain control of the paper, only to run into the ground, lose it, and get committed to a psychiatric hospital where he died. How many days after his death did the man who purchased it in 1933 die? 6563 days https://en.wikipedia.org/wiki/The_Washington_Post https://en.wikipedia.org/wiki/Edward_Beale_McLean https://en.wikipedia.org/wiki/Eugene_Meyer_(financier) Numerical reasoning | Multiple constraints | Temporal reasoning ['https://en.wikipedia.org/wiki/The_Washington_Post', 'https://en.wikipedia.org/wiki/Edward_Beale_McLean', 'https://en.wikipedia.org/wiki/Eugene_Meyer_(financier)'] 318 334 How tall does the flower get on the very first orchid David L. Jones (botanist) described and published in his career? 20–30 millimetres (0.8–1 in) https://en.wikipedia.org/wiki/David_L._Jones_(botanist) https://en.wikipedia.org/wiki/Pterostylis_aestiva#Description Multiple constraints ['https://en.wikipedia.org/wiki/David_L._Jones_(botanist)', 'https://en.wikipedia.org/wiki/Pterostylis_aestiva#Description'] 319 335 The founder of the new religious movement Thelema once attempted to summit the third highest mountain in the world, but failed. How many years did it take before this feat was successfully accomplished? 50 https://en.wikipedia.org/wiki/Thelema https://en.wikipedia.org/wiki/Aleister_Crowley https://en.wikipedia.org/wiki/List_of_highest_mountains_on_Earth https://en.wikipedia.org/wiki/1905_Kanchenjunga_expedition Numerical reasoning | Tabular reasoning | Multiple constraints | Post processing | Temporal reasoning ['https://en.wikipedia.org/wiki/Thelema', 'https://en.wikipedia.org/wiki/Aleister_Crowley', 'https://en.wikipedia.org/wiki/List_of_highest_mountains_on_Earth', 'https://en.wikipedia.org/wiki/1905_Kanchenjunga_expedition'] 320 336 Human activity by employees of a certain company caused Lake Peigneur in Louisiana to become completely drained of water. How many years passed between the founding of that company and Lake Peigneur's collapse? Convert this number to Roman numerals. LXXVIII https://en.wikipedia.org/wiki/Lake_Peigneur https://en.wikipedia.org/wiki/Texaco Numerical reasoning | Multiple constraints | Post processing | Temporal reasoning ['https://en.wikipedia.org/wiki/Lake_Peigneur', 'https://en.wikipedia.org/wiki/Texaco'] 321 337 Emory Kristof, the photographer who helped find the Titanic's wreckage, shares an alma mater with a co-creator of Seinfeld. How many years separate their births? 5 years https://en.wikipedia.org/wiki/Emory_Kristof https://en.wikipedia.org/wiki/University_of_Maryland,_College_Park#Notable_alumni https://en.wikipedia.org/wiki/Larry_David Numerical reasoning | Tabular reasoning | Post processing | Temporal reasoning ['https://en.wikipedia.org/wiki/Emory_Kristof', 'https://en.wikipedia.org/wiki/University_of_Maryland,_College_Park#Notable_alumni', 'https://en.wikipedia.org/wiki/Larry_David'] 322 338 Who manufactured the kits Tomáš Pekhart wore in 2008 in his senior career? Puma and Umbro https://en.wikipedia.org/wiki/Tom%C3%A1%C5%A1_Pekhart https://en.wikipedia.org/wiki/Tottenham_Hotspur_F.C. https://en.wikipedia.org/wiki/Southampton_F.C. Tabular reasoning ['https://en.wikipedia.org/wiki/Tom%C3%A1%C5%A1_Pekhart', 'https://en.wikipedia.org/wiki/Tottenham_Hotspur_F.C.', 'https://en.wikipedia.org/wiki/Southampton_F.C.'] 323 339 How many years before Starbucks introduced juice-blended Frappuccinos, was the company that is accredited for inventing the Frappuccino, founded? 31 years https://en.wikipedia.org/wiki/George_Howell_(entrepreneur)#The_Coffee_Connection https://en.wikipedia.org/wiki/Frappuccino Temporal reasoning ['https://en.wikipedia.org/wiki/George_Howell_(entrepreneur)#The_Coffee_Connection', 'https://en.wikipedia.org/wiki/Frappuccino'] 324 340 Which Nobel laureates in Physics between 1901 and 1920 made contributions to quantum theory, and what were their specific contributions? List the laureates and their respective contributions. Several Nobel laureates in Physics between 1901 and 1920 made significant contributions to quantum theory. Here are the laureates and their specific contributions: 1. **Max Planck (1918)** - Planck is considered the father of quantum theory. He introduced the idea of energy quanta and derived Planck's radiation law, which describes the electromagnetic radiation emitted by a black body in thermal equilibrium. 2. **Albert Einstein (1921)** - Although awarded the Nobel Prize in 1921, Einstein made crucial contributions to quantum theory with his explanation of the photoelectric effect, which demonstrated the particle nature of light. 3. **Niels Bohr (1922)** - Bohr received his Nobel Prize for his contributions to our understanding of atomic structure and radiation. His Bohr model of the atom incorporated quantum theory and explained how electrons could have stable orbits. 4. **Johannes Stark (1919)** - Stark was awarded the Nobel Prize for his discovery of the Doppler effect in canal rays and the splitting of spectral lines in electric fields (the Stark effect), both phenomena that provided insights into atomic structure and quantum mechanics. 5. **James Franck and Gustav Hertz (1925)** - Franck and Hertz's Nobel Prize was awarded for their experiments on electron impacts in atoms, which confirmed quantum energy levels within atoms. https://en.wikipedia.org/wiki/Max_Planck https://en.wikipedia.org/wiki/Albert_Einstein https://en.wikipedia.org/wiki/Niels_Bohr https://en.wikipedia.org/wiki/Johannes_Stark https://en.wikipedia.org/wiki/James_Franck https://en.wikipedia.org/wiki/Gustav_Hertz https://en.wikipedia.org/wiki/List_of_Nobel_laureates_in_Physics https://en.wikipedia.org/wiki/Quantum_mechanics https://en.wikipedia.org/wiki/Photoelectric_effect https://en.wikipedia.org/wiki/Bohr_model Numerical reasoning | Multiple constraints | Post processing ['https://en.wikipedia.org/wiki/Max_Planck', 'https://en.wikipedia.org/wiki/Albert_Einstein', 'https://en.wikipedia.org/wiki/Niels_Bohr', 'https://en.wikipedia.org/wiki/Johannes_Stark', 'https://en.wikipedia.org/wiki/James_Franck', 'https://en.wikipedia.org/wiki/Gustav_Hertz', 'https://en.wikipedia.org/wiki/List_of_Nobel_laureates_in_Physics', 'https://en.wikipedia.org/wiki/Quantum_mechanics', 'https://en.wikipedia.org/wiki/Photoelectric_effect', 'https://en.wikipedia.org/wiki/Bohr_model'] 325 341 Which Labour MP served for Bury North while Chukka Umunna was the MP for Streatham? James Frith https://en.wikipedia.org/wiki/Bury_North_(UK_Parliament_constituency) https://en.wikipedia.org/wiki/Streatham_(UK_Parliament_constituency) Tabular reasoning ['https://en.wikipedia.org/wiki/Bury_North_(UK_Parliament_constituency)', 'https://en.wikipedia.org/wiki/Streatham_(UK_Parliament_constituency)'] 326 342 How much older was the songwriter of 'The Twist' when he wrote it than Chubby Checker was when he recorded the song? 12 years https://en.wikipedia.org/wiki/The_Twist_(song) https://en.wikipedia.org/wiki/Hank_Ballard https://en.wikipedia.org/wiki/Chubby_Checker Multiple constraints | Temporal reasoning ['https://en.wikipedia.org/wiki/The_Twist_(song)', 'https://en.wikipedia.org/wiki/Hank_Ballard', 'https://en.wikipedia.org/wiki/Chubby_Checker'] 327 343 As of August 4, 2024, what other idol groups, that are not NCT-related, is the only Canadian NCT member connected to? SuperM https://en.wikipedia.org/wiki/NCT_(group)#Members https://en.wikipedia.org/wiki/Mark_Lee_(singer) Tabular reasoning ['https://en.wikipedia.org/wiki/NCT_(group)#Members', 'https://en.wikipedia.org/wiki/Mark_Lee_(singer)'] 328 344 Of the top two most watched television season finales (as of June 2024), which finale ran the longest in length and by how much? The MASH finale ran for 52 minutes longer than the Cheers finale. https://en.wikipedia.org/wiki/Series_finale https://en.wikipedia.org/wiki/Goodbye,_Farewell_and_Amen https://en.wikipedia.org/wiki/One_for_the_Road_(Cheers) Numerical reasoning | Tabular reasoning ['https://en.wikipedia.org/wiki/Series_finale', 'https://en.wikipedia.org/wiki/Goodbye,_Farewell_and_Amen', 'https://en.wikipedia.org/wiki/One_for_the_Road_(Cheers)'] 329 345 What compass direction (of the 4 cardinal and 4 ordinal directions) is the capital city of the state which houses the Jackson Hole Mountain Resort in relation to the centre of the state? Southeast https://en.wikipedia.org/wiki/Jackson_Hole_Mountain_Resort https://en.wikipedia.org/wiki/Wyoming https://en.wikipedia.org/wiki/Cheyenne,_Wyoming Multiple constraints ['https://en.wikipedia.org/wiki/Jackson_Hole_Mountain_Resort', 'https://en.wikipedia.org/wiki/Wyoming', 'https://en.wikipedia.org/wiki/Cheyenne,_Wyoming'] 330 346 How many years older is the police force that covers the City of London, than the political party that came 4th in the 2019 European Parliament election in Lombardy? Write the number of years in binary 10101110 https://en.wikipedia.org/wiki/City_of_London https://en.wikipedia.org/wiki/City_of_London_Police https://en.wikipedia.org/wiki/2019_European_Parliament_election_in_Lombardy https://en.wikipedia.org/wiki/Forza_Italia_(2013) Numerical reasoning | Multiple constraints | Post processing | Temporal reasoning ['https://en.wikipedia.org/wiki/City_of_London', 'https://en.wikipedia.org/wiki/City_of_London_Police', 'https://en.wikipedia.org/wiki/2019_European_Parliament_election_in_Lombardy', 'https://en.wikipedia.org/wiki/Forza_Italia_(2013)'] 331 347 Lauryn Hill has one older brother named Malaney. The year he was born, a famous baseball player who played his final season with the Philadelphia Athletics died in March. That player was elected to the Baseball Hall of Fame. 6 years later after his election, the only person to be inducted to the Baseball Hall of Fame had how many total career wins? 365 https://en.wikipedia.org/wiki/Lauryn_Hill#Early_life https://en.wikipedia.org/wiki/1972# https://en.wikipedia.org/wiki/Zack_Wheat# https://en.wikipedia.org/wiki/List_of_members_of_the_Baseball_Hall_of_Fame https://en.wikipedia.org/wiki/Pud_Galvin Numerical reasoning | Tabular reasoning | Multiple constraints | Post processing | Temporal reasoning ['https://en.wikipedia.org/wiki/Lauryn_Hill#Early_life', 'https://en.wikipedia.org/wiki/1972#', 'https://en.wikipedia.org/wiki/Zack_Wheat#', 'https://en.wikipedia.org/wiki/List_of_members_of_the_Baseball_Hall_of_Fame', 'https://en.wikipedia.org/wiki/Pud_Galvin'] 332 348 What is an animal that is distantly related to the capybara and has the densest fur of all land-dwelling mammals? Chinchilla https://en.wikipedia.org/wiki/Capybara# https://en.wikipedia.org/wiki/Chinchilla Multiple constraints ['https://en.wikipedia.org/wiki/Capybara#', 'https://en.wikipedia.org/wiki/Chinchilla'] 333 349 Who was the United States President when Chile won their first Copa America? Barack Obama https://en.wikipedia.org/wiki/List_of_Copa_Am%C3%A9rica_finals#Finals https://en.wikipedia.org/wiki/List_of_presidents_of_the_United_States Tabular reasoning | Temporal reasoning ['https://en.wikipedia.org/wiki/List_of_Copa_Am%C3%A9rica_finals#Finals', 'https://en.wikipedia.org/wiki/List_of_presidents_of_the_United_States'] 334 350 "Which female athlete achieved a ""world's best"" and a ""world record"" for the marathon during Tony Blair's second term as British Prime Minister." Paula Radcliffe https://en.wikipedia.org/wiki/Tony_Blair https://en.wikipedia.org/wiki/Marathon_world_record_progression Tabular reasoning | Temporal reasoning ['https://en.wikipedia.org/wiki/Tony_Blair', 'https://en.wikipedia.org/wiki/Marathon_world_record_progression'] 335 351 Who were the key members of the original lineup of the rock band Queen, and what were their primary roles in the band? Which original member of Queen was also in a band named Smile and what year did that band begin and end? The original lineup of Queen consisted of Freddie Mercury (lead vocals, piano), Brian May (guitar, vocals), Roger Taylor (drums, vocals), and John Deacon (bass guitar). Smile was an English rock band formed in London in 1968 with Brian May and Tim Staffell. It ended in 1970 when Staffell left to join another band, Humpy Bong. https://en.wikipedia.org/wiki/Queen_(band) https://en.wikipedia.org/wiki/Smile_(band) Temporal reasoning ['https://en.wikipedia.org/wiki/Queen_(band)', 'https://en.wikipedia.org/wiki/Smile_(band)'] 336 352 In the town where the most successful American Ace of World War 2 to survive that war was born, there is a college. What was the name and percentage of the largest demographic of full and part time students attending that college in 2020? Black/African American, 80%. https://en.wikipedia.org/wiki/List_of_World_War_II_aces_from_the_United_States https://en.wikipedia.org/wiki/David_McCampbell https://en.wikipedia.org/wiki/Bessemer,_Alabama https://en.wikipedia.org/wiki/Lawson_State_Community_College Tabular reasoning | Multiple constraints ['https://en.wikipedia.org/wiki/List_of_World_War_II_aces_from_the_United_States', 'https://en.wikipedia.org/wiki/David_McCampbell', 'https://en.wikipedia.org/wiki/Bessemer,_Alabama', 'https://en.wikipedia.org/wiki/Lawson_State_Community_College'] 337 353 What geographical commonality do the person who first described Apriona brunneomarginata and painter Wolfgang Hutter share? They were both born in Vienna. https://en.wikipedia.org/wiki/Apriona_brunneomarginata https://en.wikipedia.org/wiki/Stephan_von_Breuning_(entomologist) https://en.wikipedia.org/wiki/Wolfgang_Hutter Multiple constraints ['https://en.wikipedia.org/wiki/Apriona_brunneomarginata', 'https://en.wikipedia.org/wiki/Stephan_von_Breuning_(entomologist)', 'https://en.wikipedia.org/wiki/Wolfgang_Hutter'] 338 355 What is population in 1968 of the city that hosted the olympic games in which Lucy Steele competed? 15,739 https://en.wikipedia.org/wiki/Lucy_Steele https://en.wikipedia.org/wiki/1992_Winter_Olympics https://en.wikipedia.org/wiki/Albertville Multiple constraints ['https://en.wikipedia.org/wiki/Lucy_Steele', 'https://en.wikipedia.org/wiki/1992_Winter_Olympics', 'https://en.wikipedia.org/wiki/Albertville'] 339 356 There's a comedy club on Sunset in West Hollywood that opened in 1972. One of its founders opened for one of Elvis' comeback performances. Where was this founder's first wife born? Marinette, Wisconsin https://en.wikipedia.org/wiki/The_Comedy_Store https://en.wikipedia.org/wiki/Sammy_Shore https://en.wikipedia.org/wiki/Mitzi_Shore Multiple constraints | Temporal reasoning ['https://en.wikipedia.org/wiki/The_Comedy_Store', 'https://en.wikipedia.org/wiki/Sammy_Shore', 'https://en.wikipedia.org/wiki/Mitzi_Shore'] 340 357 As of August 3, 2024, which band was nominated three times for the Grammy Award for Best Metal Performance and also headlined the Opus Stage at Download Festival 2023? Ghost https://en.wikipedia.org/wiki/Grammy_Award_for_Best_Metal_Performance https://en.wikipedia.org/wiki/Download_Festival Tabular reasoning ['https://en.wikipedia.org/wiki/Grammy_Award_for_Best_Metal_Performance', 'https://en.wikipedia.org/wiki/Download_Festival'] 341 358 Spryo the Dragon released a sequel just a year after it came out. In the same year Spryo's sequel came out, what popular skateboarding based game hit shelves? Tony Hawk's Pro Skater https://en.wikipedia.org/wiki/Spyro_the_Dragon https://en.wikipedia.org/wiki/Spyro_2:_Ripto%27s_Rage! https://en.wikipedia.org/wiki/Tony_Hawk%27s_Pro_Skater Multiple constraints | Temporal reasoning ['https://en.wikipedia.org/wiki/Spyro_the_Dragon', 'https://en.wikipedia.org/wiki/Spyro_2:_Ripto%27s_Rage!', 'https://en.wikipedia.org/wiki/Tony_Hawk%27s_Pro_Skater'] 342 359 Who was the president of Kenya when Uduak Amimo took a break from her talk show and the media to pursue her interest in coaching and social impact projects? Uhuru Kenyatta https://en.wikipedia.org/wiki/Uduak_Amimo https://en.wikipedia.org/wiki/President_of_Kenya Temporal reasoning ['https://en.wikipedia.org/wiki/Uduak_Amimo', 'https://en.wikipedia.org/wiki/President_of_Kenya'] 343 360 Which month had the third-lowest mean daily minimum temperature (recorded from 1876–1905) in the Japanese city which in 1720 was estimated to be the largest in the world? December. https://en.wikipedia.org/wiki/List_of_largest_cities_throughout_history https://en.wikipedia.org/wiki/Edo https://en.wikipedia.org/wiki/Tokyo Numerical reasoning | Tabular reasoning | Temporal reasoning ['https://en.wikipedia.org/wiki/List_of_largest_cities_throughout_history', 'https://en.wikipedia.org/wiki/Edo', 'https://en.wikipedia.org/wiki/Tokyo'] 344 362 Which political leader of the countries where the Potsdam agreement and the Wellington Convention were signed was older on the 1st of July 1905? Richard Seddon https://en.wikipedia.org/wiki/Potsdam_Agreement https://en.wikipedia.org/wiki/Wellington_Convention https://en.wikipedia.org/wiki/List_of_prime_ministers_of_New_Zealand https://en.wikipedia.org/wiki/List_of_chancellors_of_Germany https://en.wikipedia.org/wiki/Richard_Seddon https://en.wikipedia.org/wiki/Bernhard_von_B%C3%BClow Numerical reasoning | Multiple constraints | Temporal reasoning ['https://en.wikipedia.org/wiki/Potsdam_Agreement', 'https://en.wikipedia.org/wiki/Wellington_Convention', 'https://en.wikipedia.org/wiki/List_of_prime_ministers_of_New_Zealand', 'https://en.wikipedia.org/wiki/List_of_chancellors_of_Germany', 'https://en.wikipedia.org/wiki/Richard_Seddon', 'https://en.wikipedia.org/wiki/Bernhard_von_B%C3%BClow'] 345 363 In the year that the film 'Moana' was released, who was the hitting coach of the team that lost the MLB World Series? Ty Van Burkleo https://en.wikipedia.org/wiki/Moana_(2016_film) https://en.wikipedia.org/wiki/2016_World_Series https://en.wikipedia.org/wiki/2016_Cleveland_Indians_season Multiple constraints ['https://en.wikipedia.org/wiki/Moana_(2016_film)', 'https://en.wikipedia.org/wiki/2016_World_Series', 'https://en.wikipedia.org/wiki/2016_Cleveland_Indians_season'] 346 364 What sea borders the Irish County where the author of the short story Foster was born? The Irish Sea https://en.wikipedia.org/wiki/Foster_(short_story) https://en.wikipedia.org/wiki/Claire_Keegan https://en.wikipedia.org/wiki/County_Wicklow Multiple constraints ['https://en.wikipedia.org/wiki/Foster_(short_story)', 'https://en.wikipedia.org/wiki/Claire_Keegan', 'https://en.wikipedia.org/wiki/County_Wicklow'] 347 365 How many times in total did the Argentina women's field hockey team and the Uruguay women's field hockey team enter the Pan American Games and the Olympic Games from 2000 to 2020? 15 https://en.wikipedia.org/wiki/Field_hockey_at_the_Pan_American_Games https://en.wikipedia.org/wiki/Field_hockey_at_the_Summer_Olympics Numerical reasoning | Tabular reasoning | Temporal reasoning ['https://en.wikipedia.org/wiki/Field_hockey_at_the_Pan_American_Games', 'https://en.wikipedia.org/wiki/Field_hockey_at_the_Summer_Olympics'] 348 366 If you add up the birth years of Emma Watson, Daniel Radcliffe and Rupert Grint, what is the sum? 5967 https://en.wikipedia.org/wiki/Daniel_Radcliffe https://en.wikipedia.org/wiki/Emma_Watson https://en.wikipedia.org/wiki/Rupert_Grint Numerical reasoning | Multiple constraints ['https://en.wikipedia.org/wiki/Daniel_Radcliffe', 'https://en.wikipedia.org/wiki/Emma_Watson', 'https://en.wikipedia.org/wiki/Rupert_Grint'] 349 367 The Eastern Shawnee Tribe of Oklahoma owns and operates the Indigo Sky Casino. Their tribal headquarters are located in a suburb of the metropolitan area of a historic 2011 tornado that killed 158 people. What is the name of the metropolitan area? Joplin, Missouri metropolitan area https://en.wikipedia.org/wiki/Eastern_Shawnee_Tribe_of_Oklahoma https://en.wikipedia.org/wiki/Wyandotte,_Oklahoma https://en.wikipedia.org/wiki/2011_Joplin_tornado Multiple constraints ['https://en.wikipedia.org/wiki/Eastern_Shawnee_Tribe_of_Oklahoma', 'https://en.wikipedia.org/wiki/Wyandotte,_Oklahoma', 'https://en.wikipedia.org/wiki/2011_Joplin_tornado'] 350 368 How many years apart was the founding of Snell & Wilmer from the birth of the man who got 3rd place at the 1954 Masters golf tournament? 16 years https://en.wikipedia.org/wiki/Snell_&_Wilmer https://en.wikipedia.org/wiki/1954_Masters_Tournament https://en.wikipedia.org/wiki/Billy_Joe_Patton Numerical reasoning ['https://en.wikipedia.org/wiki/Snell_&_Wilmer', 'https://en.wikipedia.org/wiki/1954_Masters_Tournament', 'https://en.wikipedia.org/wiki/Billy_Joe_Patton'] 351 369 Which other astronaut flew the same type of aircraft during the Korean War as the first man to step on the moon? John Glenn https://en.wikipedia.org/wiki/Moon_landing https://en.wikipedia.org/wiki/Neil_Armstrong https://en.wikipedia.org/wiki/Grumman_F9F_Panther https://en.wikipedia.org/wiki/John_Glenn Multiple constraints ['https://en.wikipedia.org/wiki/Moon_landing', 'https://en.wikipedia.org/wiki/Neil_Armstrong', 'https://en.wikipedia.org/wiki/Grumman_F9F_Panther', 'https://en.wikipedia.org/wiki/John_Glenn'] 352 370 How many days did it take for World War 2 to end after the death of Alois Burgstaller? 136 days https://en.wikipedia.org/wiki/Alois_Burgstaller https://en.wikipedia.org/wiki/World_War_II Numerical reasoning | Post processing | Temporal reasoning ['https://en.wikipedia.org/wiki/Alois_Burgstaller', 'https://en.wikipedia.org/wiki/World_War_II'] 353 371 Adolf Hitler was born exactly 110 years before which US mass shooting? Columbine https://en.wikipedia.org/wiki/Adolf_Hitler https://en.wikipedia.org/wiki/Mass_shootings_in_the_United_States https://en.wikipedia.org/wiki/Columbine_High_School_massacre Numerical reasoning | Tabular reasoning | Multiple constraints | Temporal reasoning ['https://en.wikipedia.org/wiki/Adolf_Hitler', 'https://en.wikipedia.org/wiki/Mass_shootings_in_the_United_States', 'https://en.wikipedia.org/wiki/Columbine_High_School_massacre'] 354 372 How old was the 31st President of the United States when the second nuclear weapon ever used in warfare was dropped? 70 years old https://en.wikipedia.org/wiki/List_of_presidents_of_the_United_States https://en.wikipedia.org/wiki/Herbert_Hoover https://en.wikipedia.org/wiki/Nuclear_weapon Numerical reasoning | Temporal reasoning ['https://en.wikipedia.org/wiki/List_of_presidents_of_the_United_States', 'https://en.wikipedia.org/wiki/Herbert_Hoover', 'https://en.wikipedia.org/wiki/Nuclear_weapon'] 355 373 What is the total number of pages in the first two first-edition books in the Emperyan Series by Rebecca Yarros? 1135 https://en.wikipedia.org/wiki/Fourth_Wing https://en.wikipedia.org/wiki/Iron_Flame Numerical reasoning | Multiple constraints ['https://en.wikipedia.org/wiki/Fourth_Wing', 'https://en.wikipedia.org/wiki/Iron_Flame'] 356 375 As of August 3, 2024, excluding white, what colour features on the flag of the city that houses the racecourse where Danon The Kid made his racing debut? Red https://en.wikipedia.org/wiki/Danon_The_Kid https://en.wikipedia.org/wiki/Hanshin_Racecourse https://en.wikipedia.org/wiki/Takarazuka,_Hy%C5%8Dgo Multiple constraints ['https://en.wikipedia.org/wiki/Danon_The_Kid', 'https://en.wikipedia.org/wiki/Hanshin_Racecourse', 'https://en.wikipedia.org/wiki/Takarazuka,_Hy%C5%8Dgo'] 357 376 What was the first skyscraper built in this player's hometown before he became the 160th pick of the 1980's NFL Draft? The Life & Casualty Tower https://en.wikipedia.org/wiki/1980_NFL_draft https://en.wikipedia.org/wiki/Preston_Brown_(wide_receiver) https://en.wikipedia.org/wiki/Nashville,_Tennessee https://en.wikipedia.org/wiki/Life_%26_Casualty_Tower Tabular reasoning | Temporal reasoning ['https://en.wikipedia.org/wiki/1980_NFL_draft', 'https://en.wikipedia.org/wiki/Preston_Brown_(wide_receiver)', 'https://en.wikipedia.org/wiki/Nashville,_Tennessee', 'https://en.wikipedia.org/wiki/Life_%26_Casualty_Tower'] 358 377 Regarding the award that the man who created the initial sketch for the Eiffel Tower received, how many of the award models have hanging devices that are in the shape of a crown? Eight https://en.wikipedia.org/wiki/Eiffel_Tower https://en.wikipedia.org/wiki/Maurice_Koechlin https://en.wikipedia.org/wiki/Legion_of_Honour#Legal_status_and_leadership Numerical reasoning | Tabular reasoning ['https://en.wikipedia.org/wiki/Eiffel_Tower', 'https://en.wikipedia.org/wiki/Maurice_Koechlin', 'https://en.wikipedia.org/wiki/Legion_of_Honour#Legal_status_and_leadership'] 359 378 Excluding blue, what other colour appears on the 2004 flag of the region in which Alastair Galbraith's home city is situated? Yellow https://en.wikipedia.org/wiki/Alastair_Galbraith https://en.wikipedia.org/wiki/Dunedin https://en.wikipedia.org/wiki/Otago Multiple constraints ['https://en.wikipedia.org/wiki/Alastair_Galbraith', 'https://en.wikipedia.org/wiki/Dunedin', 'https://en.wikipedia.org/wiki/Otago'] 360 379 When the San Francisco 49ers and San Diego Chargers met in Super Bowl XXIX, what was the #1 film at the box office in the U.S.? Legends of the Fall https://en.wikipedia.org/wiki/Super_Bowl_XXIX#:~:text=The%2049ers%20defeated%20the%20Chargers,ravaged%20the%20city%20in%201992. https://en.wikipedia.org/wiki/List_of_1995_box_office_number-one_films_in_the_United_States Multiple constraints ['https://en.wikipedia.org/wiki/Super_Bowl_XXIX#:~:text=The%2049ers%20defeated%20the%20Chargers,ravaged%20the%20city%20in%201992.', 'https://en.wikipedia.org/wiki/List_of_1995_box_office_number-one_films_in_the_United_States'] 361 380 What was the difference in original sale price between Tom Thomson’s “Northern River” and “Northern Lake” The difference in price was $250. https://en.wikipedia.org/wiki/Tom_Thomson https://en.wikipedia.org/wiki/Northern_River_(painting) Numerical reasoning ['https://en.wikipedia.org/wiki/Tom_Thomson', 'https://en.wikipedia.org/wiki/Northern_River_(painting)'] 362 381 What NBA team was founded three years before the Apollo 11 crew landed on the moon? The Chicago Bulls https://en.wikipedia.org/wiki/Moon_landing https://en.wikipedia.org/wiki/National_Basketball_Association Numerical reasoning | Temporal reasoning ['https://en.wikipedia.org/wiki/Moon_landing', 'https://en.wikipedia.org/wiki/National_Basketball_Association'] 363 382 How many more career home runs did the MLB player who had the highest slugging percentage in 1954 have than the player who was the the first African American to play in Major League Baseball? 519 https://en.wikipedia.org/wiki/List_of_Major_League_Baseball_titles_leaders https://en.wikipedia.org/wiki/Willie_Mays# https://en.wikipedia.org/wiki/Jackie_Robinson# Numerical reasoning | Tabular reasoning | Multiple constraints ['https://en.wikipedia.org/wiki/List_of_Major_League_Baseball_titles_leaders', 'https://en.wikipedia.org/wiki/Willie_Mays#', 'https://en.wikipedia.org/wiki/Jackie_Robinson#'] 364 383 What are the coordinates for the beach outside the UK that shares its name to a Welsh town that is home to the Great Orme? 34°0′37″S 18°20′34″E https://en.wikipedia.org/wiki/Great_Orme https://en.wikipedia.org/wiki/Llandudno https://en.wikipedia.org/wiki/Llandudno,_Western_Cape Tabular reasoning ['https://en.wikipedia.org/wiki/Great_Orme', 'https://en.wikipedia.org/wiki/Llandudno', 'https://en.wikipedia.org/wiki/Llandudno,_Western_Cape'] 365 384 I'm thinking of the Weird Al Yankovic parody of American Pie, what Weird Al album did it first appear on? Running with Scissors https://en.wikipedia.org/wiki/American_Pie_(song) https://en.wikipedia.org/wiki/The_Saga_Begins Multiple constraints ['https://en.wikipedia.org/wiki/American_Pie_(song)', 'https://en.wikipedia.org/wiki/The_Saga_Begins'] 366 385 What year was the band leader of the group who originally performed the song sampled in Kayne West's song Power born? 1946 https://en.wikipedia.org/wiki/My_Beautiful_Dark_Twisted_Fantasy https://en.wikipedia.org/wiki/King_Crimson https://en.wikipedia.org/wiki/Robert_Fripp Multiple constraints ['https://en.wikipedia.org/wiki/My_Beautiful_Dark_Twisted_Fantasy', 'https://en.wikipedia.org/wiki/King_Crimson', 'https://en.wikipedia.org/wiki/Robert_Fripp'] 367 386 Frank Jevons was the 9th Master of Hatfield College. His death came how many years after the death of his predecessor in the same role? 5 years https://en.wikipedia.org/wiki/Hatfield_College,_Durham https://en.wikipedia.org/wiki/Frank_Jevons https://en.wikipedia.org/wiki/Archibald_Robertson_(bishop) Numerical reasoning ['https://en.wikipedia.org/wiki/Hatfield_College,_Durham', 'https://en.wikipedia.org/wiki/Frank_Jevons', 'https://en.wikipedia.org/wiki/Archibald_Robertson_(bishop)'] 368 387 Renowned Abstract Expressionist painter Clyfford Still graduated from college in 1933. As of August 1, 2024, what other alumnus from that university was nominated for a Pulitzer Prize? Don Magnuson https://en.wikipedia.org/wiki/Clyfford_Still https://en.wikipedia.org/wiki/Spokane_University#Notable_alumni https://en.wikipedia.org/wiki/Don_Magnuson Multiple constraints ['https://en.wikipedia.org/wiki/Clyfford_Still', 'https://en.wikipedia.org/wiki/Spokane_University#Notable_alumni', 'https://en.wikipedia.org/wiki/Don_Magnuson'] 369 388 What is the the sum of the ages of the men who were executed by firing squad in Kilmainham Gaol, on the 12th of May, during the same year as the Battle of the Somme, when the died? 80 https://en.wikipedia.org/wiki/Battle_of_the_Somme https://en.wikipedia.org/wiki/1916 https://en.wikipedia.org/wiki/James_Connolly https://en.wikipedia.org/wiki/Se%C3%A1n_Mac_Diarmada Numerical reasoning | Multiple constraints ['https://en.wikipedia.org/wiki/Battle_of_the_Somme', 'https://en.wikipedia.org/wiki/1916', 'https://en.wikipedia.org/wiki/James_Connolly', 'https://en.wikipedia.org/wiki/Se%C3%A1n_Mac_Diarmada'] 370 390 "When the maker of the third-party console title ""ActRaiser"", merged with the makers of the console title ""Chrono Trigger"", what percentage of the company did the makers of ""ActRaiser"" make up?" 20% of the company. https://en.wikipedia.org/wiki/Square_Enix https://en.wikipedia.org/wiki/Enix https://en.wikipedia.org/wiki/Chrono_Trigger Numerical reasoning | Multiple constraints ['https://en.wikipedia.org/wiki/Square_Enix', 'https://en.wikipedia.org/wiki/Enix', 'https://en.wikipedia.org/wiki/Chrono_Trigger'] 371 391 How many years apart were the start of the Haitian and French revolutions , how many years longer did the Haitian Revolution last than the French Revolution, and how old were their respective leaders when each revolution began? The French Revolution began 2 years before the Haitian Revolution. The Haitian Revolution lasted 2 years longer. Louis XVI was 34 years old when the French Revolution began and 36 when the Haitian Revolution began. https://en.wikipedia.org/wiki/French_Revolution https://en.wikipedia.org/wiki/Haitian_Revolution https://en.wikipedia.org/wiki/Louis_XVI# Numerical reasoning | Temporal reasoning ['https://en.wikipedia.org/wiki/French_Revolution', 'https://en.wikipedia.org/wiki/Haitian_Revolution', 'https://en.wikipedia.org/wiki/Louis_XVI#'] 372 392 Are mulberries more related to raspberries or cannabis? Cannabis https://en.wikipedia.org/wiki/Morus_(plant) https://en.wikipedia.org/wiki/Rosales https://en.wikipedia.org/wiki/Cannabaceae https://en.wikipedia.org/wiki/Rosaceae Tabular reasoning ['https://en.wikipedia.org/wiki/Morus_(plant)', 'https://en.wikipedia.org/wiki/Rosales', 'https://en.wikipedia.org/wiki/Cannabaceae', 'https://en.wikipedia.org/wiki/Rosaceae'] 373 393 There is a speech-sound disorder, not caused by a structural abnormality, for which one symptom is rearranging sounds of a word. The term for the disorder was first defined in 1908 by a German neuroscientist with the first name Hugo. Why was the Nobel Peace Prize given to an American the same year Hugo died? For his crucial role in bringing about the Dawes Plan. https://en.wikipedia.org/wiki/Speech_disorder https://en.wikipedia.org/wiki/Apraxia_of_speech https://en.wikipedia.org/wiki/Hugo_Liepmann https://en.wikipedia.org/wiki/List_of_Nobel_Peace_Prize_laureates Tabular reasoning | Multiple constraints ['https://en.wikipedia.org/wiki/Speech_disorder', 'https://en.wikipedia.org/wiki/Apraxia_of_speech', 'https://en.wikipedia.org/wiki/Hugo_Liepmann', 'https://en.wikipedia.org/wiki/List_of_Nobel_Peace_Prize_laureates'] 374 394 How old was Russian vodka tycoon Yuri Shefler when Serene, the yacht he commissioned, was delivered to him? 43 https://en.wikipedia.org/wiki/Serene_(yacht) https://en.wikipedia.org/wiki/Yuri_Shefler Numerical reasoning ['https://en.wikipedia.org/wiki/Serene_(yacht)', 'https://en.wikipedia.org/wiki/Yuri_Shefler'] 375 395 Who won the World Series the same year that Andy Roddick won his only Grand Slam title in 2003? The Florida Marlins https://en.wikipedia.org/wiki/Andy_Roddick_career_statistics#Singles:_5_finals_(1–4) https://en.wikipedia.org/wiki/2003_World_Series Tabular reasoning ['https://en.wikipedia.org/wiki/Andy_Roddick_career_statistics#Singles:_5_finals_(1–4)', 'https://en.wikipedia.org/wiki/2003_World_Series'] 376 396 What was the name of the work written by Louis Pierre Vieillot published two years after he described the great blue turao as Musophaga cristata? Ornithologie https://en.wikipedia.org/wiki/Great_blue_turaco https://en.wikipedia.org/wiki/Louis_Pierre_Vieillot Multiple constraints | Temporal reasoning ['https://en.wikipedia.org/wiki/Great_blue_turaco', 'https://en.wikipedia.org/wiki/Louis_Pierre_Vieillot'] 377 397 What military awards were received by the General originally scheduled to lead Operation Torch before Lieutenant General Dwight D. Eisenhower was given command of the operation? Joseph Stilwell was originally scheduled to lead Operation Torch before Lieutenant General Dwight D. Eisenhower was given command of the operation. Stillwell received a Distinguished Service Cross, two Army Distinguished Service Medals, a Legion of Merit award, and a Bronze Star during his military career. https://en.wikipedia.org/wiki/Operation_Torch https://en.wikipedia.org/wiki/Joseph_Stilwell Multiple constraints ['https://en.wikipedia.org/wiki/Operation_Torch', 'https://en.wikipedia.org/wiki/Joseph_Stilwell'] 378 399 From United States Former President Bill Clinton through Former President Donald Trump, which president in this time period has published the greatest number of books as of August 1, 2024? Please exclude from that count any books you find that are authored by someone else about a particular former president. Former President Bill Clinton has published seven books through 2024. https://en.wikipedia.org/wiki/Bill_Clinton https://en.wikipedia.org/wiki/George_W._Bush https://en.wikipedia.org/wiki/Barack_Obama https://en.wikipedia.org/wiki/Donald_Trump Numerical reasoning | Multiple constraints ['https://en.wikipedia.org/wiki/Bill_Clinton', 'https://en.wikipedia.org/wiki/George_W._Bush', 'https://en.wikipedia.org/wiki/Barack_Obama', 'https://en.wikipedia.org/wiki/Donald_Trump'] 379 400 What US president was born in the same year that the Treaty of Resht was signed? George Washington https://en.wikipedia.org/wiki/Treaty_of_Resht https://en.wikipedia.org/wiki/List_of_presidents_of_the_United_States Tabular reasoning | Temporal reasoning ['https://en.wikipedia.org/wiki/Treaty_of_Resht', 'https://en.wikipedia.org/wiki/List_of_presidents_of_the_United_States'] 380 401 Which horse won the Kentucky Derby during the same calendar year in which John Hinckley Jr. attempted to assassinate U.S. President Ronald Reagan? Pleasant Colony https://en.wikipedia.org/wiki/Attempted_assassination_of_Ronald_Reagan https://en.wikipedia.org/wiki/Kentucky_Derby Multiple constraints ['https://en.wikipedia.org/wiki/Attempted_assassination_of_Ronald_Reagan', 'https://en.wikipedia.org/wiki/Kentucky_Derby'] 381 402 Milton Friedman won the Nobel Prize for Economics in 1976. What was the name of the Nobel Peace Prize winning wife of the economist who won the Nobel Prize for Economics two years before Friedman did? Alva Myrdal https://en.wikipedia.org/wiki/Milton_Friedman https://en.wikipedia.org/wiki/List_of_Nobel_Memorial_Prize_laureates_in_Economic_Sciences https://en.wikipedia.org/wiki/Gunnar_Myrdal Tabular reasoning | Temporal reasoning ['https://en.wikipedia.org/wiki/Milton_Friedman', 'https://en.wikipedia.org/wiki/List_of_Nobel_Memorial_Prize_laureates_in_Economic_Sciences', 'https://en.wikipedia.org/wiki/Gunnar_Myrdal'] 382 404 When Justin Trudeau was elected as Prime Minister of Canada, who was the current Prime Minister of France? Manuel Valls https://en.wikipedia.org/wiki/Justin_Trudeau https://en.wikipedia.org/wiki/List_of_prime_ministers_of_France Multiple constraints ['https://en.wikipedia.org/wiki/Justin_Trudeau', 'https://en.wikipedia.org/wiki/List_of_prime_ministers_of_France'] 383 405 This town was the original location of the tallest Christmas Tree displayed at the Rockefeller Center. This was also home to the author of a famous children's book series. What is the name of this series? Doctor Dolittle https://en.wikipedia.org/wiki/Rockefeller_Center_Christmas_Tree https://en.wikipedia.org/wiki/Killingworth,_Connecticut https://en.wikipedia.org/wiki/Hugh_Lofting https://en.wikipedia.org/wiki/Doctor_Dolittle Multiple constraints ['https://en.wikipedia.org/wiki/Rockefeller_Center_Christmas_Tree', 'https://en.wikipedia.org/wiki/Killingworth,_Connecticut', 'https://en.wikipedia.org/wiki/Hugh_Lofting', 'https://en.wikipedia.org/wiki/Doctor_Dolittle'] 384 406 "Houston, Texas had dozens of airports as of January 1, 2024. Find the three-letter IATA code of the airport in which the longest runway was exactly 7000 feet long, and rearrange those letters to match that of another airport at that time. Here are your hints: The last letter of the re-arranged code is ""J"". The new airport was located in China. With this information, what was the IATA code of this airport?" LNJ https://en.wikipedia.org/wiki/List_of_airports_in_the_Greater_Houston_Area https://en.wikipedia.org/wiki/Lists_of_airports#By_code https://en.wikipedia.org/wiki/List_of_airports_by_IATA_airport_code:_L Multiple constraints ['https://en.wikipedia.org/wiki/List_of_airports_in_the_Greater_Houston_Area', 'https://en.wikipedia.org/wiki/Lists_of_airports#By_code', 'https://en.wikipedia.org/wiki/List_of_airports_by_IATA_airport_code:_L'] 385 407 How many years earlier did Wimbledon start compared to the birthdate of the winner of the 2019 tournament. 110 years https://en.m.wikipedia.org/wiki/Wimbledon_Championships https://en.m.wikipedia.org/wiki/List_of_Wimbledon_gentlemen%27s_singles_champions https://en.m.wikipedia.org/wiki/Novak_Djokovic Numerical reasoning ['https://en.m.wikipedia.org/wiki/Wimbledon_Championships', 'https://en.m.wikipedia.org/wiki/List_of_Wimbledon_gentlemen%27s_singles_champions', 'https://en.m.wikipedia.org/wiki/Novak_Djokovic'] 386 409 The starship USS Enterprise has a registry number containing the founding year of what Ivy League university? Yale University https://en.wikipedia.org/wiki/USS_Enterprise_(NCC-1701) https://en.wikipedia.org/wiki/Ivy_League Tabular reasoning | Multiple constraints ['https://en.wikipedia.org/wiki/USS_Enterprise_(NCC-1701)', 'https://en.wikipedia.org/wiki/Ivy_League'] 387 410 What two cities hosted the Summer Olympic Games between when the television shows featuring characters Olivia Benson and Meredith Grey started airing? Sydney, Australia, and Athens, Greece https://en.wikipedia.org/wiki/Olivia_Benson https://en.wikipedia.org/wiki/Meredith_Grey https://en.wikipedia.org/wiki/List_of_Olympic_Games_host_cities Multiple constraints ['https://en.wikipedia.org/wiki/Olivia_Benson', 'https://en.wikipedia.org/wiki/Meredith_Grey', 'https://en.wikipedia.org/wiki/List_of_Olympic_Games_host_cities'] 388 411 Who had their twenty-first number one hit on the US Billboard Hot Country Songs chart the same week Carly Rae Jepsen hit #39 on the Australia ARIA Top 50 Singles chart? Kenny Chesney https://en.wikipedia.org/wiki/Call_Me_Maybe#Year-end_charts https://en.wikipedia.org/wiki/List_of_Billboard_number-one_country_songs_of_2012 https://en.wikipedia.org/wiki/Reality_(Kenny_Chesney_song) Numerical reasoning | Tabular reasoning | Multiple constraints | Temporal reasoning ['https://en.wikipedia.org/wiki/Call_Me_Maybe#Year-end_charts', 'https://en.wikipedia.org/wiki/List_of_Billboard_number-one_country_songs_of_2012', 'https://en.wikipedia.org/wiki/Reality_(Kenny_Chesney_song)'] 389 413 What was the word that featured the least in the longest recorded quotation by the chimpanzee named after the recipient of the 2011 Sydney Peace Prize? You https://en.wikipedia.org/wiki/Sydney_Peace_Prize https://en.wikipedia.org/wiki/Noam_Chomsky#In_academia https://en.wikipedia.org/wiki/Nim_Chimpsky#Quotations Numerical reasoning | Multiple constraints ['https://en.wikipedia.org/wiki/Sydney_Peace_Prize', 'https://en.wikipedia.org/wiki/Noam_Chomsky#In_academia', 'https://en.wikipedia.org/wiki/Nim_Chimpsky#Quotations'] 390 414 What song was #1 on Billboard's Hot 100 for the most days during the Cuban Missile Crisis? Monster Mash' by Bobby 'Boris' Pickett & the Crypt-Kickers https://en.wikipedia.org/wiki/Cuban_Missile_Crisis https://en.wikipedia.org/wiki/List_of_Billboard_Hot_100_number_ones_of_1962 Numerical reasoning | Tabular reasoning | Multiple constraints ['https://en.wikipedia.org/wiki/Cuban_Missile_Crisis', 'https://en.wikipedia.org/wiki/List_of_Billboard_Hot_100_number_ones_of_1962'] 391 415 Which ancient archaeoastronomical site in the U.K. is also associated with the summer solstice during which time light illuminates a quartz-rich stone in the chamber. Bryn Celli Ddu https://en.wikipedia.org/wiki/Summer_solstice https://en.wikipedia.org/wiki/List_of_archaeoastronomical_sites_by_country https://en.wikipedia.org/wiki/Bryn_Celli_Ddu Multiple constraints ['https://en.wikipedia.org/wiki/Summer_solstice', 'https://en.wikipedia.org/wiki/List_of_archaeoastronomical_sites_by_country', 'https://en.wikipedia.org/wiki/Bryn_Celli_Ddu'] 392 416 What is the price difference of an iPhone (8GB) from when it was first released compared to the price of the iPhone X when it was released? $400 https://en.wikipedia.org/wiki/IPhone_(1st_generation) https://en.wikipedia.org/wiki/IPhone_X#:~:text=The%20iPhone%20X%20(Roman%20numeral,released%20on%20November%203%2C%202017. Numerical reasoning ['https://en.wikipedia.org/wiki/IPhone_(1st_generation)', 'https://en.wikipedia.org/wiki/IPhone_X#:~:text=The%20iPhone%20X%20(Roman%20numeral,released%20on%20November%203%2C%202017.'] 393 417 What's the star sign of the author of A Court of Thorns and Roses? Pisces https://en.wikipedia.org/wiki/A_Court_of_Thorns_and_Roses https://en.wikipedia.org/wiki/Sarah_J._Maas https://en.wikipedia.org/wiki/Astrological_sign Temporal reasoning ['https://en.wikipedia.org/wiki/A_Court_of_Thorns_and_Roses', 'https://en.wikipedia.org/wiki/Sarah_J._Maas', 'https://en.wikipedia.org/wiki/Astrological_sign'] 394 418 What was the average launch mass of Apollo 11, Apollo 12, and Apollo 13 in kilograms, rounded to the nearest integer? The average launch mass in kilograms rounded to the nearest integer of Apollo 11, Apollo 12, and Apollo 13 is 47,906 kilograms. https://en.wikipedia.org/wiki/Apollo_11 https://en.wikipedia.org/wiki/Apollo_12 https://en.wikipedia.org/wiki/Apollo_13 Numerical reasoning ['https://en.wikipedia.org/wiki/Apollo_11', 'https://en.wikipedia.org/wiki/Apollo_12', 'https://en.wikipedia.org/wiki/Apollo_13'] 395 419 What was the second starring role of the actress who won an Oscar for portraying union activist Cyrstal Lee Sutton? The Flying Nun https://en.wikipedia.org/wiki/Crystal_Lee_Sutton https://en.wikipedia.org/wiki/Sally_Field Temporal reasoning ['https://en.wikipedia.org/wiki/Crystal_Lee_Sutton', 'https://en.wikipedia.org/wiki/Sally_Field'] 396 420 What is the scientific name of an amphibian that is listed as endangered by the Canadian government (as of 2024), and its only population in Canada occurs on an island which is the southernmost inhabited part of Canada? Ambystoma texanum https://en.wikipedia.org/wiki/List_of_Wildlife_Species_at_Risk_(Canada)#Amphibians https://en.wikipedia.org/wiki/Small-mouth_salamander https://en.wikipedia.org/wiki/Pelee,_Ontario Tabular reasoning | Multiple constraints ['https://en.wikipedia.org/wiki/List_of_Wildlife_Species_at_Risk_(Canada)#Amphibians', 'https://en.wikipedia.org/wiki/Small-mouth_salamander', 'https://en.wikipedia.org/wiki/Pelee,_Ontario'] 397 421 What were the names of the parents of the first overall pick in the 2007 NHL entry draft? Donna and Patrick were the names of Patrick Kane's parents. https://en.wikipedia.org/wiki/2007_NHL_entry_draft#Round_one https://en.wikipedia.org/wiki/Patrick_Kane#Early_life Tabular reasoning ['https://en.wikipedia.org/wiki/2007_NHL_entry_draft#Round_one', 'https://en.wikipedia.org/wiki/Patrick_Kane#Early_life'] 398 422 Of the counties that Wisconsin Highway 34 runs through, what is the seat of the most populous county based on 2020 census data? Wausau https://en.wikipedia.org/wiki/Wisconsin_Highway_34 https://en.wikipedia.org/wiki/Wood_County,_Wisconsin https://en.wikipedia.org/wiki/Portage_County,_Wisconsin https://en.wikipedia.org/wiki/Marathon_County,_Wisconsin Multiple constraints ['https://en.wikipedia.org/wiki/Wisconsin_Highway_34', 'https://en.wikipedia.org/wiki/Wood_County,_Wisconsin', 'https://en.wikipedia.org/wiki/Portage_County,_Wisconsin', 'https://en.wikipedia.org/wiki/Marathon_County,_Wisconsin'] 399 423 Emma Lazarus's most famous poem inspired the founding of an order of nursing nuns. What disease does this order specialize in treating? Cancer https://en.wikipedia.org/wiki/Emma_Lazarus https://en.wikipedia.org/wiki/Dominican_Sisters_of_Hawthorne Multiple constraints ['https://en.wikipedia.org/wiki/Emma_Lazarus', 'https://en.wikipedia.org/wiki/Dominican_Sisters_of_Hawthorne'] 400 425 How many times did the victor of the Immortal Game of 1851 lose a chess match to an American-born opponent? 2 https://en.wikipedia.org/wiki/Immortal_Game https://en.wikipedia.org/wiki/Adolf_Anderssen https://en.wikipedia.org/wiki/Paul_Morphy Tabular reasoning | Multiple constraints ['https://en.wikipedia.org/wiki/Immortal_Game', 'https://en.wikipedia.org/wiki/Adolf_Anderssen', 'https://en.wikipedia.org/wiki/Paul_Morphy'] 401 426 "How many years were between the publication of a book considered 'one of the seminal works of fiction of the 20th century', and the Japanese release of the retail version of the game ""Resident Evil: Revelations 2"" for the PS3, of which the author of the 20th century book was a great inspiration for the plot?" 100 https://en.wikipedia.org/wiki/Resident_Evil:_Revelations_2# https://en.wikipedia.org/wiki/Franz_Kafka#Stories Numerical reasoning | Post processing ['https://en.wikipedia.org/wiki/Resident_Evil:_Revelations_2#', 'https://en.wikipedia.org/wiki/Franz_Kafka#Stories'] 402 427 The actress who played Aunt Rose in A Brooklyn State of Mind (1997) also starred in a mafia movie where she sang a popular Sicilian song. How many years later did her version of the song occur after the earliest recording? 45 years https://en.wikipedia.org/wiki/A_Brooklyn_State_of_Mind#Cast https://en.wikipedia.org/wiki/Morgana_King#Film_debut https://en.wikipedia.org/wiki/C%27%C3%A8_la_luna_mezzo_mare#Notable_recordings https://en.wikipedia.org/wiki/The_Godfather Numerical reasoning | Multiple constraints | Post processing | Temporal reasoning ['https://en.wikipedia.org/wiki/A_Brooklyn_State_of_Mind#Cast', 'https://en.wikipedia.org/wiki/Morgana_King#Film_debut', 'https://en.wikipedia.org/wiki/C%27%C3%A8_la_luna_mezzo_mare#Notable_recordings', 'https://en.wikipedia.org/wiki/The_Godfather'] 403 428 Put in chronological order the Major League Baseball seasons in which Barry Bonds, Tony Gwynn, and Benny Kauff hit for a .370 batting average. 1914, 1987, 2002 https://en.wikipedia.org/wiki/Barry_Bonds https://en.wikipedia.org/wiki/Tony_Gwynn https://en.wikipedia.org/wiki/Benny_Kauff Temporal reasoning ['https://en.wikipedia.org/wiki/Barry_Bonds', 'https://en.wikipedia.org/wiki/Tony_Gwynn', 'https://en.wikipedia.org/wiki/Benny_Kauff'] 404 429 What famous film maker once provided editorial assistance for a 90s documentary on Mongolian-Tuvan throat singing before directing a series of superhero movies? Christopher Nolan https://en.wikipedia.org/wiki/Tuvan_throat_singing https://en.wikipedia.org/wiki/Genghis_Blues https://en.wikipedia.org/wiki/Christopher_Nolan Multiple constraints ['https://en.wikipedia.org/wiki/Tuvan_throat_singing', 'https://en.wikipedia.org/wiki/Genghis_Blues', 'https://en.wikipedia.org/wiki/Christopher_Nolan'] 405 430 How many of the first 8 Harry Potter films based on the original 7 books were released in years when a United States presidential election took place, and what movies were they? One of the original 8 Harry Potter films based on the original 7 books coincided with a U.S. presidential election, and the film was Harry Potter and the Prisoner of Azkaban. https://en.wikipedia.org/wiki/Harry_Potter_(film_series) https://en.wikipedia.org/wiki/United_States_presidential_election Tabular reasoning | Temporal reasoning ['https://en.wikipedia.org/wiki/Harry_Potter_(film_series)', 'https://en.wikipedia.org/wiki/United_States_presidential_election'] 406 431 In which year did the 4th Sheriff of Yorkshire to be part of the House of Plantagenet die? 1190 https://en.wikipedia.org/wiki/Sheriff_of_Yorkshire https://en.wikipedia.org/wiki/Ranulf_de_Glanvill Multiple constraints | Temporal reasoning ['https://en.wikipedia.org/wiki/Sheriff_of_Yorkshire', 'https://en.wikipedia.org/wiki/Ranulf_de_Glanvill'] 407 432 How many more medals did France win in the 2008 Summer Olympics than in the 2004 Summer Olympics? 10 https://en.wikipedia.org/wiki/2004_Summer_Olympics https://en.wikipedia.org/wiki/2008_Summer_Olympics Numerical reasoning | Tabular reasoning ['https://en.wikipedia.org/wiki/2004_Summer_Olympics', 'https://en.wikipedia.org/wiki/2008_Summer_Olympics'] 408 433 How many colours are on the flag of the country whose capital is the southernmost by latitude out of all landlocked countries, as of 2024? What are the names of these colours? 4 - blue, white, green, black https://en.wikipedia.org/wiki/List_of_national_capitals_by_latitude https://en.wikipedia.org/wiki/Landlocked_country https://en.wikipedia.org/wiki/Flag_of_Lesotho Numerical reasoning | Tabular reasoning | Multiple constraints ['https://en.wikipedia.org/wiki/List_of_national_capitals_by_latitude', 'https://en.wikipedia.org/wiki/Landlocked_country', 'https://en.wikipedia.org/wiki/Flag_of_Lesotho'] 409 434 This 90s rock musical depiction of La bohème, had this music, lyrics and story written by a talent who passed the day before their Off-Broadway preview performance. Aside from Broadway, what was the name of his Netflix success? Tick, Tick... Boom! https://en.wikipedia.org/wiki/La_boh%C3%A8me https://en.wikipedia.org/wiki/Rent_(musical) https://en.wikipedia.org/wiki/Jonathan_Larson https://en.wikipedia.org/wiki/Tick,_Tick..._Boom! Multiple constraints | Post processing | Temporal reasoning ['https://en.wikipedia.org/wiki/La_boh%C3%A8me', 'https://en.wikipedia.org/wiki/Rent_(musical)', 'https://en.wikipedia.org/wiki/Jonathan_Larson', 'https://en.wikipedia.org/wiki/Tick,_Tick..._Boom!'] 410 435 What German-born Author had books published in 1995, 1999, & 2005 detailing their studies and activism with what Tang's animal mascot of the time? Birutė Galdikas https://en.wikipedia.org/wiki/Tang_(drink_mix) https://en.wikipedia.org/wiki/Orangutan https://en.wikipedia.org/wiki/Birut%C4%97_Galdikas Temporal reasoning ['https://en.wikipedia.org/wiki/Tang_(drink_mix)', 'https://en.wikipedia.org/wiki/Orangutan', 'https://en.wikipedia.org/wiki/Birut%C4%97_Galdikas'] 411 436 This author won the Popular Fiction Book of the Year award in 2009 at the Irish Book Awards. What is their astrological sign? Virgo https://en.wikipedia.org/wiki/Irish_Book_Awards https://en.wikipedia.org/wiki/Marian_Keyes https://en.wikipedia.org/wiki/Astrological_sign Tabular reasoning | Multiple constraints | Temporal reasoning ['https://en.wikipedia.org/wiki/Irish_Book_Awards', 'https://en.wikipedia.org/wiki/Marian_Keyes', 'https://en.wikipedia.org/wiki/Astrological_sign'] 412 437 What is the name of the 2nd track on the album by Corinne Bailey Rae that came out 10 years before her album The Heart Speaks in Whispers? Enchantment https://en.wikipedia.org/wiki/Corinne_Bailey_Rae_discography https://en.wikipedia.org/wiki/Corinne_Bailey_Rae_(album) Multiple constraints | Temporal reasoning ['https://en.wikipedia.org/wiki/Corinne_Bailey_Rae_discography', 'https://en.wikipedia.org/wiki/Corinne_Bailey_Rae_(album)'] 413 438 Which of the following albums came out on a date closest to the date that Nancy Kerrigan was assaulted? What about which album was closest to the date of Harding's plea deal? Awake by Dream Theater Inside Out by Fates Warning Promised Land by Queensryche Dreamspace by Stratovarius Dreamspace' by Stratovarius. https://en.wikipedia.org/wiki/Assault_of_Nancy_Kerrigan https://en.wikipedia.org/wiki/Awake_(Dream_Theater_album) https://en.wikipedia.org/wiki/Inside_Out_(Fates_Warning_album) https://en.wikipedia.org/wiki/Promised_Land_(Queensrÿche_album) https://en.wikipedia.org/wiki/Dreamspace Post processing | Temporal reasoning ['https://en.wikipedia.org/wiki/Assault_of_Nancy_Kerrigan', 'https://en.wikipedia.org/wiki/Awake_(Dream_Theater_album)', 'https://en.wikipedia.org/wiki/Inside_Out_(Fates_Warning_album)', 'https://en.wikipedia.org/wiki/Promised_Land_(Queensrÿche_album)', 'https://en.wikipedia.org/wiki/Dreamspace'] 414 439 Which U.S. National Park was the first to be established after the Portland Trail Blazers won their only playoff series with LaMarcus Aldridge on the team? Gateway Arch National Park https://en.wikipedia.org/wiki/LaMarcus_Aldridge#2013%E2%80%9314_season https://en.wikipedia.org/wiki/2013%E2%80%9314_Portland_Trail_Blazers_season#Playoffs https://en.wikipedia.org/wiki/List_of_national_parks_of_the_United_States Temporal reasoning ['https://en.wikipedia.org/wiki/LaMarcus_Aldridge#2013%E2%80%9314_season', 'https://en.wikipedia.org/wiki/2013%E2%80%9314_Portland_Trail_Blazers_season#Playoffs', 'https://en.wikipedia.org/wiki/List_of_national_parks_of_the_United_States'] 415 440 As of August 3rd, 2024, which Moose Jaw Warrior with a retired number was born on May 29, 1967? Mike Keane https://en.wikipedia.org/wiki/Moose_Jaw_Warriors https://en.wikipedia.org/wiki/Mike_Keane https://en.wikipedia.org/wiki/Theoren_Fleury https://en.wikipedia.org/wiki/Kelly_Buchberger https://en.wikipedia.org/wiki/Ryan_Smyth Post processing | Temporal reasoning ['https://en.wikipedia.org/wiki/Moose_Jaw_Warriors', 'https://en.wikipedia.org/wiki/Mike_Keane', 'https://en.wikipedia.org/wiki/Theoren_Fleury', 'https://en.wikipedia.org/wiki/Kelly_Buchberger', 'https://en.wikipedia.org/wiki/Ryan_Smyth'] 416 441 As of July 2024, which protagonist of a 'shonen jump' series shares a name with a station on a West Japan Railway Company regional line? Light Yagami https://en.wikipedia.org/wiki/West_Japan_Railway_Company https://en.wikipedia.org/wiki/Geibi_Line https://en.wikipedia.org/wiki/Weekly_Sh%C5%8Dnen_Jump https://en.wikipedia.org/wiki/Death_Note Tabular reasoning | Multiple constraints ['https://en.wikipedia.org/wiki/West_Japan_Railway_Company', 'https://en.wikipedia.org/wiki/Geibi_Line', 'https://en.wikipedia.org/wiki/Weekly_Sh%C5%8Dnen_Jump', 'https://en.wikipedia.org/wiki/Death_Note'] 417 442 "What's the fifth song on the fifth album of the pop singer who was parodied in the fifth song on ""Weird Al"" Yankovic's fifth album?" Keep Walking https://en.wikipedia.org/wiki/%22Weird_Al%22_Yankovic https://en.wikipedia.org/wiki/Even_Worse https://en.wikipedia.org/wiki/Tiffany_Darwish https://en.wikipedia.org/wiki/The_Color_of_Silence Multiple constraints ['https://en.wikipedia.org/wiki/%22Weird_Al%22_Yankovic', 'https://en.wikipedia.org/wiki/Even_Worse', 'https://en.wikipedia.org/wiki/Tiffany_Darwish', 'https://en.wikipedia.org/wiki/The_Color_of_Silence'] 418 443 I'm thinking of a painting. It was done by the same man who painted The Anti-Slavery Society Convention in the 1840's. The painting is about an election. Can you tell me the name of it? Mock Election https://en.wikipedia.org/wiki/World_Anti-Slavery_Convention https://en.wikipedia.org/wiki/Benjamin_Haydon Multiple constraints ['https://en.wikipedia.org/wiki/World_Anti-Slavery_Convention', 'https://en.wikipedia.org/wiki/Benjamin_Haydon'] 419 444 In what year was the former South Korean prime minister who is from the same clan as the oldest member of the band BTS born? 1948 https://en.wikipedia.org/wiki/BTS https://en.wikipedia.org/wiki/Jin_(singer) https://en.wikipedia.org/wiki/Gwangsan_Kim_clan Multiple constraints ['https://en.wikipedia.org/wiki/BTS', 'https://en.wikipedia.org/wiki/Jin_(singer)', 'https://en.wikipedia.org/wiki/Gwangsan_Kim_clan'] 420 445 What is the birth year of the American President who once pet the cat who was buried at the Hagia Sofia in 2020? 1961 https://en.wikipedia.org/wiki/Hagia_Sophia https://en.wikipedia.org/wiki/Gli https://en.wikipedia.org/wiki/Barack_Obama Multiple constraints ['https://en.wikipedia.org/wiki/Hagia_Sophia', 'https://en.wikipedia.org/wiki/Gli', 'https://en.wikipedia.org/wiki/Barack_Obama'] 421 446 There was a popular movie that came out in 2016 starring Emma Stone and Ryan Gosling, tell me where the director of this movie was born. Providence, Rhode Island https://en.wikipedia.org/wiki/La_La_Land https://en.wikipedia.org/wiki/Damien_Chazelle https://en.wikipedia.org/wiki/Ryan_Gosling https://en.wikipedia.org/wiki/Emma_Stone Multiple constraints ['https://en.wikipedia.org/wiki/La_La_Land', 'https://en.wikipedia.org/wiki/Damien_Chazelle', 'https://en.wikipedia.org/wiki/Ryan_Gosling', 'https://en.wikipedia.org/wiki/Emma_Stone'] 422 447 By what amount was the budget of Peter Jackson's King Kong higher than the budget of John Guillermin's version of King Kong? $183 million https://en.wikipedia.org/wiki/King_Kong_(franchise) https://en.wikipedia.org/wiki/King_Kong_(2005_film) https://en.wikipedia.org/wiki/King_Kong_(1976_film) Numerical reasoning | Multiple constraints ['https://en.wikipedia.org/wiki/King_Kong_(franchise)', 'https://en.wikipedia.org/wiki/King_Kong_(2005_film)', 'https://en.wikipedia.org/wiki/King_Kong_(1976_film)'] 423 448 What was the age difference at their deaths (in years) between Edgar Allan Poe and his rival, Rufus Wilmot Griswold, multiplied by 100? 200 https://en.wikipedia.org/wiki/Rufus_Wilmot_Griswold https://en.wikipedia.org/wiki/Edgar_Allan_Poe Numerical reasoning | Post processing ['https://en.wikipedia.org/wiki/Rufus_Wilmot_Griswold', 'https://en.wikipedia.org/wiki/Edgar_Allan_Poe'] 424 449 How old was the vice president to the fifth US president when he died? 50 https://en.wikipedia.org/wiki/List_of_presidents_of_the_United_States https://en.wikipedia.org/wiki/Daniel_D._Tompkins Multiple constraints ['https://en.wikipedia.org/wiki/List_of_presidents_of_the_United_States', 'https://en.wikipedia.org/wiki/Daniel_D._Tompkins'] 425 450 Who was the manager for the MLB World Series winning team the year that another team broke the record for the longest consecutive winning streak in a regular season? Base your answer on the following: -- The team who broke the record did so in the 2010s A. J. Hinch https://en.wikipedia.org/wiki/List_of_Major_League_Baseball_longest_winning_streaks https://en.wikipedia.org/wiki/2017_Cleveland_Indians_season https://en.wikipedia.org/wiki/2017_World_Series https://en.wikipedia.org/wiki/A._J._Hinch Tabular reasoning | Multiple constraints ['https://en.wikipedia.org/wiki/List_of_Major_League_Baseball_longest_winning_streaks', 'https://en.wikipedia.org/wiki/2017_Cleveland_Indians_season', 'https://en.wikipedia.org/wiki/2017_World_Series', 'https://en.wikipedia.org/wiki/A._J._Hinch'] 426 451 What award was won in 2003 by the Swiss architecture firm that designed Roche Tower? The Stirling Prize. https://en.wikipedia.org/wiki/Roche_Tower https://en.wikipedia.org/wiki/Herzog_%26_de_Meuron Temporal reasoning ['https://en.wikipedia.org/wiki/Roche_Tower', 'https://en.wikipedia.org/wiki/Herzog_%26_de_Meuron'] 427 452 How many New Zealanders have won Australian Idol during seasons 1 to 8? 1 https://en.wikipedia.org/wiki/Australian_Idol https://en.wikipedia.org/wiki/Guy_Sebastian https://en.wikipedia.org/wiki/Casey_Donovan_(singer) https://en.wikipedia.org/wiki/Kate_DeAraugo https://en.wikipedia.org/wiki/Damien_Leith https://en.wikipedia.org/wiki/Natalie_Gauci https://en.wikipedia.org/wiki/Wes_Carr https://en.wikipedia.org/wiki/Stan_Walker https://en.wikipedia.org/wiki/Royston_Sagigi-Baira Numerical reasoning | Multiple constraints ['https://en.wikipedia.org/wiki/Australian_Idol', 'https://en.wikipedia.org/wiki/Guy_Sebastian', 'https://en.wikipedia.org/wiki/Casey_Donovan_(singer)', 'https://en.wikipedia.org/wiki/Kate_DeAraugo', 'https://en.wikipedia.org/wiki/Damien_Leith', 'https://en.wikipedia.org/wiki/Natalie_Gauci', 'https://en.wikipedia.org/wiki/Wes_Carr', 'https://en.wikipedia.org/wiki/Stan_Walker', 'https://en.wikipedia.org/wiki/Royston_Sagigi-Baira'] 428 453 Who was the mayor of France's 25th President's hometown when they were first elected President? Brigitte Fouré https://en.wikipedia.org/wiki/List_of_presidents_of_France#Presidents_2 https://en.wikipedia.org/wiki/Emmanuel_Macron https://en.wikipedia.org/wiki/Amiens Multiple constraints ['https://en.wikipedia.org/wiki/List_of_presidents_of_France#Presidents_2', 'https://en.wikipedia.org/wiki/Emmanuel_Macron', 'https://en.wikipedia.org/wiki/Amiens'] 429 454 Of the Jason Statham movies that came out the year Dennis Hopper died, which could he have lived to see the premiere of? 13 https://en.wikipedia.org/wiki/Dennis_Hopper https://en.wikipedia.org/wiki/Jason_Statham https://en.wikipedia.org/wiki/13_(2010_film) https://en.wikipedia.org/wiki/The_Expendables_(2010_film) Temporal reasoning ['https://en.wikipedia.org/wiki/Dennis_Hopper', 'https://en.wikipedia.org/wiki/Jason_Statham', 'https://en.wikipedia.org/wiki/13_(2010_film)', 'https://en.wikipedia.org/wiki/The_Expendables_(2010_film)'] 430 455 Whose memoir was co-written with the author of Pill Head: The Secret Life of a Painkiller Addict and published post-humously 2 years after her death? Edith Windsor https://en.wikipedia.org/wiki/Edith_Windsor https://en.wikipedia.org/wiki/Joshua_Lyon Multiple constraints ['https://en.wikipedia.org/wiki/Edith_Windsor', 'https://en.wikipedia.org/wiki/Joshua_Lyon'] 431 456 What day of the year do John of Lancaster (Duke of Bedford), Fritz Koenig (German Sculptor), Edith Windsor (LGBT Activist), and Ulf Merbold (German Physicist and Astronaut) all have in common? They are all born on June 20. https://en.wikipedia.org/wiki/John_of_Lancaster,_Duke_of_Bedford https://en.wikipedia.org/wiki/Fritz_Koenig https://en.wikipedia.org/wiki/Edith_Windsor https://en.wikipedia.org/wiki/Ulf_Merbold Multiple constraints ['https://en.wikipedia.org/wiki/John_of_Lancaster,_Duke_of_Bedford', 'https://en.wikipedia.org/wiki/Fritz_Koenig', 'https://en.wikipedia.org/wiki/Edith_Windsor', 'https://en.wikipedia.org/wiki/Ulf_Merbold'] 432 457 In 2024's version of the world, which country was the birthplace of the Emperor who reigned from the year 363 to 364 over the Empire that the Goths played a major part in collapsing? Serbia https://en.wikipedia.org/wiki/Goths https://en.wikipedia.org/wiki/Western_Roman_Empire https://en.wikipedia.org/wiki/Jovian_(emperor) Multiple constraints ['https://en.wikipedia.org/wiki/Goths', 'https://en.wikipedia.org/wiki/Western_Roman_Empire', 'https://en.wikipedia.org/wiki/Jovian_(emperor)'] 433 458 "Who was Prime Minister in Australia at the same time that Norman Gunston released ""Salute to ABBA""?" Malcolm Fraser https://en.wikipedia.org/wiki/Norman_Gunston https://en.wikipedia.org/wiki/List_of_prime_ministers_of_Australia# Tabular reasoning ['https://en.wikipedia.org/wiki/Norman_Gunston', 'https://en.wikipedia.org/wiki/List_of_prime_ministers_of_Australia#'] 434 459 Which one was longer and by how much? James Cameron's film Titanic (1997) or the actual sinking of the Titanic in 1912? James Cameron's film Titanic was longer than the actual sinking Titanic, with a running time of 195 minutes, 35 minutes longer than the actual sinking of the Titanic in 1912. https://en.wikipedia.org/wiki/Titanic https://en.wikipedia.org/wiki/Titanic_(1997_film) Numerical reasoning ['https://en.wikipedia.org/wiki/Titanic', 'https://en.wikipedia.org/wiki/Titanic_(1997_film)'] 435 460 I'm thinking of the screenplay, co-wrote by the same author as Lonesome Dove, that won an Oscar for Best Adapted Screenplay 20 years after Lonesome Dove won the Pulitzer Prize. What was the name of the screenplay? Brokeback Mountain https://en.wikipedia.org/wiki/Lonesome_Dove https://en.wikipedia.org/wiki/78th_Academy_Awards Numerical reasoning | Temporal reasoning ['https://en.wikipedia.org/wiki/Lonesome_Dove ', 'https://en.wikipedia.org/wiki/78th_Academy_Awards'] 436 461 What was the founding name of the company that ran the coal mining camp in the city where baseball great Willie Mays was born? Sewanee Furnace Company https://en.wikipedia.org/wiki/Willie_Mays https://en.wikipedia.org/wiki/Westfield,_Alabama https://en.wikipedia.org/wiki/Tennessee_Coal,_Iron_and_Railroad_Company Multiple constraints ['https://en.wikipedia.org/wiki/Willie_Mays', 'https://en.wikipedia.org/wiki/Westfield,_Alabama', 'https://en.wikipedia.org/wiki/Tennessee_Coal,_Iron_and_Railroad_Company'] 437 462 Which original Saturday Night Live cast member's daughter, tied with Hannah Waddingham for the Best Supporting Actress in a Streaming Series, Comedy award at the 1st Hollywood Critics Association TV Awards in 2021? Laraine Newman https://en.wikipedia.org/wiki/1st_Hollywood_Critics_Association_TV_Awards https://en.wikipedia.org/wiki/Hannah_Einbinder https://en.wikipedia.org/wiki/Hollywood_Creative_Alliance Multiple constraints ['https://en.wikipedia.org/wiki/1st_Hollywood_Critics_Association_TV_Awards', 'https://en.wikipedia.org/wiki/Hannah_Einbinder', 'https://en.wikipedia.org/wiki/Hollywood_Creative_Alliance'] 438 463 Who was the youngest climber to podium at the first year climbing was in the olympics? Alberto Ginés López (18 at the time). https://en.wikipedia.org/wiki/Sport_climbing_at_the_Summer_Olympics https://en.wikipedia.org/wiki/Alberto_Gin%C3%A9s_L%C3%B3pez https://en.wikipedia.org/wiki/Nathaniel_Coleman https://en.wikipedia.org/wiki/Jakob_Schubert https://en.wikipedia.org/wiki/Janja_Garnbret https://en.wikipedia.org/wiki/Miho_Nonaka https://en.wikipedia.org/wiki/Akiyo_Noguchi Tabular reasoning ['https://en.wikipedia.org/wiki/Sport_climbing_at_the_Summer_Olympics', 'https://en.wikipedia.org/wiki/Alberto_Gin%C3%A9s_L%C3%B3pez', 'https://en.wikipedia.org/wiki/Nathaniel_Coleman', 'https://en.wikipedia.org/wiki/Jakob_Schubert', 'https://en.wikipedia.org/wiki/Janja_Garnbret', 'https://en.wikipedia.org/wiki/Miho_Nonaka', 'https://en.wikipedia.org/wiki/Akiyo_Noguchi'] 439 464 As of January 1st, 2024, how many buildings in New York City were 750ft or taller the last time Halley's Comet came close to Earth? 12 https://en.wikipedia.org/wiki/Halley%27s_Comet https://en.wikipedia.org/wiki/List_of_tallest_buildings_in_New_York_City Tabular reasoning ['https://en.wikipedia.org/wiki/Halley%27s_Comet', 'https://en.wikipedia.org/wiki/List_of_tallest_buildings_in_New_York_City'] 440 465 What novel by Ernest Hemingway won a Pulitzer prize for fiction that was subsequently overturned twelve years before the same author won again? For Whom the Bell Tolls by Ernest Hemingway https://en.wikipedia.org/wiki/Pulitzer_Prize_for_Fiction#Repeat_winners https://en.wikipedia.org/wiki/For_Whom_the_Bell_Tolls#Pulitzer_Prize_snub https://en.wikipedia.org/wiki/The_Old_Man_and_the_Sea#Reception_and_legacy https://en.wikipedia.org/wiki/Ernest_Hemingway_bibliography Multiple constraints ['https://en.wikipedia.org/wiki/Pulitzer_Prize_for_Fiction#Repeat_winners', 'https://en.wikipedia.org/wiki/For_Whom_the_Bell_Tolls#Pulitzer_Prize_snub', 'https://en.wikipedia.org/wiki/The_Old_Man_and_the_Sea#Reception_and_legacy', 'https://en.wikipedia.org/wiki/Ernest_Hemingway_bibliography'] 441 466 Who was the successor of the Egyptian Pharaoh that was in power when the Treasury of Atreus was completed? Merneptah https://en.wikipedia.org/wiki/Treasury_of_Atreus https://en.wikipedia.org/wiki/Periodization_of_ancient_Egypt https://en.wikipedia.org/wiki/Nineteenth_Dynasty_of_Egypt#Pharaohs_of_the_19th_Dynasty https://en.wikipedia.org/wiki/Ramesses_II https://en.wikipedia.org/wiki/Merneptah Multiple constraints | Temporal reasoning ['https://en.wikipedia.org/wiki/Treasury_of_Atreus', 'https://en.wikipedia.org/wiki/Periodization_of_ancient_Egypt', 'https://en.wikipedia.org/wiki/Nineteenth_Dynasty_of_Egypt#Pharaohs_of_the_19th_Dynasty', 'https://en.wikipedia.org/wiki/Ramesses_II', 'https://en.wikipedia.org/wiki/Merneptah'] 442 467 What is the age difference between the youngest and oldest person, in the 20th Century, to win two Nobel Prizes? 20 years. https://en.wikipedia.org/wiki/Nobel_Prize#Multiple_laureates https://en.wikipedia.org/wiki/Marie_Curie https://en.wikipedia.org/wiki/Linus_Pauling https://en.wikipedia.org/wiki/John_Bardeen https://en.wikipedia.org/wiki/Frederick_Sanger Numerical reasoning | Temporal reasoning ['https://en.wikipedia.org/wiki/Nobel_Prize#Multiple_laureates', 'https://en.wikipedia.org/wiki/Marie_Curie', 'https://en.wikipedia.org/wiki/Linus_Pauling', 'https://en.wikipedia.org/wiki/John_Bardeen', 'https://en.wikipedia.org/wiki/Frederick_Sanger'] 443 468 What season of The Challenge was airing when Bridgeton premiered? Season 36, Double Agents https://en.wikipedia.org/wiki/The_Challenge_(TV_series) https://en.wikipedia.org/wiki/Bridgerton Multiple constraints ['https://en.wikipedia.org/wiki/The_Challenge_(TV_series)', 'https://en.wikipedia.org/wiki/Bridgerton'] 444 469 How many of Mark Calaway's consecutive Wrestlemania wins occurred in matches that were longer than the final match result in the same year's competition, not including years where Calaway was part of the final result? Five https://en.wikipedia.org/wiki/The_Undertaker https://en.wikipedia.org/wiki/The_Streak_(professional_wrestling) https://en.wikipedia.org/wiki/WrestleMania_VII#Results https://en.wikipedia.org/wiki/WrestleMania_VIII#Results https://en.wikipedia.org/wiki/WrestleMania_IX#Results https://en.wikipedia.org/wiki/WrestleMania_XI#Results https://en.wikipedia.org/wiki/WrestleMania_XII#Results https://en.wikipedia.org/wiki/WrestleMania_13#Results https://en.wikipedia.org/wiki/WrestleMania_XIV#Results https://en.wikipedia.org/wiki/WrestleMania_XV#Results https://en.wikipedia.org/wiki/WrestleMania_X-Seven#Results, https://en.wikipedia.org/wiki/WrestleMania_X8#Results, https://en.wikipedia.org/wiki/WrestleMania_XIX#Results, https://en.wikipedia.org/wiki/WrestleMania_XX#Results, https://en.wikipedia.org/wiki/WrestleMania_21#Results, https://en.wikipedia.org/wiki/WrestleMania_22#Results, https://en.wikipedia.org/wiki/WrestleMania_23#Results, https://en.wikipedia.org/wiki/WrestleMania_XXIV#Results, https://en.wikipedia.org/wiki/WrestleMania_25#Results, https://en.wikipedia.org/wiki/WrestleMania_XXVI#Results, https://en.wikipedia.org/wiki/WrestleMania_XXVII#Results, https://en.wikipedia.org/wiki/WrestleMania_XXVIII#Results, https://en.wikipedia.org/wiki/WrestleMania_29#Results Numerical reasoning | Tabular reasoning | Multiple constraints ['https://en.wikipedia.org/wiki/The_Undertaker', 'https://en.wikipedia.org/wiki/The_Streak_(professional_wrestling)', 'https://en.wikipedia.org/wiki/WrestleMania_VII#Results', 'https://en.wikipedia.org/wiki/WrestleMania_VIII#Results', 'https://en.wikipedia.org/wiki/WrestleMania_IX#Results', 'https://en.wikipedia.org/wiki/WrestleMania_XI#Results', 'https://en.wikipedia.org/wiki/WrestleMania_XII#Results', 'https://en.wikipedia.org/wiki/WrestleMania_13#Results', 'https://en.wikipedia.org/wiki/WrestleMania_XIV#Results', 'https://en.wikipedia.org/wiki/WrestleMania_XV#Results', 'https://en.wikipedia.org/wiki/WrestleMania_X-Seven#Results, https://en.wikipedia.org/wiki/WrestleMania_X8#Results, https://en.wikipedia.org/wiki/WrestleMania_XIX#Results, https://en.wikipedia.org/wiki/WrestleMania_XX#Results, https://en.wikipedia.org/wiki/WrestleMania_21#Results, https://en.wikipedia.org/wiki/WrestleMania_22#Results, https://en.wikipedia.org/wiki/WrestleMania_23#Results, https://en.wikipedia.org/wiki/WrestleMania_XXIV#Results, https://en.wikipedia.org/wiki/WrestleMania_25#Results, https://en.wikipedia.org/wiki/WrestleMania_XXVI#Results, https://en.wikipedia.org/wiki/WrestleMania_XXVII#Results, https://en.wikipedia.org/wiki/WrestleMania_XXVIII#Results, https://en.wikipedia.org/wiki/WrestleMania_29#Results'] 445 470 What number do you get when you add up the numbers in the postcode of the hospital John Lennon was born in? 14 https://en.wikipedia.org/wiki/John_Lennon https://en.wikipedia.org/wiki/Liverpool_Maternity_Hospital Numerical reasoning ['https://en.wikipedia.org/wiki/John_Lennon', 'https://en.wikipedia.org/wiki/Liverpool_Maternity_Hospital'] 446 471 Which set director born in 1936 won the Academy Award for Best Production Design during the 71st Academy Awards? Jill Quertier https://en.wikipedia.org/wiki/Academy_Award_for_Best_Production_Design https://en.wikipedia.org/wiki/Martin_Childs https://en.wikipedia.org/wiki/Jill_Quertier Multiple constraints ['https://en.wikipedia.org/wiki/Academy_Award_for_Best_Production_Design', 'https://en.wikipedia.org/wiki/Martin_Childs', 'https://en.wikipedia.org/wiki/Jill_Quertier'] 447 472 "The star of the TV show ""The Bear"" had his breakout role on a Showtime show. The actress who played his older sister on that Showtime show released an album in 2007. The label who released her album released their first album 27 years previous. From their first album, what number did the title track reach on the Billboard Hot 100?" Number 3 https://en.wikipedia.org/wiki/The_Bear_(TV_series) https://en.wikipedia.org/wiki/Jeremy_Allen_White https://en.wikipedia.org/wiki/Shameless_(American_TV_series) https://en.wikipedia.org/wiki/Emmy_Rossum#Awards_and_nominations https://en.wikipedia.org/wiki/Geffen_Records#History https://en.wikipedia.org/wiki/The_Wanderer_(Donna_Summer_album) Numerical reasoning | Tabular reasoning | Multiple constraints ['https://en.wikipedia.org/wiki/The_Bear_(TV_series)', 'https://en.wikipedia.org/wiki/Jeremy_Allen_White', 'https://en.wikipedia.org/wiki/Shameless_(American_TV_series)', 'https://en.wikipedia.org/wiki/Emmy_Rossum#Awards_and_nominations', 'https://en.wikipedia.org/wiki/Geffen_Records#History', 'https://en.wikipedia.org/wiki/The_Wanderer_(Donna_Summer_album)'] 448 473 "When Taylor Swift first released her studio album ""1989,"" how many studio albums had Katy Perry already released? " 4 https://en.wikipedia.org/wiki/Taylor_Swift_albums_discography https://en.wikipedia.org/wiki/Katy_Perry_discography Multiple constraints ['https://en.wikipedia.org/wiki/Taylor_Swift_albums_discography', 'https://en.wikipedia.org/wiki/Katy_Perry_discography'] 449 474 Who lived longer one of the Bronte sisters or Jane Austen? Jane Austen https://en.wikipedia.org/wiki/Bront%C3%AB_family https://en.wikipedia.org/wiki/Jane_Austen Numerical reasoning ['https://en.wikipedia.org/wiki/Bront%C3%AB_family', 'https://en.wikipedia.org/wiki/Jane_Austen'] 450 475 Who was the head coach of the team that won the Superbowl the year that the show Law & Order: SVU was released? Mike Shanahan https://en.wikipedia.org/wiki/Law_%26_Order:_Special_Victims_Unit https://en.wikipedia.org/wiki/List_of_Super_Bowl_champions https://en.wikipedia.org/wiki/1998_Denver_Broncos_season Tabular reasoning | Multiple constraints ['https://en.wikipedia.org/wiki/Law_%26_Order:_Special_Victims_Unit', 'https://en.wikipedia.org/wiki/List_of_Super_Bowl_champions', 'https://en.wikipedia.org/wiki/1998_Denver_Broncos_season'] 451 476 How many times taller is the 7th highest mountain the world than Novake, Poljčane in Slovenia? Round your answer to one decimal place. 31.5 https://en.wikipedia.org/wiki/Novake,_Polj%C4%8Dane https://en.wikipedia.org/wiki/List_of_highest_mountains_on_Earth Numerical reasoning | Tabular reasoning | Multiple constraints ['https://en.wikipedia.org/wiki/Novake,_Polj%C4%8Dane', 'https://en.wikipedia.org/wiki/List_of_highest_mountains_on_Earth'] 452 477 The Wisconsin Butter Fire led to dams being built to protect a lake that was the site of the plane crash death of which famous musician? Otis Redding https://en.wikipedia.org/wiki/Wisconsin_Butter_Fire https://en.wikipedia.org/wiki/Lake_Monona Multiple constraints ['https://en.wikipedia.org/wiki/Wisconsin_Butter_Fire', 'https://en.wikipedia.org/wiki/Lake_Monona'] 453 478 What is the English meaning of the name of the smaller of the two constellations which resides in the middle of the Summer Triangle? Arrow. https://en.wikipedia.org/wiki/Summer_Triangle https://en.wikipedia.org/wiki/Sagitta https://en.wikipedia.org/wiki/Vulpecula Numerical reasoning | Tabular reasoning ['https://en.wikipedia.org/wiki/Summer_Triangle', 'https://en.wikipedia.org/wiki/Sagitta', 'https://en.wikipedia.org/wiki/Vulpecula'] 454 479 During the year that Serbia became an independent republic after separation from Montenegro, who won the Nobel Prize for Literature? Orhan Pamuk https://en.wikipedia.org/wiki/Socialist_Republic_of_Serbia https://en.wikipedia.org/wiki/2006_Nobel_Prize_in_Literature#:~:text=The%202006%20Nobel%20Prize%20in,clash%20and%20interlacing%20of%20cultures.%22 Multiple constraints ['https://en.wikipedia.org/wiki/Socialist_Republic_of_Serbia', 'https://en.wikipedia.org/wiki/2006_Nobel_Prize_in_Literature#:~:text=The%202006%20Nobel%20Prize%20in,clash%20and%20interlacing%20of%20cultures.%22'] 455 480 How many years older is the the library where Galileo's middle finger was first exhibited than the first public library to be lit using electric lightbulbs? 222 https://en.wikipedia.org/wiki/Galileo%27s_middle_finger#Exhibition_history https://en.wikipedia.org/wiki/Laurentian_Library#Architecture https://en.wikipedia.org/wiki/Literary_and_Philosophical_Society_of_Newcastle_upon_Tyne# Numerical reasoning | Temporal reasoning ['https://en.wikipedia.org/wiki/Galileo%27s_middle_finger#Exhibition_history', 'https://en.wikipedia.org/wiki/Laurentian_Library#Architecture', 'https://en.wikipedia.org/wiki/Literary_and_Philosophical_Society_of_Newcastle_upon_Tyne#'] 456 481 "If the person who played the joker in ""The Dark Knight"" (2008) had children, how old will they be on the films 50th anniversary? " Heath Ledger had one daughter named Matilda Ledger who was born on October 28, 2005. Since the film was released on the 18th of July in 2008, Matilda will be 52 on the films 50th anniversary. https://en.wikipedia.org/wiki/Heath_Ledger https://en.wikipedia.org/wiki/Michelle_Williams_(actress) https://en.wikipedia.org/wiki/The_Dark_Knight Numerical reasoning | Post processing | Temporal reasoning ['https://en.wikipedia.org/wiki/Heath_Ledger', 'https://en.wikipedia.org/wiki/Michelle_Williams_(actress)', 'https://en.wikipedia.org/wiki/The_Dark_Knight'] 457 482 What was David Fincher's most recently released feature film when Britney Spears' first feature film was released in the U.S.? Fight Club. https://en.wikipedia.org/wiki/Crossroads_(2002_film)#Reception https://en.wikipedia.org/wiki/David_Fincher_filmography https://en.wikipedia.org/wiki/Panic_Room#Theatrical_run Numerical reasoning | Multiple constraints | Temporal reasoning ['https://en.wikipedia.org/wiki/Crossroads_(2002_film)#Reception', 'https://en.wikipedia.org/wiki/David_Fincher_filmography', 'https://en.wikipedia.org/wiki/Panic_Room#Theatrical_run'] 458 483 What was the last album the Grateful Dead released prior to the death of Doors vocalist Jim Morrison? American Beauty https://en.wikipedia.org/wiki/Jim_Morrison https://en.wikipedia.org/wiki/Grateful_Dead_discography Multiple constraints ['https://en.wikipedia.org/wiki/Jim_Morrison', 'https://en.wikipedia.org/wiki/Grateful_Dead_discography'] 459 484 David Fincher has two movies in the 90's that have numbers in the title. What is the number in the title of David Fincher's later-released movie multiplied by the ratio of the sum of all of the molars in an aardvark over the number of adult teeth in a dog? 13/3 or 4.333... (infinitely repeating decimal) https://en.wikipedia.org/wiki/David_Fincher https://en.wikipedia.org/wiki/Aardvark https://en.wikipedia.org/wiki/Puppy_teething Numerical reasoning | Tabular reasoning | Temporal reasoning ['https://en.wikipedia.org/wiki/David_Fincher', 'https://en.wikipedia.org/wiki/Aardvark', 'https://en.wikipedia.org/wiki/Puppy_teething'] 460 485 The band Franz Ferdinand is named after Archduke Franz Ferdinand of Austria and a racehorse that the band watched win a race. How many years before the assassination of Archduke Franz Ferdinand was that race established? The Northumberland Plate horse race was established 81 years before the assassination of Archduke Franz Ferdinand https://en.wikipedia.org/wiki/Assassination_of_Archduke_Franz_Ferdinand https://en.wikipedia.org/wiki/Franz_Ferdinand_(band) https://en.wikipedia.org/wiki/Northumberland_Plate Numerical reasoning | Multiple constraints | Temporal reasoning ['https://en.wikipedia.org/wiki/Assassination_of_Archduke_Franz_Ferdinand', 'https://en.wikipedia.org/wiki/Franz_Ferdinand_(band)', 'https://en.wikipedia.org/wiki/Northumberland_Plate'] 461 486 Which university did the actor who has appeared in the most movies in the American pie film series (including spinoffs, as of 2020) deliver a commencement address at in 2012? Dalhousie University, in Halifax, Nova Scotia https://en.wikipedia.org/wiki/American_Pie_(film_series) https://en.wikipedia.org/wiki/Eugene_Levy Tabular reasoning ['https://en.wikipedia.org/wiki/American_Pie_(film_series)', 'https://en.wikipedia.org/wiki/Eugene_Levy'] 462 488 Since the turn of the 21st century, there has only been one year in which the Pulitzer Prize for Fiction has not been awarded. Among the finalists that year, one novel was published posthumously. The author of this posthumous publication is best known for a different novel published 15 years prior. The opening lines of this book later inspired a song by a band. The name of this band can be determined with the following calculation: find the age of the author on January 1 of the year the latter book was published, add four, and subtract the sum from the year this author was a Pulitzer Prize finalist. The name of this band was inspired by yet another book. What is the name of this book and its author? On the Road by Jack Kerouac https://en.wikipedia.org/wiki/Pulitzer_Prize_for_Fiction https://en.wikipedia.org/wiki/David_Foster_Wallace https://en.wikipedia.org/wiki/Infinite_Jest#Adaptations https://en.wikipedia.org/wiki/The_1975 Numerical reasoning | Multiple constraints | Temporal reasoning ['https://en.wikipedia.org/wiki/Pulitzer_Prize_for_Fiction', 'https://en.wikipedia.org/wiki/David_Foster_Wallace', 'https://en.wikipedia.org/wiki/Infinite_Jest#Adaptations', 'https://en.wikipedia.org/wiki/The_1975'] 463 489 How old was Akira Toriyama when Pokemon Diamond and Pearl was released in America? Akira Toriyama was 52. https://en.wikipedia.org/wiki/Akira_Toriyama https://en.wikipedia.org/wiki/Pok%C3%A9mon_Diamond_and_Pearl Numerical reasoning | Temporal reasoning ['https://en.wikipedia.org/wiki/Akira_Toriyama', 'https://en.wikipedia.org/wiki/Pok%C3%A9mon_Diamond_and_Pearl'] 464 490 As of 2020, what was the population of the city where Dippin' Dots were founded? 27,137 https://en.wikipedia.org/wiki/Dippin%27_Dots https://en.wikipedia.org/wiki/Paducah,_Kentucky Multiple constraints ['https://en.wikipedia.org/wiki/Dippin%27_Dots', 'https://en.wikipedia.org/wiki/Paducah,_Kentucky'] 465 491 How old were the founders of the firm that designed the Empire State Building when construction on the building began? Richmond Shreve was 52 years old and William Lamb was 46 years old. https://en.wikipedia.org/wiki/Empire_State_Building https://en.wikipedia.org/wiki/Shreve,_Lamb_%26_Harmon https://en.wikipedia.org/wiki/Richmond_Shreve https://en.wikipedia.org/wiki/William_F._Lamb Numerical reasoning | Multiple constraints | Temporal reasoning ['https://en.wikipedia.org/wiki/Empire_State_Building', 'https://en.wikipedia.org/wiki/Shreve,_Lamb_%26_Harmon', 'https://en.wikipedia.org/wiki/Richmond_Shreve', 'https://en.wikipedia.org/wiki/William_F._Lamb'] 466 492 In 2015, Emmanuel Lubezki was cinematographer for a film that was directed by the man who also directed the 2014 film Birdman. What is that film? The Revenant https://en.wikipedia.org/wiki/Birdman_(film) https://en.wikipedia.org/wiki/Alejandro_González_Iñárritu https://en.wikipedia.org/wiki/Emmanuel_Lubezki#Feature_film Multiple constraints ['https://en.wikipedia.org/wiki/Birdman_(film)', 'https://en.wikipedia.org/wiki/Alejandro_González_Iñárritu', 'https://en.wikipedia.org/wiki/Emmanuel_Lubezki#Feature_film'] 467 493 A puzzle released in the 1970's gained popularity and inspired the establishment of an international speed competition. What was the average time of all winners of this competition between 2005 and 2015 rounded to the nearest 100th? 10.45 https://en.wikipedia.org/wiki/Rubik%27s_Cube https://en.wikipedia.org/wiki/Speedcubing Numerical reasoning | Tabular reasoning | Multiple constraints | Post processing | Temporal reasoning ['https://en.wikipedia.org/wiki/Rubik%27s_Cube', 'https://en.wikipedia.org/wiki/Speedcubing'] 468 494 Who was the British Prime Minister in the year that the Glastonbury Festival was launched? Edward Heath, September 1970 https://en.wikipedia.org/wiki/Glastonbury_Festival https://en.wikipedia.org/wiki/List_of_prime_ministers_of_the_United_Kingdom Multiple constraints ['https://en.wikipedia.org/wiki/Glastonbury_Festival', 'https://en.wikipedia.org/wiki/List_of_prime_ministers_of_the_United_Kingdom'] 469 495 Which South Korean Cross-country skier had the highest rank at the Olympics in the Men's 15 km, among South Korean skiers only, between 2002 and 2006? Park Byeong-ju https://en.wikipedia.org/wiki/South_Korea_at_the_2006_Winter_Olympics https://en.wikipedia.org/wiki/South_Korea_at_the_2002_Winter_Olympics Numerical reasoning | Tabular reasoning ['https://en.wikipedia.org/wiki/South_Korea_at_the_2006_Winter_Olympics', 'https://en.wikipedia.org/wiki/South_Korea_at_the_2002_Winter_Olympics'] 470 496 Which country did tennis' first Golden Slam winner represent when they achieved it? West Germany https://en.wikipedia.org/wiki/Grand_Slam_(tennis)# https://en.wikipedia.org/wiki/Tennis_at_the_1988_Summer_Olympics_%E2%80%93_Women%27s_singles Multiple constraints ['https://en.wikipedia.org/wiki/Grand_Slam_(tennis)#', 'https://en.wikipedia.org/wiki/Tennis_at_the_1988_Summer_Olympics_%E2%80%93_Women%27s_singles'] 471 497 Which two MLB teams played in the World Series just weeks after the O.J. Simpson murder trial had ended? Atlanta Braves and Cleveland Indians https://en.wikipedia.org/wiki/Murder_trial_of_O._J._Simpson https://en.wikipedia.org/wiki/1995_World_Series Temporal reasoning ['https://en.wikipedia.org/wiki/Murder_trial_of_O._J._Simpson', 'https://en.wikipedia.org/wiki/1995_World_Series'] 472 498 Who was the Vice Admiral in charge of the Carrier Division the Japanese carrier Hiyō was in when she was sunk? Vice Admiral Kakuji Kakuta https://en.wikipedia.org/wiki/Japanese_aircraft_carrier_Hiyō https://en.wikipedia.org/wiki/Battle_of_the_Philippine_Sea https://en.wikipedia.org/wiki/Philippine_Sea_order_of_battle Multiple constraints ['https://en.wikipedia.org/wiki/Japanese_aircraft_carrier_Hiyō', 'https://en.wikipedia.org/wiki/Battle_of_the_Philippine_Sea', 'https://en.wikipedia.org/wiki/Philippine_Sea_order_of_battle'] 473 499 In January of 2024, how many colleges were in the district in which the William C. Davis Science building can be found? Alamo Colleges District has 5 schools. https://en.wikipedia.org/wiki/William_Conan_Davis https://en.wikipedia.org/wiki/St._Philip%27s_College_(United_States) https://en.wikipedia.org/wiki/Alamo_Colleges_District Multiple constraints ['https://en.wikipedia.org/wiki/William_Conan_Davis', 'https://en.wikipedia.org/wiki/St._Philip%27s_College_(United_States)', 'https://en.wikipedia.org/wiki/Alamo_Colleges_District'] 474 500 Can you add one minute to the time of the title track from the album released by Ray Charles in the year before Phil Quartararo took over as president of its record label? "5:08 (""Strong Love Affair"" has an individual track time of 4:08)" https://en.wikipedia.org/wiki/Phil_Quartararo#Warner_Bros._Records https://en.wikipedia.org/wiki/Warner_Records#End_of_an_era:_Ostin_and_Waronker_depart https://en.wikipedia.org/wiki/Ray_Charles_discography https://en.wikipedia.org/wiki/Strong_Love_Affair#Track_listing Numerical reasoning | Multiple constraints | Temporal reasoning ['https://en.wikipedia.org/wiki/Phil_Quartararo#Warner_Bros._Records', 'https://en.wikipedia.org/wiki/Warner_Records#End_of_an_era:_Ostin_and_Waronker_depart', 'https://en.wikipedia.org/wiki/Ray_Charles_discography', 'https://en.wikipedia.org/wiki/Strong_Love_Affair#Track_listing'] 475 501 As of 2020, who is the longest-serving president of the university where Hannah Arendt is buried? Leon Botstein https://en.wikipedia.org/wiki/Hannah_Arendt https://en.wikipedia.org/wiki/Bard_College Multiple constraints ['https://en.wikipedia.org/wiki/Hannah_Arendt', 'https://en.wikipedia.org/wiki/Bard_College'] 476 502 What was state of the schools regarding integration in the hometown of Miller Williams when he began his master's degree in Zoology? They were segregated until 1955. Miller Williams began working on his master's in 1952. https://en.wikipedia.org/wiki/Miller_Williams https://en.wikipedia.org/wiki/Hoxie,_Arkansas Temporal reasoning ['https://en.wikipedia.org/wiki/Miller_Williams', 'https://en.wikipedia.org/wiki/Hoxie,_Arkansas'] 477 503 Do zebra, giraffe or hippos wean the earliest? Giraffe https://en.wikipedia.org/wiki/Plains_zebra https://en.wikipedia.org/wiki/Giraffe https://en.wikipedia.org/wiki/Hippopotamus Numerical reasoning | Temporal reasoning ['https://en.wikipedia.org/wiki/Plains_zebra', 'https://en.wikipedia.org/wiki/Giraffe', 'https://en.wikipedia.org/wiki/Hippopotamus'] 478 504 How many years were there between Heath Ledger's birth and the first successful use of a special human extraction technology that appeared in a Batman movie that Ledger starred in? 21 years. https://en.wikipedia.org/wiki/Fulton_surface-to-air_recovery_system https://en.wikipedia.org/wiki/The_Dark_Knight https://en.wikipedia.org/wiki/Heath_Ledger Numerical reasoning | Multiple constraints ['https://en.wikipedia.org/wiki/Fulton_surface-to-air_recovery_system', 'https://en.wikipedia.org/wiki/The_Dark_Knight', 'https://en.wikipedia.org/wiki/Heath_Ledger'] 479 505 How many feature films had James Cameron directed by the time Barack Obama was inaugurated as President of the United States? Seven. https://en.wikipedia.org/wiki/Barack_Obama https://en.wikipedia.org/wiki/James_Cameron#Filmography https://en.wikipedia.org/wiki/Avatar_(2009_film) Numerical reasoning | Multiple constraints ['https://en.wikipedia.org/wiki/Barack_Obama', 'https://en.wikipedia.org/wiki/James_Cameron#Filmography', 'https://en.wikipedia.org/wiki/Avatar_(2009_film)'] 480 506 Who was the president of the USA the year that the European Convention on Human Rights came into effect? Dwight D Eisenhower https://en.wikipedia.org/wiki/European_Convention_on_Human_Rights https://en.wikipedia.org/wiki/List_of_presidents_of_the_United_States#Presidents Tabular reasoning ['https://en.wikipedia.org/wiki/European_Convention_on_Human_Rights', 'https://en.wikipedia.org/wiki/List_of_presidents_of_the_United_States#Presidents'] 481 507 As of 1st November 2023, Which two South African drivers took part in the Formula One the year in which a micro-nation claimed territory off the coast of Suffolk, England? Dave Charlton and Luki Botha https://en.wikipedia.org/wiki/List_of_micronations https://en.wikipedia.org/wiki/1967_Formula_One_season#Teams_and_drivers Tabular reasoning | Multiple constraints | Temporal reasoning ['https://en.wikipedia.org/wiki/List_of_micronations', 'https://en.wikipedia.org/wiki/1967_Formula_One_season#Teams_and_drivers'] 482 508 How old was the first minister of the Ministry of Digital Affairs in Poland when Andrzej Halicki became minister? The first minister of the Ministry of Digital Affairs in Poland Anna Streżyńska was 47 years old when Andrzej Halicki became minister in 2014. Anna Streżyńska was born on May 11th, 1967. https://en.wikipedia.org/wiki/Ministry_of_Digital_Affairs https://en.wikipedia.org/wiki/Anna_Stre%C5%BCy%C5%84ska https://en.wikipedia.org/wiki/Ministry_of_Administration_and_Digitization_(Poland) Numerical reasoning | Tabular reasoning ['https://en.wikipedia.org/wiki/Ministry_of_Digital_Affairs', 'https://en.wikipedia.org/wiki/Anna_Stre%C5%BCy%C5%84ska', 'https://en.wikipedia.org/wiki/Ministry_of_Administration_and_Digitization_(Poland)'] 483 509 Looking at the Best Actor and Best Actress categories for the 2023 Academy Awards, how many children did all of the winners and nominees have combined as of August 1, 2024? 13 https://en.wikipedia.org/wiki/95th_Academy_Awards#Awards https://en.wikipedia.org/wiki/Brendan_Fraser https://en.wikipedia.org/wiki/Austin_Butler https://en.wikipedia.org/wiki/Colin_Farrell https://en.wikipedia.org/wiki/Paul_Mescal https://en.wikipedia.org/wiki/Bill_Nighy https://en.wikipedia.org/wiki/Michelle_Yeoh https://en.wikipedia.org/wiki/Cate_Blanchett https://en.wikipedia.org/wiki/Ana_de_Armas https://en.wikipedia.org/wiki/Andrea_Riseborough https://en.wikipedia.org/wiki/Michelle_Williams_(actress) Numerical reasoning ['https://en.wikipedia.org/wiki/95th_Academy_Awards#Awards', 'https://en.wikipedia.org/wiki/Brendan_Fraser', 'https://en.wikipedia.org/wiki/Austin_Butler', 'https://en.wikipedia.org/wiki/Colin_Farrell', 'https://en.wikipedia.org/wiki/Paul_Mescal', 'https://en.wikipedia.org/wiki/Bill_Nighy', 'https://en.wikipedia.org/wiki/Michelle_Yeoh', 'https://en.wikipedia.org/wiki/Cate_Blanchett', 'https://en.wikipedia.org/wiki/Ana_de_Armas', 'https://en.wikipedia.org/wiki/Andrea_Riseborough', 'https://en.wikipedia.org/wiki/Michelle_Williams_(actress)'] 484 510 Shakshouka and menemen are considered similar dishes. What two main ingredients do they have in common? Egg and tomato. https://en.wikipedia.org/wiki/Menemen_(food) https://en.wikipedia.org/wiki/Shakshouka Multiple constraints ['https://en.wikipedia.org/wiki/Menemen_(food)', 'https://en.wikipedia.org/wiki/Shakshouka'] 485 511 What is the title of the song that had the second highest record sales recorded on the Discography of American Historical Recordings in the same year that Fred J. Rath was elected mayor of Utica? Blue Yodel No. 1 (T for Texas) https://en.m.wikipedia.org/wiki/List_of_mayors_of_Utica,_New_York https://en.m.wikipedia.org/wiki/1928_in_music Tabular reasoning ['https://en.m.wikipedia.org/wiki/List_of_mayors_of_Utica,_New_York', 'https://en.m.wikipedia.org/wiki/1928_in_music'] 486 512 Give me the difference in time, measured in hours not days, between the first solo woman to thru-hike the Appalachian Trail and the fastest recorded solo woman to walk the Appalachian Trail before 2012. 2128 hours https://en.wikipedia.org/wiki/Appalachian_Trail https://en.wikipedia.org/wiki/Grandma_Gatewood https://en.wikipedia.org/wiki/Jennifer_Pharr_Davis Numerical reasoning | Multiple constraints | Post processing | Temporal reasoning ['https://en.wikipedia.org/wiki/Appalachian_Trail', 'https://en.wikipedia.org/wiki/Grandma_Gatewood', 'https://en.wikipedia.org/wiki/Jennifer_Pharr_Davis'] 487 513 "If Mr. Rogers were still alive, how old would he have been when the movie ""A Beautiful Day in the Neighborhood"", featuring Tom Hanks came out in the U.S.?" "Fred McFeely Rogers would have been 91 years old when ""A Beautiful Day in the Neighborhood"" was released in the U.S." https://en.wikipedia.org/wiki/Fred_Rogers https://en.wikipedia.org/wiki/A_Beautiful_Day_in_the_Neighborhood Numerical reasoning ['https://en.wikipedia.org/wiki/Fred_Rogers', 'https://en.wikipedia.org/wiki/A_Beautiful_Day_in_the_Neighborhood'] 488 514 Who became the prime minister of Canada in the same year that Jonathan Tremblay's re-election bid as representative for the electoral district of Montmorency—Charlevoix—Haute-Côte-Nord ended in defeat? Justin Trudeau https://en.wikipedia.org/wiki/Jonathan_Tremblay https://en.wikipedia.org/wiki/List_of_prime_ministers_of_Canada Multiple constraints ['https://en.wikipedia.org/wiki/Jonathan_Tremblay', 'https://en.wikipedia.org/wiki/List_of_prime_ministers_of_Canada'] 489 515 How many days after Peter Tosh died did Bunny Wailer pass away? 12,226 days https://en.wikipedia.org/wiki/Peter_Tosh https://en.wikipedia.org/wiki/Bunny_Wailer Temporal reasoning ['https://en.wikipedia.org/wiki/Peter_Tosh', 'https://en.wikipedia.org/wiki/Bunny_Wailer'] 490 516 How old was Lucy Lawless when season six of Xena: Warrior Princess first aired? 32 https://en.wikipedia.org/wiki/List_of_Xena:_Warrior_Princess_episodes https://en.wikipedia.org/wiki/Lucy_Lawless Numerical reasoning | Tabular reasoning | Temporal reasoning ['https://en.wikipedia.org/wiki/List_of_Xena:_Warrior_Princess_episodes', 'https://en.wikipedia.org/wiki/Lucy_Lawless'] 491 517 Taylor Swift's debut single is named after another famous country singer. As of August 1, 2024, when is his wife's birthday? September 21, 1967 https://en.wikipedia.org/wiki/Taylor_Swift https://en.wikipedia.org/wiki/Tim_McGraw https://en.wikipedia.org/wiki/Faith_Hill Multiple constraints ['https://en.wikipedia.org/wiki/Taylor_Swift', 'https://en.wikipedia.org/wiki/Tim_McGraw', 'https://en.wikipedia.org/wiki/Faith_Hill'] 492 518 What is the name of the rock climber/businessman who co-founded the environmental group that the co-founder of Netflix joined? Yvon Chouinard https://en.wikipedia.org/wiki/Netflix https://en.wikipedia.org/wiki/Marc_Randolph https://en.wikipedia.org/wiki/One_Percent_for_the_Planet https://en.wikipedia.org/wiki/Yvon_Chouinard Multiple constraints ['https://en.wikipedia.org/wiki/Netflix', 'https://en.wikipedia.org/wiki/Marc_Randolph', 'https://en.wikipedia.org/wiki/One_Percent_for_the_Planet', 'https://en.wikipedia.org/wiki/Yvon_Chouinard'] 493 519 Meghan Markle's veil, worn at her 2018 wedding to Prince Harry, featured a flower for each Common Wealth country. What colour is the flower that was used to represent New Zealand? Yellow https://en.wikipedia.org/wiki/Wedding_dress_of_Meghan_Markle https://en.wikipedia.org/wiki/Sophora_microphylla Multiple constraints ['https://en.wikipedia.org/wiki/Wedding_dress_of_Meghan_Markle', 'https://en.wikipedia.org/wiki/Sophora_microphylla'] 494 520 How many years did it take after the FIFA ban was lifted for someone to wear a hijab in the World Cup? 11 years https://en.wikipedia.org/wiki/Kit_(association_football) https://en.wikipedia.org/wiki/Nouhaila_Benzina Post processing ['https://en.wikipedia.org/wiki/Kit_(association_football)', 'https://en.wikipedia.org/wiki/Nouhaila_Benzina'] 495 521 What is the name of the lead role of the play that Harry C. Bradley's second wife was in, in 1906? Lord Fancourt Babberly https://w.wiki/ASFv https://en.wikipedia.org/wiki/Lottie_Alter https://en.wikipedia.org/wiki/Charley%27s_Aunt Multiple constraints ['https://w.wiki/ASFv', 'https://en.wikipedia.org/wiki/Lottie_Alter', 'https://en.wikipedia.org/wiki/Charley%27s_Aunt'] 496 522 In the largest Northern City in California, the widow of a firearms magnate built a infamous mansion that became a tourist attraction only nine months after her death in the early 1900s. The mansion is not only a magnet for ghost hunters and horror story writers, but also architecture enthusiasts as the owner of the house spent 22 years constructing and adding on additional rooms and stairways and features to the already intricate mansion. The house at one point had 500 rooms, 10,000 windows, 47 fireplaces, and 6 kitchens. What exact date was this structure added to the U.S. National Register of Historic Places? August 7, 1974 https://en.wikipedia.org/wiki/Northern_California#Cities https://en.wikipedia.org/wiki/San_Jose,_California https://en.wikipedia.org/wiki/National_Register_of_Historic_Places https://en.wikipedia.org/wiki/Winchester_Mystery_House Multiple constraints | Post processing | Temporal reasoning ['https://en.wikipedia.org/wiki/Northern_California#Cities', 'https://en.wikipedia.org/wiki/San_Jose,_California', 'https://en.wikipedia.org/wiki/National_Register_of_Historic_Places', 'https://en.wikipedia.org/wiki/Winchester_Mystery_House'] 497 523 Fossils of the extinct sea snail, Alvania belgica, were originally found in a European country. What is the capital of that country? City of Brussels https://en.wikipedia.org/wiki/Alvania_belgica https://en.wikipedia.org/wiki/City_of_Brussels Multiple constraints ['https://en.wikipedia.org/wiki/Alvania_belgica', 'https://en.wikipedia.org/wiki/City_of_Brussels'] 498 524 How many more points did Michael Jordan average in his sophomore season (regular season) in the NBA than the first black NBA player averaged during his career (regular season)? Show me a math equation to justify your answer. 14.3 PPG 22.7 - 8.4 = 14.3 https://en.wikipedia.org/wiki/Michael_Jordan https://en.wikipedia.org/wiki/Race_and_ethnicity_in_the_NBA https://en.wikipedia.org/wiki/Earl_Lloyd Numerical reasoning | Tabular reasoning | Multiple constraints | Post processing | Temporal reasoning ['https://en.wikipedia.org/wiki/Michael_Jordan', 'https://en.wikipedia.org/wiki/Race_and_ethnicity_in_the_NBA', 'https://en.wikipedia.org/wiki/Earl_Lloyd'] 499 525 What is the title of the book, written by Stephanie Meyer, in her vampire book series that was published in the same year that the first Black president of the United States was elected? Breaking Dawn https://en.wikipedia.org/wiki/African-American_presidents_of_the_United_States_in_popular_culture https://en.wikipedia.org/wiki/Barack_Obama https://en.wikipedia.org/wiki/Stephenie_Meyer https://en.wikipedia.org/wiki/Breaking_Dawn Multiple constraints ['https://en.wikipedia.org/wiki/African-American_presidents_of_the_United_States_in_popular_culture', 'https://en.wikipedia.org/wiki/Barack_Obama', 'https://en.wikipedia.org/wiki/Stephenie_Meyer', 'https://en.wikipedia.org/wiki/Breaking_Dawn'] 500 526 The US President who officially opened the Tennessee Centennial and International Exposition was married in what year? 1871 https://en.wikipedia.org/wiki/Tennessee_Centennial_and_International_Exposition https://en.wikipedia.org/wiki/William_McKinley Multiple constraints ['https://en.wikipedia.org/wiki/Tennessee_Centennial_and_International_Exposition', 'https://en.wikipedia.org/wiki/William_McKinley'] 501 527 How old was the U.S. President's wife when the Camp David Accords were signed? She was 51 years old. https://en.wikipedia.org/wiki/Camp_David_Accords https://en.wikipedia.org/wiki/Jimmy_Carter https://en.wikipedia.org/wiki/Rosalynn_Carter Numerical reasoning ['https://en.wikipedia.org/wiki/Camp_David_Accords', 'https://en.wikipedia.org/wiki/Jimmy_Carter', 'https://en.wikipedia.org/wiki/Rosalynn_Carter'] 502 528 Out of all of Steven Spielberg's Oscar winning movies up until 2020, which one has made the most money? Saving Private Ryan https://en.wikipedia.org/wiki/List_of_awards_and_nominations_received_by_Steven_Spielberg https://en.wikipedia.org/wiki/Schindler%27s_List https://en.wikipedia.org/wiki/Saving_Private_Ryan Numerical reasoning ['https://en.wikipedia.org/wiki/List_of_awards_and_nominations_received_by_Steven_Spielberg', 'https://en.wikipedia.org/wiki/Schindler%27s_List', 'https://en.wikipedia.org/wiki/Saving_Private_Ryan'] 503 529 The number I am thinking about is the atomic number of the heavy metal that shares a name with the tier of the awards programme founded and chaired by Prince Philip that takes , at most, the same number of months to complete as the duration of an average pregnancy. What is the sum of the digits in this number? 11 https://en.wikipedia.org/wiki/Prince_Philip,_Duke_of_Edinburgh https://en.wikipedia.org/wiki/The_Duke_of_Edinburgh%27s_Award https://en.wikipedia.org/wiki/Pregnancy https://en.wikipedia.org/wiki/Silver Numerical reasoning | Multiple constraints | Post processing ['https://en.wikipedia.org/wiki/Prince_Philip,_Duke_of_Edinburgh', 'https://en.wikipedia.org/wiki/The_Duke_of_Edinburgh%27s_Award ', 'https://en.wikipedia.org/wiki/Pregnancy', 'https://en.wikipedia.org/wiki/Silver'] 504 530 Concerning just the winners between 2019 and 2024, which Pulitzer Prize-winning author was born in Maryland? Barbara Kingsolver https://en.wikipedia.org/wiki/Pulitzer_Prize_for_Fiction https://en.wikipedia.org/wiki/Richard_Powers https://en.wikipedia.org/wiki/Jayne_Anne_Phillips https://en.wikipedia.org/wiki/Barbara_Kingsolver https://en.wikipedia.org/wiki/Hernan_Diaz_(writer) https://en.wikipedia.org/wiki/Joshua_Cohen_(writer) https://en.wikipedia.org/wiki/Louise_Erdrich https://en.wikipedia.org/wiki/Colson_Whitehead Multiple constraints | Temporal reasoning ['https://en.wikipedia.org/wiki/Pulitzer_Prize_for_Fiction', 'https://en.wikipedia.org/wiki/Richard_Powers', 'https://en.wikipedia.org/wiki/Jayne_Anne_Phillips', 'https://en.wikipedia.org/wiki/Barbara_Kingsolver', 'https://en.wikipedia.org/wiki/Hernan_Diaz_(writer)', 'https://en.wikipedia.org/wiki/Joshua_Cohen_(writer)', 'https://en.wikipedia.org/wiki/Louise_Erdrich', 'https://en.wikipedia.org/wiki/Colson_Whitehead'] 505 531 What rank did Hermann Goring hold in the Luftwaffe during World War II, before Robert Ritter von Greim, and how does this rank compare to the equivalent ranks in other branches of the German military? "Reichsmarschall, which was a rank above ""General de Luftwaffe."" This rank does not exist in other branches of the German military and was unique to Goring himself as Robert Ritter von Greim held the title of Generalfeldmarschall after Hermann. " https://en.wikipedia.org/wiki/General_der_Luftwaffe https://en.wikipedia.org/wiki/General_of_the_branch https://en.wikipedia.org/wiki/Luftwaffe https://en.wikipedia.org/wiki/Hermann_G%C3%B6ring Numerical reasoning | Multiple constraints | Temporal reasoning ['https://en.wikipedia.org/wiki/General_der_Luftwaffe', 'https://en.wikipedia.org/wiki/General_of_the_branch', 'https://en.wikipedia.org/wiki/Luftwaffe', 'https://en.wikipedia.org/wiki/Hermann_G%C3%B6ring'] 506 532 What is the full name of the district where the Memory of Mankind project is located? Bezirk Gmunden https://en.wikipedia.org/wiki/Memory_of_Mankind https://en.wikipedia.org/wiki/Hallstatt https://en.wikipedia.org/wiki/Gmunden_District Multiple constraints ['https://en.wikipedia.org/wiki/Memory_of_Mankind', 'https://en.wikipedia.org/wiki/Hallstatt', 'https://en.wikipedia.org/wiki/Gmunden_District'] 507 533 How old was the journalist who reviewed the first iPad for the Wall St Journal when the first iPad came out? Walt Mossberg was 63 when the first iPad was released. https://en.wikipedia.org/wiki/IPad# https://en.wikipedia.org/wiki/Walt_Mossberg Numerical reasoning ['https://en.wikipedia.org/wiki/IPad#', 'https://en.wikipedia.org/wiki/Walt_Mossberg'] 508 534 Mass Effect 2 once won the D.I.C.E. Award for Game of the Year. Who was the director for the game that won the same award twelve years earlier? Martin Hollis https://en.wikipedia.org/wiki/List_of_Game_of_the_Year_awards https://en.wikipedia.org/wiki/GoldenEye_007_(1997_video_game) https://en.wikipedia.org/wiki/D.I.C.E._Award_for_Game_of_the_Year Numerical reasoning | Temporal reasoning ['https://en.wikipedia.org/wiki/List_of_Game_of_the_Year_awards', 'https://en.wikipedia.org/wiki/GoldenEye_007_(1997_video_game)', 'https://en.wikipedia.org/wiki/D.I.C.E._Award_for_Game_of_the_Year'] 509 535 As of August 3, 2024, what is the capital of the country with the largest energy complex in South Eastern Europe? Sofia is the capital of Bulgaria, which is home to the largest energy complex in South Eastern Europe, the Maritsa Iztok Complex in Galabovo. https://en.wikipedia.org/wiki/Maritsa_Iztok_Complex https://en.wikipedia.org/wiki/Bulgaria Multiple constraints ['https://en.wikipedia.org/wiki/Maritsa_Iztok_Complex', 'https://en.wikipedia.org/wiki/Bulgaria'] 510 536 As of August 2024, which cast member fired from Saturday Night Live appeared on the show Hot Ones? Shane Gillis https://en.wikipedia.org/wiki/Saturday_Night_Live#Controversies https://en.wikipedia.org/wiki/List_of_Hot_Ones_episodes#Season_23_(2024) Tabular reasoning | Multiple constraints ['https://en.wikipedia.org/wiki/Saturday_Night_Live#Controversies', 'https://en.wikipedia.org/wiki/List_of_Hot_Ones_episodes#Season_23_(2024)'] 511 537 How many more solo studio albums did Taylor Swift release than Beyonce between the years 2003-2023 (not including re-recorded albums)? 3 https://en.wikipedia.org/wiki/Beyonc%C3%A9# https://en.wikipedia.org/wiki/Taylor_Swift# Numerical reasoning | Multiple constraints ['https://en.wikipedia.org/wiki/Beyonc%C3%A9#', 'https://en.wikipedia.org/wiki/Taylor_Swift#'] 512 538 Andre the Giant's favorite acting role was for a 1987 film. Who was the director of that film? Rob Reiner https://en.wikipedia.org/wiki/Andr%C3%A9_the_Giant https://en.wikipedia.org/wiki/The_Princess_Bride_(film) Multiple constraints ['https://en.wikipedia.org/wiki/Andr%C3%A9_the_Giant', 'https://en.wikipedia.org/wiki/The_Princess_Bride_(film)'] 513 539 What lasted longer: the reign of Queen Elizabeth II or the life of Julius Caesar? The reign of Queen Elizabeth II https://en.wikipedia.org/wiki/Elizabeth_II https://en.wikipedia.org/wiki/Julius_Caesar Temporal reasoning ['https://en.wikipedia.org/wiki/Elizabeth_II', 'https://en.wikipedia.org/wiki/Julius_Caesar'] 514 540 What is the birthday of the actor that portrayed the character David Cronenberg based on Marshall McLuhan? March 6, 1926 https://en.wikipedia.org/wiki/Marshall_McLuhan https://en.wikipedia.org/wiki/Videodrome https://en.wikipedia.org/wiki/Jack_Creley Multiple constraints ['https://en.wikipedia.org/wiki/Marshall_McLuhan', 'https://en.wikipedia.org/wiki/Videodrome', 'https://en.wikipedia.org/wiki/Jack_Creley'] 515 541 The first president of the International Olympic Committee was born on a Greek island, belonging to which island group? Cyclades https://en.m.wikipedia.org/wiki/President_of_the_International_Olympic_Committee https://en.m.wikipedia.org/wiki/Demetrios_Vikelas https://en.m.wikipedia.org/wiki/Ermoupoli Multiple constraints ['https://en.m.wikipedia.org/wiki/President_of_the_International_Olympic_Committee', 'https://en.m.wikipedia.org/wiki/Demetrios_Vikelas', 'https://en.m.wikipedia.org/wiki/Ermoupoli'] 516 542 Which film featuring a solar eclipse in its opening scene is adapted from the same source material as a David Lynch movie? Dune: Part Two https://en.wikipedia.org/wiki/List_of_films_featuring_eclipses https://en.wikipedia.org/wiki/David_Lynch_filmography https://en.wikipedia.org/wiki/Dune_(novel) Multiple constraints ['https://en.wikipedia.org/wiki/List_of_films_featuring_eclipses', 'https://en.wikipedia.org/wiki/David_Lynch_filmography', 'https://en.wikipedia.org/wiki/Dune_(novel)'] 517 543 What age was the Director of Inception (2010) when the film was released in the UK? 39 https://en.wikipedia.org/wiki/Inception https://en.wikipedia.org/wiki/Christopher_Nolan Numerical reasoning ['https://en.wikipedia.org/wiki/Inception', 'https://en.wikipedia.org/wiki/Christopher_Nolan'] 518 544 During the same year the Orlando Magic lost to the Los Angeles Lakers in their second NBA Finals appearance, what strain of flu spread into pandemic status throughout the world? The Swine Flu. https://en.wikipedia.org/wiki/Orlando_Magic https://en.wikipedia.org/wiki/2009_swine_flu_pandemic Temporal reasoning ['https://en.wikipedia.org/wiki/Orlando_Magic', 'https://en.wikipedia.org/wiki/2009_swine_flu_pandemic'] 519 545 Who won the Academy Award for Best Actor the year that John Steinbeck was awarded the Nobel prize for Literature? Gregory Peck https://en.wikipedia.org/wiki/List_of_Nobel_laureates_in_Literature#1960 https://en.wikipedia.org/wiki/Academy_Award_for_Best_Actor#1960s Multiple constraints ['https://en.wikipedia.org/wiki/List_of_Nobel_laureates_in_Literature#1960', 'https://en.wikipedia.org/wiki/Academy_Award_for_Best_Actor#1960s'] 520 546 What was the population in 2020 of the city that is in the name of the football team that won the Super Bowl five years before Matthew McConaughey won best actor at the Academy Awards? 302,971 https://en.wikipedia.org/wiki/List_of_awards_and_nominations_received_by_Matthew_McConaughey https://en.wikipedia.org/wiki/Super_Bowl_XLIII https://en.wikipedia.org/wiki/Pittsburgh Numerical reasoning | Multiple constraints | Temporal reasoning ['https://en.wikipedia.org/wiki/List_of_awards_and_nominations_received_by_Matthew_McConaughey', 'https://en.wikipedia.org/wiki/Super_Bowl_XLIII', 'https://en.wikipedia.org/wiki/Pittsburgh'] 521 547 How many England caps were won by university rugby teammates Will Carling, Chris Oti, and Andy Mullins? Round your answer to the nearest ten. 90 https://en.wikipedia.org/wiki/Will_Carling https://en.wikipedia.org/wiki/Chris_Oti https://en.wikipedia.org/wiki/Andy_Mullins_(rugby_union) Numerical reasoning | Tabular reasoning | Post processing ['https://en.wikipedia.org/wiki/Will_Carling', 'https://en.wikipedia.org/wiki/Chris_Oti', 'https://en.wikipedia.org/wiki/Andy_Mullins_(rugby_union)'] 522 548 What two actors starred in both The Craft and Scream in the same year? Neve Cambell and Skeet Ulrich https://en.wikipedia.org/wiki/Scream_(1996_film) https://en.wikipedia.org/wiki/The_Craft_(film) Multiple constraints ['https://en.wikipedia.org/wiki/Scream_(1996_film)', 'https://en.wikipedia.org/wiki/The_Craft_(film)'] 523 549 How many letters were in the name of the first single by the artist who played the first concert at Principality Stadium? 12 https://en.wikipedia.org/wiki/Millennium_Stadium https://en.wikipedia.org/wiki/List_of_concerts_at_the_Millennium_Stadium https://en.wikipedia.org/wiki/Manic_Street_Preachers Numerical reasoning ['https://en.wikipedia.org/wiki/Millennium_Stadium', 'https://en.wikipedia.org/wiki/List_of_concerts_at_the_Millennium_Stadium', 'https://en.wikipedia.org/wiki/Manic_Street_Preachers'] 524 550 How many more letters are in the first name of the eighth Director of Special Forces (United Kingdom) than the runner who won Silver in the 1985 UK Athletics Championship 10,000 meters event? Give the answer in morse code. ....- https://en.wikipedia.org/wiki/Director_Special_Forces https://en.wikipedia.org/wiki/1985_UK_Athletics_Championships https://en.wikipedia.org/wiki/Morse_code Numerical reasoning | Tabular reasoning | Multiple constraints | Post processing ['https://en.wikipedia.org/wiki/Director_Special_Forces', 'https://en.wikipedia.org/wiki/1985_UK_Athletics_Championships', 'https://en.wikipedia.org/wiki/Morse_code'] 525 551 What popular ice cream dessert shares its birthplace with Fred Rogers? The Banana Split https://en.wikipedia.org/wiki/Fred_Rogers https://en.wikipedia.org/wiki/Latrobe,_Pennsylvania Multiple constraints ['https://en.wikipedia.org/wiki/Fred_Rogers', 'https://en.wikipedia.org/wiki/Latrobe,_Pennsylvania'] 526 552 If the author of the philosophical magnum opus Being and Time were to adopt the last name of the winner of the Nobel Prize for literature in 1964 and the middle name of the person to first break the 4-minute mile as the middle name, what would the full name be? Martin Gilbert Sartre https://en.wikipedia.org/wiki/Being_and_Time https://en.wikipedia.org/wiki/1964_Nobel_Prize_in_Literature https://en.wikipedia.org/wiki/Four-minute_mile https://en.wikipedia.org/wiki/Roger_Bannister Multiple constraints | Temporal reasoning ['https://en.wikipedia.org/wiki/Being_and_Time', 'https://en.wikipedia.org/wiki/1964_Nobel_Prize_in_Literature', 'https://en.wikipedia.org/wiki/Four-minute_mile', 'https://en.wikipedia.org/wiki/Roger_Bannister'] 527 553 What is the percentage increase of total deaths of Japanese during World War One versus World War Two? Use the largest number count found to calculate as these numbers are often ranges. 66409.3% increase https://en.wikipedia.org/wiki/World_War_I_casualties https://en.wikipedia.org/wiki/World_War_II_casualties Numerical reasoning | Tabular reasoning ['https://en.wikipedia.org/wiki/World_War_I_casualties', 'https://en.wikipedia.org/wiki/World_War_II_casualties'] 528 554 Baldur's Gate 3 was released in 2023 and the game and its staff have received countless awards. How old will Astarion's voice actor be on August 15, 2035? 57 years old https://en.wikipedia.org/wiki/Baldur%27s_Gate_3# https://en.wikipedia.org/wiki/Neil_Newbon Numerical reasoning | Post processing | Temporal reasoning ['https://en.wikipedia.org/wiki/Baldur%27s_Gate_3#', 'https://en.wikipedia.org/wiki/Neil_Newbon'] 529 555 How old would Olivia Newton-John have been at the release of Grease 2 in the United States? 33 years old. https://en.wikipedia.org/wiki/Olivia_Newton-John https://en.wikipedia.org/wiki/Grease_2 Numerical reasoning ['https://en.wikipedia.org/wiki/Olivia_Newton-John', 'https://en.wikipedia.org/wiki/Grease_2'] 530 556 As of 2024, what is the street address of the headquarters of the American online video sharing platform that was launched in the same year as the founding of the Vancouver Cherry Blossom festival? 901 Cherry Avenue, San Bruno, California, United States https://en.wikipedia.org/wiki/Vancouver_Cherry_Blossom_Festival https://en.wikipedia.org/wiki/YouTube Multiple constraints ['https://en.wikipedia.org/wiki/Vancouver_Cherry_Blossom_Festival', 'https://en.wikipedia.org/wiki/YouTube'] 531 557 Coached in the 2023/24 season by Dusty May, this college basketball player was the 2023/24 co-winner of the AAC Player of the Year Award. Who is that player? Johnell Davis https://en.wikipedia.org/wiki/Florida_Atlantic_Owls_men%27s_basketball https://en.wikipedia.org/wiki/American_Athletic_Conference_Men%27s_Basketball_Player_of_the_Year https://en.wikipedia.org/wiki/Johnell_Davis Tabular reasoning | Multiple constraints ['https://en.wikipedia.org/wiki/Florida_Atlantic_Owls_men%27s_basketball', 'https://en.wikipedia.org/wiki/American_Athletic_Conference_Men%27s_Basketball_Player_of_the_Year', 'https://en.wikipedia.org/wiki/Johnell_Davis'] 532 558 Who was number 3 on the 1976-77 NBA World Champions team? Herm Gilliam https://en.wikipedia.org/wiki/1977_NBA_Finals https://en.wikipedia.org/wiki/1976%E2%80%9377_Portland_Trail_Blazers_season Multiple constraints | Post processing ['https://en.wikipedia.org/wiki/1977_NBA_Finals', 'https://en.wikipedia.org/wiki/1976%E2%80%9377_Portland_Trail_Blazers_season'] 533 559 As of August 3rd 2024, how many countries are full members of the council that oversees the sport that Henry Simonds plays? 12 https://en.wikipedia.org/wiki/Henry_Simonds https://en.wikipedia.org/wiki/Cricket#Governance Numerical reasoning | Tabular reasoning ['https://en.wikipedia.org/wiki/Henry_Simonds', 'https://en.wikipedia.org/wiki/Cricket#Governance'] 534 560 How many pages do the first edition harry potter series books have combined? 3407 https://en.wikipedia.org/wiki/Harry_Potter https://en.wikipedia.org/wiki/Harry_Potter_and_the_Philosopher%27s_Stone https://en.wikipedia.org/wiki/Harry_Potter_and_the_Chamber_of_Secrets https://en.wikipedia.org/wiki/Harry_Potter_and_the_Prisoner_of_Azkaban https://en.wikipedia.org/wiki/Harry_Potter_and_the_Goblet_of_Fire https://en.wikipedia.org/wiki/Harry_Potter_and_the_Order_of_the_Phoenix https://en.wikipedia.org/wiki/Harry_Potter_and_the_Half-Blood_Prince https://en.wikipedia.org/wiki/Harry_Potter_and_the_Deathly_Hallows Numerical reasoning | Multiple constraints | Post processing ['https://en.wikipedia.org/wiki/Harry_Potter', 'https://en.wikipedia.org/wiki/Harry_Potter_and_the_Philosopher%27s_Stone', 'https://en.wikipedia.org/wiki/Harry_Potter_and_the_Chamber_of_Secrets', 'https://en.wikipedia.org/wiki/Harry_Potter_and_the_Prisoner_of_Azkaban', 'https://en.wikipedia.org/wiki/Harry_Potter_and_the_Goblet_of_Fire', 'https://en.wikipedia.org/wiki/Harry_Potter_and_the_Order_of_the_Phoenix', 'https://en.wikipedia.org/wiki/Harry_Potter_and_the_Half-Blood_Prince', 'https://en.wikipedia.org/wiki/Harry_Potter_and_the_Deathly_Hallows'] 535 561 I was one of Bad Religion's drummers between 1990 and 2023. I am from California, but not Los Angeles. Who am I? Brooks Wackerman https://en.wikipedia.org/wiki/List_of_Bad_Religion_members https://en.wikipedia.org/wiki/Jamie_Miller_(drummer) https://en.wikipedia.org/wiki/Lucky_Lehrer https://en.wikipedia.org/wiki/Bobby_Schayer https://en.wikipedia.org/wiki/Brooks_Wackerman Multiple constraints ['https://en.wikipedia.org/wiki/List_of_Bad_Religion_members', 'https://en.wikipedia.org/wiki/Jamie_Miller_(drummer)', 'https://en.wikipedia.org/wiki/Lucky_Lehrer', 'https://en.wikipedia.org/wiki/Bobby_Schayer', 'https://en.wikipedia.org/wiki/Brooks_Wackerman'] 536 562 What was the pseudonym of one of the co-founders of the Eagle Awards that won Favourite Specialist Comics Publication/Trade Publication 1977 and 1978? Burt https://en.wikipedia.org/wiki/Eagle_Awards https://en.wikipedia.org/wiki/Richard_Burton_(comics) Multiple constraints ['https://en.wikipedia.org/wiki/Eagle_Awards', 'https://en.wikipedia.org/wiki/Richard_Burton_(comics)'] 537 563 I am thinking of a movie where Hans Zimmer won a Grammy Award for his work. He won the Grammy award the same year that he did his first musical score for film director Michael Bay. Can you please tell me the name of that movie? Crimson Tide https://en.wikipedia.org/wiki/Michael_Bay https://en.wikipedia.org/wiki/Hans_Zimmer#Grammy_Awards https://en.wikipedia.org/wiki/Bad_Boys_(1995_film) https://en.wikipedia.org/wiki/The_Rock_(film) https://en.wikipedia.org/wiki/Armageddon_(1998_film) https://en.wikipedia.org/wiki/Pearl_Harbor_(film) https://en.wikipedia.org/wiki/Bad_Boys_II https://en.wikipedia.org/wiki/The_Island_(2005_film) https://en.wikipedia.org/wiki/Transformers_(film) https://en.wikipedia.org/wiki/Transformers:_Revenge_of_the_Fallen https://en.wikipedia.org/wiki/Transformers:_Dark_of_the_Moon, https://en.wikipedia.org/wiki/Pain_%26_Gain, https://en.wikipedia.org/wiki/Transformers:_Age_of_Extinction, https://en.wikipedia.org/wiki/13_Hours:_The_Secret_Soldiers_of_Benghazi, https://en.wikipedia.org/wiki/Transformers:_The_Last_Knight, https://en.wikipedia.org/wiki/6_Underground_(film), https://en.wikipedia.org/wiki/Ambulance_(2022_film) Multiple constraints ['https://en.wikipedia.org/wiki/Michael_Bay', 'https://en.wikipedia.org/wiki/Hans_Zimmer#Grammy_Awards', 'https://en.wikipedia.org/wiki/Bad_Boys_(1995_film)', 'https://en.wikipedia.org/wiki/The_Rock_(film)', 'https://en.wikipedia.org/wiki/Armageddon_(1998_film)', 'https://en.wikipedia.org/wiki/Pearl_Harbor_(film)', 'https://en.wikipedia.org/wiki/Bad_Boys_II', 'https://en.wikipedia.org/wiki/The_Island_(2005_film)', 'https://en.wikipedia.org/wiki/Transformers_(film)', 'https://en.wikipedia.org/wiki/Transformers:_Revenge_of_the_Fallen', 'https://en.wikipedia.org/wiki/Transformers:_Dark_of_the_Moon, https://en.wikipedia.org/wiki/Pain_%26_Gain, https://en.wikipedia.org/wiki/Transformers:_Age_of_Extinction, https://en.wikipedia.org/wiki/13_Hours:_The_Secret_Soldiers_of_Benghazi, https://en.wikipedia.org/wiki/Transformers:_The_Last_Knight, https://en.wikipedia.org/wiki/6_Underground_(film), https://en.wikipedia.org/wiki/Ambulance_(2022_film)'] 538 564 As of July 4th, 2024, what is the ratio of the number of years in the Early Dynastic Period of Egypt to the number of years since the United States declared independence? Round your answer to the nearest whole number. 2 https://en.wikipedia.org/wiki/Ancient_Egypt https://en.wikipedia.org/wiki/United_States Numerical reasoning | Tabular reasoning ['https://en.wikipedia.org/wiki/Ancient_Egypt', 'https://en.wikipedia.org/wiki/United_States'] 539 565 Who was the British monarch when Michigan was admitted as a state in the United States of America? King William IV. https://en.wikipedia.org/wiki/Michigan https://en.wikipedia.org/wiki/List_of_British_monarchs Temporal reasoning ['https://en.wikipedia.org/wiki/Michigan', 'https://en.wikipedia.org/wiki/List_of_British_monarchs'] 540 566 Of the participants in the first round of the 2024 Yucatán Open (Doubles), which Mexican player had received two wild cards during their career prior to August 4, 2024? Alan Fernando Rubio Fierros https://en.wikipedia.org/wiki/2024_Yucat%C3%A1n_Open_%E2%80%93_Doubles https://en.wikipedia.org/wiki/George_Goldhoff https://en.wikipedia.org/wiki/James_Trotter_(tennis) https://en.wikipedia.org/wiki/Alan_Fernando_Rubio_Fierros https://en.wikipedia.org/wiki/Rodrigo_Pacheco_M%C3%A9ndez https://en.wikipedia.org/wiki/Roberto_Cid_Subervi https://en.wikipedia.org/wiki/Nick_Hardt https://en.wikipedia.org/wiki/Juan_Pablo_Paz_(tennis) https://en.wikipedia.org/wiki/Jack_Vance_(tennis) https://en.wikipedia.org/wiki/Stefan_Kozlov https://en.wikipedia.org/wiki/Murkel_Dellien, https://en.wikipedia.org/wiki/Facundo_Mena, https://en.wikipedia.org/wiki/Louis_Wessels, https://en.wikipedia.org/wiki/Gabi_Adrian_Boitan, https://en.wikipedia.org/wiki/Trey_Hilderbrand, https://en.wikipedia.org/wiki/Oliver_Crawford_(tennis), https://en.wikipedia.org/wiki/Tristan_Boyer, https://en.wikipedia.org/wiki/Thomas_Fancutt, https://en.wikipedia.org/wiki/Hunter_Reese, https://en.wikipedia.org/wiki/Alex_Hern%C3%A1ndez_(tennis), Tabular reasoning | Multiple constraints ['https://en.wikipedia.org/wiki/2024_Yucat%C3%A1n_Open_%E2%80%93_Doubles', 'https://en.wikipedia.org/wiki/George_Goldhoff', 'https://en.wikipedia.org/wiki/James_Trotter_(tennis)', 'https://en.wikipedia.org/wiki/Alan_Fernando_Rubio_Fierros', 'https://en.wikipedia.org/wiki/Rodrigo_Pacheco_M%C3%A9ndez', 'https://en.wikipedia.org/wiki/Roberto_Cid_Subervi', 'https://en.wikipedia.org/wiki/Nick_Hardt', 'https://en.wikipedia.org/wiki/Juan_Pablo_Paz_(tennis)', 'https://en.wikipedia.org/wiki/Jack_Vance_(tennis)', 'https://en.wikipedia.org/wiki/Stefan_Kozlov', 'https://en.wikipedia.org/wiki/Murkel_Dellien, https://en.wikipedia.org/wiki/Facundo_Mena, https://en.wikipedia.org/wiki/Louis_Wessels, https://en.wikipedia.org/wiki/Gabi_Adrian_Boitan, https://en.wikipedia.org/wiki/Trey_Hilderbrand, https://en.wikipedia.org/wiki/Oliver_Crawford_(tennis), https://en.wikipedia.org/wiki/Tristan_Boyer, https://en.wikipedia.org/wiki/Thomas_Fancutt, https://en.wikipedia.org/wiki/Hunter_Reese, https://en.wikipedia.org/wiki/Alex_Hern%C3%A1ndez_(tennis), '] 541 567 Which Kiwi author died the same year as the first British Prime Minister to be elected after the end of World War 1? Katherine Mansfield https://en.wikipedia.org/wiki/List_of_prime_ministers_of_the_United_Kingdom https://en.wikipedia.org/wiki/Bonar_Law https://en.wikipedia.org/wiki/1923#Deaths https://en.wikipedia.org/wiki/World_War_I Tabular reasoning | Multiple constraints ['https://en.wikipedia.org/wiki/List_of_prime_ministers_of_the_United_Kingdom', 'https://en.wikipedia.org/wiki/Bonar_Law', 'https://en.wikipedia.org/wiki/1923#Deaths', 'https://en.wikipedia.org/wiki/World_War_I'] 542 568 What war memorial was constructed and completed at the same time as the beginning of WWII, located on an island in the Potomac River in the US capital, and honors four of the eight federal uniformed services of the US that lost their lives at sea during the war and other previous conflicts? Navy-Merchant Marine Memorial https://en.wikipedia.org/wiki/Navy_%E2%80%93_Merchant_Marine_Memorial https://en.wikipedia.org/wiki/Columbia_Island_(Washington,_D.C.) https://en.wikipedia.org/wiki/Uniformed_services_of_the_United_States https://en.wikipedia.org/wiki/World_War_II Numerical reasoning | Multiple constraints | Temporal reasoning ['https://en.wikipedia.org/wiki/Navy_%E2%80%93_Merchant_Marine_Memorial', 'https://en.wikipedia.org/wiki/Columbia_Island_(Washington,_D.C.)', 'https://en.wikipedia.org/wiki/Uniformed_services_of_the_United_States', 'https://en.wikipedia.org/wiki/World_War_II'] 543 569 As of 2024, which islands in the Indonesian archipelago are home to the UNESCO World Heritage Site for the largest species of extant lizard? The Lesser Sundra Islands https://en.wikipedia.org/wiki/List_of_largest_extant_lizards https://en.wikipedia.org/wiki/Indonesia https://en.wikipedia.org/wiki/Komodo_National_Park Tabular reasoning | Multiple constraints ['https://en.wikipedia.org/wiki/List_of_largest_extant_lizards', 'https://en.wikipedia.org/wiki/Indonesia', 'https://en.wikipedia.org/wiki/Komodo_National_Park'] 544 570 Which of these series was published earliest? Wedding Peach, Tokyo Mew Mew, Sailor Moon Sailor Moon https://en.wikipedia.org/wiki/Sailor_Moon https://en.wikipedia.org/wiki/Wedding_Peach https://en.wikipedia.org/wiki/Tokyo_Mew_Mew Multiple constraints ['https://en.wikipedia.org/wiki/Sailor_Moon', 'https://en.wikipedia.org/wiki/Wedding_Peach', 'https://en.wikipedia.org/wiki/Tokyo_Mew_Mew'] 545 571 How many published literary works had 1998's Nobel Prize in Literatures' recipient have at the time they received the award? Only count publications in the author's native language. 21 https://en.wikipedia.org/wiki/List_of_Nobel_laureates_in_Literature https://en.wikipedia.org/wiki/José_Saramago# Numerical reasoning | Tabular reasoning | Temporal reasoning ['https://en.wikipedia.org/wiki/List_of_Nobel_laureates_in_Literature', 'https://en.wikipedia.org/wiki/José_Saramago#'] 546 572 What is the name of the high school and hometown of the Olympic Gold Medalist winner who won 4-3 against Hassan Yazdani at the Japan Olympics in 2020? Graham High School in St. Paris Ohio https://en.wikipedia.org/wiki/2020_Summer_Olympics https://en.wikipedia.org/wiki/Wrestling_at_the_2020_Summer_Olympics#Medalists https://en.wikipedia.org/wiki/Wrestling_at_the_2020_Summer_Olympics_%E2%80%93_Men%27s_freestyle_86_kg https://en.wikipedia.org/wiki/David_Taylor_(wrestler,_born_1990) https://en.wikipedia.org/wiki/Graham_High_School_(St._Paris,_Ohio) Multiple constraints ['https://en.wikipedia.org/wiki/2020_Summer_Olympics', 'https://en.wikipedia.org/wiki/Wrestling_at_the_2020_Summer_Olympics#Medalists', 'https://en.wikipedia.org/wiki/Wrestling_at_the_2020_Summer_Olympics_%E2%80%93_Men%27s_freestyle_86_kg', 'https://en.wikipedia.org/wiki/David_Taylor_(wrestler,_born_1990)', 'https://en.wikipedia.org/wiki/Graham_High_School_(St._Paris,_Ohio)'] 547 573 Sworn in for his second term, who was US President during the only year in history to feature Triple Crown winners in both horse racing AND baseball (War Admiral and St. Louis Cardinals' left fielder Joe Medwick, respectively), as of August 3, 2024? Franklin D. Roosevelt (1937) https://en.wikipedia.org/wiki/Triple_Crown_of_Thoroughbred_Racing_(United_States)#Winners_of_the_Triple_Crown https://en.wikipedia.org/wiki/Triple_Crown_(baseball) https://en.wikipedia.org/wiki/Franklin_D._Roosevelt Multiple constraints ['https://en.wikipedia.org/wiki/Triple_Crown_of_Thoroughbred_Racing_(United_States)#Winners_of_the_Triple_Crown', 'https://en.wikipedia.org/wiki/Triple_Crown_(baseball)', 'https://en.wikipedia.org/wiki/Franklin_D._Roosevelt'] 548 574 How many years after Anton Grylewicz's date of birth was the second SpongeBob Squarepants movie released? Round down to the nearest year (e.g. January 1999 to December 2000 = 1 year, despite being closer to 2). 130 https://en.wikipedia.org/wiki/Anton_Grylewicz https://en.wikipedia.org/wiki/SpongeBob_SquarePants#Franchise Numerical reasoning | Temporal reasoning ['https://en.wikipedia.org/wiki/Anton_Grylewicz', 'https://en.wikipedia.org/wiki/SpongeBob_SquarePants#Franchise'] 549 575 Out of the following man-portable launchers, which entered service with their respective military last? A) FGM-148 Javelin B) 9K38 Igla C) FIM-92 Stinger. A) FGM-148 Javelin https://en.wikipedia.org/wiki/FGM-148_Javelin https://en.wikipedia.org/wiki/9K38_Igla https://en.wikipedia.org/wiki/FIM-92_Stinger Temporal reasoning ['https://en.wikipedia.org/wiki/FGM-148_Javelin', 'https://en.wikipedia.org/wiki/9K38_Igla', 'https://en.wikipedia.org/wiki/FIM-92_Stinger'] 550 576 "As of August 4, 2024, Rosamund Pike voices a character in a podcast about the so-called ""First Female President."" How many siblings does the character have?" 10 https://en.wikipedia.org/wiki/Edith! https://en.wikipedia.org/wiki/Edith_Wilson Multiple constraints | Post processing ['https://en.wikipedia.org/wiki/Edith!', 'https://en.wikipedia.org/wiki/Edith_Wilson'] 551 577 Which angle is greater: the recommended angle a glass should be tilted when pouring a pint of Guinness, or the angle the nose of a light aircraft is tilted to effect liftoff? The recommended angle a glass should be tilted when pouring a pint of Guinness https://en.wikipedia.org/wiki/Guinness https://en.wikipedia.org/wiki/Takeoff Multiple constraints ['https://en.wikipedia.org/wiki/Guinness', 'https://en.wikipedia.org/wiki/Takeoff'] 552 578 Which NFL team won the second Super Bowl after Donald Trump was elected to be President during the term directly after Obama? The Philadelphia Eagles won the second Super Bowl during Trump's term in 2018. https://en.wikipedia.org/wiki/Philadelphia_Eagles https://en.wikipedia.org/wiki/Donald_Trump Multiple constraints ['https://en.wikipedia.org/wiki/Philadelphia_Eagles', 'https://en.wikipedia.org/wiki/Donald_Trump'] 553 579 The city which has India's most famous biryani dish was from 1956 until 2014 the capital of a state which subsequently split into two new states; what is the capital of the state which does not contain the city from where the famous biryani originates? Amaravati. https://en.wikipedia.org/wiki/Biryani https://en.wikipedia.org/wiki/Hyderabadi_biryani https://en.wikipedia.org/wiki/Hyderabad https://en.wikipedia.org/wiki/Andhra_Pradesh_(1956%E2%80%932014) https://en.wikipedia.org/wiki/Telangana https://en.wikipedia.org/wiki/Andhra_Pradesh https://en.wikipedia.org/wiki/Amaravati Multiple constraints ['https://en.wikipedia.org/wiki/Biryani', 'https://en.wikipedia.org/wiki/Hyderabadi_biryani', 'https://en.wikipedia.org/wiki/Hyderabad', 'https://en.wikipedia.org/wiki/Andhra_Pradesh_(1956%E2%80%932014)', 'https://en.wikipedia.org/wiki/Telangana', 'https://en.wikipedia.org/wiki/Andhra_Pradesh', 'https://en.wikipedia.org/wiki/Amaravati'] 554 580 I live in a US state that was admitted to the union in January 1959. What was the approximate population of the capital of my state according to the last census of the 20th century? 26,751 https://en.wikipedia.org/wiki/List_of_U.S._states_by_date_of_admission_to_the_Union https://en.wikipedia.org/wiki/Alaska https://en.wikipedia.org/wiki/Juneau,_Alaska Tabular reasoning | Multiple constraints | Temporal reasoning ['https://en.wikipedia.org/wiki/List_of_U.S._states_by_date_of_admission_to_the_Union', 'https://en.wikipedia.org/wiki/Alaska', 'https://en.wikipedia.org/wiki/Juneau,_Alaska'] 555 581 "When Metallica released its album ""Master of Puppets,"" how old were founding members James Hetfield and Lars Ulrich?" 22 https://en.wikipedia.org/wiki/Master_of_Puppets https://en.wikipedia.org/wiki/James_Hetfield https://en.wikipedia.org/wiki/Lars_Ulrich Temporal reasoning ['https://en.wikipedia.org/wiki/Master_of_Puppets', 'https://en.wikipedia.org/wiki/James_Hetfield', 'https://en.wikipedia.org/wiki/Lars_Ulrich'] 556 582 As of August 3rd 2024, the only documented genus of poison dart frog to be used in blow darts by Aboriginal South Americans contains many alkaloids, the most toxic of which, when exposed, irreversibly opens what kind of channels within nerve cells? Sodium https://en.wikipedia.org/wiki/Poison_dart_frog https://en.wikipedia.org/wiki/Phyllobates https://en.wikipedia.org/wiki/Batrachotoxin Multiple constraints ['https://en.wikipedia.org/wiki/Poison_dart_frog', 'https://en.wikipedia.org/wiki/Phyllobates', 'https://en.wikipedia.org/wiki/Batrachotoxin'] 557 584 What's the name of the third track on the third studio album of the KPop girl group that started in 1997 and had 5 members. Missing You https://en.wikipedia.org/wiki/List_of_South_Korean_idol_groups_(1990s) https://en.wikipedia.org/wiki/Baby_Vox#Studio_albums https://en.wikipedia.org/wiki/Come_Come_Come_Baby Tabular reasoning | Multiple constraints | Temporal reasoning ['https://en.wikipedia.org/wiki/List_of_South_Korean_idol_groups_(1990s)', 'https://en.wikipedia.org/wiki/Baby_Vox#Studio_albums', 'https://en.wikipedia.org/wiki/Come_Come_Come_Baby'] 558 585 Use this information: -The Dragonfly Sea is a novel by Yvonne Adhiambo Owuor. -It has a publisher. -The publisher has two co-founders. What was the original name of the university where the male co-founder of this publisher studied? King's College https://en.wikipedia.org/wiki/The_Dragonfly_Sea https://en.wikipedia.org/wiki/Alfred_A._Knopf https://en.wikipedia.org/wiki/Alfred_A._Knopf_Sr. https://en.wikipedia.org/wiki/Columbia_University Multiple constraints | Post processing ['https://en.wikipedia.org/wiki/The_Dragonfly_Sea', 'https://en.wikipedia.org/wiki/Alfred_A._Knopf', 'https://en.wikipedia.org/wiki/Alfred_A._Knopf_Sr.', 'https://en.wikipedia.org/wiki/Columbia_University'] 559 586 What was the most-sold album in the year that the Dallas Stars won the Stanley Cup in the 1990s? Millennium - Backstreet Boys (1999) https://en.wikipedia.org/wiki/List_of_Stanley_Cup_champions https://en.wikipedia.org/wiki/List_of_Billboard_200_number-one_albums_of_1999#:~:text=Millennium%20became%20the%20best%2Dselling,nomination%20at%20the%20Grammy%20Awards. Multiple constraints ['https://en.wikipedia.org/wiki/List_of_Stanley_Cup_champions', 'https://en.wikipedia.org/wiki/List_of_Billboard_200_number-one_albums_of_1999#:~:text=Millennium%20became%20the%20best%2Dselling,nomination%20at%20the%20Grammy%20Awards.'] 560 587 "Counting Crows burst onto the scene with hits like ""Mr. Jones,"" and ""Round Here."" Which of their albums came out the same year as the first Atlanta Summer Olympics?" Recovering the Satellites https://en.wikipedia.org/wiki/Summer_Olympic_Games https://en.wikipedia.org/wiki/Counting_Crows https://en.wikipedia.org/wiki/Recovering_the_Satellites Multiple constraints ['https://en.wikipedia.org/wiki/Summer_Olympic_Games', 'https://en.wikipedia.org/wiki/Counting_Crows', 'https://en.wikipedia.org/wiki/Recovering_the_Satellites'] 561 588 What are the lyrics to the popular, unofficial fight song of the university that the main character of Gilmore Girls attended? Boola boola, boola boola, boola boola, boola, boola Oh when we're through with those poor fellows They will holler boola, boo Rah, rah! Oh Yale, Eli Yale Oh Yale, Eli Yale Oh Yale, Eli Yale Oh Yale, Eli Yale https://en.wikipedia.org/wiki/Gilmore_Girls https://en.wikipedia.org/wiki/Yale_University https://en.wikipedia.org/wiki/Boola_Boola Multiple constraints | Post processing ['https://en.wikipedia.org/wiki/Gilmore_Girls', 'https://en.wikipedia.org/wiki/Yale_University', 'https://en.wikipedia.org/wiki/Boola_Boola'] 562 589 A unified team competed in the 1992 Summer Olympic games. How many years old was the first leader of the largest member nation of that team at the time of the game? 61 years old. https://en.wikipedia.org/wiki/Unified_Team_at_the_Olympics https://en.wikipedia.org/wiki/Soviet_Union https://en.wikipedia.org/wiki/List_of_presidents_of_Russia Numerical reasoning | Multiple constraints | Temporal reasoning ['https://en.wikipedia.org/wiki/Unified_Team_at_the_Olympics', 'https://en.wikipedia.org/wiki/Soviet_Union', 'https://en.wikipedia.org/wiki/List_of_presidents_of_Russia'] 563 590 In 1994, Linus Roache starred in Priest. Who composed the music on his next film? Edward Shearmur https://en.wikipedia.org/wiki/Priest_(1994_film) https://en.wikipedia.org/wiki/Linus_Roache https://en.wikipedia.org/wiki/The_Wings_of_the_Dove_(1997_film) Tabular reasoning | Temporal reasoning ['https://en.wikipedia.org/wiki/Priest_(1994_film)', 'https://en.wikipedia.org/wiki/Linus_Roache', 'https://en.wikipedia.org/wiki/The_Wings_of_the_Dove_(1997_film)'] 564 591 "How many years elapsed between the release of the song ""I Think I'm Go Go"" by the band Squeeze and the theatrical premier of E.T. the movie?" 2 https://en.wikipedia.org/wiki/I_Think_I'm_Go_Go https://en.wikipedia.org/wiki/E.T._the_Extra-Terrestrial Numerical reasoning | Temporal reasoning "[""https://en.wikipedia.org/wiki/I_Think_I'm_Go_Go"", 'https://en.wikipedia.org/wiki/E.T._the_Extra-Terrestrial']" 565 592 Who was the Super Bowl MVP's wife the year the building Rodney Gordon designed was sold to Capital and City Group? Abby McGrew https://en.wikipedia.org/wiki/Rodney_Gordon https://en.wikipedia.org/wiki/Target_House,_London https://en.wikipedia.org/wiki/Super_Bowl https://en.wikipedia.org/wiki/Super_Bowl_XLVI https://en.wikipedia.org/wiki/Eli_Manning Multiple constraints ['https://en.wikipedia.org/wiki/Rodney_Gordon', 'https://en.wikipedia.org/wiki/Target_House,_London', 'https://en.wikipedia.org/wiki/Super_Bowl', 'https://en.wikipedia.org/wiki/Super_Bowl_XLVI', 'https://en.wikipedia.org/wiki/Eli_Manning'] 566 593 Who designed the first 5 figurines in Wroclaw, Poland which now number in the hundreds and are a popularly looked for by tourists? Tomasz Moczek https://en.wikipedia.org/wiki/Wroc%C5%82aw https://en.wikipedia.org/wiki/Wroc%C5%82aw_Dwarfs Multiple constraints ['https://en.wikipedia.org/wiki/Wroc%C5%82aw', 'https://en.wikipedia.org/wiki/Wroc%C5%82aw_Dwarfs'] 567 594 What language is the main character's name in in the Disney film that came out in 1994? Swahili https://en.wikipedia.org/wiki/List_of_Walt_Disney_Animation_Studios_films#ep29 https://en.wikipedia.org/wiki/The_Lion_King https://en.wikipedia.org/wiki/Simba Tabular reasoning | Multiple constraints ['https://en.wikipedia.org/wiki/List_of_Walt_Disney_Animation_Studios_films#ep29', 'https://en.wikipedia.org/wiki/The_Lion_King', 'https://en.wikipedia.org/wiki/Simba'] 568 595 How many fewer races did Sebastian Vettel complete before he retired compared to his hero, Michael Schumacher? Seven https://en.wikipedia.org/wiki/Sebastian_Vettel https://en.wikipedia.org/wiki/Michael_Schumacher Numerical reasoning ['https://en.wikipedia.org/wiki/Sebastian_Vettel', 'https://en.wikipedia.org/wiki/Michael_Schumacher'] 569 596 Who was the MVP in the season that Cam Plante played in the National Hockey League? Wayne Gretzky https://en.wikipedia.org/wiki/Cam_Plante https://en.wikipedia.org/wiki/1984%E2%80%9385_NHL_season Tabular reasoning | Multiple constraints ['https://en.wikipedia.org/wiki/Cam_Plante', 'https://en.wikipedia.org/wiki/1984%E2%80%9385_NHL_season'] 570 597 How old was Katie Couric when Oprah Winfrey was 8 years old? Katie Couric was 5 years old when Oprah Winfrey was 8 years old. https://en.wikipedia.org/wiki/Oprah_Winfrey https://en.wikipedia.org/wiki/Katie_Couric Numerical reasoning ['https://en.wikipedia.org/wiki/Oprah_Winfrey', 'https://en.wikipedia.org/wiki/Katie_Couric'] 571 598 In 2003, Audible entered an agreement of exclusivity with a major brand. Who founded that brand? Steve Jobs, Steve Wozniak and Ronald Wayne https://en.wikipedia.org/wiki/Audible_(service) https://en.wikipedia.org/wiki/Apple_Inc. Multiple constraints ['https://en.wikipedia.org/wiki/Audible_(service)', 'https://en.wikipedia.org/wiki/Apple_Inc.'] 572 599 What is the difference in elevation between Mount Rainier and Condor Mountain? What is the answer in feet? 2,689 feet. https://en.wikipedia.org/wiki/Condor_(mountain) https://en.wikipedia.org/wiki/Mount_Rainier Multiple constraints ['https://en.wikipedia.org/wiki/Condor_(mountain)', 'https://en.wikipedia.org/wiki/Mount_Rainier'] 573 600 As of August 3, 2024, what is the main specialization of the hospital designed by Vasco Morais Palmeiro Regaleira that is located in the civil parish where the Monteiro-Mor Palace resides? Pulmonary medicine https://en.wikipedia.org/wiki/Monteiro-Mor_Palace https://en.wikipedia.org/wiki/Lumiar https://en.wikipedia.org/wiki/Hospital_Pulido_Valente Multiple constraints ['https://en.wikipedia.org/wiki/Monteiro-Mor_Palace', 'https://en.wikipedia.org/wiki/Lumiar', 'https://en.wikipedia.org/wiki/Hospital_Pulido_Valente'] 574 601 If the man that the SS Edmund Fitzgerald was named after was living at the time of the ship's sinking, how old was he? If he was already deceased, how long had he been dead? You may just use the year without regard for the date of birth. Edmund Fitzgerald, the man for whom the ill-fated ship SS Edmund Fitzgerald was named was born in 1895, he was 80 years old in 1975 when she sank. https://en.wikipedia.org/wiki/SS_Edmund_Fitzgerald https://en.wikipedia.org/wiki/Edmund_Fitzgerald_(disambiguation) Numerical reasoning | Post processing ['https://en.wikipedia.org/wiki/SS_Edmund_Fitzgerald', 'https://en.wikipedia.org/wiki/Edmund_Fitzgerald_(disambiguation)'] 575 602 How old was Benjamin Franklin when Wolfgang Amadeus Mozart was born? 50 years old. https://en.wikipedia.org/wiki/Benjamin_Franklin https://en.wikipedia.org/wiki/Wolfgang_Amadeus_Mozart Numerical reasoning ['https://en.wikipedia.org/wiki/Benjamin_Franklin', 'https://en.wikipedia.org/wiki/Wolfgang_Amadeus_Mozart'] 576 603 "The female of the pair who have been called ""the worst guests in 'Below Deck Sailing Yacht' history"" made her TV debut five seasons before which Bachelor?" Jake Pavelka https://en.wikipedia.org/wiki/Erica_Rose_(television_personality) https://en.wikipedia.org/wiki/The_Bachelor_(American_TV_series)_season_14 Numerical reasoning | Temporal reasoning ['https://en.wikipedia.org/wiki/Erica_Rose_(television_personality)', 'https://en.wikipedia.org/wiki/The_Bachelor_(American_TV_series)_season_14'] 577 604 As of 1 August 2024, How many more seasons did Outrageous Fortune have compared to bro'Town? 1 https://en.wikipedia.org/wiki/Bro%27Town https://en.wikipedia.org/wiki/Outrageous_Fortune_(TV_series) Numerical reasoning | Post processing ['https://en.wikipedia.org/wiki/Bro%27Town', 'https://en.wikipedia.org/wiki/Outrageous_Fortune_(TV_series)'] 578 605 What attraction in Walt Disney World opened exactly 50 years after the theme park originally opened? Remy's Ratatouille Adventure https://en.wikipedia.org/wiki/Walt_Disney_World https://en.wikipedia.org/wiki/Remy%27s_Ratatouille_Adventure Temporal reasoning ['https://en.wikipedia.org/wiki/Walt_Disney_World', 'https://en.wikipedia.org/wiki/Remy%27s_Ratatouille_Adventure'] 579 606 Which of these statements is true as of August 3rd, 2024? a) 221 Eos is roughly double the diameter of 1844 Susilva. b) 1844 Susilva is roughly double the diameter of 221 Eos. c) 221 Eos's diameter is roughly 150% of the diameter of 1844 Susilva. d) 1844 Susilva's diameter is roughly 20% the diameter of 221 Eos. d) 1844 Susilva's diameter is roughly 20% the diameter of 221 Eos. https://en.wikipedia.org/wiki/1844_Susilva https://en.wikipedia.org/wiki/221_Eos Numerical reasoning | Tabular reasoning ['https://en.wikipedia.org/wiki/1844_Susilva', 'https://en.wikipedia.org/wiki/221_Eos'] 580 607 How old was the Miss Miss Venezuela 1970 winner on the day the 68th Academy Awards was held? Bella La Rosa was 46 years old on March 25, 1996. https://en.wikipedia.org/wiki/Miss_Venezuela_1970 https://en.wikipedia.org/wiki/Bella_La_Rosa https://en.wikipedia.org/wiki/68th_Academy_Awards Numerical reasoning | Multiple constraints | Post processing | Temporal reasoning ['https://en.wikipedia.org/wiki/Miss_Venezuela_1970', 'https://en.wikipedia.org/wiki/Bella_La_Rosa', 'https://en.wikipedia.org/wiki/68th_Academy_Awards'] 581 608 Among Aristotle, Alexander the Great, Socrates, and Plato, which of them taught the others? What was their order chronologically? Socrates taught Plato and Plato taught Aristotle. Aristotle taught Alexander the Great. So, chronologically, it was Socrates, Plato, Aristotle, and finally Alexander the Great. https://en.wikipedia.org/wiki/Aristotle https://en.wikipedia.org/wiki/Socrates https://en.wikipedia.org/wiki/Plato https://en.wikipedia.org/wiki/Alexander_the_Great Multiple constraints | Temporal reasoning ['https://en.wikipedia.org/wiki/Aristotle', 'https://en.wikipedia.org/wiki/Socrates', 'https://en.wikipedia.org/wiki/Plato', 'https://en.wikipedia.org/wiki/Alexander_the_Great'] 582 609 Where was Robert Vesco living when Bank of Credit and Commerce International was formally liquidated? Cuba https://en.wikipedia.org/wiki/Bank_of_Credit_and_Commerce_International https://en.wikipedia.org/wiki/Robert_Vesco Multiple constraints ['https://en.wikipedia.org/wiki/Bank_of_Credit_and_Commerce_International', 'https://en.wikipedia.org/wiki/Robert_Vesco'] 583 610 Who was the Prime Minister of Canada the first time that The Toronto Maple Leafs won The Stanley Cup? R. B. Bennett https://en.wikipedia.org/wiki/List_of_Stanley_Cup_champions https://en.wikipedia.org/wiki/List_of_prime_ministers_of_Canada Multiple constraints ['https://en.wikipedia.org/wiki/List_of_Stanley_Cup_champions', 'https://en.wikipedia.org/wiki/List_of_prime_ministers_of_Canada'] 584 611 Which political party held the most seats in the Leeds City Council election during the year that philosopher John Wall was born? Conservatives held the most seats in the Leeds City Council election in 1965 when John Wall was born. https://en.wikipedia.org/wiki/John_Wall_(philosopher) https://en.wikipedia.org/wiki/1965_Leeds_City_Council_election Tabular reasoning ['https://en.wikipedia.org/wiki/John_Wall_(philosopher)', 'https://en.wikipedia.org/wiki/1965_Leeds_City_Council_election'] 585 613 What was the release date of the movie directed by Gordon Douglas which featured American decathlete who was a 1984 olympic torch runner and first African American to light the Olympic Cauldron? April 2, 1961 https://en.wikipedia.org/wiki/1984_Summer_Olympics https://en.wikipedia.org/wiki/Rafer_Johnson https://en.wikipedia.org/wiki/The_Sins_of_Rachel_Cade Multiple constraints ['https://en.wikipedia.org/wiki/1984_Summer_Olympics', 'https://en.wikipedia.org/wiki/Rafer_Johnson', 'https://en.wikipedia.org/wiki/The_Sins_of_Rachel_Cade'] 586 615 How long after Archduke Franz Ferdinand received Artstetten Castle did he have his first child? Twelve Years https://en.wikipedia.org/wiki/Artstetten_Castle https://en.wikipedia.org/wiki/Archduke_Franz_Ferdinand_of_Austria Numerical reasoning | Temporal reasoning ['https://en.wikipedia.org/wiki/Artstetten_Castle', 'https://en.wikipedia.org/wiki/Archduke_Franz_Ferdinand_of_Austria'] 587 616 Who was the manager of the team that won the first Football League after the death of Queen Victoria? Tom Watson was the manager of Liverpool F.C. in 1901. https://en.wikipedia.org/wiki/Queen_Victoria https://en.wikipedia.org/wiki/1900%E2%80%9301_Football_League#Final_league_tables https://en.wikipedia.org/wiki/List_of_Liverpool_F.C._managers#Managers Tabular reasoning | Multiple constraints | Temporal reasoning ['https://en.wikipedia.org/wiki/Queen_Victoria', 'https://en.wikipedia.org/wiki/1900%E2%80%9301_Football_League#Final_league_tables', 'https://en.wikipedia.org/wiki/List_of_Liverpool_F.C._managers#Managers'] 588 619 What is the nickname for the city where Mette Solli was born? Give the answer in Norwegian. Rosenes by https://en.wikipedia.org/wiki/Mette_Solli https://en.wikipedia.org/wiki/Molde Post processing ['https://en.wikipedia.org/wiki/Mette_Solli', 'https://en.wikipedia.org/wiki/Molde'] 589 620 Tell me the singer that I am thinking about. Use this information to determine who it is: The song hit #1 on the billboard in 2015. The singer is from Canada. The artist was born before the dissolution of Czechoslovakia. The Weeknd https://en.wikipedia.org/wiki/List_of_Billboard_Hot_100_number_ones_of_2015 https://en.wikipedia.org/wiki/List_of_Canadian_musicians https://en.wikipedia.org/wiki/Dissolution_of_Czechoslovakia Tabular reasoning | Multiple constraints | Temporal reasoning ['https://en.wikipedia.org/wiki/List_of_Billboard_Hot_100_number_ones_of_2015', 'https://en.wikipedia.org/wiki/List_of_Canadian_musicians', 'https://en.wikipedia.org/wiki/Dissolution_of_Czechoslovakia'] 590 622 The Nintendo Entertainment System shipped with a main processor that was a derivative of the CPU in an Apple Computer that was released after the Apple 1 and before 1980. When was the original model Apple device I am referring to discontinued? 1979 https://en.wikipedia.org/wiki/Nintendo_Entertainment_System https://en.wikipedia.org/wiki/Ricoh_2A03 https://en.wikipedia.org/wiki/MOS_Technology_6502 https://en.wikipedia.org/wiki/List_of_Apple_products https://en.wikipedia.org/wiki/Apple_II_(original) Multiple constraints | Temporal reasoning ['https://en.wikipedia.org/wiki/Nintendo_Entertainment_System', 'https://en.wikipedia.org/wiki/Ricoh_2A03', 'https://en.wikipedia.org/wiki/MOS_Technology_6502', 'https://en.wikipedia.org/wiki/List_of_Apple_products', 'https://en.wikipedia.org/wiki/Apple_II_(original)'] 591 623 What is the name of the home town of the top scorer for the 2018-19 Brisbane Roar Football Club? Perth. https://en.wikipedia.org/wiki/2018%E2%80%9319_Brisbane_Roar_FC_season https://en.wikipedia.org/wiki/Adam_Taggart Tabular reasoning ['https://en.wikipedia.org/wiki/2018%E2%80%9319_Brisbane_Roar_FC_season', 'https://en.wikipedia.org/wiki/Adam_Taggart'] 592 624 As of 2020, who has experienced the coldest record temperature provided in Celsius, the Canadian territory, Yukon, or Yellowstone National Park? The Canadian territory of Yukon experienced a colder temperature of -63C. https://en.wikipedia.org/wiki/Yellowstone_National_Park https://en.wikipedia.org/wiki/List_of_extreme_temperatures_in_Canada Tabular reasoning ['https://en.wikipedia.org/wiki/Yellowstone_National_Park', 'https://en.wikipedia.org/wiki/List_of_extreme_temperatures_in_Canada'] 593 625 What is the name of the sequel to this comedy movie that shares a name with Obie Trice's fourth studio album? The Hangover Part II https://en.wikipedia.org/wiki/Obie_Trice https://en.wikipedia.org/wiki/The_Hangover https://en.wikipedia.org/wiki/The_Hangover_Part_II Multiple constraints ['https://en.wikipedia.org/wiki/Obie_Trice', 'https://en.wikipedia.org/wiki/The_Hangover', 'https://en.wikipedia.org/wiki/The_Hangover_Part_II'] 594 626 What does Lose Your Way by the British rock band Love Amongst Ruin have in common with Sally Lunn Buns? They're both made in Bath https://en.wikipedia.org/wiki/Lose_Your_Way_(album) https://en.wikipedia.org/wiki/Bath,_Somerset#Culture https://en.wikipedia.org/wiki/Sally_Lunn_bun Multiple constraints | Post processing ['https://en.wikipedia.org/wiki/Lose_Your_Way_(album)', 'https://en.wikipedia.org/wiki/Bath,_Somerset#Culture', 'https://en.wikipedia.org/wiki/Sally_Lunn_bun'] 595 627 In the same year that HMS Holland 1 was launched, a British monarch died. How long had their predecessor ruled for? 6 years, 11 months, 25 days. https://en.wikipedia.org/wiki/HMS_Holland_1 https://en.wikipedia.org/wiki/1901 https://en.wikipedia.org/wiki/Queen_Victoria https://en.wikipedia.org/wiki/William_IV Numerical reasoning ['https://en.wikipedia.org/wiki/HMS_Holland_1', 'https://en.wikipedia.org/wiki/1901', 'https://en.wikipedia.org/wiki/Queen_Victoria', 'https://en.wikipedia.org/wiki/William_IV'] 596 628 What BTS member turned 5 years old the soonest after BoA's first album was released? Jimin https://en.wikipedia.org/wiki/BoA#Discography https://en.wikipedia.org/wiki/ID;_Peace_B https://en.wikipedia.org/wiki/BTS https://en.wikipedia.org/wiki/Jimin Numerical reasoning | Multiple constraints ['https://en.wikipedia.org/wiki/BoA#Discography', 'https://en.wikipedia.org/wiki/ID;_Peace_B', 'https://en.wikipedia.org/wiki/BTS', 'https://en.wikipedia.org/wiki/Jimin'] 597 629 As of August 3rd 2024, what's the name of the university in the city right off exit 183 on I-94 in Michigan? Eastern Michigan University https://en.wikipedia.org/wiki/Interstate_94_in_Michigan# https://en.wikipedia.org/wiki/Ypsilanti,_Michigan Tabular reasoning ['https://en.wikipedia.org/wiki/Interstate_94_in_Michigan#', 'https://en.wikipedia.org/wiki/Ypsilanti,_Michigan'] 598 630 A certain singer won the Grammy Award for Song of the Year in 2008 and in the same year, sang at a prominent person's 90th Birthday Party concert in London. How many vowels are in the given first name of the person who the birthday party was for? 4 https://en.wikipedia.org/wiki/Grammy_Award_for_Song_of_the_Year#2000s https://en.wikipedia.org/wiki/Amy_Winehouse https://en.wikipedia.org/wiki/Nelson_Mandela Numerical reasoning | Multiple constraints | Temporal reasoning ['https://en.wikipedia.org/wiki/Grammy_Award_for_Song_of_the_Year#2000s', 'https://en.wikipedia.org/wiki/Amy_Winehouse', 'https://en.wikipedia.org/wiki/Nelson_Mandela'] 599 631 Of Louis XVI, Louis XIV, and Louis XV, who was the youngest at their coronation? Louis XV https://en.wikipedia.org/wiki/Louis_XIV https://en.wikipedia.org/wiki/Louis_XV https://en.wikipedia.org/wiki/Louis_XVI Numerical reasoning ['https://en.wikipedia.org/wiki/Louis_XIV', 'https://en.wikipedia.org/wiki/Louis_XV', 'https://en.wikipedia.org/wiki/Louis_XVI'] 600 632 Who was the building named after on South Forest Avenue, which was built around 1959-1964 and designed by the architect married to Olga Lazovic? Grady Gammage https://en.wikipedia.org/wiki/Olgivanna_Lloyd_Wright https://en.wikipedia.org/wiki/Frank_Lloyd_Wright https://en.wikipedia.org/wiki/Gammage_Memorial_Auditorium Multiple constraints ['https://en.wikipedia.org/wiki/Olgivanna_Lloyd_Wright', 'https://en.wikipedia.org/wiki/Frank_Lloyd_Wright', 'https://en.wikipedia.org/wiki/Gammage_Memorial_Auditorium'] 601 633 Where did Marion Couthouy Smith publish her books and poems between 1906 and 1918? Which years did each of these magazine companies first start? Marion Couthouy Smith published her books and poems in Harper's Magazine, Century Magazine, Atlantic Monthly, and The New England Magazine. Harpers Magazine was first published in 1850. Century Magazine was published in 1881. The Atlantic was founded in 1857. Lastly, The New England Magazine was first published in 1884. https://en.wikipedia.org/wiki/Marion_Couthouy_Smith https://en.wikipedia.org/wiki/Harper%27s_Magazine https://en.wikipedia.org/wiki/The_Century_Magazine https://en.wikipedia.org/wiki/The_Atlantic https://en.wikipedia.org/wiki/The_New_England_Magazine Multiple constraints ['https://en.wikipedia.org/wiki/Marion_Couthouy_Smith', 'https://en.wikipedia.org/wiki/Harper%27s_Magazine', 'https://en.wikipedia.org/wiki/The_Century_Magazine', 'https://en.wikipedia.org/wiki/The_Atlantic', 'https://en.wikipedia.org/wiki/The_New_England_Magazine'] 602 634 What famous playable video game character is killed in their story for the purpose of creating a huge feeling of emptiness? To give a little help, this character also helps her family out by selling flowers she has grown, she knows and has traveled with a known eco-terrorist leader and also a owner of a bar in the slums. Aerith Gainsborough. https://en.wikipedia.org/wiki/Aerith_Gainsborough https://en.wikipedia.org/wiki/Final_Fantasy_VII https://en.wikipedia.org/wiki/Barret_Wallace https://en.wikipedia.org/wiki/Tifa_Lockhart Multiple constraints ['https://en.wikipedia.org/wiki/Aerith_Gainsborough', 'https://en.wikipedia.org/wiki/Final_Fantasy_VII', 'https://en.wikipedia.org/wiki/Barret_Wallace', 'https://en.wikipedia.org/wiki/Tifa_Lockhart'] 603 635 "What is the name of the popular vantage point that is featured in the 1980 comedy film ""The Gods Must Be Crazy"", and which provincial nature reserve is it located in as of 2024?" God's Window in Blyde River Canyon Nature Reserve https://en.wikipedia.org/wiki/The_Gods_Must_Be_Crazy https://en.wikipedia.org/wiki/Blyde_River_Canyon_Nature_Reserve#God.27s_Window https://en.wikipedia.org/wiki/Blyde_River_Canyon_Nature_Reserve Multiple constraints ['https://en.wikipedia.org/wiki/The_Gods_Must_Be_Crazy', 'https://en.wikipedia.org/wiki/Blyde_River_Canyon_Nature_Reserve#God.27s_Window', 'https://en.wikipedia.org/wiki/Blyde_River_Canyon_Nature_Reserve'] 604 636 During which year did the actor who played George Falconer in the film A Single Man receive his first Academy Award? Include the name of the film for which he won. Colin Firth won his first Academy Award in 2011 for The King's Speech. https://en.wikipedia.org/wiki/A_Single_Man https://en.wikipedia.org/wiki/Colin_Firth https://en.wikipedia.org/wiki/List_of_awards_and_nominations_received_by_Colin_Firth Multiple constraints ['https://en.wikipedia.org/wiki/A_Single_Man', 'https://en.wikipedia.org/wiki/Colin_Firth', 'https://en.wikipedia.org/wiki/List_of_awards_and_nominations_received_by_Colin_Firth'] 605 637 "How many years before the founding of Google, was George Orwell's book ""1984"" published?" 49 years. https://en.wikipedia.org/wiki/Nineteen_Eighty-Four https://en.wikipedia.org/wiki/Google Numerical reasoning | Tabular reasoning ['https://en.wikipedia.org/wiki/Nineteen_Eighty-Four', 'https://en.wikipedia.org/wiki/Google'] 606 638 How many more knock-outs did Joe Louis have than Muhammad Ali? 15 https://en.wikipedia.org/wiki/Muhammad_Ali https://en.wikipedia.org/wiki/Joe_Louis Numerical reasoning | Tabular reasoning ['https://en.wikipedia.org/wiki/Muhammad_Ali', 'https://en.wikipedia.org/wiki/Joe_Louis'] 607 639 In the same city of California that is home to Walt Disney Imagineering is a famous retail shopping mall that opened with 1.6 million square feet of retail space. What is the name of that mall? The Glendale Galleria. https://en.wikipedia.org/wiki/Walt_Disney_Imagineering https://en.wikipedia.org/wiki/Glendale_Galleria Multiple constraints ['https://en.wikipedia.org/wiki/Walt_Disney_Imagineering', 'https://en.wikipedia.org/wiki/Glendale_Galleria'] 608 640 The parish church of Renče, Slovenia, is dedicated to two saints. What would be the tropical zodiac sign of someone born on the date of their annual festival? Cancer https://en.wikipedia.org/wiki/Ren%C4%8De https://en.wikipedia.org/wiki/Hermagoras_of_Aquileia https://en.wikipedia.org/wiki/Zodiac Tabular reasoning | Multiple constraints | Temporal reasoning ['https://en.wikipedia.org/wiki/Ren%C4%8De', 'https://en.wikipedia.org/wiki/Hermagoras_of_Aquileia', 'https://en.wikipedia.org/wiki/Zodiac'] 609 641 What is the name of the river in the city where Ikea's headquarters are? the Oude Rijn. https://en.wikipedia.org/wiki/IKEA https://en.wikipedia.org/wiki/Leiden Multiple constraints ['https://en.wikipedia.org/wiki/IKEA', 'https://en.wikipedia.org/wiki/Leiden'] 610 642 How much did the film in which Jake Gyllenhaal played his second lead role gross in its initial run at the box office? Donnie Darko grossed $517,375 in its initial run at the box office. https://en.wikipedia.org/wiki/Jake_Gyllenhaal https://en.wikipedia.org/wiki/Donnie_Darko Multiple constraints ['https://en.wikipedia.org/wiki/Jake_Gyllenhaal', 'https://en.wikipedia.org/wiki/Donnie_Darko'] 611 643 The actor who played Oliver Quick in Saltburn appeared in a music video for an artist who opened for Taylor Swift during the Latin American leg of the Eras Tour. What is the name of this music video? Please Please Please https://en.wikipedia.org/wiki/Saltburn_(film) https://en.wikipedia.org/wiki/The_Eras_Tour https://en.wikipedia.org/wiki/Please_Please_Please_(Sabrina_Carpenter_song) Multiple constraints ['https://en.wikipedia.org/wiki/Saltburn_(film)', 'https://en.wikipedia.org/wiki/The_Eras_Tour', 'https://en.wikipedia.org/wiki/Please_Please_Please_(Sabrina_Carpenter_song)'] 612 644 "In the region known as Sulawesi Selatan, which includes the Selayar Islands, what Austronesian language is predominantly spoken by the local population and how does the name ""Sulawesi Selatan"" relate to the location of the region?" "The predominant Austronesian language spoken in Sulawesi Selatan, including the Selayar Islands, is Makassarese. The name ""Sulawesi Selatan"" refers to the southern part of Sulawesi." https://en.wikipedia.org/wiki/Selatan https://en.wikipedia.org/wiki/South_Sulawesi https://en.wikipedia.org/wiki/Selayar_Islands https://en.wikipedia.org/wiki/Makassarese_language Multiple constraints | Temporal reasoning ['https://en.wikipedia.org/wiki/Selatan', 'https://en.wikipedia.org/wiki/South_Sulawesi', 'https://en.wikipedia.org/wiki/Selayar_Islands', 'https://en.wikipedia.org/wiki/Makassarese_language'] 613 645 How many different Prime Ministers of the United Kingdom were there during the first term of Grover Cleveland's presidency, and who were they? There were two different Prime Ministers of the United Kingdom during Grover Cleveland's first term as president, and they were William Ewart Gladstone and Robert Gascoyne-Cecil. https://en.wikipedia.org/wiki/List_of_prime_ministers_of_the_United_Kingdom https://en.wikipedia.org/wiki/Grover_Cleveland#:~:text=Stephen%20Grover%20Cleveland%20(March%2018,serve%20non%2Dconsecutive%20presidential%20terms. Tabular reasoning | Multiple constraints | Temporal reasoning ['https://en.wikipedia.org/wiki/List_of_prime_ministers_of_the_United_Kingdom', 'https://en.wikipedia.org/wiki/Grover_Cleveland#:~:text=Stephen%20Grover%20Cleveland%20(March%2018,serve%20non%2Dconsecutive%20presidential%20terms.'] 614 646 Which American high school can boast of an alumnus for whom the following is true: -Was inducted into the Pro Football Hall of Fame in 2018 -Played in 13 Pro Bowls -Played his first season of professional football for a team playing their first season in the NFL Kathleen Senior High School in Lakeland, Florida https://en.wikipedia.org/wiki/List_of_Pro_Football_Hall_of_Fame_inductees https://en.wikipedia.org/wiki/Pro_Bowl#Players_with_most_invitations https://en.wikipedia.org/wiki/Baltimore_Ravens https://en.wikipedia.org/wiki/Ray_Lewis https://en.wikipedia.org/wiki/Kathleen_Senior_High_School Tabular reasoning | Multiple constraints ['https://en.wikipedia.org/wiki/List_of_Pro_Football_Hall_of_Fame_inductees', 'https://en.wikipedia.org/wiki/Pro_Bowl#Players_with_most_invitations', 'https://en.wikipedia.org/wiki/Baltimore_Ravens', 'https://en.wikipedia.org/wiki/Ray_Lewis', 'https://en.wikipedia.org/wiki/Kathleen_Senior_High_School'] 615 647 Which 1963 Disney film starred the same actress who played a dual role in a Disney film two years earlier about twins who plot to reunite their separated parents? Summer Magic https://en.wikipedia.org/wiki/List_of_Walt_Disney_Pictures_films https://en.wikipedia.org/wiki/One_Hundred_and_One_Dalmatians https://en.wikipedia.org/wiki/The_Absent-Minded_Professor https://en.wikipedia.org/wiki/The_Parent_Trap_(1961_film) https://en.wikipedia.org/wiki/Nikki:_Wild_Dog_of_the_North https://en.wikipedia.org/wiki/Greyfriars_Bobby_(film) https://en.wikipedia.org/wiki/Babes_in_Toyland_(1961_film) https://en.wikipedia.org/wiki/Son_of_Flubber https://en.wikipedia.org/wiki/Miracle_of_the_White_Stallions https://en.wikipedia.org/wiki/Savage_Sam_(film) https://en.wikipedia.org/wiki/Summer_Magic_(film), https://en.wikipedia.org/wiki/The_Incredible_Journey_(film), https://en.wikipedia.org/wiki/The_Sword_in_the_Stone_(1963_film) Numerical reasoning | Tabular reasoning | Multiple constraints | Temporal reasoning ['https://en.wikipedia.org/wiki/List_of_Walt_Disney_Pictures_films', 'https://en.wikipedia.org/wiki/One_Hundred_and_One_Dalmatians', 'https://en.wikipedia.org/wiki/The_Absent-Minded_Professor', 'https://en.wikipedia.org/wiki/The_Parent_Trap_(1961_film)', 'https://en.wikipedia.org/wiki/Nikki:_Wild_Dog_of_the_North', 'https://en.wikipedia.org/wiki/Greyfriars_Bobby_(film)', 'https://en.wikipedia.org/wiki/Babes_in_Toyland_(1961_film)', 'https://en.wikipedia.org/wiki/Son_of_Flubber', 'https://en.wikipedia.org/wiki/Miracle_of_the_White_Stallions', 'https://en.wikipedia.org/wiki/Savage_Sam_(film)', 'https://en.wikipedia.org/wiki/Summer_Magic_(film), https://en.wikipedia.org/wiki/The_Incredible_Journey_(film), https://en.wikipedia.org/wiki/The_Sword_in_the_Stone_(1963_film)'] 616 648 As of 2024, how many times could the country where shogi was invented fit inside the country where xiangqi was invented? Round to the nearest whole number. 25 https://en.wikipedia.org/wiki/Shogi https://en.wikipedia.org/wiki/Xiangqi https://en.wikipedia.org/wiki/China https://en.wikipedia.org/wiki/Japan Numerical reasoning | Post processing ['https://en.wikipedia.org/wiki/Shogi', 'https://en.wikipedia.org/wiki/Xiangqi', 'https://en.wikipedia.org/wiki/China', 'https://en.wikipedia.org/wiki/Japan'] 617 649 How many player entries were in the event that the winner of the 2008 Aussie Millions also won in 2010 at the PokerStars World Championship of Online Poker? 1,240 https://en.wikipedia.org/wiki/Crown_Australian_Poker_Championship https://en.wikipedia.org/wiki/Alexander_Kostritsyn https://en.wikipedia.org/wiki/World_Championship_of_Online_Poker Multiple constraints ['https://en.wikipedia.org/wiki/Crown_Australian_Poker_Championship', 'https://en.wikipedia.org/wiki/Alexander_Kostritsyn', 'https://en.wikipedia.org/wiki/World_Championship_of_Online_Poker'] 618 650 How many votes did the opposition party get in the Brant riding the election before Justin Trudeau was elected Prime Minister? 16,351 https://en.wikipedia.org/wiki/Justin_Trudeau https://en.wikipedia.org/wiki/List_of_Canadian_federal_general_elections https://en.wikipedia.org/wiki/Results_of_the_2011_Canadian_federal_election Post processing | Temporal reasoning ['https://en.wikipedia.org/wiki/Justin_Trudeau', 'https://en.wikipedia.org/wiki/List_of_Canadian_federal_general_elections', 'https://en.wikipedia.org/wiki/Results_of_the_2011_Canadian_federal_election'] 619 651 Who was the screenwriter of the first collaboration film between Sunrise and the studio who animates Full Metal Alchemist? Keiko Nobumoto https://en.wikipedia.org/wiki/Fullmetal_Alchemist https://en.wikipedia.org/wiki/Bones_(studio) https://en.wikipedia.org/wiki/Cowboy_Bebop:_Knockin%27_on_Heaven%27s_Door Tabular reasoning | Multiple constraints ['https://en.wikipedia.org/wiki/Fullmetal_Alchemist', 'https://en.wikipedia.org/wiki/Bones_(studio)', 'https://en.wikipedia.org/wiki/Cowboy_Bebop:_Knockin%27_on_Heaven%27s_Door'] 620 652 If Anastasia Romanov had still been alive when the 90s cartoon movie based on her was released, how old would she be? Duchess Anastasia Romanov would have been 96 years old. https://en.m.wikipedia.org/wiki/Grand_Duchess_Anastasia_Nikolaevna_of_Russia https://en.m.wikipedia.org/wiki/Anastasia_(1997_film) Numerical reasoning ['https://en.m.wikipedia.org/wiki/Grand_Duchess_Anastasia_Nikolaevna_of_Russia', 'https://en.m.wikipedia.org/wiki/Anastasia_(1997_film)'] 621 653 Only one of the founding members of the superhero team 'The Defenders' was not a doctor. How many letters are in his name? Five (the name is Namor') https://en.wikipedia.org/wiki/List_of_Defenders_members https://en.wikipedia.org/wiki/Doctor_Strange https://en.wikipedia.org/wiki/Hulk https://en.wikipedia.org/wiki/Namor Multiple constraints | Post processing ['https://en.wikipedia.org/wiki/List_of_Defenders_members', 'https://en.wikipedia.org/wiki/Doctor_Strange', 'https://en.wikipedia.org/wiki/Hulk', 'https://en.wikipedia.org/wiki/Namor'] 622 654 If Alice turned 36 on the day John F. Kennedy was assassinated, how old would she be on the day the Berlin Wall fell? 61 https://en.wikipedia.org/wiki/Assassination_of_John_F._Kennedy https://en.wikipedia.org/wiki/Fall_of_the_Berlin_Wall#:~:text=The%20fall%20of%20the%20Berlin,restrictions%20were%20overwhelmed%20and%20discarded. Numerical reasoning ['https://en.wikipedia.org/wiki/Assassination_of_John_F._Kennedy', 'https://en.wikipedia.org/wiki/Fall_of_the_Berlin_Wall#:~:text=The%20fall%20of%20the%20Berlin,restrictions%20were%20overwhelmed%20and%20discarded.'] 623 655 What career was shared by one of the world's oldest fathers, who had a child at the age of 96, and a man who killed his wife, named Nancy, and their son, named Daniel? professional wrestler https://en.wikipedia.org/wiki/List_of_oldest_fathers https://en.wikipedia.org/wiki/Ramjit_Raghav https://en.wikipedia.org/wiki/Chris_Benoit#Death https://en.wikipedia.org/wiki/Professional_wrestling#Occupational_hazards Multiple constraints ['https://en.wikipedia.org/wiki/List_of_oldest_fathers', 'https://en.wikipedia.org/wiki/Ramjit_Raghav', 'https://en.wikipedia.org/wiki/Chris_Benoit#Death', 'https://en.wikipedia.org/wiki/Professional_wrestling#Occupational_hazards'] 624 656 Which two rivers pass through the hometown of a famous Chinese philosopher who was born as Kong Qiu The Si River and the Yi River pass through Qufu, Confucius' home town. https://en.wikipedia.org/wiki/Confucius# https://en.wikipedia.org/wiki/Qufu#Geography Multiple constraints ['https://en.wikipedia.org/wiki/Confucius#', 'https://en.wikipedia.org/wiki/Qufu#Geography'] 625 657 "How much larger was the concert capacity of the venue where Led Zeppelin recorded ""The Song Remains the Same"" than the venue where AC/DC recorded their first live album?" 16,500 https://en.wikipedia.org/wiki/The_Song_Remains_the_Same_(album) https://en.wikipedia.org/wiki/Madison_Square_Garden https://en.wikipedia.org/wiki/AC/DC_discography#Live_albums https://en.wikipedia.org/wiki/If_You_Want_Blood_You%27ve_Got_It https://en.wikipedia.org/wiki/The_Apollo,_Glasgow Numerical reasoning ['https://en.wikipedia.org/wiki/The_Song_Remains_the_Same_(album)', 'https://en.wikipedia.org/wiki/Madison_Square_Garden', 'https://en.wikipedia.org/wiki/AC/DC_discography#Live_albums', 'https://en.wikipedia.org/wiki/If_You_Want_Blood_You%27ve_Got_It', 'https://en.wikipedia.org/wiki/The_Apollo,_Glasgow'] 626 658 Five Nights at Freddy's initial game release came one day short of the 19 year anniversary of the death of which Grateful Dead band member? Jerry Garcia https://en.wikipedia.org/wiki/Five_Nights_at_Freddy%27s https://en.wikipedia.org/wiki/Grateful_Dead#Main_career_(1967–1995) Numerical reasoning | Tabular reasoning | Multiple constraints | Post processing | Temporal reasoning ['https://en.wikipedia.org/wiki/Five_Nights_at_Freddy%27s', 'https://en.wikipedia.org/wiki/Grateful_Dead#Main_career_(1967–1995)'] 627 659 One episode title from classic Doctor Who series 12 features the name of a Pentateuch book. Can you tell me which part of this episode had the lowest viewing figures in roman numerals? III (the answer is Genesis of the Daleks, the 3rd part had the least amount of viewers when broadcast) https://en.wikipedia.org/wiki/Old_Testament https://en.wikipedia.org/wiki/Doctor_Who_season_12 https://en.wikipedia.org/wiki/Genesis_of_the_Daleks Tabular reasoning | Multiple constraints | Post processing ['https://en.wikipedia.org/wiki/Old_Testament', 'https://en.wikipedia.org/wiki/Doctor_Who_season_12', 'https://en.wikipedia.org/wiki/Genesis_of_the_Daleks'] 628 660 "Which Naruto characters from before the TV series ""Boruto: Naruto the next generation"", can perfectly use the Rasengan technique?" Naruto Uzumaki, Minato Namikaze and Jiraiya. https://en.wikipedia.org/wiki/Naruto_Shippuden_the_Movie:_The_Lost_Tower https://en.wikipedia.org/wiki/Naruto_Uzumaki https://en.wikipedia.org/wiki/Jiraiya_(Naruto) Multiple constraints ['https://en.wikipedia.org/wiki/Naruto_Shippuden_the_Movie:_The_Lost_Tower', 'https://en.wikipedia.org/wiki/Naruto_Uzumaki', 'https://en.wikipedia.org/wiki/Jiraiya_(Naruto)'] 629 661 Twin brothers and former linebackers Ricardo and Devon McDonald were drafted into the NFL in 1992 and 1993, respectively. How many more games did one twin play than the other during their NFL career? 56 https://en.wikipedia.org/wiki/Devon_McDonald https://en.wikipedia.org/wiki/Ricardo_McDonald Numerical reasoning ['https://en.wikipedia.org/wiki/Devon_McDonald', 'https://en.wikipedia.org/wiki/Ricardo_McDonald'] 630 662 How old was the United States Air Force when the 317th Fighter-Interceptor Squadron was inactivated, rounded to the nearest year? 22 years old https://en.wikipedia.org/wiki/317th_Fighter-Interceptor_Squadron https://en.wikipedia.org/wiki/United_States_Air_Force Numerical reasoning | Temporal reasoning ['https://en.wikipedia.org/wiki/317th_Fighter-Interceptor_Squadron', 'https://en.wikipedia.org/wiki/United_States_Air_Force'] 631 663 Who was the Argentinian president who resigned from the position the same year the José Martín Olaeta Stadium was inaugurated? Pedro Ramírez https://en.wikipedia.org/wiki/Estadio_Jos%C3%A9_Mart%C3%ADn_Olaeta https://en.wikipedia.org/wiki/1944_in_Argentina Multiple constraints ['https://en.wikipedia.org/wiki/Estadio_Jos%C3%A9_Mart%C3%ADn_Olaeta', 'https://en.wikipedia.org/wiki/1944_in_Argentina'] 632 664 How many days is it from Damon Wayans's first episode as a cast member of Saturday Night Live to Damon Wayans's first episode as a cast member of In Living Color, including the days the first episodes premiered? 1619 days. https://en.wikipedia.org/wiki/Damon_Wayans https://en.wikipedia.org/wiki/Saturday_Night_Live_season_11 https://en.wikipedia.org/wiki/In_Living_Color https://en.wikipedia.org/wiki/List_of_In_Living_Color_episodes Numerical reasoning | Multiple constraints ['https://en.wikipedia.org/wiki/Damon_Wayans', 'https://en.wikipedia.org/wiki/Saturday_Night_Live_season_11', 'https://en.wikipedia.org/wiki/In_Living_Color', 'https://en.wikipedia.org/wiki/List_of_In_Living_Color_episodes'] 633 665 There is a famous Texas city where a Stadium built in 1930 hosted six games for the 1994 World Cup, but only after widening the field and permanently installing natural grass on the playing surface. What was the average (mean) number of attendance for these six games? 58,692 https://en.wikipedia.org/wiki/1994_FIFA_World_Cup https://en.wikipedia.org/wiki/Cotton_Bowl_(stadium) https://en.wikipedia.org/wiki/Dallas Numerical reasoning | Multiple constraints | Post processing ['https://en.wikipedia.org/wiki/1994_FIFA_World_Cup', 'https://en.wikipedia.org/wiki/Cotton_Bowl_(stadium)', 'https://en.wikipedia.org/wiki/Dallas'] 634 666 The September Declaration has a rough equivalent in the Netherlands that takes place on the same day every year. In what century was the current name of this day (as of August 2024) chosen? the 19th century https://en.wikipedia.org/wiki/September_Declaration https://en.wikipedia.org/wiki/Speech_from_the_throne#Netherlands https://en.wikipedia.org/wiki/Prinsjesdag#History Multiple constraints ['https://en.wikipedia.org/wiki/September_Declaration', 'https://en.wikipedia.org/wiki/Speech_from_the_throne#Netherlands', 'https://en.wikipedia.org/wiki/Prinsjesdag#History'] 635 667 Who was team captain of the team that won the Stanley Cup the year Connor McDavid was born? Steve Yzerman https://en.wikipedia.org/wiki/Connor_McDavid https://en.wikipedia.org/wiki/List_of_Stanley_Cup_champions https://en.wikipedia.org/wiki/1996%E2%80%9397_Detroit_Red_Wings_season Temporal reasoning ['https://en.wikipedia.org/wiki/Connor_McDavid', 'https://en.wikipedia.org/wiki/List_of_Stanley_Cup_champions', 'https://en.wikipedia.org/wiki/1996%E2%80%9397_Detroit_Red_Wings_season'] 636 668 From the date the Soviet Union first used their veto power in the UN security council, how many more years would the then-Soviet leader live? Seven https://en.wikipedia.org/wiki/List_of_vetoed_United_Nations_Security_Council_resolutions#Resolutions https://en.wikipedia.org/wiki/List_of_leaders_of_the_Soviet_Union https://en.wikipedia.org/wiki/Joseph_Stalin Numerical reasoning ['https://en.wikipedia.org/wiki/List_of_vetoed_United_Nations_Security_Council_resolutions#Resolutions', 'https://en.wikipedia.org/wiki/List_of_leaders_of_the_Soviet_Union', 'https://en.wikipedia.org/wiki/Joseph_Stalin'] 637 669 How many years had the station that preceded the Salthill and Monkstown railway station on the historic Dublin and South Eastern Railway line been open on Christmas Day of 2005? 143 years https://en.wikipedia.org/wiki/Salthill_and_Monkstown_railway_station https://en.wikipedia.org/wiki/Dublin_Area_Rapid_Transit https://en.wikipedia.org/wiki/Seapoint_railway_station Numerical reasoning | Multiple constraints | Post processing | Temporal reasoning ['https://en.wikipedia.org/wiki/Salthill_and_Monkstown_railway_station', 'https://en.wikipedia.org/wiki/Dublin_Area_Rapid_Transit', 'https://en.wikipedia.org/wiki/Seapoint_railway_station'] 638 670 How many countries were part of the commonwealth on the date Prince Charles ascended? On 8 September 2022, there were 56 countries as part of the Commonwealth. https://en.wikipedia.org/wiki/Charles_III https://en.wikipedia.org/wiki/Member_states_of_the_Commonwealth_of_Nations#:~:text=The%20Republic%20of%20Ireland%20(as,former%20members%20of%20the%20Commonwealth. Temporal reasoning ['https://en.wikipedia.org/wiki/Charles_III', 'https://en.wikipedia.org/wiki/Member_states_of_the_Commonwealth_of_Nations#:~:text=The%20Republic%20of%20Ireland%20(as,former%20members%20of%20the%20Commonwealth.'] 639 671 "In feet, subtract the diameter of the pitching plate (""rubber"") in softball, from the distance between the ""points"" of the bases in baseball, and multiply that figure by the year that Joe DiMaggio married Marilyn Monroe. " 144,596. The distance between the points of bases in baseball is 90 feet, subtract the diameter of the pitching plate in softball (16 feet), to get 74, and multiply that by 1954. https://en.wikipedia.org/wiki/Softball https://en.wikipedia.org/wiki/Baseball_field https://en.wikipedia.org/wiki/Joe_DiMaggio Numerical reasoning | Multiple constraints ['https://en.wikipedia.org/wiki/Softball', 'https://en.wikipedia.org/wiki/Baseball_field', 'https://en.wikipedia.org/wiki/Joe_DiMaggio'] 640 672 What are the sizes of the two islands in Crater Lake in acres? Phantom Ship is 2.3 acres. Wizard Island is 315.85 acres. https://en.wikipedia.org/wiki/Wizard_Island https://en.wikipedia.org/wiki/Phantom_Ship_(island) Tabular reasoning | Post processing ['https://en.wikipedia.org/wiki/Wizard_Island', 'https://en.wikipedia.org/wiki/Phantom_Ship_(island)'] 641 673 As of August 3rd 2024, which movie using the Technicolor dye-transfer process was the last to win the Academy Award for Best Picture? "The movie using the Technicolor dye-transfer process that was the last to win the Academy Award for Best Picture was ""The Godfather Part II.""" https://en.wikipedia.org/wiki/Technicolor https://en.wikipedia.org/wiki/Dye-transfer_process https://en.wikipedia.org/wiki/The_Godfather_Part_II https://en.wikipedia.org/wiki/Academy_Award_for_Best_Picture Multiple constraints | Temporal reasoning ['https://en.wikipedia.org/wiki/Technicolor', 'https://en.wikipedia.org/wiki/Dye-transfer_process', 'https://en.wikipedia.org/wiki/The_Godfather_Part_II', 'https://en.wikipedia.org/wiki/Academy_Award_for_Best_Picture'] 642 674 How much older than Michael B. Jordan is Michael Jordan? 24 years https://en.wikipedia.org/wiki/Michael_Jordan https://en.wikipedia.org/wiki/Michael_B._Jordan Numerical reasoning ['https://en.wikipedia.org/wiki/Michael_Jordan', 'https://en.wikipedia.org/wiki/Michael_B._Jordan'] 643 675 The University that Cillian Murphy attended was founded how many years before he began studying? 151 years. https://en.wikipedia.org/wiki/Cillian_Murphy https://en.wikipedia.org/wiki/University_College_Cork Numerical reasoning ['https://en.wikipedia.org/wiki/Cillian_Murphy', 'https://en.wikipedia.org/wiki/University_College_Cork'] 644 676 The incorporation of this company happened in the same year titled on Taylor Swift 5th studio album. It was incorporated by a man born in Setagaya, Tokyo, Japan that worked with the illustrator who largely designed the original 151 Pokémon. What is “this company”? Game Freak https://en.wikipedia.org/wiki/1989_(album) https://en.wikipedia.org/wiki/Satoshi_Tajiri https://en.wikipedia.org/wiki/Pok%C3%A9mon (NOT REQUIRED, BUT HELPFUL) Multiple constraints ['https://en.wikipedia.org/wiki/1989_(album)', 'https://en.wikipedia.org/wiki/Satoshi_Tajiri', 'https://en.wikipedia.org/wiki/Pok%C3%A9mon (NOT REQUIRED, BUT HELPFUL) '] 645 677 What is the name of the only Texan radio station on frequency 89.5 FM that is operated by a university, as of the death of Jerry West? KACU https://en.wikipedia.org/wiki/89.5_FM https://en.wikipedia.org/wiki/Abilene_Christian_University https://en.wikipedia.org/wiki/Jerry_West Tabular reasoning | Multiple constraints | Temporal reasoning ['https://en.wikipedia.org/wiki/89.5_FM', 'https://en.wikipedia.org/wiki/Abilene_Christian_University', 'https://en.wikipedia.org/wiki/Jerry_West'] 646 678 How much money would be left of Elon Musk's net worth in January of 2021, if you subtracted the 2022 average yearly gross salary of 10,000 people, working in the Philippines, written in words? one hundred eighty-four billion nine hundred fifty-nine million four hundred forty thousand. https://en.wikipedia.org/wiki/Wealth_of_Elon_Musk https://en.wikipedia.org/wiki/Economy_of_the_Philippines Numerical reasoning | Tabular reasoning | Post processing ['https://en.wikipedia.org/wiki/Wealth_of_Elon_Musk', 'https://en.wikipedia.org/wiki/Economy_of_the_Philippines'] 647 679 What city was the capital of the United States on the day that the first president of the United States died? Philadelphia, Pennsylvania https://en.wikipedia.org/wiki/George_Washington#Personal_life https://en.wikipedia.org/wiki/List_of_capitals_in_the_United_States Temporal reasoning ['https://en.wikipedia.org/wiki/George_Washington#Personal_life', 'https://en.wikipedia.org/wiki/List_of_capitals_in_the_United_States'] 648 680 How old was Joel McHale the first time the Philadelphia Waterdogs won the PLL Championship? Joel McHale was 50 years old. https://en.wikipedia.org/wiki/Joel_McHale#:~:text=Joel%20Edward%20McHale%20(born%20November,actor%2C%20comedian%20and%20television%20presenter. https://en.wikipedia.org/wiki/Philadelphia_Waterdogs#Season_results Temporal reasoning ['https://en.wikipedia.org/wiki/Joel_McHale#:~:text=Joel%20Edward%20McHale%20(born%20November,actor%2C%20comedian%20and%20television%20presenter.', 'https://en.wikipedia.org/wiki/Philadelphia_Waterdogs#Season_results'] 649 681 If somebody was born on the day the Anglo-Dutch Treaty of 1814 was signed, how old would they have been while Albert A. Michelson and Edward W. Morley were performing their Michelson-Morley experiment? 72 https://en.wikipedia.org/wiki/Anglo-Dutch_Treaty_of_1814 https://en.wikipedia.org/wiki/Michelson%E2%80%93Morley_experiment Numerical reasoning | Temporal reasoning ['https://en.wikipedia.org/wiki/Anglo-Dutch_Treaty_of_1814', 'https://en.wikipedia.org/wiki/Michelson%E2%80%93Morley_experiment'] 650 682 Who won Britain's Got Talent in the same year that London hosted the Olympics for the third time? Ashleigh and Pudsey https://en.wikipedia.org/wiki/Category:Summer_Olympics_in_London https://en.wikipedia.org/wiki/2012_Summer_Olympics https://en.wikipedia.org/wiki/Britain%27s_Got_Talent Temporal reasoning ['https://en.wikipedia.org/wiki/Category:Summer_Olympics_in_London', 'https://en.wikipedia.org/wiki/2012_Summer_Olympics', 'https://en.wikipedia.org/wiki/Britain%27s_Got_Talent'] 651 683 What key signature was the song that was number one on the Billboard Hot 100 on June 8, 2002 performed in? C major https://en.wikipedia.org/wiki/List_of_Billboard_Hot_100_number_ones_of_2002 https://en.wikipedia.org/wiki/Foolish_(Ashanti_song) Tabular reasoning ['https://en.wikipedia.org/wiki/List_of_Billboard_Hot_100_number_ones_of_2002', 'https://en.wikipedia.org/wiki/Foolish_(Ashanti_song)'] 652 684 Itanihomi is a municipality in Brazil. What is the state that sits directly south of the state Itanihomi belongs to? Itanihomi is in Minas Gerais, and the state directly south of this is Sao Paulo. https://en.wikipedia.org/wiki/Itanhomi https://en.wikipedia.org/wiki/Federative_units_of_Brazil#/media/File:Brazil,_administrative_divisions_(states)_-_en_-_colored.svg Multiple constraints ['https://en.wikipedia.org/wiki/Itanhomi', 'https://en.wikipedia.org/wiki/Federative_units_of_Brazil#/media/File:Brazil,_administrative_divisions_(states)_-_en_-_colored.svg'] 653 686 Which Greek pole vaulter who participated in the 2020 Summer Olympics in Tokyo also won gold at the 2015 IAAF Diamond League? Nikoleta Kyriakopoulou https://en.wikipedia.org/wiki/Athletics_at_the_2020_Summer_Olympics https://en.wikipedia.org/wiki/2015_Diamond_League https://en.wikipedia.org/wiki/Nikoleta_Kyriakopoulou Multiple constraints ['https://en.wikipedia.org/wiki/Athletics_at_the_2020_Summer_Olympics', 'https://en.wikipedia.org/wiki/2015_Diamond_League', 'https://en.wikipedia.org/wiki/Nikoleta_Kyriakopoulou'] 654 688 "Please consider the following clues and answer the question that follows: 1. This mosque is located in the city dubbed the ""Jerusalem of the Balkans."" 2, The mosque was commissioned by an authoritarian dictator born in a small village 10 km west of Yogyakarta. Question: What is the height difference between the twin towers of the mosque and its dome? " 21 meters https://en.wikipedia.org/wiki/Istiklal_Mosque,_Sarajevo https://en.wikipedia.org/wiki/Sarajevo https://en.wikipedia.org/wiki/Suharto https://en.wikipedia.org/wiki/Kemusuk Numerical reasoning | Multiple constraints ['https://en.wikipedia.org/wiki/Istiklal_Mosque,_Sarajevo', 'https://en.wikipedia.org/wiki/Sarajevo', 'https://en.wikipedia.org/wiki/Suharto', 'https://en.wikipedia.org/wiki/Kemusuk'] 655 689 What is the birthday of the basketball player turned wrestler who shares a nickname with the YouTuber who created the brand Feastables? May 23, 1985 https://en.wikipedia.org/wiki/Feastables https://en.wikipedia.org/wiki/Peter_John_Ramos Multiple constraints ['https://en.wikipedia.org/wiki/Feastables', 'https://en.wikipedia.org/wiki/Peter_John_Ramos'] 656 690 In 2000, Michel von Tell drove a Toyota and placed sixth in an event. Who placed first that same year, and what car were they driving? Charles Muhanji in a Subaru Impreza WRX https://en.wikipedia.org/wiki/Michel_von_Tell https://en.wikipedia.org/wiki/Rwanda_Mountain_Gorilla_Rally Tabular reasoning | Multiple constraints | Temporal reasoning ['https://en.wikipedia.org/wiki/Michel_von_Tell', 'https://en.wikipedia.org/wiki/Rwanda_Mountain_Gorilla_Rally'] 657 691 "There's an episode of a TV series that was directed by the person who won the 2011 Dorothy Arzner Directors award. The series started in 2005. The season that contains my episode included 20 episodes in total. The episode is one word that starts with an ""M."" " Miracles https://en.wikipedia.org/wiki/Women_in_Film_Crystal_%2B_Lucy_Awards#Dorothy_Arzner_Directors_award https://en.wikipedia.org/wiki/Pamela_Fryman https://en.wikipedia.org/wiki/How_I_Met_Your_Mother https://en.wikipedia.org/wiki/List_of_How_I_Met_Your_Mother_episodes#Season_3_(2007%E2%80%9308) Tabular reasoning | Multiple constraints ['https://en.wikipedia.org/wiki/Women_in_Film_Crystal_%2B_Lucy_Awards#Dorothy_Arzner_Directors_award', 'https://en.wikipedia.org/wiki/Pamela_Fryman', 'https://en.wikipedia.org/wiki/How_I_Met_Your_Mother', 'https://en.wikipedia.org/wiki/List_of_How_I_Met_Your_Mother_episodes#Season_3_(2007%E2%80%9308)'] 658 692 What is the birth town of the absent athlete from the Victory Salute statue in San Jose, California? Peter Norman was born in Coburg, Victoria, Australia. https://en.wikipedia.org/wiki/Victory_Salute_(statue) https://en.wikipedia.org/wiki/Peter_Norman Multiple constraints ['https://en.wikipedia.org/wiki/Victory_Salute_(statue)', 'https://en.wikipedia.org/wiki/Peter_Norman'] 659 693 What is the capacity of the Olympic stadium used during the first Winter Games attended by a tropical nation? This nation was visited by the 5th Cavalry Regiment (US) in 1901. 17,324 https://en.wikipedia.org/wiki/5th_Cavalry_Regiment https://en.wikipedia.org/wiki/Philippines https://en.wikipedia.org/wiki/1972_Winter_Olympics https://en.wikipedia.org/wiki/Makomanai_Open_Stadium Multiple constraints | Temporal reasoning ['https://en.wikipedia.org/wiki/5th_Cavalry_Regiment', 'https://en.wikipedia.org/wiki/Philippines', 'https://en.wikipedia.org/wiki/1972_Winter_Olympics', 'https://en.wikipedia.org/wiki/Makomanai_Open_Stadium'] 660 694 In 2010, the WWE Hall of Fame took place in the same stadium as a famous comedy movie done a few years before. Who are the four comedians that starred in this comedy movie? jeff Foxworthy, Bill Engvall, Ron White and Larry the Cable Guy. https://en.wikipedia.org/wiki/WWE_Hall_of_Fame_(2010) https://en.wikipedia.org/wiki/Arizona_Financial_Theatre https://en.wikipedia.org/wiki/Blue_Collar_Comedy_Tour:_The_Movie Multiple constraints | Temporal reasoning ['https://en.wikipedia.org/wiki/WWE_Hall_of_Fame_(2010)', 'https://en.wikipedia.org/wiki/Arizona_Financial_Theatre', 'https://en.wikipedia.org/wiki/Blue_Collar_Comedy_Tour:_The_Movie'] 661 695 What year was the first Uber employee born? 1983 https://en.wikipedia.org/wiki/Uber https://en.wikipedia.org/wiki/Ryan_Graves_(businessman) Multiple constraints ['https://en.wikipedia.org/wiki/Uber', 'https://en.wikipedia.org/wiki/Ryan_Graves_(businessman)'] 662 696 Which British prime minister of the 1990s had the most children? Tony Blair https://en.wikipedia.org/wiki/List_of_prime_ministers_of_the_United_Kingdom https://en.wikipedia.org/wiki/Margaret_Thatcher https://en.wikipedia.org/wiki/John_Major https://en.wikipedia.org/wiki/Tony_Blair Numerical reasoning ['https://en.wikipedia.org/wiki/List_of_prime_ministers_of_the_United_Kingdom', 'https://en.wikipedia.org/wiki/Margaret_Thatcher', 'https://en.wikipedia.org/wiki/John_Major', 'https://en.wikipedia.org/wiki/Tony_Blair'] 663 697 What was the daily average passenger count in 2011 of the first station on the train line that serves Hiraka Train Station in Japan? 2,851 Passengers https://en.wikipedia.org/wiki/Hiraka_Station https://en.wikipedia.org/wiki/K%C5%8Dnan_Railway_K%C5%8Dnan_Line https://en.wikipedia.org/wiki/Hirosaki_Station Multiple constraints ['https://en.wikipedia.org/wiki/Hiraka_Station', 'https://en.wikipedia.org/wiki/K%C5%8Dnan_Railway_K%C5%8Dnan_Line', 'https://en.wikipedia.org/wiki/Hirosaki_Station'] 664 698 How many Red Hot Chili Peppers albums were released while Nelson Mandela was in prison? 4 https://en.wikipedia.org/wiki/Red_Hot_Chili_Peppers#Discography https://en.wikipedia.org/wiki/Nelson_Mandela#Imprisonment Temporal reasoning ['https://en.wikipedia.org/wiki/Red_Hot_Chili_Peppers#Discography', 'https://en.wikipedia.org/wiki/Nelson_Mandela#Imprisonment'] 665 699 How many years after the first album release by The Beatles was the first solo album released by one of its members? Five years https://en.wikipedia.org/wiki/The_Beatles_discography https://en.wikipedia.org/wiki/George_Harrison#Discography Multiple constraints | Temporal reasoning ['https://en.wikipedia.org/wiki/The_Beatles_discography', 'https://en.wikipedia.org/wiki/George_Harrison#Discography'] 666 700 "Who starred in a movie about the subject of a song on the compilation album ""Friends and Relatives,"" and also played themselves on an episode of ""Friends""?" Isabella Rossellini https://en.wikipedia.org/wiki/Friends_%26_Relatives https://en.wikipedia.org/wiki/Beethoven_in_film https://en.wikipedia.org/wiki/Immortal_Beloved_(1994_film) https://en.wikipedia.org/wiki/Isabella_Rossellini Multiple constraints ['https://en.wikipedia.org/wiki/Friends_%26_Relatives', 'https://en.wikipedia.org/wiki/Beethoven_in_film', 'https://en.wikipedia.org/wiki/Immortal_Beloved_(1994_film)', 'https://en.wikipedia.org/wiki/Isabella_Rossellini'] 667 701 In 1966, the Lower Klamath National Wildlife Refuge became part of the U.S. National Register of Historic Places (NRHP). What is another natural site with water added during that year, also located in California? Lake Merritt https://en.wikipedia.org/wiki/Old_Mission_Dam https://en.wikipedia.org/wiki/Pecos_National_Historical_Park https://en.wikipedia.org/wiki/Walden_Pond https://en.wikipedia.org/wiki/Seal_Island_Historic_District https://en.wikipedia.org/wiki/Russian_Bishop%27s_House https://en.wikipedia.org/wiki/Luther_Burbank_Home_and_Gardens Multiple constraints ['https://en.wikipedia.org/wiki/Old_Mission_Dam', 'https://en.wikipedia.org/wiki/Pecos_National_Historical_Park', 'https://en.wikipedia.org/wiki/Walden_Pond', 'https://en.wikipedia.org/wiki/Seal_Island_Historic_District', 'https://en.wikipedia.org/wiki/Russian_Bishop%27s_House', 'https://en.wikipedia.org/wiki/Luther_Burbank_Home_and_Gardens'] 668 702 What was the magnitude of the earthquake that was the catalyst for the charitable U.S. speedrunning marathon that took place in April 2011? Mw 9.0–9.1 https://en.wikipedia.org/wiki/Speedrunning https://en.wikipedia.org/wiki/Games_Done_Quick https://en.wikipedia.org/wiki/2011_T%C5%8Dhoku_earthquake_and_tsunami Tabular reasoning | Multiple constraints ['https://en.wikipedia.org/wiki/Speedrunning', 'https://en.wikipedia.org/wiki/Games_Done_Quick', 'https://en.wikipedia.org/wiki/2011_T%C5%8Dhoku_earthquake_and_tsunami'] 669 703 What Volkswagen former car model has nearly the same name as a Bangkok rooftop restaurant? The car has one additional letter. Scirocco (The Bangkok restaurant is called Sirocco) https://en.wikipedia.org/wiki/Lebua_at_State_Tower https://en.wikipedia.org/wiki/List_of_Volkswagen_vehicles https://en.wikipedia.org/wiki/Volkswagen_Scirocco Multiple constraints | Post processing ['https://en.wikipedia.org/wiki/Lebua_at_State_Tower', 'https://en.wikipedia.org/wiki/List_of_Volkswagen_vehicles', 'https://en.wikipedia.org/wiki/Volkswagen_Scirocco'] 670 705 As of 1st August 2024 The three British Olympic Class ocean liners were manufactured in a city that lies at the mouth of what river? River Lagan https://en.wikipedia.org/wiki/HMHS_Britannic https://en.wikipedia.org/wiki/Belfast Multiple constraints ['https://en.wikipedia.org/wiki/HMHS_Britannic', 'https://en.wikipedia.org/wiki/Belfast'] 671 706 The Mossi King who passed away in Thailand in 2016, would have spoken what language? Mòoré https://en.wikipedia.org/wiki/Tenkodogo https://en.wikipedia.org/wiki/Mossi_Kingdoms https://en.wikipedia.org/wiki/List_of_rulers_of_the_Mossi_state_of_Tenkodogo Multiple constraints | Temporal reasoning ['https://en.wikipedia.org/wiki/Tenkodogo', 'https://en.wikipedia.org/wiki/Mossi_Kingdoms', 'https://en.wikipedia.org/wiki/List_of_rulers_of_the_Mossi_state_of_Tenkodogo'] 672 707 Which Colombian cyclist was born on the same day as Edmonton Oilers captain Connor McDavid? Egan Bernal https://en.wikipedia.org/wiki/Connor_McDavid https://en.wikipedia.org/wiki/January_13 Numerical reasoning | Tabular reasoning ['https://en.wikipedia.org/wiki/Connor_McDavid', 'https://en.wikipedia.org/wiki/January_13'] 673 708 From 1924 to August 2024, how many times did Texas's and California's electoral colleges elect the same nominee during the presidential election? 13 times. https://en.wikipedia.org/wiki/Politics_of_California https://en.wikipedia.org/wiki/Politics_of_Texas Tabular reasoning | Temporal reasoning ['https://en.wikipedia.org/wiki/Politics_of_California', 'https://en.wikipedia.org/wiki/Politics_of_Texas'] 674 710 Who composed the Broadway musical that premiered in 2003 and starred the actress who would later voice Elsa in Disney's Frozen? Stephen Schwartz https://en.wikipedia.org/wiki/Wicked_(musical) https://en.wikipedia.org/wiki/Idina_Menzel Multiple constraints ['https://en.wikipedia.org/wiki/Wicked_(musical)', 'https://en.wikipedia.org/wiki/Idina_Menzel'] 675 711 In physics, when speaking of classical mechanics, there is an infamous problem that involves taking the initial positions and velocities of three point masses that orbit each other and attempting to calculate their trajectories. There is no general closed-form solution for this infamous problem. French mathematicians in the 18th century focused on solving this problem in regards to astronomical motion, specifically how the Moon rotates on its apsides. Their work led to a solution using Newton's laws of physics and the Discrete Fourier Transformation (DFT), which ultimately changed how sailors were able to determine longitude at sea. The inventor of the device that would revolutionize naval navigation using these new principles and proofs spent how many years testing and perfecting his work? 31 years (1730-1761) https://en.wikipedia.org/wiki/Three-body_problem https://en.wikipedia.org/wiki/Alexis_Clairaut#Focus_on_astronomical_motion https://en.wikipedia.org/wiki/Marine_chronometer https://en.wikipedia.org/wiki/John_Harrison Tabular reasoning | Multiple constraints | Post processing | Temporal reasoning ['https://en.wikipedia.org/wiki/Three-body_problem', 'https://en.wikipedia.org/wiki/Alexis_Clairaut#Focus_on_astronomical_motion', 'https://en.wikipedia.org/wiki/Marine_chronometer', 'https://en.wikipedia.org/wiki/John_Harrison'] 676 712 "What is the country of origin of the football coach with the first initial ""P"" for the Thailand national men's football team who coached 54 years after the country's name officially changed?" German. https://en.wikipedia.org/wiki/Thailand https://en.wikipedia.org/wiki/Thailand_national_football_team#Coaching_history Numerical reasoning | Tabular reasoning | Post processing | Temporal reasoning ['https://en.wikipedia.org/wiki/Thailand', 'https://en.wikipedia.org/wiki/Thailand_national_football_team#Coaching_history'] 677 713 Archibald Sinclair had an American mom who was a half-sister. The half-sister had a life partner who had a painting of her by Walter Sickert. How many words is the title of that painting? 5 ( Miss Hudson at Rowlandson House) https://en.wikipedia.org/wiki/Archibald_Sinclair,_1st_Viscount_Thurso https://en.wikipedia.org/wiki/Ethel_Sands https://en.wikipedia.org/wiki/Anna_Hope_Hudson Post processing ['https://en.wikipedia.org/wiki/Archibald_Sinclair,_1st_Viscount_Thurso', 'https://en.wikipedia.org/wiki/Ethel_Sands', 'https://en.wikipedia.org/wiki/Anna_Hope_Hudson'] 678 714 What movie won the Academy Award for Best Picture the same year that Argentina won its first World Cup? The Deer Hunter https://en.wikipedia.org/wiki/Argentina_at_the_FIFA_World_Cup#:~:text=Argentina%20is%20one%20of%20the,in%201930%2C%201990%20and%202014. https://en.wikipedia.org/wiki/Academy_Award_for_Best_Picture#1970s Tabular reasoning ['https://en.wikipedia.org/wiki/Argentina_at_the_FIFA_World_Cup#:~:text=Argentina%20is%20one%20of%20the,in%201930%2C%201990%20and%202014.', 'https://en.wikipedia.org/wiki/Academy_Award_for_Best_Picture#1970s'] 679 715 "What is the other name, beginning and ending with the letter ""d"", for the substance, often deployed by means of artillery shells, which was also instrumental in developing a treatment for a condition that dogs of the same breed as Mayor Max II are unusually prone to?" Distilled mustard https://en.wikipedia.org/wiki/Mayor_Max_II https://en.wikipedia.org/wiki/Golden_Retriever#Health https://en.wikipedia.org/wiki/Hemangiosarcoma#Treatments https://en.wikipedia.org/wiki/Chemotherapy#History https://en.wikipedia.org/wiki/Mustard_gas Multiple constraints ['https://en.wikipedia.org/wiki/Mayor_Max_II', 'https://en.wikipedia.org/wiki/Golden_Retriever#Health', 'https://en.wikipedia.org/wiki/Hemangiosarcoma#Treatments', 'https://en.wikipedia.org/wiki/Chemotherapy#History', 'https://en.wikipedia.org/wiki/Mustard_gas'] 680 716 As of May 2024, which female New Zealand Prime Minister was the oldest when they took office? Helen Clark. https://en.wikipedia.org/wiki/List_of_prime_ministers_of_New_Zealand https://en.wikipedia.org/wiki/Jenny_Shipley https://en.wikipedia.org/wiki/Helen_Clark https://en.wikipedia.org/wiki/Jacinda_Ardern Numerical reasoning | Multiple constraints | Post processing | Temporal reasoning ['https://en.wikipedia.org/wiki/List_of_prime_ministers_of_New_Zealand', 'https://en.wikipedia.org/wiki/Jenny_Shipley', 'https://en.wikipedia.org/wiki/Helen_Clark', 'https://en.wikipedia.org/wiki/Jacinda_Ardern'] 681 717 What is the capital of the country where the Treaty on European Union was signed? Amsterdam https://en.wikipedia.org/wiki/Maastricht_Treaty https://en.wikipedia.org/wiki/Netherlands Multiple constraints ['https://en.wikipedia.org/wiki/Maastricht_Treaty', 'https://en.wikipedia.org/wiki/Netherlands'] 682 718 Which happened earlier: Diageo reducing the volume of Red stripe beer bottles in the US from 12 fl. oz. to 11.2 fl. oz. or Philip Morris making a bid for the company Swedish Match? The Red Stripe US bottle volume reduction happened earlier. https://en.m.wikipedia.org/wiki/Red_Stripe https://en.m.wikipedia.org/wiki/Swedish_Match Temporal reasoning ['https://en.m.wikipedia.org/wiki/Red_Stripe', 'https://en.m.wikipedia.org/wiki/Swedish_Match'] 683 719 Which 2024 college/university president has a degree from Harvard University: the president from the one that organizes the Miami Book Fair or from the one that organizes the Kentucky Women Writers Conference? Eli Capilouto https://en.wikipedia.org/wiki/Miami_Book_Fair_International https://en.wikipedia.org/wiki/Miami_Dade_College https://en.wikipedia.org/wiki/Kentucky_Women_Writers_Conference https://en.wikipedia.org/wiki/University_of_Kentucky https://en.wikipedia.org/wiki/Eli_Capilouto https://en.wikipedia.org/wiki/Madeline_Pumariega Multiple constraints ['https://en.wikipedia.org/wiki/Miami_Book_Fair_International', 'https://en.wikipedia.org/wiki/Miami_Dade_College', 'https://en.wikipedia.org/wiki/Kentucky_Women_Writers_Conference', 'https://en.wikipedia.org/wiki/University_of_Kentucky', 'https://en.wikipedia.org/wiki/Eli_Capilouto', 'https://en.wikipedia.org/wiki/Madeline_Pumariega'] 684 720 What Doctor Who episode aired on a date closest to the 441st launch of the Skylark rocket? Dalek https://en.wikipedia.org/wiki/Doctor_Who_series_1 https://en.wikipedia.org/wiki/Skylark_(rocket) Tabular reasoning | Temporal reasoning ['https://en.wikipedia.org/wiki/Doctor_Who_series_1', 'https://en.wikipedia.org/wiki/Skylark_(rocket)'] 685 722 The shonen manga that won the 35th Kodansha Manga Award has how many chapters in its final volume? 5 https://en.wikipedia.org/wiki/Kodansha_Manga_Award https://en.wikipedia.org/wiki/List_of_Attack_on_Titan_chapters Tabular reasoning ['https://en.wikipedia.org/wiki/Kodansha_Manga_Award', 'https://en.wikipedia.org/wiki/List_of_Attack_on_Titan_chapters'] 686 723 How many years apart did a Kim Jong Un impersonator who was interviewed by USA today, and the man who founded Playboy attend the University of Illinois Urbana-Champaign? 60 https://en.wikipedia.org/wiki/University_of_Illinois_Urbana-Champaign#Notable_alumni_and_faculty https://en.wikipedia.org/wiki/Hugh_Hefner#Early_life_and_education https://en.wikipedia.org/wiki/Minyong_Kim Numerical reasoning | Multiple constraints | Temporal reasoning ['https://en.wikipedia.org/wiki/University_of_Illinois_Urbana-Champaign#Notable_alumni_and_faculty', 'https://en.wikipedia.org/wiki/Hugh_Hefner#Early_life_and_education', 'https://en.wikipedia.org/wiki/Minyong_Kim'] 687 724 In 2024, assuming that their family has been in the same line of work since they took a surname, what is the major use of the product they make if their last name is Kalkbrenner? Making steel. https://en.wikipedia.org/wiki/Kalkbrenner https://en.wikipedia.org/wiki/Lime_kiln https://en.wikipedia.org/wiki/Calcium_oxide Multiple constraints ['https://en.wikipedia.org/wiki/Kalkbrenner', 'https://en.wikipedia.org/wiki/Lime_kiln', 'https://en.wikipedia.org/wiki/Calcium_oxide'] 688 725 The person who posted a photo with Rahul Ligma and Daniel Johnson at the headquarters of a social media company claims to have a certain syndrome, despite never receiving a formal diagnosis. Who was this syndrome named after? Hans Asperger https://en.wikipedia.org/wiki/Rahul_Ligma https://en.wikipedia.org/wiki/Elon_Musk#Personal_life https://en.wikipedia.org/wiki/Asperger_syndrome Multiple constraints ['https://en.wikipedia.org/wiki/Rahul_Ligma', 'https://en.wikipedia.org/wiki/Elon_Musk#Personal_life', 'https://en.wikipedia.org/wiki/Asperger_syndrome'] 689 726 Emma Stone was the highest paid actress in 2017. How much did breakthrough movie for the highest paid actress 6 years after Emma Stone make in its theatrical run? $406.9 million https://en.wikipedia.org/wiki/Emma_Stone https://en.wikipedia.org/wiki/List_of_highest-paid_film_actors https://en.wikipedia.org/wiki/Margot_Robbie https://en.wikipedia.org/wiki/The_Wolf_of_Wall_Street_(2013_film) Tabular reasoning | Temporal reasoning ['https://en.wikipedia.org/wiki/Emma_Stone', 'https://en.wikipedia.org/wiki/List_of_highest-paid_film_actors', 'https://en.wikipedia.org/wiki/Margot_Robbie', 'https://en.wikipedia.org/wiki/The_Wolf_of_Wall_Street_(2013_film)'] 690 727 The lead actor who plays the regional manager of this popular mockumentary sitcom released in 2005 has the same initials as Santa Claus. What is the name of the voice character for Flower in the latest animated film this actor starred in in 2024? Matt Damon https://en.wikipedia.org/wiki/The_Office https://en.wikipedia.org/wiki/Steve_Carell#2004%E2%80%932013:_The_Office_and_comedic_roles https://en.wikipedia.org/wiki/IF_(film) Tabular reasoning | Multiple constraints ['https://en.wikipedia.org/wiki/The_Office', 'https://en.wikipedia.org/wiki/Steve_Carell#2004%E2%80%932013:_The_Office_and_comedic_roles', 'https://en.wikipedia.org/wiki/IF_(film)'] 691 728 This blood pressure drug commonly used to treat gestational hypertension was patented the same year the first black student at the University of Mississippi was shot. Labetalol https://en.wikipedia.org/wiki/University_of_Mississippi https://en.wikipedia.org/wiki/James_Meredith https://en.wikipedia.org/wiki/Gestational_hypertension https://en.wikipedia.org/wiki/Labetalol Temporal reasoning ['https://en.wikipedia.org/wiki/University_of_Mississippi', 'https://en.wikipedia.org/wiki/James_Meredith', 'https://en.wikipedia.org/wiki/Gestational_hypertension', 'https://en.wikipedia.org/wiki/Labetalol'] 692 729 Little River Canyon National Preserve has three waterfalls, one of which is the tallest in Alabama. How much shorter is Alabama's tallest waterfall than the tallest waterfall in the continental US? 2,452 ft https://en.wikipedia.org/wiki/Grace%27s_High_Falls https://en.wikipedia.org/wiki/List_of_waterfalls_by_height#By_overall_height Numerical reasoning | Tabular reasoning | Post processing ['https://en.wikipedia.org/wiki/Grace%27s_High_Falls', 'https://en.wikipedia.org/wiki/List_of_waterfalls_by_height#By_overall_height'] 693 730 What is the name of the retired Swiss tennis player who made the 4th round of Wimbledon in 2002? Michel Kratochvil https://en.wikipedia.org/wiki/2002_Wimbledon_Championships_%E2%80%93_Men%27s_singles https://en.wikipedia.org/wiki/Michel_Kratochvil Multiple constraints ['https://en.wikipedia.org/wiki/2002_Wimbledon_Championships_%E2%80%93_Men%27s_singles', 'https://en.wikipedia.org/wiki/Michel_Kratochvil'] 694 731 Who did the Canadian swimmer Eric Lamont compete against in Heat 3 of the freestyle competition that he was older than? No one, he was the youngest. https://en.wikipedia.org/wiki/Eric_Jubb https://en.wikipedia.org/wiki/Swimming_at_the_1948_Summer_Olympics_%E2%80%93_Men%27s_100_metre_freestyle https://en.wikipedia.org/wiki/Sachin_Nag https://en.wikipedia.org/wiki/Ali_Mustafa_Baghdady https://en.wikipedia.org/wiki/Warren_Boyd_(swimmer) https://en.wikipedia.org/wiki/Alberto_Isaac https://en.wikipedia.org/wiki/G%C3%A9za_K%C3%A1das Numerical reasoning | Multiple constraints | Temporal reasoning ['https://en.wikipedia.org/wiki/Eric_Jubb', 'https://en.wikipedia.org/wiki/Swimming_at_the_1948_Summer_Olympics_%E2%80%93_Men%27s_100_metre_freestyle', 'https://en.wikipedia.org/wiki/Sachin_Nag', 'https://en.wikipedia.org/wiki/Ali_Mustafa_Baghdady', 'https://en.wikipedia.org/wiki/Warren_Boyd_(swimmer)', 'https://en.wikipedia.org/wiki/Alberto_Isaac', 'https://en.wikipedia.org/wiki/G%C3%A9za_K%C3%A1das'] 695 732 I'm a spooky tourist and I'm visiting Savannah, GA. I'm going to visit two of the most well-known cemeteries, what Cemetery in Savannah is famous for being in a book and on a book cover? What was the book? There is another old cemetery downtown I want to visit, I heard epidemic victims were buried there. What epidemic and what years did it take place? How many victims are buried there? "Bonaventure Cemetery, ""Midnight in the Garden of Good and Evil."" Colonial Park, Yellow Fever; 1820s, estimated 700 victims." https://en.wikipedia.org/wiki/Bonaventure_Cemetery https://en.wikipedia.org/wiki/Colonial_Park_Cemetery https://en.wikipedia.org/wiki/Midnight_in_the_Garden_of_Good_and_Evil Multiple constraints ['https://en.wikipedia.org/wiki/Bonaventure_Cemetery', 'https://en.wikipedia.org/wiki/Colonial_Park_Cemetery', 'https://en.wikipedia.org/wiki/Midnight_in_the_Garden_of_Good_and_Evil'] 696 733 Of the 6 main cast members of the series Friends, which have appeared in music videos? Of the 6 main cast members of Friends, Jennifer Anniston, Courtney Cox, and Matt LeBlanc have all appeared in music videos. https://en.wikipedia.org/wiki/Friends https://en.wikipedia.org/wiki/Jennifer_Aniston https://en.wikipedia.org/wiki/Courteney_Cox https://en.wikipedia.org/wiki/Lisa_Kudrow https://en.wikipedia.org/wiki/Matt_LeBlanc https://en.wikipedia.org/wiki/Matthew_Perry https://en.wikipedia.org/wiki/David_Schwimmer Multiple constraints ['https://en.wikipedia.org/wiki/Friends', 'https://en.wikipedia.org/wiki/Jennifer_Aniston', 'https://en.wikipedia.org/wiki/Courteney_Cox', 'https://en.wikipedia.org/wiki/Lisa_Kudrow', 'https://en.wikipedia.org/wiki/Matt_LeBlanc', 'https://en.wikipedia.org/wiki/Matthew_Perry', 'https://en.wikipedia.org/wiki/David_Schwimmer'] 697 734 In what city were the Summer Olympic Games held in the year the RMS _Titanic_ sank? Stockholm (Sweden) https://en.wikipedia.org/wiki/Titanic https://en.wikipedia.org/wiki/1912_Summer_Olympics Multiple constraints ['https://en.wikipedia.org/wiki/Titanic', 'https://en.wikipedia.org/wiki/1912_Summer_Olympics'] 698 735 The manager Pep Guardiola won the Premier League four years in a row with a football club whose owners are from a country whose main export in 2009 was which raw material? Oil. ('Oil' is fine here but related words from the Wikipedia article (link 6) are also acceptable, such as 'petroleum') https://en.wikipedia.org/wiki/Pep_Guardiola https://en.wikipedia.org/wiki/Manchester_City_F.C. https://en.wikipedia.org/wiki/City_Football_Group https://en.wikipedia.org/wiki/Abu_Dhabi_United_Group https://en.wikipedia.org/wiki/United_Arab_Emirates https://en.wikipedia.org/wiki/Economy_of_the_United_Arab_Emirates Multiple constraints ['https://en.wikipedia.org/wiki/Pep_Guardiola', 'https://en.wikipedia.org/wiki/Manchester_City_F.C.', 'https://en.wikipedia.org/wiki/City_Football_Group', 'https://en.wikipedia.org/wiki/Abu_Dhabi_United_Group', 'https://en.wikipedia.org/wiki/United_Arab_Emirates', 'https://en.wikipedia.org/wiki/Economy_of_the_United_Arab_Emirates'] 699 736 As of August 3 2024, which surviving building of the World's Columbian Exposition of 1893 sits on the same street as a skyscraper over 1,000 feet tall? Give the surviving building's current name. Art Institute of Chicago https://en.wikipedia.org/wiki/World%27s_Columbian_Exposition https://en.wikipedia.org/wiki/Art_Institute_of_Chicago https://en.wikipedia.org/wiki/John_Hancock_Center Multiple constraints ['https://en.wikipedia.org/wiki/World%27s_Columbian_Exposition', 'https://en.wikipedia.org/wiki/Art_Institute_of_Chicago', 'https://en.wikipedia.org/wiki/John_Hancock_Center'] 700 737 How many more wins did the team with the number one seed from the NBA Western Conference in the 2020-2021 season have than the team with the fifth seed from the NBA Western Conference in the 2019-2020 season? 8 https://en.wikipedia.org/wiki/2020%E2%80%9321_NBA_season https://en.wikipedia.org/wiki/2019%E2%80%9320_NBA_season Numerical reasoning | Tabular reasoning ['https://en.wikipedia.org/wiki/2020%E2%80%9321_NBA_season', 'https://en.wikipedia.org/wiki/2019%E2%80%9320_NBA_season'] 701 738 "Cut It by O.T Genasis was released in 2015. What is the name of the streaming service that exclusively hosted the music video of the song that ranked one position above ""Cut It"" on the US Billboard Hot 100 of 2016?" Tidal https://en.wikipedia.org/wiki/Cut_It https://en.wikipedia.org/wiki/Billboard_Year-End_Hot_100_singles_of_2016 https://en.wikipedia.org/wiki/No_Limit_(Usher_song) Tabular reasoning | Temporal reasoning ['https://en.wikipedia.org/wiki/Cut_It', 'https://en.wikipedia.org/wiki/Billboard_Year-End_Hot_100_singles_of_2016', 'https://en.wikipedia.org/wiki/No_Limit_(Usher_song)'] 702 740 An episode of the first season of the show Digimon had an English air date exactly 2 years before 9/11 - what Digimon destroyed the Black Gear in that episode? Kabuterimon https://en.wikipedia.org/wiki/Digimon https://en.wikipedia.org/wiki/Digimon_Adventure_(1999_TV_series) https://en.wikipedia.org/wiki/List_of_Digimon_Adventure_(1999_TV_series)_episodes https://en.wikipedia.org/wiki/September_11_attacks Numerical reasoning | Tabular reasoning | Multiple constraints ['https://en.wikipedia.org/wiki/Digimon', 'https://en.wikipedia.org/wiki/Digimon_Adventure_(1999_TV_series)', 'https://en.wikipedia.org/wiki/List_of_Digimon_Adventure_(1999_TV_series)_episodes', 'https://en.wikipedia.org/wiki/September_11_attacks'] 703 741 How many years after Prohibition ended was Gone With The Wind released? 6 years. https://en.wikipedia.org/wiki/Prohibition_in_the_United_States https://en.wikipedia.org/wiki/Gone_with_the_Wind_(film) Temporal reasoning ['https://en.wikipedia.org/wiki/Prohibition_in_the_United_States', 'https://en.wikipedia.org/wiki/Gone_with_the_Wind_(film)'] 704 742 Who was on the British throne when the England Men’s Cricket Team first beat Australia in a Test Series? Queen Victoria, 1882. https://en.wikipedia.org/wiki/The_Ashes https://en.wikipedia.org/wiki/List_of_British_monarchs Multiple constraints ['https://en.wikipedia.org/wiki/The_Ashes', 'https://en.wikipedia.org/wiki/List_of_British_monarchs'] 705 743 How many years had the then-Prime Minister of France been in office as PM when the first Shelby GT500 was built? 5 https://en.wikipedia.org/wiki/Shelby_Mustang https://en.wikipedia.org/wiki/Georges_Pompidou Numerical reasoning | Multiple constraints | Temporal reasoning ['https://en.wikipedia.org/wiki/Shelby_Mustang', 'https://en.wikipedia.org/wiki/Georges_Pompidou'] 706 745 Azamat Satybaldy's appearance in the film Road to Mother occurred in the same year as the Trace Gas Orbiter's launch from what location? Baikonur Cosmodrome https://en.wikipedia.org/wiki/Azamat_Satybaldy https://en.wikipedia.org/wiki/Trace_Gas_Orbiter Tabular reasoning ['https://en.wikipedia.org/wiki/Azamat_Satybaldy', 'https://en.wikipedia.org/wiki/Trace_Gas_Orbiter'] 707 746 Who was the winner of the Nobel Peace Prize the year that U.S. President Barack Obama awarded baseball player Willie Mays the Presidential Medal of Freedom? The Tunisian National Dialogue Quartet https://en.wikipedia.org/wiki/Presidential_Medal_of_Freedom https://en.wikipedia.org/wiki/Willie_Mays https://en.wikipedia.org/wiki/Nobel_Peace_Prize https://en.wikipedia.org/wiki/List_of_Nobel_Peace_Prize_laureates https://en.wikipedia.org/wiki/Tunisian_National_Dialogue_Quartet Multiple constraints ['https://en.wikipedia.org/wiki/Presidential_Medal_of_Freedom', 'https://en.wikipedia.org/wiki/Willie_Mays', 'https://en.wikipedia.org/wiki/Nobel_Peace_Prize', 'https://en.wikipedia.org/wiki/List_of_Nobel_Peace_Prize_laureates', 'https://en.wikipedia.org/wiki/Tunisian_National_Dialogue_Quartet'] 708 747 "What song did Christina Aguilera release after Britney Spears released ""...Baby One More Time.""?" Genie in a Bottle https://en.wikipedia.org/wiki/...Baby_One_More_Time_(song) https://en.wikipedia.org/wiki/List_of_songs_recorded_by_Christina_Aguilera https://en.wikipedia.org/wiki/Genie_in_a_Bottle Temporal reasoning ['https://en.wikipedia.org/wiki/...Baby_One_More_Time_(song)', 'https://en.wikipedia.org/wiki/List_of_songs_recorded_by_Christina_Aguilera', 'https://en.wikipedia.org/wiki/Genie_in_a_Bottle'] 709 748 What primate species, known for its large population in China and presence in Florida, could potentially carry the Herpes B virus? Rhesus Macaques https://en.wikipedia.org/wiki/Wildlife_of_China https://en.wikipedia.org/wiki/Macaque Multiple constraints ['https://en.wikipedia.org/wiki/Wildlife_of_China', 'https://en.wikipedia.org/wiki/Macaque'] 710 749 What drug did the male founder of the company that first cloned a U.S. endangered species help study with the International Foundation for Advanced Study? LSD https://en.wikipedia.org/wiki/Elizabeth_Ann https://en.wikipedia.org/wiki/Revive_%26_Restore https://en.wikipedia.org/wiki/Stewart_Brand Multiple constraints ['https://en.wikipedia.org/wiki/Elizabeth_Ann', 'https://en.wikipedia.org/wiki/Revive_%26_Restore', 'https://en.wikipedia.org/wiki/Stewart_Brand'] 711 750 What made-for-TV movie did Dolly Parton have a role in the same year that Dolly the sheep was cloned? "In 1996, the year Dolly the sheep was cloned, Dolly Parton had a role in the made-for-tv movie ""Unlikely Angel""." https://en.wikipedia.org/wiki/Dolly_(sheep) https://en.wikipedia.org/wiki/Dolly_Parton Multiple constraints ['https://en.wikipedia.org/wiki/Dolly_(sheep)', 'https://en.wikipedia.org/wiki/Dolly_Parton'] 712 751 Before the COVID-19 pandemic, how many Juno Awards nominations did the Saskatoon bands The Northern Pikes, Wide Mouth Mason, and The Sheepdogs have combined? 18 https://en.wikipedia.org/wiki/The_Northern_Pikes https://en.wikipedia.org/wiki/Wide_Mouth_Mason https://en.wikipedia.org/wiki/The_Sheepdogs Numerical reasoning ['https://en.wikipedia.org/wiki/The_Northern_Pikes', 'https://en.wikipedia.org/wiki/Wide_Mouth_Mason', 'https://en.wikipedia.org/wiki/The_Sheepdogs'] 713 752 How many years did Cardi B's rap career overlap with Tupac's rap career? Zero https://en.wikipedia.org/wiki/Cardi_B https://en.wikipedia.org/wiki/Tupac_Shakur Temporal reasoning ['https://en.wikipedia.org/wiki/Cardi_B', 'https://en.wikipedia.org/wiki/Tupac_Shakur'] 714 753 What date did the Lego Avatar theme debut? How many years are there between the release of the original movie and the release of the Lego theme? The debut date was October 1st, 2022. It debuted 13 years after the release of the movie. https://en.wikipedia.org/wiki/Lego_Avatar https://en.wikipedia.org/wiki/Avatar_(franchise) Numerical reasoning | Temporal reasoning ['https://en.wikipedia.org/wiki/Lego_Avatar', 'https://en.wikipedia.org/wiki/Avatar_(franchise)'] 715 754 "What year did the author of ""The Conquest for Bread"" write about ""Mutual Aid""? Who was the author?" "Peter Kropotkin wrote the series of essays ""Mutual Aid"" in 1902. " https://en.wikipedia.org/wiki/The_Conquest_of_Bread https://en.wikipedia.org/wiki/Peter_Kropotkin https://en.wikipedia.org/wiki/Mutual_Aid:_A_Factor_of_Evolution Multiple constraints ['https://en.wikipedia.org/wiki/The_Conquest_of_Bread', 'https://en.wikipedia.org/wiki/Peter_Kropotkin', 'https://en.wikipedia.org/wiki/Mutual_Aid:_A_Factor_of_Evolution'] 716 755 "What were the last names of the players selected for the Pro Bowl from the NFL team that was featured in the movie ""Ace Ventura: Pet Detective""? Base the answer on the following specifications: -- These players were on the team while Wayne Huizenga was the owner -- The team that these players were on achieved a 10-6 regular season record while still making the playoffs during their Pro Bowl season" Bowens, Thomas, & Madison https://en.wikipedia.org/wiki/Ace_Ventura:_Pet_Detective https://en.wikipedia.org/wiki/Miami_Dolphins https://en.wikipedia.org/wiki/1995_Miami_Dolphins_season https://en.wikipedia.org/wiki/1997_Miami_Dolphins_season https://en.wikipedia.org/wiki/1998_Miami_Dolphins_season https://en.wikipedia.org/wiki/1999_Miami_Dolphins_season https://en.wikipedia.org/wiki/2000_Miami_Dolphins_season https://en.wikipedia.org/wiki/2001_Miami_Dolphins_season https://en.wikipedia.org/wiki/2008_Miami_Dolphins_season Tabular reasoning | Multiple constraints ['https://en.wikipedia.org/wiki/Ace_Ventura:_Pet_Detective', 'https://en.wikipedia.org/wiki/Miami_Dolphins', 'https://en.wikipedia.org/wiki/1995_Miami_Dolphins_season', 'https://en.wikipedia.org/wiki/1997_Miami_Dolphins_season', 'https://en.wikipedia.org/wiki/1998_Miami_Dolphins_season', 'https://en.wikipedia.org/wiki/1999_Miami_Dolphins_season', 'https://en.wikipedia.org/wiki/2000_Miami_Dolphins_season', 'https://en.wikipedia.org/wiki/2001_Miami_Dolphins_season', 'https://en.wikipedia.org/wiki/2008_Miami_Dolphins_season'] 717 756 Who placed 2nd and 3rd against Katharina Molitor in her World Winning Championship, and what was the difference between the final gold and silver throws, and silver and bronze throws? Katharina Molitor - 67.69m 1.56m difference Lü Huihui - 66.13m 0.34m difference Sunette Viljoen - 65.79m https://en.wikipedia.org/wiki/Katharina_Molitor https://en.wikipedia.org/wiki/2015_World_Championships_in_Athletics_%E2%80%93_Women%27s_javelin_throw Numerical reasoning | Temporal reasoning ['https://en.wikipedia.org/wiki/Katharina_Molitor', 'https://en.wikipedia.org/wiki/2015_World_Championships_in_Athletics_%E2%80%93_Women%27s_javelin_throw'] 718 757 How many years had passed since the Commonwealth of Pennsylvania was admitted to the Union by the time Rep. Robert D. Heaton was born? 85 https://en.wikipedia.org/wiki/Pennsylvania https://en.wikipedia.org/wiki/Robert_D._Heaton Temporal reasoning ['https://en.wikipedia.org/wiki/Pennsylvania', 'https://en.wikipedia.org/wiki/Robert_D._Heaton'] 719 758 Who was the president of the United States when the resort housing the BomBora steel roller coaster first opened? Grover Cleveland https://en.wikipedia.org/wiki/BomBora_(Lagoon) https://en.wikipedia.org/wiki/Lagoon_(amusement_park) https://en.wikipedia.org/wiki/List_of_presidents_of_the_United_States Multiple constraints ['https://en.wikipedia.org/wiki/BomBora_(Lagoon)', 'https://en.wikipedia.org/wiki/Lagoon_(amusement_park)', 'https://en.wikipedia.org/wiki/List_of_presidents_of_the_United_States'] 720 760 Which movie starring Meryl Streep was nominated for Best Picture at the Academy Awards the year that the Pioneer 11 visited Saturn? Kramer vs. Kramer https://en.wikipedia.org/wiki/Meryl_Streep https://en.wikipedia.org/wiki/Kramer_vs._Kramer https://en.wikipedia.org/wiki/Pioneer_11 https://en.wikipedia.org/wiki/The_Seduction_of_Joe_Tynan#Awards Multiple constraints ['https://en.wikipedia.org/wiki/Meryl_Streep', 'https://en.wikipedia.org/wiki/Kramer_vs._Kramer', 'https://en.wikipedia.org/wiki/Pioneer_11', 'https://en.wikipedia.org/wiki/The_Seduction_of_Joe_Tynan#Awards'] 721 761 In 1973, Edward Fox starred in The Day of the Jackal. He beat out a James Bond actor to the part, but what was the name of another James Bond actor he appeared with in an 'unofficial' 80s Bond film? Sean Connery https://en.wikipedia.org/wiki/The_Day_of_the_Jackal_(film) https://en.wikipedia.org/wiki/Edward_Fox_(actor) https://en.wikipedia.org/wiki/Roger_Moore https://en.wikipedia.org/wiki/Never_Say_Never_Again Multiple constraints ['https://en.wikipedia.org/wiki/The_Day_of_the_Jackal_(film)', 'https://en.wikipedia.org/wiki/Edward_Fox_(actor)', 'https://en.wikipedia.org/wiki/Roger_Moore', 'https://en.wikipedia.org/wiki/Never_Say_Never_Again'] 722 762 "In September 1607, two Earls and their followers left Rathmullan for Rome. The event was first called a ""flight"" in a book published in 1868. What is the name of the book?" "The Fate and Fortunes of Hugh O'Neill, Earl of Tyrone and Rory O'Donnel, Earl of Tyrconnel; their flight from Ireland, and death in exile (or ""Fate and Fortunes of the Earls of Tyrone and Tyrconnell"") by Charles Patrick Meehan" https://en.wikipedia.org/wiki/Rathmullan https://en.wikipedia.org/wiki/Flight_of_the_Earls#Journey https://en.wikipedia.org/wiki/Charles_Patrick_Meehan Multiple constraints | Temporal reasoning ['https://en.wikipedia.org/wiki/Rathmullan', 'https://en.wikipedia.org/wiki/Flight_of_the_Earls#Journey', 'https://en.wikipedia.org/wiki/Charles_Patrick_Meehan'] 723 763 The flavored ice brand Slush Puppie is actually named after a food made from cornmeal-batter. That food was itself originally named after a fish native to South Carolina. What’s the genus name of the fish? Moxostoma https://en.wikipedia.org/wiki/Slush_Puppie https://en.wikipedia.org/wiki/Hushpuppy https://en.wikipedia.org/wiki/Moxostoma Multiple constraints ['https://en.wikipedia.org/wiki/Slush_Puppie', 'https://en.wikipedia.org/wiki/Hushpuppy', 'https://en.wikipedia.org/wiki/Moxostoma'] 724 764 Gifts to children from Krampus are principally composed of what element? Krampus may give children a wooden rute or coal. They are both mostly composed of carbon. https://en.m.wikipedia.org/wiki/Krampus https://en.m.wikipedia.org/wiki/Coal https://en.m.wikipedia.org/wiki/Wood Multiple constraints ['https://en.m.wikipedia.org/wiki/Krampus', 'https://en.m.wikipedia.org/wiki/Coal', 'https://en.m.wikipedia.org/wiki/Wood'] 725 765 As noted in the 2020 census, what is the population of the county in which Waterville, Maine, resides? 123,642 https://en.wikipedia.org/wiki/Waterville,_Maine https://en.wikipedia.org/wiki/Kennebec_County,_Maine Multiple constraints ['https://en.wikipedia.org/wiki/Waterville,_Maine', 'https://en.wikipedia.org/wiki/Kennebec_County,_Maine'] 726 766 How many years apart were the Canadian Museum of History and the National Gallery of Canada established? 24 years https://en.wikipedia.org/wiki/Canadian_Museum_of_History https://en.wikipedia.org/wiki/National_Gallery_of_Canada Numerical reasoning ['https://en.wikipedia.org/wiki/Canadian_Museum_of_History', 'https://en.wikipedia.org/wiki/National_Gallery_of_Canada'] 727 767 Who was the team manager for Lee Jae-won's only football season as of January 1, 2024? Yasuyuki Kishino https://en.wikipedia.org/wiki/Lee_Jae-won_(footballer,_born_1992) https://en.wikipedia.org/wiki/2015_J3_League https://en.wikipedia.org/wiki/Kataller_Toyama Tabular reasoning | Temporal reasoning ['https://en.wikipedia.org/wiki/Lee_Jae-won_(footballer,_born_1992)', 'https://en.wikipedia.org/wiki/2015_J3_League', 'https://en.wikipedia.org/wiki/Kataller_Toyama'] 728 768 What medal was won in 1979 by the famous physicist who attended the oldest college in London? Albert Einstein medal https://en.wikipedia.org/wiki/University_of_Oxford#Mathematics_and_sciences https://en.wikipedia.org/wiki/Stephen_Hawking#1975%E2%80%931990 https://en.wikipedia.org/wiki/Albert_Einstein_Medal Multiple constraints ['https://en.wikipedia.org/wiki/University_of_Oxford#Mathematics_and_sciences', 'https://en.wikipedia.org/wiki/Stephen_Hawking#1975%E2%80%931990', 'https://en.wikipedia.org/wiki/Albert_Einstein_Medal'] 729 769 "What is the name of the ""pseudo label"" that collected the early collaborations of English architect Sir Peter Cook's son? " Gamsonite https://en.wikipedia.org/wiki/Peter_Cook_(architect) https://en.wikipedia.org/wiki/A._G._Cook Multiple constraints ['https://en.wikipedia.org/wiki/Peter_Cook_(architect)', 'https://en.wikipedia.org/wiki/A._G._Cook'] 730 770 Who won the third-place playoff at the UEFA World Cup while Donald Trump was in office as the 45th President of the United States? Belgium https://en.wikipedia.org/wiki/List_of_presidents_of_the_United_States https://en.wikipedia.org/wiki/FIFA_World_Cup Tabular reasoning ['https://en.wikipedia.org/wiki/List_of_presidents_of_the_United_States', 'https://en.wikipedia.org/wiki/FIFA_World_Cup'] 731 771 How old were the winners of the Men's Pairs division at the 1988 World Indoor Bowls Championship? 35 and 53 years old. https://en.wikipedia.org/wiki/1988_World_Indoor_Bowls_Championship https://en.wikipedia.org/wiki/Ian_Schuback https://en.wikipedia.org/wiki/Jim_Yates_(bowls) Numerical reasoning | Multiple constraints | Temporal reasoning ['https://en.wikipedia.org/wiki/1988_World_Indoor_Bowls_Championship', 'https://en.wikipedia.org/wiki/Ian_Schuback', 'https://en.wikipedia.org/wiki/Jim_Yates_(bowls)'] 732 772 In 1908 a fireboat operated by the Chicago Fire Department sank and was later refloated. When was the state that bears the same name as the fireboat founded? 1818 https://en.m.wikipedia.org/wiki/Illinois_(fireboat) https://en.m.wikipedia.org/wiki/Illinois Multiple constraints ['https://en.m.wikipedia.org/wiki/Illinois_(fireboat)', 'https://en.m.wikipedia.org/wiki/Illinois'] 733 773 What were the top 5 Billboard songs by musical groups in the year 1985? "1. ""Careless Whisper"" by Wham! 2. ""Wake Me Up Before You Go-Go"" by Wham! 3. ""I Want to Know What Love Is"" by Foreigner 4. ""Out of Touch"" by Hall & Oats. 5. ""Everybody Wants to Rule the World"" by Tears for Fears" https://en.wikipedia.org/wiki/Billboard_Year-End_Hot_100_singles_of_1985#:~:text=Article,11 https://en.wikipedia.org/wiki/Wham! https://en.wikipedia.org/wiki/I_Want_to_Know_What_Love_Is https://en.wikipedia.org/wiki/Out_of_Touch https://en.wikipedia.org/wiki/Everybody_Wants_to_Rule_the_World Multiple constraints ['https://en.wikipedia.org/wiki/Billboard_Year-End_Hot_100_singles_of_1985#:~:text=Article,11', 'https://en.wikipedia.org/wiki/Wham!', 'https://en.wikipedia.org/wiki/I_Want_to_Know_What_Love_Is', 'https://en.wikipedia.org/wiki/Out_of_Touch', 'https://en.wikipedia.org/wiki/Everybody_Wants_to_Rule_the_World'] 734 774 Tell me the name of the place I am thinking of based on these clues: - I am a metropolitan borough in the West Midlands, UK - I am not a city - My boroughs name does not relate to a town Sandwell. https://en.wikipedia.org/wiki/West_Midlands_(county) https://en.wikipedia.org/wiki/Metropolitan_Borough_of_Dudley https://en.wikipedia.org/wiki/Sandwell https://en.wikipedia.org/wiki/Metropolitan_Borough_of_Solihull https://en.wikipedia.org/wiki/Metropolitan_Borough_of_Walsall https://en.wikipedia.org/wiki/Birmingham https://en.wikipedia.org/wiki/Coventry https://en.wikipedia.org/wiki/Wolverhampton Multiple constraints ['https://en.wikipedia.org/wiki/West_Midlands_(county)', 'https://en.wikipedia.org/wiki/Metropolitan_Borough_of_Dudley', 'https://en.wikipedia.org/wiki/Sandwell', 'https://en.wikipedia.org/wiki/Metropolitan_Borough_of_Solihull', 'https://en.wikipedia.org/wiki/Metropolitan_Borough_of_Walsall', 'https://en.wikipedia.org/wiki/Birmingham', 'https://en.wikipedia.org/wiki/Coventry', 'https://en.wikipedia.org/wiki/Wolverhampton'] 735 775 What was the name of the worker-owned cooperative in Spain that recently started working with the United Steelworkers in the late 2000s and was associated with a social activist priest? Mondragon Corporation https://en.wikipedia.org/wiki/Jos%C3%A9_Mar%C3%ADa_Arizmendiarrieta https://en.wikipedia.org/wiki/Mondragon_Corporation https://en.wikipedia.org/wiki/United_Steelworkers Multiple constraints ['https://en.wikipedia.org/wiki/Jos%C3%A9_Mar%C3%ADa_Arizmendiarrieta', 'https://en.wikipedia.org/wiki/Mondragon_Corporation', 'https://en.wikipedia.org/wiki/United_Steelworkers'] 736 776 What was the former name of the brand of sneakers worn by the Heaven's Gate members who committed suicide? Blue Ribbon Sports Inc. https://en.wikipedia.org/wiki/Heaven%27s_Gate_(religious_group)#Nike_Decades https://en.wikipedia.org/wiki/Nike,_Inc. Multiple constraints ['https://en.wikipedia.org/wiki/Heaven%27s_Gate_(religious_group)#Nike_Decades', 'https://en.wikipedia.org/wiki/Nike,_Inc.'] 737 777 During World War I, the French designed a new military decoration to recognize French and allied soldiers for their service. The sculptor who designed the medal also worked on two war monuments one year after WWI ended. What is the name of the monument that he began work on, but was completed by someone else? Le Creusot War Memorial (monument aux morts) https://en.wikipedia.org/wiki/Croix_de_guerre_1914%E2%80%931918_(France)#Award_description https://en.wikipedia.org/wiki/World_War_I https://en.wikipedia.org/wiki/Albert_Bartholom%C3%A9#Main_works_(continued) Numerical reasoning | Tabular reasoning | Multiple constraints | Temporal reasoning ['https://en.wikipedia.org/wiki/Croix_de_guerre_1914%E2%80%931918_(France)#Award_description', 'https://en.wikipedia.org/wiki/World_War_I', 'https://en.wikipedia.org/wiki/Albert_Bartholom%C3%A9#Main_works_(continued)'] 738 778 Who wrote the first movie that Chris Columbus ever directed? David Simkins. https://en.wikipedia.org/wiki/Chris_Columbus_(filmmaker)#Filmography https://en.wikipedia.org/wiki/Adventures_in_Babysitting Tabular reasoning | Multiple constraints ['https://en.wikipedia.org/wiki/Chris_Columbus_(filmmaker)#Filmography', 'https://en.wikipedia.org/wiki/Adventures_in_Babysitting'] 739 779 What was the earliest known media use of the theme song used by the show The Last Leg? 2007 https://en.wikipedia.org/wiki/The_Last_Leg https://en.wikipedia.org/wiki/Harder_Than_You_Think https://en.wikipedia.org/wiki/Fully_Flared Multiple constraints | Temporal reasoning ['https://en.wikipedia.org/wiki/The_Last_Leg', 'https://en.wikipedia.org/wiki/Harder_Than_You_Think', 'https://en.wikipedia.org/wiki/Fully_Flared'] 740 780 Frank Lampard scored 5 league goals in his debut season at Chelsea. How many more league goals did Didier Drogba score during his debut season at Chelsea? 5 more goals https://en.wikipedia.org/wiki/Frank_Lampard https://en.wikipedia.org/wiki/Didier_Drogba Numerical reasoning | Tabular reasoning ['https://en.wikipedia.org/wiki/Frank_Lampard', 'https://en.wikipedia.org/wiki/Didier_Drogba'] 741 781 What do the inventor of the marine chronometer, the US president with the shortest tenure in history, and the president who was sworn in by Chief Justice Melville Fuller all have in common? "They all share the last name ""Harrison."" The scientist is John Harrison, the US president with the shortest tenure is William Henry Harrison, who is the grandfather of the US president sworn in by Chief Justice Melville Fuller, Benjamin Harrison. " https://en.wikipedia.org/wiki/Marine_chronometer https://en.wikipedia.org/wiki/List_of_presidents_of_the_United_States https://en.wikipedia.org/wiki/Melville_Fuller https://en.wikipedia.org/wiki/William_Henry_Harrison Tabular reasoning | Multiple constraints ['https://en.wikipedia.org/wiki/Marine_chronometer', 'https://en.wikipedia.org/wiki/List_of_presidents_of_the_United_States', 'https://en.wikipedia.org/wiki/Melville_Fuller', 'https://en.wikipedia.org/wiki/William_Henry_Harrison'] 742 782 Of the top five all-time scoring leaders of the National Basketball Association (NBA) and the Women's National Basketball Association (WNBA), which players were still playing professionally as of the 2024 season? LeBron James, Diana Taurasi, Tina Charles, and DeWanna Bonner https://en.wikipedia.org/wiki/List_of_NBA_career_scoring_leaders https://en.wikipedia.org/wiki/List_of_WNBA_career_scoring_leaders Tabular reasoning ['https://en.wikipedia.org/wiki/List_of_NBA_career_scoring_leaders', 'https://en.wikipedia.org/wiki/List_of_WNBA_career_scoring_leaders'] 743 783 Who was the Captain of the Toronto Maple Leafs when Morgan Rielly played his first game? Dion Phaneuf https://en.wikipedia.org/wiki/Morgan_Rielly https://en.wikipedia.org/wiki/Toronto_Maple_Leafs#Season-by-season_record Temporal reasoning ['https://en.wikipedia.org/wiki/Morgan_Rielly', 'https://en.wikipedia.org/wiki/Toronto_Maple_Leafs#Season-by-season_record'] 744 784 What is the famous novel by the wife of the 19th-century poet who wrote about an Egyptian pharaoh who reigned sometime between 1290 and 1200 B.C.? Frankenstein https://en.wikipedia.org/wiki/List_of_pharaohs https://en.wikipedia.org/wiki/Seti_I https://en.wikipedia.org/wiki/Ramesses_II https://en.wikipedia.org/wiki/Merneptah https://en.wikipedia.org/wiki/Amenmesse https://en.wikipedia.org/wiki/Percy_Bysshe_Shelley Tabular reasoning | Multiple constraints | Temporal reasoning ['https://en.wikipedia.org/wiki/List_of_pharaohs', 'https://en.wikipedia.org/wiki/Seti_I', 'https://en.wikipedia.org/wiki/Ramesses_II', 'https://en.wikipedia.org/wiki/Merneptah', 'https://en.wikipedia.org/wiki/Amenmesse', 'https://en.wikipedia.org/wiki/Percy_Bysshe_Shelley'] 745 785 How many Best Director winners from the Academy Awards in the 1990s were born before 1950? Three. Steven Spielberg, Jonathan Demme, Clint Eastwood. https://en.wikipedia.org/wiki/Academy_Award_for_Best_Director https://en.wikipedia.org/wiki/Sam_Mendes https://en.wikipedia.org/wiki/Steven_Spielberg https://en.wikipedia.org/wiki/James_Cameron https://en.wikipedia.org/wiki/Anthony_Minghella https://en.wikipedia.org/wiki/Robert_Zemeckis https://en.wikipedia.org/wiki/Jonathan_Demme https://en.wikipedia.org/wiki/Kevin_Costner https://en.wikipedia.org/wiki/Clint_Eastwood Numerical reasoning | Multiple constraints ['https://en.wikipedia.org/wiki/Academy_Award_for_Best_Director', 'https://en.wikipedia.org/wiki/Sam_Mendes', 'https://en.wikipedia.org/wiki/Steven_Spielberg', 'https://en.wikipedia.org/wiki/James_Cameron', 'https://en.wikipedia.org/wiki/Anthony_Minghella', 'https://en.wikipedia.org/wiki/Robert_Zemeckis', 'https://en.wikipedia.org/wiki/Jonathan_Demme', 'https://en.wikipedia.org/wiki/Kevin_Costner', 'https://en.wikipedia.org/wiki/Clint_Eastwood'] 746 786 who won the formula one season in the year nine people were killed on the track at the argentine grand prix, and how old were they two years before sliced bread was first sold? Alberto Ascari, 8 years old https://en.wikipedia.org/wiki/Argentine_Grand_Prix https://en.wikipedia.org/wiki/1953_Formula_One_season https://en.wikipedia.org/wiki/Alberto_Ascari https://en.wikipedia.org/wiki/Sliced_bread Numerical reasoning | Multiple constraints | Temporal reasoning ['https://en.wikipedia.org/wiki/Argentine_Grand_Prix', 'https://en.wikipedia.org/wiki/1953_Formula_One_season', 'https://en.wikipedia.org/wiki/Alberto_Ascari', 'https://en.wikipedia.org/wiki/Sliced_bread'] 747 787 What is the burial place of the most successful racehorse in the Grand National's history, as of 2024? The winning post at Aintree Racecourse https://en.wikipedia.org/wiki/Grand_National https://en.wikipedia.org/wiki/Red_Rum Multiple constraints ['https://en.wikipedia.org/wiki/Grand_National ', 'https://en.wikipedia.org/wiki/Red_Rum'] 748 788 Who is the male cousin, whose name begins with an L, of the actor who played the murderer in the ITV series White House Farm, and how old was he when he stood in the 2021 London mayoral election? Laurence Fox, 42 https://en.wikipedia.org/wiki/White_House_Farm_murders https://en.wikipedia.org/wiki/Freddie_Fox_(actor) https://en.wikipedia.org/wiki/Laurence_Fox https://en.wikipedia.org/wiki/2021_London_mayoral_election Numerical reasoning | Temporal reasoning ['https://en.wikipedia.org/wiki/White_House_Farm_murders', 'https://en.wikipedia.org/wiki/Freddie_Fox_(actor)', 'https://en.wikipedia.org/wiki/Laurence_Fox', 'https://en.wikipedia.org/wiki/2021_London_mayoral_election'] 749 789 In the 1984 Olympics, what sport did the country that got 5 total medals win a gold medal in? Sailing https://en.wikipedia.org/wiki/1984_Summer_Olympics_medal_table https://en.wikipedia.org/wiki/Spain_at_the_1984_Summer_Olympics https://en.wikipedia.org/wiki/Sailing_at_the_1984_Summer_Olympics_%E2%80%93_470 Tabular reasoning | Multiple constraints ['https://en.wikipedia.org/wiki/1984_Summer_Olympics_medal_table', 'https://en.wikipedia.org/wiki/Spain_at_the_1984_Summer_Olympics', 'https://en.wikipedia.org/wiki/Sailing_at_the_1984_Summer_Olympics_%E2%80%93_470'] 750 790 When Tom Hanks received his first Oscar, how many Grammys had Alan Menken won? 9 https://en.m.wikipedia.org/wiki/List_of_awards_and_nominations_received_by_Alan_Menken https://en.m.wikipedia.org/wiki/List_of_awards_and_nominations_received_by_Tom_Hanks https://en.m.wikipedia.org/wiki/36th_Annual_Grammy_Awards https://en.m.wikipedia.org/wiki/66th_Academy_Awards Temporal reasoning ['https://en.m.wikipedia.org/wiki/List_of_awards_and_nominations_received_by_Alan_Menken', 'https://en.m.wikipedia.org/wiki/List_of_awards_and_nominations_received_by_Tom_Hanks', 'https://en.m.wikipedia.org/wiki/36th_Annual_Grammy_Awards', 'https://en.m.wikipedia.org/wiki/66th_Academy_Awards'] 751 791 As of August 3rd, 2024, How high is the ancient standing stone located next to the A92 road? 3.5 metres https://en.wikipedia.org/wiki/A92_road https://en.wikipedia.org/wiki/Stone_of_Morphie Multiple constraints ['https://en.wikipedia.org/wiki/A92_road', 'https://en.wikipedia.org/wiki/Stone_of_Morphie'] 752 792 As of August 04, 2024, what is the exact age difference between Daniel Radcliff and his current partner in days? 1777 days https://en.wikipedia.org/wiki/Daniel_Radcliffe https://en.wikipedia.org/wiki/Erin_Darke Numerical reasoning | Multiple constraints | Temporal reasoning ['https://en.wikipedia.org/wiki/Daniel_Radcliffe', 'https://en.wikipedia.org/wiki/Erin_Darke'] 753 793 How long after the incorporation of Ottawa was the designer of Ottawa's Justice Building born? 10 years. https://en.wikipedia.org/wiki/Ottawa https://en.wikipedia.org/wiki/Justice_Building https://en.wikipedia.org/wiki/Thomas_W._Fuller Numerical reasoning | Temporal reasoning ['https://en.wikipedia.org/wiki/Ottawa', 'https://en.wikipedia.org/wiki/Justice_Building', 'https://en.wikipedia.org/wiki/Thomas_W._Fuller'] 754 794 How many times larger was the population of the city of Paris, 19 years after the year designated as The International Year for the Culture of Peace by the United Nations, than the population of Brown County, Kansas according to its 2020 census? Round the answer to the nearest whole number. 228 times larger https://en.wikipedia.org/wiki/Brown_County,_Kansas https://en.wikipedia.org/wiki/Demographics_of_Paris https://en.wikipedia.org/wiki/International_Year_for_the_Culture_of_Peace Numerical reasoning | Post processing | Temporal reasoning ['https://en.wikipedia.org/wiki/Brown_County,_Kansas', 'https://en.wikipedia.org/wiki/Demographics_of_Paris', 'https://en.wikipedia.org/wiki/International_Year_for_the_Culture_of_Peace'] 755 795 After Edward II, who was the next English monarch to have been born outside of England? Richard II https://en.wikipedia.org/wiki/Edward_II_of_England https://en.wikipedia.org/wiki/Edward_III_of_England https://en.wikipedia.org/wiki/Richard_II_of_England Temporal reasoning ['https://en.wikipedia.org/wiki/Edward_II_of_England', 'https://en.wikipedia.org/wiki/Edward_III_of_England', 'https://en.wikipedia.org/wiki/Richard_II_of_England'] 756 796 Who was the president of the USA when the wife of the former HEB Grocery Company CEO Howard Edward Butt Sr. died? Bill Clinton was president beginning in 1993, the same year that Mary Elizabeth Butt passed away. https://en.wikipedia.org/wiki/Mary_Elizabeth_Butt https://en.wikipedia.org/wiki/Bill_Clinton Multiple constraints | Temporal reasoning ['https://en.wikipedia.org/wiki/Mary_Elizabeth_Butt', 'https://en.wikipedia.org/wiki/Bill_Clinton'] 757 797 What was the population of the province which the town Robat-e Morad is located, in 2016? 1,429,475 https://en.wikipedia.org/wiki/Robat-e_Morad https://en.wikipedia.org/wiki/Markazi_province Multiple constraints ['https://en.wikipedia.org/wiki/Robat-e_Morad', 'https://en.wikipedia.org/wiki/Markazi_province'] 758 798 Who was the Mayor of Quincy, Massachusetts when the Quincy Medical Center opened and who was the Mayor when it closed? Henry O. Fairbanks and Thomas P. Koch https://en.wikipedia.org/wiki/Quincy_Medical_Center https://en.wikipedia.org/wiki/List_of_mayors_of_Quincy,_Massachusetts Numerical reasoning | Tabular reasoning | Temporal reasoning ['https://en.wikipedia.org/wiki/Quincy_Medical_Center', 'https://en.wikipedia.org/wiki/List_of_mayors_of_Quincy,_Massachusetts'] 759 800 I can't recall who I'm trying to think of. This person was the partner of someone in the 1984 Olympic keelboat competition. Their partner's sister was the only American who placed in the 2003 Pan American Games for sailing in the men's or women's competition (not including the open events). Richard Coxon. https://en.wikipedia.org/wiki/Sailing_at_the_2003_Pan_American_Games https://en.wikipedia.org/wiki/Lanee_Butler https://en.wikipedia.org/wiki/Colin_Beashel Multiple constraints ['https://en.wikipedia.org/wiki/Sailing_at_the_2003_Pan_American_Games', 'https://en.wikipedia.org/wiki/Lanee_Butler', 'https://en.wikipedia.org/wiki/Colin_Beashel'] 760 801 There's a famous children's book about a King Elephant. At what age did the French author die? 37 years old. https://en.wikipedia.org/wiki/Babar_the_Elephant https://en.wikipedia.org/wiki/Jean_de_Brunhoff Multiple constraints ['https://en.wikipedia.org/wiki/Babar_the_Elephant', 'https://en.wikipedia.org/wiki/Jean_de_Brunhoff'] 761 802 How many ballets did Frederick Ashton choreograph by the time he was 25 years old? 8 https://en.wikipedia.org/wiki/Frederick_Ashton https://en.wikipedia.org/wiki/List_of_ballets_choreographed_by_Frederick_Ashton Numerical reasoning | Tabular reasoning | Temporal reasoning ['https://en.wikipedia.org/wiki/Frederick_Ashton', 'https://en.wikipedia.org/wiki/List_of_ballets_choreographed_by_Frederick_Ashton'] 762 803 In the season before Jamal Murray won the NBA Championship, who was the fourth overall draft pick? Scottie Barnes https://en.wikipedia.org/wiki/Jamal_Murray https://en.wikipedia.org/wiki/2021_NBA_draft Multiple constraints | Temporal reasoning ['https://en.wikipedia.org/wiki/Jamal_Murray', 'https://en.wikipedia.org/wiki/2021_NBA_draft'] 763 805 What band had a male lead singer born on the day Mohammad Mosaddegh announced the dissolution of the Iranian parliament? Kool & the Gang https://en.wikipedia.org/wiki/1953_Iranian_parliamentary_dissolution_referendum https://en.wikipedia.org/wiki/August_16#1901%E2%80%93present_2 https://en.wikipedia.org/wiki/James_%22J.T.%22_Taylor Multiple constraints | Temporal reasoning ['https://en.wikipedia.org/wiki/1953_Iranian_parliamentary_dissolution_referendum', 'https://en.wikipedia.org/wiki/August_16#1901%E2%80%93present_2', 'https://en.wikipedia.org/wiki/James_%22J.T.%22_Taylor'] 764 806 Who won the French Open Men’s Singles tournament the year that New York City FC won their first MLS Cup title? Novak Djokovic https://en.wikipedia.org/wiki/List_of_MLS_Cup_finals#Results_by_team https://en.wikipedia.org/wiki/2021_French_Open_%E2%80%93_Men%2527s_singles Tabular reasoning ['https://en.wikipedia.org/wiki/List_of_MLS_Cup_finals#Results_by_team', 'https://en.wikipedia.org/wiki/2021_French_Open_%E2%80%93_Men%2527s_singles'] 765 807 In what year did a great fire destroy over 100 buildings in the North American city which hosted the 2010 G20 summit? 1904 (Great Toronto Fire of 1904) https://en.wikipedia.org/wiki/2010_G20_Toronto_summit https://en.wikipedia.org/wiki/Toronto Multiple constraints ['https://en.wikipedia.org/wiki/2010_G20_Toronto_summit', 'https://en.wikipedia.org/wiki/Toronto'] 766 808 What was the difference in population in the most populous town in the Isère department from 1946 to 1975? 63876 https://en.wikipedia.org/wiki/Is%C3%A8re#Principal_towns https://en.wikipedia.org/wiki/Grenoble#Population Numerical reasoning | Tabular reasoning | Temporal reasoning ['https://en.wikipedia.org/wiki/Is%C3%A8re#Principal_towns', 'https://en.wikipedia.org/wiki/Grenoble#Population'] 767 810 What team scored the most points in an NBA finals game while Ronald Reagan was president of the United States of America? Game 1 of the Finals in 1985 the Boston Celtics scored 148 points. https://en.wikipedia.org/wiki/Ronald_Reagan https://en.wikipedia.org/wiki/1981_NBA_Finals https://en.wikipedia.org/wiki/1982_NBA_Finals https://en.wikipedia.org/wiki/1983_NBA_Finals https://en.wikipedia.org/wiki/1984_NBA_Finals https://en.wikipedia.org/wiki/1985_NBA_Finals https://en.wikipedia.org/wiki/1986_NBA_Finals https://en.wikipedia.org/wiki/1987_NBA_Finals https://en.wikipedia.org/wiki/1988_NBA_Finals https://en.wikipedia.org/wiki/1989_NBA_Finals Tabular reasoning ['https://en.wikipedia.org/wiki/Ronald_Reagan', 'https://en.wikipedia.org/wiki/1981_NBA_Finals', 'https://en.wikipedia.org/wiki/1982_NBA_Finals', 'https://en.wikipedia.org/wiki/1983_NBA_Finals', 'https://en.wikipedia.org/wiki/1984_NBA_Finals', 'https://en.wikipedia.org/wiki/1985_NBA_Finals', 'https://en.wikipedia.org/wiki/1986_NBA_Finals', 'https://en.wikipedia.org/wiki/1987_NBA_Finals', 'https://en.wikipedia.org/wiki/1988_NBA_Finals', 'https://en.wikipedia.org/wiki/1989_NBA_Finals'] 768 811 The founder of the eponymous music school at the University of Rochester held the patent for an item that later earned him a star on the Hollywood Walk of Fame. How many years passed between his initial patent and the placement of his star? 75 https://en.wikipedia.org/wiki/Eastman_School_of_Music https://en.wikipedia.org/wiki/George_Eastman https://en.wikipedia.org/wiki/List_of_stars_on_the_Hollywood_Walk_of_Fame Numerical reasoning | Multiple constraints | Temporal reasoning ['https://en.wikipedia.org/wiki/Eastman_School_of_Music', 'https://en.wikipedia.org/wiki/George_Eastman', 'https://en.wikipedia.org/wiki/List_of_stars_on_the_Hollywood_Walk_of_Fame'] 769 813 What teammates were inducted into the college football hall of fame who played the same year as the first Native American to get a gold medal in the Olympics for the United States? " Gus Welch, and William ""Lone Star"" Dietz." https://en.wikipedia.org/wiki/Jim_Thorpe https://en.wikipedia.org/wiki/1911_Carlisle_Indians_football_team Multiple constraints ['https://en.wikipedia.org/wiki/Jim_Thorpe', 'https://en.wikipedia.org/wiki/1911_Carlisle_Indians_football_team'] 770 816 What is the the origin of the mascot of the school district that Donahoe, Texas is located within? England https://en.wikipedia.org/wiki/Donahoe,_Texas https://en.wikipedia.org/wiki/Bartlett_Independent_School_District https://en.wikipedia.org/wiki/Bulldog https://en.wikipedia.org/wiki/England Multiple constraints | Post processing ['https://en.wikipedia.org/wiki/Donahoe,_Texas', 'https://en.wikipedia.org/wiki/Bartlett_Independent_School_District', 'https://en.wikipedia.org/wiki/Bulldog', 'https://en.wikipedia.org/wiki/England'] 771 817 How many Mount Katahdins makes up the height of Mount Kilimanjaro? Mount Kilimanjaro stands at a height of 5,895 meters whereas Mount Katahdn stands at a height of 1,606 meters. Therefore, Mount Kilimanjaro is the height of approximately 3.7 Mount Katahdins. https://en.wikipedia.org/wiki/Mount_Kilimanjaro https://en.wikipedia.org/wiki/Mount_Katahdin Numerical reasoning ['https://en.wikipedia.org/wiki/Mount_Kilimanjaro', 'https://en.wikipedia.org/wiki/Mount_Katahdin'] 772 818 What's the name of Goku's iconic transformation and what episode number does it first appear in the anime? Goku becomes a Super Saiyan in episode 95 of Dragon Ball Z. https://en.wikipedia.org/wiki/Goku https://en.wikipedia.org/wiki/Dragon_Ball_Z https://en.wikipedia.org/wiki/List_of_Dragon_Ball_Z_episodes https://en.wikipedia.org/wiki/Dragon_Ball_Z_season_3 Tabular reasoning | Multiple constraints ['https://en.wikipedia.org/wiki/Goku', 'https://en.wikipedia.org/wiki/Dragon_Ball_Z', 'https://en.wikipedia.org/wiki/List_of_Dragon_Ball_Z_episodes', 'https://en.wikipedia.org/wiki/Dragon_Ball_Z_season_3'] 773 819 When was the last team that Miloš Beleslin played for dissolved? 1945 https://en.wikipedia.org/wiki/Milo%C5%A1_Beleslin https://en.wikipedia.org/wiki/%C5%BDAK_Subotica Numerical reasoning | Tabular reasoning | Multiple constraints | Temporal reasoning ['https://en.wikipedia.org/wiki/Milo%C5%A1_Beleslin', 'https://en.wikipedia.org/wiki/%C5%BDAK_Subotica'] 774 820 Of the six main/principal cast of The Simpsons, who was born first? Harry Shearer https://en.wikipedia.org/wiki/The_Simpsons https://en.wikipedia.org/wiki/List_of_The_Simpsons_cast_members https://en.wikipedia.org/wiki/Dan_Castellaneta https://en.wikipedia.org/wiki/Julie_Kavner https://en.wikipedia.org/wiki/Nancy_Cartwright https://en.wikipedia.org/wiki/Yeardley_Smith https://en.wikipedia.org/wiki/Hank_Azaria https://en.wikipedia.org/wiki/Harry_Shearer Multiple constraints ['https://en.wikipedia.org/wiki/The_Simpsons', 'https://en.wikipedia.org/wiki/List_of_The_Simpsons_cast_members', 'https://en.wikipedia.org/wiki/Dan_Castellaneta', 'https://en.wikipedia.org/wiki/Julie_Kavner', 'https://en.wikipedia.org/wiki/Nancy_Cartwright', 'https://en.wikipedia.org/wiki/Yeardley_Smith', 'https://en.wikipedia.org/wiki/Hank_Azaria', 'https://en.wikipedia.org/wiki/Harry_Shearer'] 775 821 How old was the Commander-in-Chief of India from 1865-1870 when he died? How old was his wife when she died? Average these two numbers, rounding up to the nearest whole integer if necessary. William Mansfield, 1st Baron Sandhurst was 57 at the time of his death. His wife, Margaret Mansfield, Baroness Sandhurst was aged 64 at the time of her death. Their average lifespan was 61 years old. https://en.wikipedia.org/wiki/William_Mansfield,_1st_Baron_Sandhurst https://en.wikipedia.org/wiki/Margaret_Mansfield,_Baroness_Sandhurst Numerical reasoning ['https://en.wikipedia.org/wiki/William_Mansfield,_1st_Baron_Sandhurst', 'https://en.wikipedia.org/wiki/Margaret_Mansfield,_Baroness_Sandhurst'] 776 822 One of Jane Auten's novels was released in 1813, which was later adapted into a film in 2005. When was the director of that film born? Joe Wright was born on August 25th, 1972. https://en.wikipedia.org/wiki/Jane_Austen#List_of_works https://en.wikipedia.org/wiki/Pride_and_Prejudice#Film,_television_and_theatre https://en.wikipedia.org/wiki/Pride_%26_Prejudice_(2005_film) https://en.wikipedia.org/wiki/Joe_Wright Multiple constraints ['https://en.wikipedia.org/wiki/Jane_Austen#List_of_works', 'https://en.wikipedia.org/wiki/Pride_and_Prejudice#Film,_television_and_theatre', 'https://en.wikipedia.org/wiki/Pride_%26_Prejudice_(2005_film)', 'https://en.wikipedia.org/wiki/Joe_Wright'] 777 823 Which leap year happened exactly halfway between the birth of Shoghi Effendi and the founding of the college he attended? 1580 https://en.wikipedia.org/wiki/Shoghi_Effendi https://en.wikipedia.org/wiki/Balliol_College,_Oxford https://en.wikipedia.org/wiki/1580 Numerical reasoning | Multiple constraints | Temporal reasoning ['https://en.wikipedia.org/wiki/Shoghi_Effendi', 'https://en.wikipedia.org/wiki/Balliol_College,_Oxford', 'https://en.wikipedia.org/wiki/1580'] 778 824 "What was the original name of the band the male lead of ""The Song"" founded?" Anthem Lights was originally known as Yellow Cavalier. https://en.wikipedia.org/wiki/The_Song_(2014_film) https://en.wikipedia.org/wiki/Anthem_Lights Multiple constraints ['https://en.wikipedia.org/wiki/The_Song_(2014_film)', 'https://en.wikipedia.org/wiki/Anthem_Lights'] 779 825 How old was the future 34th president 5 years after the founding of the National Football League? 35 https://en.wikipedia.org/wiki/National_Football_League https://en.wikipedia.org/wiki/List_of_presidents_of_the_United_States https://en.wikipedia.org/wiki/Dwight_D._Eisenhower Numerical reasoning | Multiple constraints | Post processing | Temporal reasoning ['https://en.wikipedia.org/wiki/National_Football_League', 'https://en.wikipedia.org/wiki/List_of_presidents_of_the_United_States', 'https://en.wikipedia.org/wiki/Dwight_D._Eisenhower'] 780 826 Of the three producers of the tv sitcom Friends, which two had ties to Philadelphia? David Crane and Marta Kauffman https://en.wikipedia.org/wiki/Friends https://en.wikipedia.org/wiki/Kevin_S._Bright https://en.wikipedia.org/wiki/Marta_Kauffman https://en.wikipedia.org/wiki/David_Crane_(producer) Multiple constraints ['https://en.wikipedia.org/wiki/Friends', 'https://en.wikipedia.org/wiki/Kevin_S._Bright', 'https://en.wikipedia.org/wiki/Marta_Kauffman', 'https://en.wikipedia.org/wiki/David_Crane_(producer)'] 781 827 "What movie won the Teen Choice Award for ""Choice Movie Liplock"" the same year George W. Bush gave his ""Mission Accomplished"" speech?" Sweet Home Alabama https://en.wikipedia.org/wiki/Mission_Accomplished_speech https://en.wikipedia.org/wiki/2003_Teen_Choice_Awards Multiple constraints ['https://en.wikipedia.org/wiki/Mission_Accomplished_speech', 'https://en.wikipedia.org/wiki/2003_Teen_Choice_Awards'] 782 828 Using only the Winter and Summer Olympic events that occurred in the same country in the same year, which year had the most total combined women competing in the olympics? 1936 https://en.wikipedia.org/wiki/Winter_Olympic_Games https://en.wikipedia.org/wiki/Summer_Olympic_Games https://en.wikipedia.org/wiki/1932_Winter_Olympics https://en.wikipedia.org/wiki/1932_Summer_Olympics https://en.wikipedia.org/wiki/1936_Winter_Olympics https://en.wikipedia.org/wiki/1936_Summer_Olympics Numerical reasoning | Multiple constraints | Temporal reasoning ['https://en.wikipedia.org/wiki/Winter_Olympic_Games', 'https://en.wikipedia.org/wiki/Summer_Olympic_Games', 'https://en.wikipedia.org/wiki/1932_Winter_Olympics', 'https://en.wikipedia.org/wiki/1932_Summer_Olympics', 'https://en.wikipedia.org/wiki/1936_Winter_Olympics', 'https://en.wikipedia.org/wiki/1936_Summer_Olympics'] 783 829 Of all states within New England in the United States, which had a population between 400,000 and 800,000 in 1920? Of the 6 states within New England, Maine, New Hampshire, and Rhode Island had populations between 400,000 and 800,000 in 1920. https://en.wikipedia.org/wiki/New_England https://en.wikipedia.org/wiki/Connecticut https://en.wikipedia.org/wiki/Maine https://en.wikipedia.org/wiki/Massachusetts https://en.wikipedia.org/wiki/New_Hampshire https://en.wikipedia.org/wiki/Rhode_Island https://en.wikipedia.org/wiki/Vermont Numerical reasoning | Tabular reasoning | Temporal reasoning ['https://en.wikipedia.org/wiki/New_England', 'https://en.wikipedia.org/wiki/Connecticut', 'https://en.wikipedia.org/wiki/Maine', 'https://en.wikipedia.org/wiki/Massachusetts', 'https://en.wikipedia.org/wiki/New_Hampshire', 'https://en.wikipedia.org/wiki/Rhode_Island', 'https://en.wikipedia.org/wiki/Vermont'] 784 830 In the three summer Olympics held in Europe between 1984 and 2020, how many more times did Australia place above South Korea in the final medal rankings? Once. https://en.wikipedia.org/wiki/Australia_at_the_Olympics https://en.wikipedia.org/wiki/South_Korea_at_the_Olympics Numerical reasoning | Tabular reasoning ['https://en.wikipedia.org/wiki/Australia_at_the_Olympics', 'https://en.wikipedia.org/wiki/South_Korea_at_the_Olympics'] 785 831 What was the difference in receiving yards from 2022 and 2023 for number 11 on the 49ers? Calculate the same 2022/2023 difference for the closest ranked WR for number 11's draft class and compare their numbers. The yard difference for Brandon Aiyuk is 327. The yard difference for Justin Jefferson is -735. Brandon improved more in 2023 but Justin's 2022 numbers were astronomical. https://en.wikipedia.org/wiki/Brandon_Aiyuk https://en.wikipedia.org/wiki/2020_NFL_draft https://en.wikipedia.org/wiki/Justin_Jefferson Numerical reasoning | Tabular reasoning | Multiple constraints | Post processing | Temporal reasoning ['https://en.wikipedia.org/wiki/Brandon_Aiyuk', 'https://en.wikipedia.org/wiki/2020_NFL_draft', 'https://en.wikipedia.org/wiki/Justin_Jefferson'] 786 832 Which element of the periodic table is a homonym of a synonym of a tool commonly used in dog walking? Lead https://en.wikipedia.org/wiki/Periodic_table https://en.wikipedia.org/wiki/Dog_walking https://en.wikipedia.org/wiki/Leash Tabular reasoning | Multiple constraints ['https://en.wikipedia.org/wiki/Periodic_table', 'https://en.wikipedia.org/wiki/Dog_walking', 'https://en.wikipedia.org/wiki/Leash'] 787 833 Which German Fairytale did one of the directors of Trolls voice act as a character in a franchise Movie? Walt Dohrn voice acted Rumpelstiltskin https://en.wikipedia.org/wiki/Trolls_(film) https://en.wikipedia.org/wiki/Mike_Mitchell_(director) https://en.wikipedia.org/wiki/Walt_Dohrn Numerical reasoning | Tabular reasoning | Multiple constraints | Post processing | Temporal reasoning ['https://en.wikipedia.org/wiki/Trolls_(film)', 'https://en.wikipedia.org/wiki/Mike_Mitchell_(director)', 'https://en.wikipedia.org/wiki/Walt_Dohrn'] 788 834 A Japanese aircraft carrier that was first built to become a luxury ocean liner was sunk by the U.S. torpedos in a battle four months prior to the Battle of Leyte Gulf in World War II. What is the name of the aircraft carrier and what battle did it sink in? The aircraft carrier Hiyō was sunk in the Battle of the Philippine Sea. https://en.wikipedia.org/wiki/Japanese_aircraft_carrier_Hiy%C5%8D https://en.wikipedia.org/wiki/Battle_of_the_Philippine_Sea Multiple constraints | Temporal reasoning ['https://en.wikipedia.org/wiki/Japanese_aircraft_carrier_Hiy%C5%8D', 'https://en.wikipedia.org/wiki/Battle_of_the_Philippine_Sea'] 789 835 The creator of the animated series Family Guy was supposed to be on one of the planes that was involved in the 9/11 attacks but he arrived too late to board. Altogether, how many letters are in the name of the city from which his missed fight departed that day? 6 (Boston) https://en.wikipedia.org/wiki/Family_Guy https://en.wikipedia.org/wiki/Seth_MacFarlane https://en.wikipedia.org/wiki/American_Airlines_Flight_11 Multiple constraints | Post processing ['https://en.wikipedia.org/wiki/Family_Guy', 'https://en.wikipedia.org/wiki/Seth_MacFarlane', 'https://en.wikipedia.org/wiki/American_Airlines_Flight_11'] 790 836 "On the same day Roald Dahl first published a work, a famous guitar player and singer was born. This person was once ranked 13th in Rolling Stone's ""100 Greatest Guitarists of All Time"" cover story. In the same year that this person was ranked 13th, who was ranked number 2 in Rolling Stone magazine's list of the 100 greatest guitarists of all time?" Duane Allman https://en.wikipedia.org/wiki/Roald_Dahl https://en.wikipedia.org/wiki/August_1942 https://en.wikipedia.org/wiki/Jerry_Garcia https://en.wikipedia.org/wiki/Duane_Allman Numerical reasoning | Multiple constraints ['https://en.wikipedia.org/wiki/Roald_Dahl', 'https://en.wikipedia.org/wiki/August_1942', 'https://en.wikipedia.org/wiki/Jerry_Garcia', 'https://en.wikipedia.org/wiki/Duane_Allman'] 791 837 I lost a final against Greg Rusedski on grass, who won his 4th career singles title on grass against a player that once defeated me in the opening round of the US Open. I achieved my highest ranking in singles on what date? 10 July 2000 https://en.wikipedia.org/wiki/Greg_Rusedski https://en.wikipedia.org/wiki/Karol_Kučera https://en.wikipedia.org/wiki/2003_US_Open_%E2%80%93_Men%27s_singles https://en.wikipedia.org/wiki/Alexander_Popp Tabular reasoning | Multiple constraints ['https://en.wikipedia.org/wiki/Greg_Rusedski', 'https://en.wikipedia.org/wiki/Karol_Kučera', 'https://en.wikipedia.org/wiki/2003_US_Open_%E2%80%93_Men%27s_singles', 'https://en.wikipedia.org/wiki/Alexander_Popp'] 792 838 As of 1st August 2024 Queens of the Stone Age, Them Crooked Vultures, Screaming Trees and Kyuss have one band member in common. What does his last name mean in English? Man https://en.wikipedia.org/wiki/Queens_of_the_Stone_Age https://en.wikipedia.org/wiki/Them_Crooked_Vultures https://en.wikipedia.org/wiki/Kyuss https://en.wikipedia.org/wiki/Homme https://en.wikipedia.org/wiki/Screaming_Trees Multiple constraints | Post processing ['https://en.wikipedia.org/wiki/Queens_of_the_Stone_Age', 'https://en.wikipedia.org/wiki/Them_Crooked_Vultures', 'https://en.wikipedia.org/wiki/Kyuss', 'https://en.wikipedia.org/wiki/Homme', 'https://en.wikipedia.org/wiki/Screaming_Trees'] 793 839 Joel Oshiro Dyck played professional ice hockey for three teams. Which of those teams were dissolved for financial difficulties after the 2018-2019 season? Joel Oshiro Dyck played for the Chatham Wheels, the Wheeling Thunderbirds, and the Nippon Paper Cranes. After the 2018-2019 ice hockey season, Nippon Paper Cranes were dissolved and replaced by the East Hokkaido Cranes. https://en.wikipedia.org/wiki/Joel_Dyck https://en.wikipedia.org/wiki/Nippon_Paper_Cranes Multiple constraints | Temporal reasoning ['https://en.wikipedia.org/wiki/Joel_Dyck', 'https://en.wikipedia.org/wiki/Nippon_Paper_Cranes'] 794 841 There is a statue, Avukana Buddha, that overlooks a reservoir in Sri Lanka. The reservoir was built by a king in 460 A.D. What is the name of the king's uncle who raised him? Mahanama https://en.wikipedia.org/wiki/Avukana_Buddha_statue#Location_and_appearance https://en.wikipedia.org/wiki/Kala_Wewa#History https://en.wikipedia.org/wiki/Dhatusena_of_Anuradhapura#Early_life_and_becoming_king Multiple constraints ['https://en.wikipedia.org/wiki/Avukana_Buddha_statue#Location_and_appearance', 'https://en.wikipedia.org/wiki/Kala_Wewa#History', 'https://en.wikipedia.org/wiki/Dhatusena_of_Anuradhapura#Early_life_and_becoming_king'] 795 842 This Canadian mountain, located in Cypress Provincial Park, got its name because one didn't have to cross it to reach The Lions (formerly Ch'ich'iyúy Elxwíkn'). What is the name of this mountain? Unnecessary Mountain https://en.wikipedia.org/wiki/Cypress_Provincial_Park https://en.wikipedia.org/wiki/The_Two_Sisters_(British_Columbia) https://en.wikipedia.org/wiki/Unnecessary_Mountain Multiple constraints ['https://en.wikipedia.org/wiki/Cypress_Provincial_Park', 'https://en.wikipedia.org/wiki/The_Two_Sisters_(British_Columbia)', 'https://en.wikipedia.org/wiki/Unnecessary_Mountain'] 796 843 Who won the FIFA World Cup in the year the Falklands War broke out? Italy https://en.wikipedia.org/wiki/Falklands_War https://en.wikipedia.org/wiki/1982_FIFA_World_Cup Temporal reasoning ['https://en.wikipedia.org/wiki/Falklands_War', 'https://en.wikipedia.org/wiki/1982_FIFA_World_Cup'] 797 844 John F. Kennedy was inaugurated into the presidency quite some time ago. Which song was number 1 on the Billboard Hot 100 chart on that same day, but 30 years after his inauguration? "Kennedy was inaugurated on January 20, 1961 In 1991, the number 1 song on the Billboard Hot 100 was ""Love Will Never Do (Without You)"" by Janet Jackson." https://en.wikipedia.org/wiki/Inauguration_of_John_F._Kennedy https://en.wikipedia.org/wiki/List_of_Billboard_Hot_100_number_ones_of_1991 Numerical reasoning | Tabular reasoning | Multiple constraints | Temporal reasoning ['https://en.wikipedia.org/wiki/Inauguration_of_John_F._Kennedy', 'https://en.wikipedia.org/wiki/List_of_Billboard_Hot_100_number_ones_of_1991'] 798 845 What genus does the animal that features in the upper left of the coat of arms of the area that the family of Jürgen Warnke moved to in 1945 belong to? Panthera https://en.wikipedia.org/wiki/J%C3%BCrgen_Warnke https://en.wikipedia.org/wiki/Upper_Franconia#Coat_of_arms https://en.wikipedia.org/wiki/Lion Multiple constraints ['https://en.wikipedia.org/wiki/J%C3%BCrgen_Warnke', 'https://en.wikipedia.org/wiki/Upper_Franconia#Coat_of_arms', 'https://en.wikipedia.org/wiki/Lion'] 799 846 How many minutes longer is the men's marathon record time (as of June 2024) than the duration of the shortest war in history? If multiple conflicting durations are given for the war, use the shortest one. Round the answer to the nearest whole minute. 83 https://en.wikipedia.org/wiki/Marathon_world_record_progression https://en.wikipedia.org/wiki/Anglo-Zanzibar_War Numerical reasoning | Post processing ['https://en.wikipedia.org/wiki/Marathon_world_record_progression', 'https://en.wikipedia.org/wiki/Anglo-Zanzibar_War'] 800 847 Which cities hosted the Olympics in 1988, and where were the opening ceremonies held in each city? Calgary- Winter Olympics, opening ceremony held at McMahon Stadium. Seoul- Summer Olympics, opening ceremony held at Seoul Olympic Stadium. https://en.wikipedia.org/wiki/List_of_Olympic_Games_host_cities https://en.wikipedia.org/wiki/1988_Summer_Olympics https://en.wikipedia.org/wiki/1988_Winter_Olympics Tabular reasoning ['https://en.wikipedia.org/wiki/List_of_Olympic_Games_host_cities', 'https://en.wikipedia.org/wiki/1988_Summer_Olympics', 'https://en.wikipedia.org/wiki/1988_Winter_Olympics'] 801 848 Which actor in the movie Nadja has a Golden Palm Star on the Walk of Stars in Palm Springs, California? Peter Fonda https://en.wikipedia.org/wiki/Nadja_(film) https://en.wikipedia.org/wiki/Palm_Springs_Walk_of_Stars https://en.wikipedia.org/wiki/Peter_Fonda Multiple constraints ['https://en.wikipedia.org/wiki/Nadja_(film)', 'https://en.wikipedia.org/wiki/Palm_Springs_Walk_of_Stars', 'https://en.wikipedia.org/wiki/Peter_Fonda'] 802 849 "The artist, MadLib, released Mind Fusion Vol. 1 as a collaboration with several other artists. The track, ""I Got A Right Ta (Madlib Remix)"" features an artist other than MadLib. This artist received a Bachelor's of Science degree from a university in Florida. How many years after the establishment of this university was the album Mind Fusion Vol. 1 released?" 117 https://en.wikipedia.org/wiki/Mind_Fusion https://en.wikipedia.org/wiki/Common_(rapper) https://en.wikipedia.org/wiki/Florida_A%26M_University Numerical reasoning | Multiple constraints ['https://en.wikipedia.org/wiki/Mind_Fusion', 'https://en.wikipedia.org/wiki/Common_(rapper)', 'https://en.wikipedia.org/wiki/Florida_A%26M_University'] 803 850 During the month that GEMA Global Engine Alliance LLC was founded as a joint venture of Chrysler, Mitsubishi Motors, and Hyundai Motor Company, which international arms treaty was signed, who signed it, where, and on what date? On May 24th, 2002, the Strategic Offensive Reductions Treaty was signed in Moscow by Vladimir Putin and George W. Bush. https://en.wikipedia.org/wiki/Global_Engine_Alliance https://en.wikipedia.org/wiki/Strategic_Offensive_Reductions_Treaty Multiple constraints ['https://en.wikipedia.org/wiki/Global_Engine_Alliance', 'https://en.wikipedia.org/wiki/Strategic_Offensive_Reductions_Treaty'] 804 851 Which Pope served the longest between the Battle of the Milvian Bridge and the end of the Civil Wars of the Tetrarchy? St. Sylvester I, whose Latin name was Silvester https://en.wikipedia.org/wiki/Battle_of_the_Milvian_Bridge https://en.wikipedia.org/wiki/Civil_wars_of_the_Tetrarchy https://en.wikipedia.org/wiki/List_of_popes Numerical reasoning | Multiple constraints | Post processing | Temporal reasoning ['https://en.wikipedia.org/wiki/Battle_of_the_Milvian_Bridge', 'https://en.wikipedia.org/wiki/Civil_wars_of_the_Tetrarchy', 'https://en.wikipedia.org/wiki/List_of_popes'] 805 852 Who won the season of the dance show that Tate McRae placed third in back in 2016? "Leon ""Kida"" Burns" https://en.wikipedia.org/wiki/Tate_McRae https://en.wikipedia.org/wiki/So_You_Think_You_Can_Dance:_The_Next_Generation_(American_TV_series) Tabular reasoning | Multiple constraints ['https://en.wikipedia.org/wiki/Tate_McRae', 'https://en.wikipedia.org/wiki/So_You_Think_You_Can_Dance:_The_Next_Generation_(American_TV_series)'] 806 853 What is greater: the combined 2011 populations of Rennington (Northumberland), Lydbrook (Gloucestershire), Stow-on-the-Wold (Gloucestershire) and Witney (Oxfordshire), or the 2022 population of London? The 2022 population of London https://en.wikipedia.org/wiki/Rennington https://en.wikipedia.org/wiki/Lydbrook https://en.wikipedia.org/wiki/Stow-on-the-Wold https://en.wikipedia.org/wiki/Witney https://en.wikipedia.org/wiki/London Numerical reasoning ['https://en.wikipedia.org/wiki/Rennington', 'https://en.wikipedia.org/wiki/Lydbrook', 'https://en.wikipedia.org/wiki/Stow-on-the-Wold', 'https://en.wikipedia.org/wiki/Witney', 'https://en.wikipedia.org/wiki/London'] 807 854 How many years old was The Real Housewives of New York City franchise when Jenna Lyons premiered on the show? 15 years old https://en.wikipedia.org/wiki/Jenna_Lyons https://en.wikipedia.org/wiki/The_Real_Housewives_of_New_York_City Numerical reasoning ['https://en.wikipedia.org/wiki/Jenna_Lyons', 'https://en.wikipedia.org/wiki/The_Real_Housewives_of_New_York_City'] 808 855 Two famous modernist writers were born and died on the same year. Who were they, which of them was alive for the longest, and by how many days? Virginia Woolf and James Joyce. Virginia Woolf lived 82 days longer. https://en.wikipedia.org/wiki/List_of_modernist_writers https://en.wikipedia.org/wiki/James_Joyce https://en.wikipedia.org/wiki/Virginia_Woolf Numerical reasoning | Multiple constraints | Temporal reasoning ['https://en.wikipedia.org/wiki/List_of_modernist_writers', 'https://en.wikipedia.org/wiki/James_Joyce', 'https://en.wikipedia.org/wiki/Virginia_Woolf'] 809 856 Which governor of Shizuoka resigned due to the delayed opening of the airport? Yoshinobu Ishikawa https://en.wikipedia.org/wiki/Shizuoka_Prefecture https://en.wikipedia.org/wiki/Yoshinobu_Ishikawa Tabular reasoning | Post processing ['https://en.wikipedia.org/wiki/Shizuoka_Prefecture', 'https://en.wikipedia.org/wiki/Yoshinobu_Ishikawa'] 810 857 According to topographical summit prominence, how many years were there between the first ascent of the United State's second most prominent mountain and the first ascent of Russia's second most prominent mountain? 35 years. https://en.wikipedia.org/wiki/List_of_mountain_peaks_by_prominence https://en.wikipedia.org/wiki/Mauna_Kea https://en.wikipedia.org/wiki/Klyuchevskaya_Sopka Multiple constraints | Temporal reasoning ['https://en.wikipedia.org/wiki/List_of_mountain_peaks_by_prominence', 'https://en.wikipedia.org/wiki/Mauna_Kea', 'https://en.wikipedia.org/wiki/Klyuchevskaya_Sopka'] 811 858 What is the difference between the number of years served in the seventh-ratified US state's House of Delegates between that state's senator elected in 2007 and his uncle? The seventh-ratified US state is Maryland. The senator of Maryland elected in 2007 is Ben Cardin. Ben Cardin served 20 years (1967 to 1987) and his uncle, Maurice Cardin, served 15 years (1951 to 1966) in the Maryland House of Delegates. 20 - 15 = 5 years difference, with Ben serving 5 more years. https://en.wikipedia.org/wiki/List_of_states_and_territories_of_the_United_States https://en.wikipedia.org/wiki/List_of_United_States_senators_from_Maryland https://en.wikipedia.org/wiki/Ben_Cardin https://en.wikipedia.org/wiki/Maurice_Cardin Numerical reasoning | Tabular reasoning | Multiple constraints | Temporal reasoning ['https://en.wikipedia.org/wiki/List_of_states_and_territories_of_the_United_States', 'https://en.wikipedia.org/wiki/List_of_United_States_senators_from_Maryland', 'https://en.wikipedia.org/wiki/Ben_Cardin', 'https://en.wikipedia.org/wiki/Maurice_Cardin'] 812 859 "What is the name of the father of the first cousin of the mother of the man whose name inspired the naming of the lunar mountain ""Mons Hansteen""?" Peter Treschow https://en.wikipedia.org/wiki/Mons_Hansteen https://en.wikipedia.org/wiki/Christopher_Hansteen https://en.wikipedia.org/wiki/Niels_Treschow Multiple constraints | Temporal reasoning ['https://en.wikipedia.org/wiki/Mons_Hansteen', 'https://en.wikipedia.org/wiki/Christopher_Hansteen', 'https://en.wikipedia.org/wiki/Niels_Treschow'] 813 860 What happened at the Dyatlov Pass Incident and how did it inspire the plot of 2013 horror film Devil Pass? The Dyatlov Pass Incident was an event in 1959 where nine Soviet Hiker's died in the Northern Ural Mountains after cutting open their tents and running into the snow for a reason without explanation. Devil Pass is a found footage film that takes place in the decades following the Dyatlov Pass Incident about a group of American students who travel to Russia to investigate the event. https://en.wikipedia.org/wiki/Dyatlov_Pass_incident https://en.wikipedia.org/wiki/Devil%27s_Pass Multiple constraints ['https://en.wikipedia.org/wiki/Dyatlov_Pass_incident', 'https://en.wikipedia.org/wiki/Devil%27s_Pass'] 814 861 Who was the winner of Tour de France the same year that a major catastrophic crash happened at Circuit de la Sarthe in Le Mans, France? Louison Bobet https://en.wikipedia.org/wiki/1955_Le_Mans_disaster https://en.wikipedia.org/wiki/List_of_Tour_de_France_general_classification_winners Multiple constraints ['https://en.wikipedia.org/wiki/1955_Le_Mans_disaster', 'https://en.wikipedia.org/wiki/List_of_Tour_de_France_general_classification_winners'] 815 862 A 2002 science fiction novel by an American author references La Llorona and themes of personal identity. What is the name of the trilogy that this author wrote under the same publisher? The Sea of Trolls trilogy https://en.wikipedia.org/wiki/La_Llorona#Literature https://en.wikipedia.org/wiki/The_House_of_the_Scorpion https://en.wikipedia.org/wiki/Nancy_Farmer#Bibliography Tabular reasoning | Multiple constraints ['https://en.wikipedia.org/wiki/La_Llorona#Literature', 'https://en.wikipedia.org/wiki/The_House_of_the_Scorpion', 'https://en.wikipedia.org/wiki/Nancy_Farmer#Bibliography'] 816 863 Mary Gaulden Jagger worked in the Biology Division of the Oak Ridge National Laboratory; what type of supercomputer, ranked by the TOP500 as the world's most powerful in June 2022, is present on the campus, and in what year did this supercomputer become operational? Frontier, 2022 https://en.wikipedia.org/wiki/Mary_Gaulden_Jagger https://en.wikipedia.org/wiki/Oak_Ridge_National_Laboratory https://en.wikipedia.org/wiki/Frontier_(supercomputer) Multiple constraints ['https://en.wikipedia.org/wiki/Mary_Gaulden_Jagger', 'https://en.wikipedia.org/wiki/Oak_Ridge_National_Laboratory', 'https://en.wikipedia.org/wiki/Frontier_(supercomputer)'] 817 864 "I grew up in a village on Long Island in the Town of Oyster Bay. The name of this town is made up of two words, the first starts with the letter ""S"" and the second with the letter ""C."" I went to a public elementary school in this village in the year 1999. What was the name of my school? " Sea Cliff Elementary School https://en.wikipedia.org/wiki/Oyster_Bay_(town),_New_York https://en.wikipedia.org/wiki/Sea_Cliff,_New_York https://en.wikipedia.org/wiki/North_Shore_School_District Multiple constraints ['https://en.wikipedia.org/wiki/Oyster_Bay_(town),_New_York', 'https://en.wikipedia.org/wiki/Sea_Cliff,_New_York', 'https://en.wikipedia.org/wiki/North_Shore_School_District'] 818 865 Who was the Catholic Pope eleven years after Emperor Charlemagne died? Eugene II https://en.wikipedia.org/wiki/Charlemagne https://en.wikipedia.org/wiki/List_of_popes Temporal reasoning ['https://en.wikipedia.org/wiki/Charlemagne', 'https://en.wikipedia.org/wiki/List_of_popes'] 819 866 How many years after publishing his paper *On the Inhalation of the Vapor of Ether* did John Snow make the connection between cholera, kidney failure, and contaminated water sources? Seven https://en.wikipedia.org/wiki/Miasma_theory# https://en.wikipedia.org/wiki/John_Snow# https://en.wikipedia.org/wiki/1854_Broad_Street_cholera_outbreak# Numerical reasoning | Multiple constraints | Temporal reasoning ['https://en.wikipedia.org/wiki/Miasma_theory#', 'https://en.wikipedia.org/wiki/John_Snow#', 'https://en.wikipedia.org/wiki/1854_Broad_Street_cholera_outbreak#'] 820 867 This singer represented Sweden in Eurovision four years before the Sweden Democrats entered Parliament for the first time. What astrological sign was the real person behind the character she played in her first musical? Aquarius https://en.wikipedia.org/wiki/Sweden_Democrats https://en.wikipedia.org/wiki/Eurovision_Song_Contest_2006 https://en.wikipedia.org/wiki/Carola_H%C3%A4ggkvist https://en.wikipedia.org/wiki/The_Sound_of_Music https://en.wikipedia.org/wiki/Maria_von_Trapp https://en.wikipedia.org/wiki/Astrological_sign Numerical reasoning | Temporal reasoning ['https://en.wikipedia.org/wiki/Sweden_Democrats', 'https://en.wikipedia.org/wiki/Eurovision_Song_Contest_2006', 'https://en.wikipedia.org/wiki/Carola_H%C3%A4ggkvist', 'https://en.wikipedia.org/wiki/The_Sound_of_Music', 'https://en.wikipedia.org/wiki/Maria_von_Trapp', 'https://en.wikipedia.org/wiki/Astrological_sign'] 821 868 Who was the king of England when Isaac Newton first published his Principia? James II of England https://en.wikipedia.org/wiki/Philosophi%C3%A6_Naturalis_Principia_Mathematica https://en.wikipedia.org/wiki/Monarchy_of_the_United_Kingdom https://en.wikipedia.org/wiki/James_II_of_England Temporal reasoning ['https://en.wikipedia.org/wiki/Philosophi%C3%A6_Naturalis_Principia_Mathematica', 'https://en.wikipedia.org/wiki/Monarchy_of_the_United_Kingdom', 'https://en.wikipedia.org/wiki/James_II_of_England'] 822 869 Which movie musical produced a song that was inspired by poetry from an American poet, who was born a week after Queen Victoria? Fame https://en.wikipedia.org/wiki/Queen_Victoria https://en.wikipedia.org/wiki/1819 https://en.wikipedia.org/wiki/Walt_Whitman Multiple constraints | Temporal reasoning ['https://en.wikipedia.org/wiki/Queen_Victoria', 'https://en.wikipedia.org/wiki/1819', 'https://en.wikipedia.org/wiki/Walt_Whitman'] 823 870 Diago Costa played for which club when he was awarded the first FIFA World Cup Goal based on a VAR Decision? Atlético Madrid https://simple.wikipedia.org/wiki/Video_assistant_referee https://en.wikipedia.org/wiki/2018_FIFA_World_Cup#Officiating https://en.wikipedia.org/wiki/Diego_Costa#Spain Tabular reasoning | Multiple constraints ['https://simple.wikipedia.org/wiki/Video_assistant_referee', 'https://en.wikipedia.org/wiki/2018_FIFA_World_Cup#Officiating', 'https://en.wikipedia.org/wiki/Diego_Costa#Spain']